You are on page 1of 307

Mastering

Psychiatry:
A core textbook for undergraduates

Melvyn WB Zhang
Cyrus SH Ho
Roger CM Ho
MCQ exam (Paper 1)

Questions Answers
History, mental state exam and
psychopathology
1. A 40-year-old man admitted to the The answer is B.
medical ward for upper gastrointestinal
tract bleeding is a known patient Explanation: He exhibits denial and denies the
dependent on alcohol. His amylase level reality that he is dependent on alcohol and
is high. During the interview, he leading to medical complication.
mentions that, ‘I don’t have a problem
with alcohol.’ What is the defence
mechanism? Year: 2013.

A. Acting out
B. Denial
C. Projection
D. Rationalization
E. Reaction formation.

2. A 50-year-old woman is referred by her The answer is C.


family doctor because she suffers from
depressive disorder. During the
Explanation: This patient has difficulty to
interview, she has difficulty in
verbalizing her emotions. The express her emotion and this phenomenon is
phenomenon is BEST described as: known as alexithymia.

A. Ambivalence
B. Affective flattening Year: 2013
C. Alexithymia
D. Alogia
E. Anhedonia.

3. Visual hallucinations are LEAST The answer is E.


likely to be found in which of the
following? Explanation: The other options A to D are
associated with visual hallucination. Patients
A. Alcohol withdrawal delirium suffering from Parkinson’s disease may present
B. Central anticholinergic delirium with visual hallucinations as a result of side
C. Lewy body dementia effect of madopar. As most patients suffering
D. Parkinson’s disease from schizophrenia in Singapore do not use
E. Schizophrenia. illicit drugs, visual hallucinations are relatively
less common.

Year: 2013

4. Which of the following is NOT a The answer is A.


common sign or symptom of panic
attack?
Explanation: As a result of sympathetic drive,
A. Hypotension the patient may experience hypertension
B. Palpitation rather than hypotension.
C. Shortness of breath
D. Sweating
E. Trembling. Year: 2013.

1
Cognitive assessment
5. Which of the following is NOT a The answer is B.
prominent cognitive feature of cortical
dementia such as Alzheimer’s disease?
Explanation: Anhedonia (loss of interest) and
A. Amnesia depression are more common in subcortical
B. Anhedonia dementia. The other 4 As: amnesia, apathy,
C. Apathy agnosia and apraxia are common in cortical
D. Agnosia dementia.
E. Apraxia.
Year: 2013

This is an advanced –level question.


Psychiatric epidemiology
6. In the community, the MOST common The answer is C.
psychiatric diagnosis over age 65 is:
Explanation: The National Institute of Mental
A. Alzheimer’s disease Health's Epidemiologic Catchment Area (ECA)
B. Late-onset schizophrenia program has found that the most common mental
C. Mood disorder disorder of old age is depressive disorder.
D. Delirium
E. Vascular dementia.
Year: 2013
Psychiatric aetiology, diagnosis and
classification
7. Which of the following is LEAST LIKELY to The answer is A.
be a risk factor for delusional disorder?
Explanation: The risk of delusional disorder
A. Anxiety increases with age. Hearing loss is a risk factor.
B. Increased age Low socioeconomic status and severe stress
C. Immigration are also risk factors.
D. Sensory impairment
E. Social isolation.
Year: 2013.

8. A female adolescent was raised by an The answer is A.


abusive father when she was young.
Although he has changed and is no longer Explanation: The father is a conditioned stimuli
abusive towards her, she still becomes and abuse is an unconditioned stimuli. The fear
anxious as soon as she sees him. What is is the conditioned response. Even without the
the BEST explanation based on abuse, the patient is fearful of the father. This
psychological theory? is known as classical conditioning.

A. Classical conditioning
B. Operant conditioning Year: 2013.
C. Free association
D. Reaction formation This is an advanced- level question.
E. Transference.

9. Which of the following factors is the MOST The answer is A.


IMPORTANT predictor of the development
of Post-Traumatic Stress Disorder (PTSD) in Explanation: Severity of event or accident is best
a 30-year-old man who has just been
predictor. Death of a friend in the accident indicates
involved in a road traffic accident?
severe accident.

2
a. Death of a friend in the accident
b. Duration of medical leave
c. His age and gender Year: 2013.
d. History of panic disorder
e. History of substance abuse.

10. Paul, a 20-year-old man, suffers from The answer is D.


schizophrenia. He has a monozygotic
twin brother called Peter. Based on the Explanation: Based on the genetic studies
findings from genetic studies, what is focusing on familial transmission of
the risk (in %) that Peter will develop schizophrenia, the risk is 47% for monozygotic
schizophrenia? twins of schizophrenia patients developing
schizophrenia.
A. 17%
B. 27% Risk of schizophrenia in Specific Populations
C. 37% Population Risk (%)
D. 47% General Population 1
E. 57%. Nontwin sibling of a 8
schizophrenic patient
Child with one schizophrenic 12
parent
Dizygotic twin of a schizophrenic 12
patient
Child of two schizophrenic 40
parents
Monozygotic twin of a 47
schizophrenic patient

Year: 2013.
11. The MOST important predisposing The answer is D.
factor for Attention Deficit and
Hyperactivity Disorder (ADHD) is:
Explanation: Foetal alcohol syndrome and
antenatal exposure to nicotine is the most
A. Adverse social economic status
B. Development of autism before the important predisposing factor for ADHD.
onset of ADHD. Option A is non-specific. Male gender, rather
C. Female gender than female gender is more common for
D. Prenatal exposure to alcohol and ADHD. Option E is an important predisposing
nicotine by mother. factor for conduct disorder.
E. Presence of antisocial personality
disorder in father.
Year: 2013.
12. A 40-year-old widow, said the following, The answer is D.
‘I can’t stop thinking about my husband.
The pain is unbearable. It’s been eight Explanation: She suffers from intense grief
months and it’s like it happened which persists for longer than 6 months
yesterday. I still can’t believe it. I can’t after the loss, traumatic distress, sense of
look at his picture, it’s too painful. I feel disbelief, anger and bitterness, distressing,
numb. And my friends don’t
intrusive thoughts related to the death,
understand. They still have their
husbands. And, I want to kill his doctor. avoidance of reminders of deceased and
It’s his fault my husband died. My life is separation distress.
completely empty. It’s just not fair’.
Based on her description, what is the
MOST correct diagnosis? Year: 2013.

A. Adjustment disorder
B. Antisocial personality disorder

3
C. Delusional disorder
D. Intense grief reaction
E. Schizophrenia.

General adult psychiatry


13. All of the following statements regarding The answer is E.
inpatient suicide are true EXCEPT:
A. Hanging is a common method used. Explanation: Option A, B, C, D are correct.
B. Inpatient suicide most often occurs during Hanging in bathroom is common and
home leave. bathroom needs to be specially designed. It is
C. Male inpatients are at higher risk of
important for nurses or ward staff to give
committing suicide during hospitalization.
patients or family a call during home leave or
D. The first week of admission is a high risk
immediately after discharge as these are high
period.
risk periods. Option D is incorrect. The risk for
E. The risk for women increases as they get
women plateaus or decreases with age. The
older.
risk for men shows two peaks: young age and
old age.

Year: 2013.
14. Which of the following is the MOST The answer is B.
important component of maintenance
treatment for schizophrenia? Explanation: Without maintenance
pharmacological treatment, 60-70% of
A. Cognitive behaviour therapy schizophrenia patients relapse within 1 year,
B. Monitored compliance in antipsychotic and almost 90% relapse in 2 years.
treatment Maintenance on antipsychotic therapy is the
C. Occupational rehabilitation single most important factor in preventing
D. Psychosocial rehabilitation rehospitalisation.
E. Family therapy.

Year: 2013.

15. Which of the following statements The answer is B.


regarding schizophrenia is FALSE?
A. Eye movement dysfunction may be a trait Explanation: Hallucination can occur in other
marker. psychiatric disorders. Eye movement
B. Hallucination is pathognomonic for dysfunction occurs in 50-85% of schizophrenia
schizophrenia. patients.
C. Intelligence continues to deteriorate with
the progression of the disorder.
D. Post schizophrenia depression occurs in Year: 2013.
25% of patients.
E. Prodromal signs and symptoms can be
non-specific.

16. Which of the following is NOT a sign or The answer is A.


symptom of atypical depression?
Explanation: Excessive guilt is part of
A. Excessive guilt melancholia.
B. Increased appetite
C. Interpersonal rejection sensitivity
D. Leaden paralysis Year: 2013.
E. Mood reactivity.

4
17. Which of the following change is Answer is E.
LEAST likely to occur in anorexia
nervosa, binge-purging type? Explanation: It should be hypophosphatemia
a. Hypoestrogenemia and hypocalcaemia. As binge-puruging type is
b. Hypomagnesemia
associated with recurrent vomiting, it will lead
c. Hypokalaemia
d. Hyperamylasemia to hypokalaemia and inflammation in salivary
e. Hyperphosphatemia. gland, i.e. hyperamylasemia. Anorexia nervosa
in general is associated with hypoestrogenemia
and hypomagnesemia.

Year: 2013

This is an advanced – level question.


18. Which of the following is classified as The answer is A.
circadian rhythm sleep disorder?
Explanation: Delayed sleep phase syndrome is
A. Delayed sleep phase syndrome
B. Obstructive sleep apnoea classified as circadian rhythm sleep disorder.
C. Kleine – Levin syndrome
D. Late-onset insomnia
E. Narcolepsy. Year: 2013

19. A 23-year-old medical student comes to The answer is E.


the clinic with a complaint of feeling
“worried.” He states that he may fail his Explanation: He suffers from social phobia
clinical exam in Medicine because he is because he develops marked fear which is
nervous about presenting a case and brought by being the focus of attention.
performing a physical examination in
front of examiners. When he thinks
about the exam, he feels nervous. He Year: 2013
skipped a lot of bedside teaching when
he knew it would be his turn to present a
case in front of his classmates and tutors.
He worries about criticism from his
classmates and tutor. The MOST likely
diagnosis is:
A. Agoraphobia
B. Generalized anxiety disorder
C. Normal shyness
D. Panic disorder
E. Social phobia.
Old age psychiatry
20. The MOST COMMON cause of psychotic The answer is C.
symptoms in elderly outpatients in
Singapore is: Explanation: Dementia is the most common
cause for psychotic symptoms in elderly
A. Alcohol abuse outpatients.
B. Anxiety
C. Dementia
D. Depression Year: 2013.
E. Delirium.

5
21. Which of the following statements is The answer is E.
TRUE about a 76-year-old woman
suffering from an acute confusional state Explanation: In elderly, the mortality of
after a total hip replacement? delirium and acute confusional state is
between 6 to 18%. Generalized slowing is the
A. Anticholinergic drug is the treatment of most common finding on EEG. Her age is a risk
choice. factor.
B. Her age and gender are not a risk factor.
C. She should be allowed to leave the ward
against doctor advice should she wish to. Year: 2013
D. Fast, spike waves are often seen on the
EEG.
E. There is a mortality of up to 15%.

22. A 65-year-old person is referred for The answer is A.


early dementia. Which of the following
is classified as basic activities of daily
Explanation: A is considered to be basic
living?
activities of daily living while the others are
A. Bathing classified as instrumental activities of daily
B. Doing household chores and laundry living.
C. Managing medication
D. Preparing food
E. Shopping and managing finances.
Year: 2013
23. Which of the following The answer is B.
drugs/substances is LEAST likely to be
abused by old people?
Explanation: Old people are most likely to
A. Alcohol misuse alcohol or prescribed drugs than
B. Amphetamine street/illicit drugs such as stimulant.
C. Analgesics
D. Anticholinergics
E. Cough mixture. Year: 2013.

24. You are the resident working at the The answer is A.


Accident and Emergency Department. A
70-year-old woman with a history of Explanation: Elderly with dementia are
Alzheimer’s disease presents to you after sensitive to side effects. As a result, the doctor
a fall. She had been experiencing should adjust the dose slowly with long
difficulty sleeping at night for several intervals between increments. Routine
months and was increasingly suspicious prescription of benzodiazepine to elderly with
of her husband for harming her. You dementia is not recommended as it will lead to
have decided to start an antipsychotic. confusion.
Which of the following practices is MOST
appropriate?
Year: 2013.
A. Adjust doses of antipsychotics
slowly with long intervals
between dose increments.
B. Augmentation with
benzodiazepine is highly
recommended.

6
C. Prescribe small doses of
several different antipsychotics
rather than using one
antipsychotic.
D. Use a long-acting injectable
antipsychotic to aid adherence.
E. Use a rapid loading dose of
antipsychotic to speed up the
response.
Child and adolescent psychiatry
25. Which of the following is the MOST The answer is B.
COMMON disorder for children and
adolescents to be referred to see a child Explanation: Children with externalizing
and adolescent psychiatrist? disorders (e.g. conduct disorder) are more
likely to be referred and treated. 10% of
A. Child-onset schizophrenia children and adolescents seeing a psychiatrist
B. Conduct disorder have conduct problem. Separation anxiety
C. Down syndrome disorder and Down Syndrome may not need to
D. Separation anxiety disorder see a psychiatrist. 0.1% have psychotic disorder
E. Somatization disorder. and 1% have somatization disorder.

Year: 2013
26. An 8-year-old boy presents with The answer is A.
stereotypies, pronoun reversal, word
substitution, social problems, and Explanation: Repetitive movement,
echolalia, the most likely diagnosis is:
communication problem and social problems
indicate this boy suffers from autism.
A. Autism
B. Attention deficit and hyperactivity
disorder
Year: 2013.
C. Conduct disorder
D. Rett’s syndrome
E. Separation anxiety disorder.

27. A 14-year-old adolescent continues to The answer is D.


be significantly depressed despite
actively participating in psychotherapy
Explanation: Starting an antidepressant as an
for 3 months. Which of the following is
the best treatment approach? adjunct to psychotherapy after the adolescent
has failed to respond to psychotherapy alone is
A. Change psychotherapy approach a standard approach to the treatment of
B. Start amitriptyline adolescent depression.
C. Start electroconvulsive therapy
D. Start fluoxetine
E. Start olanzapine.
Year: 2013

28. Which of the listed disorders is the The answer is D.


MOST common co-morbidity with ADHD
in children?
Explanation: sleep disturbance, conduct
A. Autism
B. Learning disorders in mathematics disorder, oppositional defiant disorder,
C. Learning disorders in expressive depression, anxiety and substance abuse are
language common comorbidity with ADHD.

7
D. Oppositional defiant disorder
E. Gender identity disorder of
childhood. Year: 2013.

Substance abuse
29. Based on clinical severity, men from The answer is C.
which of the following ethnicities are
MOST vulnerable to alcohol
Explanation: Indian men seem to be most
dependence in Singapore?
vulnerable to alcohol dependence in
A. Chinese Singapore. Malay men have the lowest risk.
B. Eurasian
C. Indian
D. Malay Year: 2013.
E. Other ethnic groups.

30. The aspartate aminotransferase (AST) The answer is E.


/alanine aminotransferase (ALT) ratio in
alcohol hepatitis is: Explanation: When greater than 1.0 but less
than 2.0, it is likely to be associated with
1. < 0.5
cirrhosis. If <1, it indicates viral hepatitis.
2. 0.6 – 0.9
3. 1 – 1.4
4. 1.5 – 1.9
5. > 2. Year: 2013.

This is an advanced –level question.


31. A 20-year-old man is suspected of The answer is C.
acute stimulant intoxication. His
symptoms are MOST LIKELY TO The syndrome picture of acute cocaine or
resemble which of the following amphetamine intoxication can include
conditions? hyperawareness, hypersexuality,
hypervigilance, agitation, paranoia, and
A. Alcohol intoxication delusions. It resembles mania or psychosis.
B. Antisocial personality disorder
C. Mania
D. Obsessive-compulsive disorder Year: 2013.
E. Panic disorder.

32. A 30-year-old man is dependent on 3 Answer: A


mg alprazolam (Xanax) on a daily
basis. He wants to stop his Explanation: Diazepam is the longest
dependence on alprazolam. Which acting benzodiazepine and this will prevent
of the following is the BEST withdrawal symptoms such as withdrawal
management? fit.
A. Change to 15 mg diazepam and
then taper off gradually
B. Change to 100mg hydroxyzine and Year: 2013
then taper off gradually
C. Change to 400mg lithium and then
taper off gradually
D. Change to 10 mg olanzapine and
then taper off gradually
E. Change to 200mg quetiapine and
then taper off gradually.

8
33. A 30-year-old man drinks alcohol and The answer is A.
develops flushing and tachycardia. He
took disulfiram 2 hours ago.
Explanation: The mechanism of actions of
Accumulation of which of the following
is accountable for the above disulfiram is to inhibit aldehyde dehydrogenase
phenomenon? and lead to acetaldehyde accumulation.

A. Acetaldehyde
B. Acetylcholine Year: 2013
C. Adrenaline
D. Alanine
E. Aspartic acid. This is an advanced-level question.

Psychopharmacology
34. Which of the following tricyclic The answer is B.
antidepressants is MOST effective in
the treatment of obsessive compulsive
Explanation: Among all TCAs, clomipramine is
disorder?
most potent is blocking 5HT reuptake.
A. Amitriptyline
B. Clomipramine
C. Desipramine Year: 2013
D. Doxepin
E. Imipramine.

35. A depressed patient does not want to The answer is B.


take medication on a daily basis. The
selective serotonin reuptake inhibitor
The have life of fluoxetine is longer than 72
(SSRI) with the LONGEST half-life is?
hours and patient can take the medication
A. Escitalporam every other day.
B. Fluoxetine
C. Fluvoxamine
D. Paroxetine Year: 2013
E. Sertraline.

36. The antidepressant, mirtazapine is less The answer is C.


likely to cause nausea because of its
effects on which of the follow
Explanation: Its effect on 5HT3 receptors is less
receptors?
likely to cause nausea side effect as compared
to SSRIs.
A. Serotonin 5HT1 receptors
B. Serotonin 5HT2 receptors
C. Serotonin 5HT3 receptors
Year: 2013.
D. Serotonin 5HT4 receptors
E. Serotonin 5HT5 receptors.
This is an advanced –level question.

37. Acetylcholinesterase inhibitors The answer is E.


(AChEIs) are BEST conceptualized
as: Explanation: AChEIs are best
conceptualized as drugs that stabilize
A. Contraindicated in the treatment of cognition, activities of daily living, and
Lewy body dementia. behavioural function and slow clinical
B. Only indicated for severe stages of deterioration in Alzheimer's disease but not
Alzheimer's disease. significant improvement from baseline.
C. Predominately associated with the side AChEI e.g. rivastigmine can be used for
effect of metabolic syndrome. dementia with Lewy bodies.

9
D. Improving cognitive function Gastrointestinal side effects e.g. nausea,
significantly from baseline. vomiting and diarrhoea are the most
E. Stabilizing cognition, activities of daily common adverse effects.
living, and behavioural function.

Year: 2013.
38. Which of the following is FALSE The answer is D.
regarding the first generation
antipsychotics (FGAs)? Explanation: The second generation
antipsychotics have higher risk for metabolic
A. FGAs have high D2 receptor blocking syndrome as compared to the FGAs.
effects.
B. FGAs have increased risk of
extrapyramidal side effects as Year: 2013.
compared with the second generation
antipsychotics.
C. FGAs have increased risk of tardive
dyskinesia as compared with the
second generation antipsychotics.
D. FGAs have increased risk of causing
metabolic syndrome as compared with
the second generation antipsychotics.
E. FGAs have proven efficacy as
pharmacological treatment for
schizophrenia.

39. All of the following are true of a patient The answer is D.


on risperidone 6mg daily who gets
parkinsonism side effects, EXCEPT:
Explanation: Increase the dose will worsen
A. 75% occupancy at D2 receptors is parkinsonism.
associated with parkinsonism.
B. This patient is at higher risk for developing
secondary negative symptoms. Year: 2013.
C. The dose is above the “antipsychotic
threshold” in this case.
D. Raising the dose of risperidone leads to a
paradoxical reduction in parkinsonism.
E. Anticholinergic drug can reduce
parkinsonism in this patient.

40. Which of the following herbs has The answer is E.


been MOST commonly used by
patients to treat mild to moderate Explanation: St John’s Wort is the most
depression in developed countries? commonly used herb to treat depression in
western countries. It shares similar properties
A. Gingko
as other SSRIs such as increase in bleeding.
B. Ginseng
C. Kava kava
D. Passion flower
Year: 2013.
E. St. John’s Wort.

41. A 40-year-old female patient with The answer is E.


schizophrenia has been treated with
haloperidol and trifluoperazine for many Explanation: Tardive dyskinesia is characterised

10
years in the past. Currently, she exhibits by lip smacking, chewing and fly catching
a non-rhythmical hyperkinetic movement tongue protrusion after long-term treatment of
disorder of the lips, jaw and tongue. The antipsychotics.
movement disorder MOST likely
consistent with this finding is:
Year: 2013.
A. Acute dystonia
B. Akathisia
C. Aphasia
D. Pseudoparkinsonism
E. Tardive dyskinesia.

42. A 35-year-old accountant presents The answer is C.


to a psychiatrist 8 months after a
motor vehicle accident. He has Explanation: SSRIs are generally most
difficulty sleeping because he has appropriate medication of choice for post
frequent nightmares about the traumatic stress disorder. Examples include
accident. He has not been able to paroxetine, sertraline and fluoxetine.
drive since the accident, and his
wife usually drives for him. Even
then, he finds it very difficult to be in
Year: 2013
a car such as a taxi, after panicking
if another car is near them on the
road. Which of the following
medications would be the MOST
appropriate for him?

A. Amitriptyline
B. Clonazepam
C. Paroxetine
D. Propranolol
E. Quetiapine.

Psychotherapy
43. A 66-year-old man who worked as an The answer is D.
executive accountant previously. He has
retired recently and presents with a Explanation: IPT and CBT are the evidence-
major depressive disorder. What is the based psychotherapy indicated for major
best choice of psychotherapy? depressive disorder. IPT can help him to work
on role transition, grief and loss.
A. Biofeedback
B. Eye movement desensitization and
reprocessing Year: 2013
C. Hypnosis
D. Interpersonal psychotherapy
E. Systemic desensitization.

44. A 40-year-old man with obsessive- The answer is A.


compulsive disorder presents for
treatment. He would prefer not using a
Explanation: Exposure with response
medication due to possible side effects.
Which of the following psychological prevention (ERP) is part of CBT, the
interventions would be the treatment of psychotherapeutic treatment of choice for
choice for his disorder? obsessive-compulsive disorder. In ERP, patients

11
are exposed to the feared stimuli and
A. Exposure and response prevention obsessions while rituals that typically serve to
B. Hypnotherapy reduce anxiety are prevented.
C. Interpersonal psychotherapy
D. Psychoanalysis
E. Supportive psychotherapy.
Year: 2013
45. Effective psychotherapy techniques The answer is D.
for Borderline Personality Disorder
include all of the following features Explanation: Distant therapist may have
EXCEPT: difficulty to establish trust and therapeutic
A. A treatment contract should be alliance with patient suffering from
established in the beginning borderline personality disorder.
phase of psychotherapy.
B. Adverse effects of self-
laceration are identified. Year: 2013.
C. Focus of treatment is to
establish connection between
actions and feelings.
D. Therapist is quiet, distant and
does not get too involved.
E. Therapist pays careful attention
to his or her own feelings in
order to manage
countertransference.

Ethics and laws


46. A woman with a history of Answer: The answer is A.
depression that responded well to
antidepressant medication is now Explanation: Autonomy refers to the obligation
depressed again. She does not want of a doctor to respect patient’s right to make
to take medications due to concerns her own choice of treatment (i.e.
about side effects and risks. She psychotherapy instead of medication in this
would like to try psychotherapy case).
instead. Her husband disagrees
with her. Which of the following
Fiduciary duty refers to the duty that a doctor
ethical principles is MOST relevant
in this situation? should act as the best interest of the patient. In
A. Autonomy this scenario, it is less relevant as compared to
B. Beneficence autonomy.
C. Fiduciary duty
D. Non-maleficence
E. Justice. Year: 2013

Liaison and neuropsychiatry


47. You are the resident working in the The answer is E.
oncology ward. Which of the following
factors is MOST predictive of high suicide Explanation: Hopelessness or cognitive
risk in a palliative cancer patient? symptoms are most predictive of high suicide
risk in a palliative cancer patient.
A. Alopecia associated with chemotherapy
B. Low energy level
C. Poor appetite Year: 2013.

12
D. Financial difficulty
E. Hopelessness.

48. A 40-year-old woman presents with Answer is C.


chronic headache and blurred vision.
She wants symptomatic relief but is not
Explanation: Patients suffering from
concerned about the underlying
condition. Which of the following hypochondriasis are more concerned about the
psychiatric diagnoses is LEAST diagnosis rather than symptomatic relief. The
relevant? other options are associated with concern of
symptoms as stated in this case.
A. Conversion disorder
B. Moderate depressive episode with
somatic complaints Year: 2013.
C. Hypochondriasis
D. Pain disorder
E. Somatization disorder.

13
MCQ exam (Paper 2): Answer Key

History, mental state exam and psychopathology


1. Which of the following is the BEST example of The best answer is D.
inattention? A. Impulsive interruptions may
A. The patient interrupts the conversation to ask or may not indicate
when he will be discharged. inattentiveness.
B. The patient is oriented and aware of his recent B. Falling asleep may indicate
medical problems but falls asleep during the inattentiveness, but further
conversation. information would be needed
C. The patient suddenly bursts into tears when you to rule out other explanations
are discussing his recent amputation. such as recent administration
D. The patient watches a fly buzzing on the ceiling of a sedating medication.
while you are discussing the prognosis for his lung C. Sudden bursts of affect have a
cancer, then falls asleep. significant differential beyond
E. The patient cannot remember what you have told inattention.
him 3 minutes ago. Then he tries to make up the D. This patient seems distracted
answer. despite discussion of an issue
of vital personal importance
to him. In the context also of
apparent drowsiness, the
clinician should suspect the
presence of delirium.
E. The patient may suffer from
dementia and cannot register
the information.

Year: 2013.
2. A 30-year-old man firmly believes that the alien The answer is B.
has put an implant in his body and he feels there
is a pushing sensation on his aorta. Which types of Explanation: Cenesthetic
hallucinations BEST suited his description? hallucinations are unfounded
sensations of altered states in bodily
A. Auditory organs. Examples of cenesthetic
B. Cenesthetic hallucinations are a burning sensation
C. Gustatory in the brain, a pushing sensation in
D. Kinesthetic the blood vessels, and a cutting
E. Visual. sensation in the bone marrow. Bodily
distortion may also occur.

Year: 2013.

This is an advanced-level question.

3. Perseveration is MOST commonly seen in which of The answer is D.


the following disorders?
Explanation: Perseveration is a
A. Autism persisting response to a previous
B. Attention deficit and hyperactivity disorder stimulus after a new stimulus has
C. Bipolar disorder been presented; it is often associated
D. Frontal lobe dementia with frontal lobe lesion.
E. Obsessive compulsive disorder.
Year: 2013

4. A patient presents with persistence, The answer is A.


perfectionism and body image distortion. Which
of the following disorders is MOST likely to be
Explanation: Persistence and
associated with the above clinical features?
perfectionism are associated with
A. Anorexia nervosa obsessive-compulsive personality
B. Borderline personality disorder traits and disorders, along with body
C. Delusional disorder image distortion occurring in many
D. Hypochondriasis patients with anorexia nervosa.
E. Somatization disorder.

Year: 2013

Cognitive assessment
5. Which of the following statement is FALSE The answer is E.
about Mini-mental state examination (MMSE)?
Explanation: MMSE is a screening test
A. A total score of 5 indicates severe dementia.
B. Assessment of orientation to time and place is part and there is no gold standard to
of the MMSE. establish the diagnosis of dementia in
C. The MMSE can be administered in Chinese or clinical practice. Clinical assessment
English. by a psychiatrist or geriatrician is
D. The MMSE can be used to monitor treatment more important than MMSE in
progress after initiation of acetylcholinesterase
establishing diagnosis of dementia.
inhibitors (AChEIs).
E. The MMSE is the gold standard in establishing the
diagnosis of dementia.
Year: 2013
Psychiatric epidemiology
6. Which of the following settings have the HIGHEST The answer is D.
prevalence of delirium?
Explanation: General surgical patients:
A. Accident and emergency department 10-15%; Cardiac surgery patients:
B. Cardiac surgery ward 30%; Hip fractures: 50%; Age >65
C. General surgery ward admitted to ICU: 70%; Palliative
D. Hospice with palliative advanced cancer patients advanced cancer patients: 88%.
E. Nursing home looking after dementia patients.

Year: 2013.
7. The worldwide point prevalence of The answer is A.
schizophrenia is BEST estimated as:
Explanation: The worldwide point
A. 1% prevalence is 1% and Singapore is
B. 2.5% 0.6%.
C. 5%
D. 7.5%
E. 10%. Year: 2013
Psychiatric aetiology, diagnosis and classification
8. Advanced paternal age is a well- established The answer is E.
risk factor for which of the following psychiatry
illnesses?
Explanation: Advanced paternal age is
A. Alcoholism a risk factor for schizophrenia.
B. Major depressive disorder
C. Obsessive compulsive disorder
D. Generalized anxiety disorder Year: 2013.
E. Schizophrenia.

9. Which of the following social factors is The answer is D.


MOST LIKELY to be associated with relapse
of schizophrenia after hospitalization? High levels of expressed emotion
A. Discrimination (EE) in the families of patients with
B. Financial problems schizophrenia predicted relapse
C. Homelessness following hospital discharge.
D. Increased expressed emotion Psychoeducational family therapy
E. Isolation and absence of family aims at helping the family reduce
member. the factors that constitute
expressed emotion. The therapist
also educates the family about
schizophrenia and the need to
continue antipsychotic medication
indefinitely.

Year: 2013.
10. Individuals with which one of the following genes The answer is B.
have the HIGHEST risk of developing Alzheimer’s
disease? Explanation: Homozygous APO E4/E4
increases the risk of developing
A. Homozygous apolipoproteins (APO) E2/E2 Alzhimer’s disease by 8 times.
B. Homozygous APO E4/E4
C. Homozygous APO E6/E6
D. Heterozygous APO E2/E4
E. Heterozygous APO E2/E6. Year: 2013.

General adult psychiatry


11. A 40-year-old man suffered from a heart attack The answer is D.
under general anaesthesia. The doctors in the
operation theatre successfully resuscitated him. Explanation: In the new diagnostic
After recovery, he wants to sue the hospital for criteria, it is possible for a PTSD
causing him post-traumatic stress disorder (PTSD). patient not to directly witness a
He claims the resuscitation process was traumatic event and learn about it
extremely traumatic. Which of the following is the from others after the accident. The
MOST IMPORTANT feature to exclude the most crucial feature is whether
diagnosis of PTSD? resuscitation itself is not a traumatic
process and it is necessary for doctors
A. He does not have past history of PTSD. to save patients’ life by resuscitation.
B. He has hidden agenda of suing the hospital
and tries to seek compensation.
C. He was under general anaesthesia and he was Year: 2013
not conscious during resuscitation.
D. The process of resuscitation is not considered This is an advanced – level question.
to be traumatic in general and doctor
resuscitated him in his best interest.
E. He did not seek treatment from a psychiatrist
after the operation.

12. Which of the following schizophrenia patients The answer is E.


has the HIGHEST suicide risk?
Explanation: The profile of the patient
A. An adolescent with prodromal symptoms and vague at greatest risk is a young man who
paranoid idea. once had high expectations, declined
B. An elderly retired man with late onset schizophrenia. from a higher level of functioning,
C. A middle-aged man with negative symptoms. realizes that his dreams are not likely
D. A middle-aged woman who suffers from simple to come true, and has lost faith in the
schizophrenia. effectiveness of treatment.
E. A young male university student who once had high
expectations.
Year: 2013.
13. Which of the following is NOT a recognized The answer is D.
alteration of sleep disturbance associated with
major depressive disorder?
Explanation: It should be reduction of
A. Difficulty in falling asleep latency of REM sleep.
B. Increase in nocturnal awakening
C. Increase in density of Rapid Eye Movement (REM)
sleep Year: 2013.
D. Increase in the latency of Rapid Eye Movement
(REM) sleep
This is an advanced level question.
E. Reduction of total sleep time.

14. A 40-year-old woman is staying with her god- The answer is B.


brother. She was rejected by her family and she
felt helpless in the past. She needs her god-
Explanation: She is best described to
brother to make decision for her and her god-
brother asks her to be the housekeeper in suffer from dependent personality
return. One day, her god-brother needs to go to because she cannot make decision on
Vietnam for 1 week and she feels extremely her own, uncomfortable when left
uncomfortable. Which of the following alone and subordination to her god-
personality traits BEST describes this person? brother’s need (e.g. to be a
housekeeper).
A. Borderline personality
B. Dependent personality
C. Histrionic personality
D. Schizoid personality Year: 2013
E. Schizotypal personality.

15. A 30-year-old man comes to see you and he The answer is A.


worries about premature ejaculation. The MOST
appropriate time frame to meet the diagnosis of
Explanation: Premature ejaculation is
premature ejaculation is:
A. Ejaculation occurs within 60 seconds following defined as ejaculation occurs within
vaginal penetration. 60 second following vaginal
B. Ejaculation occurs within 90 seconds following penetration and the patient wishes it.
vaginal penetration.
C. Ejaculation occurs within 120 seconds following
vaginal penetration.
Year: 2013
D. Ejaculation occurs within 150 seconds following
vaginal penetration.
E. Ejaculation occurs within 180 seconds following
vaginal penetration.
16. A 30-year-old man suffered from eight episodes of The answer is D.
mood disturbances within a single year. These
mood episodes met the diagnostic criteria for Explanation: This patient has more
major depression, mania and hypomania. These than 4 episodes of mania and
episodes were separated by remission. Which of depressive disorder in 1 year. He is
the following is the MOST likely diagnosis? too severe for cyclothymic disorder.

A. Cyclothymic disorder
B. Emotionally unstable personality disorder Year: 2013
C. Mixed affective disorder
D. Rapid cycling bipolar disorder
E. Schizoaffective disorder.

Old age psychiatry


17. Which of the following is NOT a predisposing The answer is D.
factor for depression in the elderly?
Explanation: Women have longer life
A. Cerebrovascular accident span compared to men. As a result,
B. Poly-pharmacy
women are more likely to be a widow
C. Looking after a spouse with chronic illness
D. Male gender and correspond to option E.
E. Widowhood.

Year: 2013.

18. Dementia in general is BEST described as: The answer is E.

A. Behavioural disturbance Explanation: dementia is a progressive


B. Global impairment deterioration of cognition, including
C. Impaired memory for personal events (e.g. memory and/or one of four other
marriage, past occupation) cognitive symptoms (aphasia, apraxia,
D. Personality change agnosia, impaired executive
E. Progressive deterioration. functioning) but it may not be a global
impairment. Impairment memory for
recent events is more common than
personal events.

Year: 2013.
19. A 65-year-old man presents with memory The answer is C.
loss, which of the following is the LEAST important
indication for a computerized tomography (CT) Explanation: The other factors
scan for his brain: suggest an organic cause.

A. A sudden decrease in cognitive function over a one-


month period Year: 2013.
B. Gait abnormalities
C. His age
D. Urinary incontinence
E. Use of anticoagulants.

Child and adolescent psychiatry


20. Which of the following is THE MOST IMPORTANT The answer is B.
predictor of bipolar disorder in adolescent
depression? Explanation: Antidepressant is the
most common cause of hypomania
A. Diurnal variation of mood and hypomania is part of the bipolar
B. Hypomanic symptoms after taking antidepressants disorder.
C. Hypersomnia
D. Increased weight
E. Multiple suicide attempts. Year: 2013.

21. The MOST COMMON reason for children with The answer is A.
autism are brought to medical attention by their
parents is: Explanation: Language delay is the
most obvious sign and most disturbing
A. Language delays to parents when they discover their
B. Lack of toilet control children are lacking behind in
C. Odd play language development compared to
D. School difficulties other children.
E. Stereotyped behaviour.

Year: 2013.
22. Which of the following statements is FALSE The answer is B.
regarding separation anxiety disorder?
Explanation: Separation anxiety disorder
A. It involves recurrent distress when separate from
is more common in children and it can
attachment figures.
occur in adolescents.
B. Separation anxiety disorder does not occur in
adolescents.
C. Separation anxiety disorder is associated with school
refusal. Year: 2013.
D. The child has difficulty falling asleep at night.
E. The child may complain of somatic symptoms.

23. Which of the following symptoms is MOST


LIKELY to persist when a young person suffers The answer is D.
from Attention Deficit and Hyperactivity
Disorder (ADHD) becoming an adult?
Answer: Although symptoms of
A. Accident risk hyperactivity and impulsivity often
B. Hyperactivity improve as the child grows older,
C. Impulsivity inattentive symptoms are likely to
D. Inattention persist.
E. Learning difficulties.

Year: 2013
24. A baby is born with low birth weight, microcephaly, The answer is A.
small eyes, upturned nose and a smooth, undeveloped
philtrum. He fails to thrive and develops seizures. Explanation: This is a classical
Which of the following was he MOST likely exposed
to during pregnancy? description of the facial features of
foetal alcohol syndrome
A. Alcohol
B. Cocaine
C. LSD Year: 2013.
D. Methamphetamine
E. Inhalants.
Substance abuse
25. The term which BEST describes an altered The answer is B.
physiological state and neuro- adaptation caused
by repeated administration of a drug is: Explanation: The term dependence
indicates physiological changes.
A. Addiction Misuse refers to use of any drug,
B. Dependence usually by self-administration, in a
C. Misuse manner that deviates from approved
D. Withdrawal social or medical patterns. Addiction is
E. Tolerance. non-specific and ignores the concept
of substance dependence is a medical
condition.

Year: 2013.
26. While driving, a 40-year-old man with no previous The answer is A.
history refused to stop for the traffic police. He
was subsequently brought into the accident and Explanation: Auditory hallucinosis is
emergency department (AED) by them. You are usually composed of auditory
the resident working in the AED. When you assess hallucination. Sensorium is clear and it
him, he states that he was hearing voices in clear is different from delirium tremens.
consciousness. He admits to a history of alcohol There are not enough symptoms to
use. What is the most appropriate diagnosis? conclude that he suffers from
schizophrenia.
A. Alcoholic hallucinosis
B. Delirium tremens Year: 2013.
C. Korsakoff psychosis
D. Schizophrenia
E. Wernicke’s encephalopathy.

27. Which of the following BEST describes the The answer is B.


mechanism of action with naltrexone in the treatment
of alcohol dependence? Explanation: Option A refers to
A. Naltrexone acts on the benzodiazepine receptor and benzodiazepine. Option C refers to
prevents alcohol withdrawal. the action of disulfiram. Option D and
B. Naltrexone blocks opioid receptors to decrease alcohol E refer to the action of acamprosate.
craving.
C. Naltrexone inhibits aldehyde dehydrogenase to decrease
alcohol craving.
Year: 2013.
D. Naltrexone is a GABA agonist and decreases alcohol
craving.
E. Naltrexone is a glutamate antagonist and decreases
alcohol craving.

28. Which of the following complications is LEAST The answer is D.


likely to occur in cocaine abusers?
A. Arrhythmia Explanation: Cocaine is least likely to
B. Hyperthermia
C. Myocardial infarction be injected via IV route. As a result,
D. Phlebitis phlebitis is relatively less likely to
E. Seizure. occur in patients misusing cocaine.
Year: 2013

This is an advanced-level question.

29. The primary neurotransmitter involved with The answer is E.


Lysergic acid diethylamide (LSD) is:
a. Acetylcholine Explanation: LSD increases the level of
b. Dopamine
serotonin and overdose of LSD is
c. γ-Aminobutyric acid
associated with serotonin syndrome.
d. Norepinephrine
e. Serotonin.

Year: 2013.

Psychopharmacology
30. Monoamine oxidase inhibitor (MAOI) is LEAST The answer is B.
useful in treating which of the following
disorders?
Explanation: MAOI is least useful to
A. Atypical depression treat OCD.
B. Obsessive compulsive disorder
C. Panic disorder
D. Severe depressive disorder not responding to Year: 2013.
selective serotonin reuptake inhibitor
E. Severe depressive disorder with hyponatraemia This is an advanced-level question.
31. Which of the following antidepressants is LEAST The answer is A.
likely to contribute to the development of
serotonin syndrome? Explanation: Bupoprion works on
dopamine and norepinephrine. As a
A. Bupropion result, it is least likely to cause
B. Fluoxetine serotonin syndrome.
C. Moclobemide
D. Paroxetine
E. Venlafaxine. Year: 2013.

This is an advanced-level question.

32. Which of the following antipsychotics exhibit a The answer is A.


novel mechanism as a partial antagonist?
Explanation: Aripiprazole exhibits a
A. Aripiprazole
B. Clozapine novel mechanism by definition while
C. Olanzapine other antipsychotics listed do not
D. Quetiapine have this property.
E. Ziprasidone.

Year: 2013.
33. Regarding lamotrigine, which of the following The answer is C.
statements is FALSE?
A. It acts at voltage-sensitive sodium channels. Explanation: Double-blind, placebo-
B. It inhibits the release of excitatory amino acid controlled trials have shown that it
neurotransmitters. has acute and prophylactic anti-
C. It is not effective in the treatment of bipolar depressant effects. It can lead to
depression. severe dermatological condition such
D. Its use can lead to toxic epidermal necrolysis. as Steven Johnson syndrome/ toxic
E. There is no need to monitor lamotrigine blood epidermal necrolysis. In Singapore,
levels during treatment. there is no blood test to check its
level.

Year: 2013

This is an advanced-level question.

34. Anticholinergic side effects include all of The answer is A.


the following EXCEPT:
Explanation: Anticholinergic side
A. Bradycardia effect include tachycardia.
B. Constipation
C. Dry mouth
D. Exacerbation of open-angle glaucoma Year: 2013.
E. Urinary retention.

35. In a schizophrenia patient without history of The answer is D.


asthma, the BEST treatment for chronic
akathisia is:
Explanation: Propranolol (beta
A. Alprazolam blocker) is the best treatment in this
B. Benzhexol patient without history of asthma as it
C. Clonazepam does not cause dependence as in
D. Propranolol Option A and C. Beta blockers can
E. Vitamin E. reduce restlessness and tremor.
Option B is more useful for
pseudoparkinsonism.

Year: 2013.
36. Which of the following psychotropic The answer is D.
medications is MOST EFFECTIVE in preventing
relapse in rapid cycling bipolar disorder?
Explanation: Lithium is less effective in
A. Haloperidol preventing relapse in rapid cycling
B. Lithium bipolar disorder. Valproate and
C. Olanzapine carbamazepine are preferred over
D. Valproate lithium.
E. Risperidone.

2013
Psychotherapy
37. In conducting psychotherapy with individuals who The answer is A.
have experienced a traumatic event and suffer
from post-traumatic stress disorder (PTSD), the Explanation: A is the answer. Affect or
following are all recommended techniques emotion should not be avoided in
EXCEPT: psychotherapy.

A. Encouraging avoidance of emotion


B. Examining feelings of guilt Year 2013.
C. Examining the patient’s response to the trauma
D. Offering consolation
E. Overcoming avoidance or phobia related to the
trauma.

38. A 30-year-old woman with panic disorder does The answer is B.


not respond to an initial treatment with an
selective serotonin reuptake inhibitor (SSRI).
Explanation: The effect of CBT is as
Which of the following treatment is considered
the best approach? efficacious as SSRI.

A. Benzodiazepine
B. Cognitive behaviour therapy Year: 2013.
C. Serotonin-noradrenaline reuptake inhibitor
D. Olanzapine
E. Psychodynamic psychotherapy.

39. Which of the following statements is The answer is C.


INCORRECT about Interpersonal therapy?
Explanation: Interpersonal therapy is
A. Has been shown to be efficacious in adult outpatients
slower than the effect of
suffering from depression.
B. Is more efficacious than psychodynamic psychotherapy
antidepressant because the therapist
C. Works as quickly as antidepressant medication. needs time to assess the patient and
D. Works best in mild to moderate depression. formulate strategies (e.g.
E. Works best in people facing problems in role transition. interpersonal inventory and role play).

Year: 2013.
40. A 23-year-old student is seen in the University The answer is D.
counselling centre because of relationship
problems. She has had one serious relationship Explanation: This patient
with her boyfriend that lasted about six months. demonstrates overgenearlization
She terminated the relationship because she because she believes in and follows a
discovered that her boyfriend was seeing general rule on the basis of limited
someone else. During psychotherapy, she examples.
mentions that ‘All men just can’t be trusted.’
Which of the following BEST describes this
thinking error? Year: 2013.

A. Arbitrary inference
B. Dichotomous thinking
C. Magnification
D. Overgeneralization
E. Selective abstraction.

Ethics and laws


41. All medical students are required to study The answer is B.
psychiatry so that they are qualified to assess and
manage common psychiatric disorders after Explanation: With competent
graduation. The compulsory psychiatric training is psychiatric knowledge, the future
BEST described by which of the following ethical doctors can provide benefits to
principles? patients when making decision on
diagnosis and management. The other
A. Autonomy concept that is relevant is dereliction.
B. Beneficence Fiduciary duty refers to the duty that
C. Morality the doctor must act in the patient’s
D. Justice best interest and it is not as relevant
E. Fiduciary duty. as beneficence in this case.

Year: 2013

Liaison and neuropsychiatry


42. You are a medical resident. A 40-year-old The answer is B.
schizophrenia patient was admitted as a result of
polydipsia. Which of the following statements Explanation: Psychogenic polydispsia
regarding polydipsia in schizophrenia is NOT true? is a common condition for chronic
schizophrenia. The estimates of
A. It is associated with low urine osmolality. prevalence range from 5 to 20%. The
B. It is associated with high sodium in urine. patient drinks excess water and
C. It is associated with low serum sodium. results in diluted urine. The body tries
D. Patients should be investigated for Syndrome of to preserve sodium and results in low
Inappropriate Secretion of Anti-Diuretic Hormone urine sodium. The serum is diluted.
(SIADH). SIADH should be considered for DDX.
E. Polydipsia occurs in between 5 to 20% of patients
suffering from chronic schizophrenia.
Year: 2013.

This is an advanced-level question.


43. A 40-year-old British man is admitted to the The answer is B.
medical ward. He tries to run away from the ward.
He is very aggressive and the security guards have Explanation: He suffers from delirium
difficulty to stop him. He claims that he is tremens. Intramuscular diazepam has
currently working in the office. The medical poor absorption. Option C and D are
records show that he has been drinking half of a not available in Singapore. Option E is
bottle of whisky every night. You need to order a a depot antipsychotics and it is not
medication to sedate him. He refuses to take any useful in acute setting.
oral medication. Which of the following
medications is the BEST alternative in Singapore?
Year: 2013
A. Intramuscular diazepam
B. Intramuscular lorazepam
C. Intramuscular olanzapine
D. Intramuscular quetiapine
E. Intramuscular risperidone.

44. You are the resident working for the liver The answer is B.
transplant team. The MOST common psychiatric
disorder in patients with liver cancer in the week Explanation: Delirium is the most
before and after transplantation is: common disorder and it can be
caused by systemic infection, side
A. Adjustment disorder effect of immunosuppressant,
B. Delirium impairment of liver function, hepatic
C. Generalized anxiety disorder encephalopathy.
D. Major depressive disorder
E. Somatoform pain disorder.
Year: 2013.
45. You are the resident working in the obstetrics The answer is B.
department. Which of the following is the BEST
treatment strategies to treat hyperemesis during Explanation: Benzodiazepine may
pregnancy? cause cleft palate in the first
trimester. Psychoanalysis is not for
A. Benzodiazepines e.g. alprazolam symptomatic relief. SSRI like
B. Relaxation therapy fluvoxamine is associated with nausea
C. Psychoanalysis side effect. Hyperemesis is not
D. Selective serotonin reuptake inhibitor e.g. associated with stimulus. Relaxation
fluvoxamine therapy is the best treatment.
E. Stimulus deprivation.

Year: 2013.
MCQ exam (Paper 3)

History and mental state exam


1. A 40-year-old woman is brought by her The answer is D.
husband brought to see you as a result of
abnormal behaviour. During the interview, Explanation: Labile affect refers to rapid
she laughs and cries with a very short period fluctuation of affect (e.g. from tearfulness and
of time. In your record, her affect is BEST laughter in a 10 – minute interview, seen in
described as: patients with bipolar disorder).

A. Apathy
B. Blunted affect Year: 2013.
C. Euthymic affect
D. Labile affect
E. Flat affect.
Cognitive assessment
2. Which of the following is the presentation The answer is B.
of cortical dementia?
Explanation: Option B refers to frontal lobe
A. A 40-year-old man presents with memory loss
dementia which is cortical dementia.
and writhing movements. His father suffered
from the same condition.
Option A, C, D and E are considered to be
B. A 50-year-old man presents with change in
subcortical dementia.
personality, poor judgment, disinhibition and
perservation. He made a lot of mistakes in
Option A = Hungtington’s disease
Luria’s hand test.
Option C = Vascular dementia
C. A 60-year-old woman presents with memory
loss and depression after a cerebrovascular Option D = Parkinson’s disease
accident. Option E = Lewy body dementia
D. A 70-year-old man presents with memory loss,
rigidity, slowness of movement and difficulty
with walking and gait. Year: 2013.
E. A 75-year-old woman presents with visual
hallucination. The GP gave her haloperidol This is an advanced-level question.
and she develops severe extrapyramidal side
effects.

Psychiatric epidemiology
3. After the age of 65, the prevalence of The answer is A.
dementia increases by how many times
every 5 years?
A. 2 times
B. 4 times Year: 2013
C. 6 times
D. 8 times
E. 10 times.
Psychiatric aetiology, diagnosis and classification
4. An 18-year-old woman presents with The answer is E
questions about her risk of developing
schizophrenia. She is worried because her
Explanation: The risk of general population is
23-year-old brother has recently been
diagnosed with schizophrenia and required 1%. The risk of a sibling is 10%. Hence, the risk is
hospitalization for psychosis. There are no increased by 10 times.
other family members who have a history
of schizophrenia. Having a sibling with
schizophrenia increases her risk of
Year: 2013.
developing schizophrenia by?

A. 2 times
B. 4 times
C. 6 times
D. 8 times
E. 10 times.

5. Which of the following neuroanatomical The answer is A.


structures is the critical region for fear
conditioning?
Explanation: Option B for Alzheimer’s disease,
A. Amgydala Option C for schizophrenia, Option D for
B. Basal nucleus of Meynert addiction and Option E for depression.
C. Mesocortical region
D. Nucleus accumbens
E. Raphe nucleus. Year: 2013.
6. The most likely neuroanatomical The answer is D.
substrate of Tourette's syndrome is
A. The thalamus. Explanation: Lesions in basal ganglia can
B. The pituitary. cause motor tic and compulsive behaviour.
C. The prefrontal cortex.
D. The basal ganglia.
E. The amygdala. Year: 2013

This is an advanced-level question.

Psychopathology
7. Agoraphobia without panic disorder can be Answer is A.
described as:
Explanation: A fear of enclosed space is one of
A. A fear of enclosed spaces the most common complaints among patients
B. A fear of speaking in public in Singapore (e.g. crowded MRT trains). Option
C. A fear of height B is social phobia.
D. Avoidance of authority figures
E. Worrying excessively about physical health.
Year: 2013.

8. Uncomplicated bereavement is NOT The answer is C.


characterized by:
A. Anger Indicators that bereavement may have
B. Depressive mood for one month progressed into depression include:
C. Feelings of worthlessness 1. symptoms still present two months
D. Sensations of seeing or hearing the after the loss
deceased 2. guilt about things other than actions
E. Transient guilt. taken or not taken by the survivor at
the time of death
3. thoughts of death other than the
survivor feeling that he or she would be
better off dead or should have died
with the deceased person
4. morbid preoccupation with
worthlessness
5. marked psychomotor retardation
6. prolonged and marked functional
impairment
7. hallucinatory experiences other than
thinking that he or she hears the voice
of, or transiently sees the image of, the
deceased person

9. The common clinical symptoms for The answer is C.


pre-menstrual dysphoric disorder
includes all of the following EXCEPT: Explanation: Guilt is not considered to be a
a. Bloating common clinical symptom of premenstrual
b. Difficulty concentrating dysmorphic disorder.
c. Guilt
d. Sleep disturbance
e. Lethargy and fatigue Year: 2013

10. A 40-year-old air-conditioner technician The answer is B.


is referred to you because of change in
behaviour and deterioration in Explanation: As this person works as an air-
functioning. When he feels the gust of conditioner technician, he is used to feel gust of
cold air coming from the air- cold air from air conditioner. This is a normal
conditioner, he interprets the cold air perception. He adds a delusional interpretation
as poison gas from Mars. This and claims that the cold air is a poison gas from
phenomenon is BEST described as: another planet. This is illogical and impossible.
As a result, there is delusional interpretation of
A. Delusional memory a real perception. This phenomenon is known
B. Delusional perception as delusional perception.
C. Tactile hallucination
D. Primary delusion
E. Secondary delusion. Year: 2013

11. Which of the following behaviour is The answer is B.


NOT considered to be purging?
Explanation: In medicine, purging is defined
A. Self-induced vomiting as to cause evacuation of the
B. Excessive exercise gastrointestinal or urinary system. Option B
C. Excessive use of laxative is not related to the gastrointestinal or
D. Excessive use of diuretic urinary system.
E. Excessive use of enema.

Year: 2013

General adult psychiatry


12. In schizophrenia, which of the following The answer is A.
factors is LEAST Likely to restrict full
rehabilitation potential? Explanation: Option A is positive symptom of
schizophrenia and is least likely to restrict full
A. Florid delusions and hallucinations rehabilitation potential as compared to
B. Lack of pleasure from social intervention and negative symptoms.
physical activity
C. Loss of interest in rehabilitation
D. Poor motivation
E. Restricted affect. Year: 2013

13. Somnambulism (Sleepwalking disorder) The answer is E.


occurs in
Explanation: Parasomnia is divided into slow-
A. Active sleep wave sleep disorders (e.g. sleep terrors,
B. Earliest phase of sleep sleepwalking disorder) and rapid eye movement
C. Paradoxical sleep sleep disorders (e.g. nightmare disorder, REM
D. Rapid eye movement (REM) sleep behaviour disorder such as kicking in the middle
E. Slow-wave sleep. of the night).

Somnabulism or sleepwalking disorder is associated


with slow wave sleep. Because sleepwalking disorder
arises from SWS
sleep, the patient is difficult to be
awaken during sleepwalking.

Year: 2013.
14. Which of the following is a poor prognostic The answer is E.

factor for schizophrenia? Explanation: Option E suggests that the aetiology of schizophrenia
is biological and resulted from a neurological lesion. Hence, the
A. Family history of mood disorder prognosis poor. Good prognostic and poor prognostic factors are
summarized as follows:
B. Female gender
C. Late onset Good Prognosis Poor Prognosis
Late onset Young onset
D. Precipitating factor Obvious precipitating factors No precipitating factors
E. Presence of neurological soft sign. Acute onset Insidious onset
Good premorbid function Poor premorbid function
Married Single, divorced, or widowed
Family history of mood disorders Family history of schizophrenia
Good support systems Poor support systems
Positive symptoms Negative symptoms

Year: 2013.

15. All of the following medical disorders are The answer is E.


more common in schizophrenia patients as
compared to the general population
Explanation: The risk for schizophrenia patients
EXCEPT?
developing RA is one-third the risk of general
A. Chronic obstructive pulmonary disease population. The reason is due to reduction in
B. Diabetes mellitus immune responses in schizophrenia patients.
C. Human immunodeficiency virus infection
D. Metabolic syndrome
E. Rheumatoid arthritis.
Year: 2013.
16. A 40-year-old executive is afraid of flying by The answer is E.
aeroplane. She has this problem since young. As
a result, she has to quit her job because it Explanation: Fear of shadow is specific phobia.
requires frequent travelling. What is the MOST
LIKELY diagnosis?
Year: 2013.
A. Agoraphobia
B. Generalized anxiety disorder
C. Panic disorder
D. Social phobia
E. Specific phobia.

Old age psychiatry


17. Which is the MOST preferred antidepressant The answer is D.
in elderly?
A. Amitriptyline Explanation: Sertraline, a SSRI, is the most
B. Imipramine preferred antidepressants as compared to
C. Moclobemide imipramine and amitriptyline which are TCA.
D. Sertraline Venlafaxine is associated with dose-related
E. Venlafaxine. hypertension.

Year: 2013

This is an advanced-level question.

18. Which of the following statements is TRUE The answer is B.


regarding risk factors for Alzheimer’s
disease? Explanation: Studies have suggested that APOE4
genotype increases risk of Alzheimer’s disease,
A. APO E2 genotype increases the risk of and especially in women. APOE2 genotype is
Alzheimer’s disease. protective. Female gender is a risk factor and
B. APO E4 genotype increases the risk of smoking is not protective.
Alzheimer’s disease.
C. APO E6 genotype increases the risk of
Alzheimer’s disease. Year: 2013
D. Male gender is a risk factor
E. Smoking is protective against Alzheimer’s
disease.

19. Which of the following is the MOST common The answer is E.


type of hallucination in people suffering
from dementia? Explanation: Visual hallucinations are the most
A. Functional common type of hallucination in dementia. It is
B. Gustatory because one type of dementia, lewy body
C. Olfactory dementia is associated with visual
D. Tactile hallucinations.
E. Visual.

Year: 2013
20. Late-onset psychotic disorder such as The answer is B.
schizophrenia or delusional disorder is
associated with:
Explanation: Late-onset psychotic disorder is
A. Less paranoia more likely to be associated with sensory
B. More likely to have sensory deficits deficit.
C. More loosening of associations
D. More thought disorders
E. More violent behaviour. Year: 2013.
21. Which of the following statements about The answer is D.
sleep disorders in old people is FALSE?
Explanation: Diazepam is long-acting and cause
A. Excessing napping is a common cause of
insomnia. oversedation or hang-over effect. This will
B. Insomnia is common in old people in increase fall risk among the elderly.
Singapore.
C. Insomnia is associated with increased irks of
depression. Year: 2013
D. If there is a need for benzodiazepine,
diazepam is the preferred sedative for old
people.
E. The first line treatment is sleep hygiene.

22. An 80-year-old man presents with The answer is E.


terminal insomnia, weight loss,
decreased appetite and pervasive Explanation: This elderly man suffers from
anhedonia. Physical examination, depressive disorder based on the history
laboratory investigations and imaging provided. Hence, suicidal ideation is the
reveal normal findings. The MOST most specific symptom.
specific symptom to his likely
diagnosis is:
Year: 2013.
A. Anxiety
B. Decreased concentration
C. Decreased energy
D. Psychomotor changes
E. Suicidal ideation.

Child and adolescent psychiatry


23. Which of the following childhood psychiatric The answer is E.
disorders has the EARLIEST mean age of
onset? Explanation: Encopresis: 4 years by definition.
ADHD: 12 years, CD: 10 years, ODD: 8 years,
A. Attention deficit hyperactivity disorder PTSD: can be childhood or adult.
B. Conduct disorder
C. Oppositional defiant disorder
D. Post-traumatic stress disorder Year: 2013
E. Secondary encopresis.

24. Which of the following is MOST CORRECT The answer is C.


about Asperger’s disorder?
Explanation: They have better verbal
A. They are good at sport. communications as compared to autism and no
B. They have a normal IQ. delay in speech. They may have lower IQ. Some
C. They have impaired non-verbal have high intelligence in specific areas but not
communication. all.
D. They have delay in speech.
E. They have high intelligence in specific areas.
Year: 2013.
25. A 15-year-old girl presents with fatigue The answer is B.
and dehydration. On physical
examination she is noted to have Explanation: This patient suffers from bulimia
swollen salivary glands and calluses nervosa. She has self-induced vomiting and the
on her knuckles. She is also gastric acid causes inflammation in the salivary
hypokalemic. These findings are glands. She uses her fingers to induce vomiting
MOST consistent with which of the and leave calluses on her knuckles, which is
following diagnoses? known as Russell’s sign. Repeated self-induced
A. Anorexia nervosa vomiting is associated with hypokaelemia.
B. Bulimia nervosa
C. Chronic fatigue syndrome
D. Obsessive-compulsive disorder
Year: 2013.
E. Rumination disorder
26. Which of the following antidepressants has The answer C
the GREATEST body of evidence for efficacy
in treating major depressive disorder in Explanation: Fluoxetine and escitalopram have
children and adolescents? the greatest body of evidence for efficacy in
A. Bupoprion treating major depressive disorder in children
B. Fluvoxamine and adolescents.
C. Fluoxetine
D. Paroxetine
E. Sertraline. Year: 2013
Substance abuse
27. A 55-year-old man who has been The answer is C.
drinking Chinese wine every night
says, ’I don’t think it is a problem.’ Explanation: He is unaware that his alcohol
Which Prochaska’s and Diclemente’s dependence is a problem.
stage of change BEST describes his
current status?
A. Action Year: 2013.
B. Contemplation
C. Precontemplation
D. Maintenance
E. Relapse.

28. A 23-year-old medical student is brought to The answer is C.


the Accident and Emergency Department by
his parents. He admits to having recently Explanation: Dilated pupil, arrhythmia, nausea
used a recreational drug during his overseas and suspiciousness are common signs
elective in the United States. He returned associated with cocaine intoxication.
two days ago. He is suspicious and sensitive
to what others are saying. Physical
examination reveals dilated pupils and Year: 2013.
electrocardiogram (ECG) shows cardiac
arrhythmias. Which drug is MOST likely to be This is an advanced-level question.
responsible?

a. Alcohol
b. Cannabis
c. Cocaine
d. Mescaline
e. Heroin.
29. A 24-year-old man presents to the The answer is B.
Accident and Emergency Department
with fever, hypotension, agitation, Explanation: Disulfiram blocks this reaction at
tachycardia and odour of alcohol on the acetaldehyde dehydrogenase. After alcohol
his breath. His friend with him stated intake under the influence of disulfiram, the
that he was prescribed a medication by concentration of acetaldehyde in the blood may
a psychiatrist 4 weeks ago. The most be five to 10 times higher than that found
likely medication responsible is:
during metabolism of the same amount of
A. Diazepam alcohol alone. As acetaldehyde is one of the
B. Disulfiram major causes of the symptoms experienced by
C. Fluoxetine this man.
D. Haloperidol
E. Moclobemide.
Year: 2013.
30. A 40-year-old heroin addict reports The answer is C.
continued carving. He is on
methadone100mg daily. His methadone
Explanation: Methodone is associated with
dose is increased to 110mg daily. After one
week, he has a syncope and is taken to the prolonged QTc. This occurs after an increase in
Accident and Emergency Department. The the dose of methadone. This may lead to
most likely finding to explain his syncope cardiac arrhythmia and results in syncope.
is:
A. Electroencephalogram (EEG) shows evidence
of seizure activity. Year: 2013
B. Liver function tests reveal marked
transaminitis.
This is an advanced level question.
C. Prolonged QTc interval
D. Ultrasound Doppler shows deep vein
thrombosis.
E. Urine toxicology screen is positive for opioid.

31. A 50-year-old with history of major The answer is A.


depressive disorder and ischaemic heart
disease. He has not received any psychiatric Explanation: Bupoprion is the best option in this
treatment. Which of the following case as it can overcome depression and
medications is the BEST option to help him smoking. Option C and E can tackle smoking but
to quit smoking? not depression.

A. Bupropion
B. Naltrexone Year: 2013.
C. Nicotine replacement treatment
D. Sertraline
E. Varenicline.

Psychopharmacology
32. The WEAKEST evidence for efficacy for The answer is B.
Post-traumatic Stress Disorder (PTSD) is
for which class of pharmacological
Explanation: Diazepam, a long term
agents?
a. Amitriptyline benzodiazepine may cause dependence and
b. Diazepam does not help to relieve anxiety symptoms in
c. Fluoxetine long run as it may cause withdrawal.
d. Fluvoxamine
e. Paroxetine.
Year: 2013.

33. Regarding escitalopram, all of the following The answer is D.


are true EXCEPT:
A. Absorption is not affected by food. Explanation: Escitalopram causes the least drug
B. It has better tolerability than other SSRIs. interaction among all SSRIs. Sertraline’s
C. It shows linear pharmacokinetics at clinically absorption is significantly affected by food.
relevant doses. Escitalopram is better tolerated than other
D. It causes a lot of drug interaction. SSRIs. Fluoxetine and paroxetine are the only
E. Peak plasma levels occur 2-4 hours after a SSRIs that do not show linear pharmacokinetics
single dose. at clinically relevant doses.

Year: 2013
34. The MOST common side effect of selective The answer is C.
serotonin reuptake inhibitor (SSRI) is:
Explanation: GI disturbances such as nausea
A. Acne
B. Erectile dysfunction and vomiting are the most common side
C. Gastrointestinal disturbances effects.
D. Metabolic syndrome
E. Tinnitus.
Year: 2013.
35. Which of the following antipsychotic drugs The answer is A.
is MOST LIKELY to be associated with
adverse haematological side effects?
Explanation: Clozapine is the second generation
A. Clozapine antipsychotics which is most likely to be
B. Olanzapine associated with agranulocytosis.
C. Quetiapine
D. Risperidone
E. Ziprasidone. Year: 2013
36. Which of the following is the LEAST common The answer is E.
side effects of acetylcholinesterase
inhibitors? Explanation: The AChEI causes bradycardia as a
result of increased vagal tone.
A. Dizziness
B. Nausea
C. Nightmares Year: 2013.
D. Seizures
E. Tachycardia. This is an advanced-level question

37. Priapism is MOST likely to be associated with The answer is E.


which of the following medications?
A. Bupoprion
Explanation: Trazodone is a serotonin
B. Duloxetine
C. Moclobemide antagonist and reuptake inhibitor. It is good for
D. Phenelzine depression and insomnia. It is available in SGH
E. Trazodone. pharmacy but not NUH pharmacy. It is
associated with priapism in men.

Year: 2013
This is an advanced-level question.

38. A patient presents with involuntary The answer is E.


frowning, blinking grimacing, and
choreoathetoid movements of the Explanation: This patient develops involuntary
upper extremities after several years of movements after taking medication for several
antipsychotic medication treatment. years. The best description is tardive dyskinesia.
Which one of the following is the MOST
likely diagnosis?
Year: 2013.
A. Akinesia
B. Akathisia
C. Acute dystonia
D. Parkinsonism
E. Tardive dyskinesia

Psychotherapy
39. At the one year follow-up of panic disorder, The answer is D.
which of the following treatments results
with the BEST outcome and the LEAST Explanation: CBT has better outcome than brief
functional impairment? dynamic psychotherapy and supportive
psychotherapy. Benzodiazepines and beta-
A. Benzodiazepines blockers only provide symptomatic relief.
B. Beta-blockers
C. Brief dynamic psychotherapy
D. Cognitive behaviour therapy Year: 2013.
E. Supportive psychotherapy.

40. A 20-year-old with patient suffers from The answer is E.


agoraphobia and she has phobia of
using MRT train. Which is the BEST Explanation: The main principle of treatment
psychological treatment for this is to not have any form of avoidance but to
patient? extinguish anxiety in response to the feared
stimulus, i.e. the MRT train.
A. No exposure to MRT train and encourage
alternative transportation mode
B. Short exposure to MRT train with coping Year: 2013.
avoidance
C. Short exposure to MRT train without coping
avoidance
D. Graded exposure to MRT train with coping
avoidance
E. Graded exposure to MRT train without coping
avoidance.

41. Defining high-risk situations, covert The answer is B


antecedents, and stimulus control
techniques are a focus of which of the Explanation: Defining high risk situations, covert
following therapeutic modalities? antecedents and stimulus control techniques
A. Brief psychodynamic psychotherapy are part of the dysfunctional thought diary used
B. Cognitive behaviour therapy in CBT.
C. Supportive therapy
D. Interpersonal therapy
E. Relapse prevention therapy. Year: 2013.

42. A security guard attacks a government The answer A.


building, killing 3 individuals and significantly
injuring another 10 before he takes his own Explanation: Debriefing is shown to be harmful
life. Psychologists from a general hospital to individuals who are the witness of a
are dispatched to the scene to provide traumatic event. The individuals should be
immediate psychological debriefing to civil given access to psychological help and they can
servants working in the building to prevent consult a psychologist on a voluntary basis.
post-traumatic stress disorder (PTSD)
symptoms. Using an evidence-based
framework, what is the most likely impact of Year: 2013
this intervention on direct survivors?
This is an advanced-level question.
A. It will be ineffective or harmful to the
direct survivors.
B. It will decrease the probability of
developing PTSD symptoms.
C. It will decrease the severity of PTSD
symptoms.
D. It will decrease the time period for
recovery from PTSD symptoms.
E. It will delay the onset of PTSD symptoms.
Ethics and laws
43. When managing a chronically suicidal The answer is A.
patient, respecting the patient’s preference
to remain at home and not be admitted to Explanation: As this question emphasizes on
the psychiatric unit for additional care respecting the patient’s preference, the best
reflects which of the following ethical answer is respecting patient’s autonomy.
principles?

A. Autonomy Year: 2013.


B. Beneficence
C. Non-maleficence
D. Egalitarianism
E. Fiduciary duty.

Liaison and neuropsychiatry


44. Features of neuroleptic malignant The answer is E.
syndrome include all of the following
EXCEPT? Explanation: It should be rigidity and not
a. Diaphoresis flaccidity.
b. Elevated temperature
c. Extrapyramidal side-effects
d. Labile blood pressure Year: 2013.
e. Flaccidity.

45. You are working as a resident in the weight The answer is B.


management programme. A 40-year-old
schizophrenia patient is referred to you for Explanation: Meformin has the best evidence in
weight management issue. Which of the reducing weight gain associated with the second
following medications has the BEST generation antipsychotics.
evidence as an adjunct for reducing weight
gain associated with the second
generation antipsychotics: Year 2013
A. Fluoxetine
B. Metformin This is an advanced-level question.
C. Sibutramine
D. Simvastatin
E. Topiramate.

46. In medically ill patients who suffer from The answer is C.


psychiatric illness, which of the following
pharmacological properties is MOST Explanation: Narrow therapeutic index like
appropriate? lithium is dangerous to medically ill patients.
A. Long half-life psychotropic drug.
B. Psychotropic drugs with active metabolites.
C. Psychotropic drug with wide therapeutic index. Year 2013
D. Psychotropic drug which inhibits metabolic
enzymes.
E. Psychotropic drug which induces metabolic
enzymes.

47. A 47-year-old woman is brought for a The answer is A.


psychiatric evaluation to assess recent
changes in her behaviour. Over the past six Explanation: This patient suffers from Wilson’s
months, she has become increasingly disease which is autosomal recessive.
disinhibited and impulsive in her behaviour.
Physical examination reveals mild dysarthria,
dysphagia, and drooling. Slit lamp Year: 2013.
examination of the eyes indicates the
presence of Kayser Fleischer rings. Which of This is an advanced-level question.
the following BEST describes the genetic
basis for her illness?
A. Autosomal recessive
B. Co-dominant
C. Mitochondrial
D. Polygenetic
E. X-linked dominant.
MCQ exam (Paper 4)

History and mental state exam


1. You are a medical resident. The nurse informs The answer is A.
you that an 80-year-old man who was admitted
for cellulitis seems to be depressed and Explanation: This man may suffer
mentions about passive suicidal thought. from delirium and acute
When you assess him, he sees ghost in the confusional state. It is most
ward and expresses paranoid ideation against important to assess orientation.
nursing staff. Which of the following
assessment is MOST important to establish Assessing somatic symptoms is
the diagnosis? not useful as it is caused by
medical conditions.
A. Assess orientation
B. Assess judgement Year: 2013.
C. Assess recognition
D. Assess registration and short-term recall
E. Assess somatic symptoms.

Cognitive assessment
2. Which of the following cognitive tasks is NOT The answer is B.
an assessment of short term or long term
memory? Explanation: Option B refers to
verbal fluency which is part of the
A. Ask the patient to name the current Prime frontal lobe assessment. Option D
Minister of Singapore. refers to registration and it is an
B. Ask the patient to name as many animals as important component before
possible that can be found in the Singapore assessing the short-term recall
Zoo. (Option E). To be fair to the
C. Ask the patient to tell you his or her address patient, the patient should be given
and later, you check the answer with patient’s multiple attempts to register the 3
medical record. items before testing short-term
D. Inform the patient 3 objects (e.g. Apple, recall in Option E.
Newspaper and Train) and ask the patient to
name the 3 objects immediately. Year: 2013.
E. Inform the patient 3 objects (e.g. Apple,
Newspaper and Train) and ask the patient to
name the 3 objects after 5 minutes.

Psychiatric, epidemiology, etiology, diagnosis and


classification
3. A 20-year-old man with an identical twin is The answer is B.
diagnosed with major depressive disorder.
His twin brother asks you to comment on the Explanation:
chance he will develop depressive disorder.
Which of the following is the CORRECT Family studies show that a person
response to his twin brother? has 40-70% chance to develop
A. ‘Major depressive disorder is a strongly depressive episode if a first degree
inheritable disease. You will definitely develop relative suffer from depressive
it as well.’ episode.
B. ‘The likelihood of you developing depressive
disorder is higher than other siblings.’ Twin studies show that the
C. ‘The likelihood of you developing depressive concordance rate for monozygotic
disorder is same as non-identical twins.’ twins is 40 – 50% and for dizygotic
D. ‘There is only a slightly increased risk that you twins is 20%.
will become depressed as compared to the
general population.’ Year: 2013.
E. ‘You do not need to worry because
environmental factor is an important factor to
determine whether you will develop depressive
disorder.’

4. A 35-year-old woman suffers from low mood, The answer is C.


low energy, poor sleep, poor appetite and
recurrent suicidal thoughts. She firmly Explanation: This man suffers from
believes that she deserves the death sentence severe depressive disorder with
for minor mistakes made in the past. Which of psychotic features. Option A, B, D
the following statements is FALSE? and E are correct.

A. For severe depressive episode with psychotic There is not enough evidence to
features, psychotic symptoms usually occur suggest the diagnosis of
after manifestation of depressive symptoms. schizoaffective disorder because
B. Her psychotic symptoms are considered to be her symptoms do not suggest that
mood congruent. she suffers from schizophrenia.
C. Based on the case scenario, there is enough
evidence to suggest that she suffers from Year: 2013.
schizoaffective disorder.
D. The prescription of antipsychotic drug should
be considered.
E. Electroconvulsive therapy (ECT) is a
recognised treatment for his condition.

5. Which of the following disorders has been The answer is C.


shown to have the GREATEST degree of
heritability? Explanation:
A. Alcoholism Heritability of autism to be more
B. Attention-deficit/hyperactivity disorder (ADHD) than 90%. 90% of the differences
C. Autism between autistic and non-autistic
D. Major depressive disorder individuals is due to genetic
E. Schizophrenia. effects.

Year: 2013.

6. Which of the following is the MOST common The answer is D.


cause of insomnia among psychiatric patients
in Singapore? Explanation: Underlying psychiatric
disorder is the most common
A. Sleep apnea cause of insomnia among
B. Shift work psychiatric patients. Students
C. Stimulant use should observe this during their
D. Underlying psychiatric illness clinical attachment. Depressive
E. Use of sleeping pills. disorder, bipolar disorder and
generalized anxiety disorder are
associated with insomnia.

7. Regarding the age of onset in schizophrenia, The answer is D.


which of the following is CORRECT?
Explanation: Current research
A. European women have earlier age of onset findings in schizophrenia show that
than Asian women. men have earlier onset as
B. Current research findings remain compared to women.
inconclusive about the age of onset in
schizophrenia.
C. There is no difference in the age of onset
between men and women.
D. Men usually have earlier age of onset as
compared to women.
E. Women usually have earlier age of onset as
compared to men.

Psychopathology
8. Agoraphobia without panic attack is BEST The answer is D.
referred to:
Explanation: Patients suffering
A. Concern about physical appearance from agoraphobia has fear that
B. Constant worried about physical health they cannot escape from the
C. Fear of blood and needle enclosed space.
D. Fear of collapse in an enclosed space
E. Fear of authority figures. Year: 2013.

9. Which of the following is MOST suggestive of The answer is D.


the diagnosis of schizophrenia?
A. Acute onset of psychosis Explanation: Hearing own thoughts
B. Apathy or echo de la pense only occurs in
C. Cognitive impairment schizophrenia and is most
D. Hearing own thoughts suggestive of such diagnosis.
E. Persistent deterioration of personality.
Option A can occur in brief
psychosis or acute/transient
psychosis, not necessarily
schizophrenia.

Option B, D and E occur in both


schizophrenia and dementia.

Year: 2013.
General adult psychiatry
10. Peter is 27-year-old and lives with his parents. The answer is A.
He has been employed as a delivery man for
most of the time since leaving school, but has Explanation: Peter develops
recently left his job as a salesman. He has grandiose delusion because he
never taken any illicit drugs. His parents state does not have the capacity to
that in the last three weeks, he has been make a machine to cure cancer
extremely active, requiring less sleep and not based on his background. The
appearing tired, being over-talkative and most appropriate diagnosis is
disinhibited and – on occasions – quite bipolar disorder with manic
irritable. He claimed to have invented a features.
machine for curing cancer and wished to go to
the U.S. to sell it. When stopped by his Year: 2013.
parents, he became violent, and they called
the police. Which of the following diagnosis is
MOST appropriate for this patient?

A. Bipolar I disorder with manic features


B. Bipolar II disorder with hypomanic features
C. Cyclothymia
D. Mixed affective disorder
E. Schizoaffective disorder.

11. Which of the following personality disorders The answer is B.


is LEAST likely to be considered as a
differential diagnosis to schizophrenia? Explanation: Obsessive
compulsive personality disorder is
A. Borderline personality disorder least likely to be associated with
B. Obsessive compulsive personality disorder psychotic features.
C. Paranoid personality disorder
D. Schizotypal personality disorder Year: 2013
E. Schizoid personality disorder.

12. Which of the following is LEAST likely to be The answer is C.


found in research in the relationship between
depressive disorder and hormones? Explanation: There is no
suppression of cortisol with
A. Blunted adrenocorticotropic hormone (ACTH) dexamethasone in research finding
response to the corticotropin-releasing in depressive disorder. Option A
hormone (CRH) indicates the failure of negative
B. Blunted thyroid stimulating hormone (TSH) feedback and causes an increase
response to thyrotropin-releasing hormone in cortisol (Option D and E). Option
(TRH) B is associated with low T3 or T4
C. Cortisol suppression with dexamethasone level.
D. There is an increased cortisol.
E. There is an increased adrenal sensitivity to Year: 2013.
ACTH.
This is an advanced-level
question.
13. Which of the following disorders is LEAST The answer is D.
likely to be considered as a differential
diagnosis of a 35-year-old driver develops Explanation: OCD is least likely to
nightmare, flashback and hypervigilance after share symptoms with post-
a road traffic accident? traumatic stress disorder (PTSD).
A. Factitious disorder As a result, OCD is least like to be
B. Generalized anxiety disorder a DDX of PTSD.
C. Major depressive disorder
D. Obsessive compulsive disorder Year: 2013.
E. Phobic disorder.

14. Rapid eye movement (REM) sleep is The answer is A.


associated with all of the following EXCEPT:
Explanation: REM sleep is
A. Increased parasympathetic activity associated with an increase in
B. Increased cerebral blood flow sympathetic activity.
C. Increased complexity of dreams
D. Maximal loss of muscle tone Year: 2013.
E. Transient runs of conjugate eye
movements. This is an advanced level question.

15. A 17-year-old male comes to see you because The answer is B.


he is experiencing tremendous stress in his
new job. He has finished his “O” level Explanation:
examination and waiting to start the National
Service in 6 months. He got an offer to work in This man suffers from fetishism
a shoe shop which specializes in selling because he derives sexual
female shoes. He realizes that he feels stimulation from an inanimate
sexually aroused by female shoes. He spends object (i.e. female shoes) and his
a lot of time fantasizing about female shoes. fetishism causes social and
He stole multiple pairs of shoes from the shop occupational dysfunction because
and used them for masturbation. He used to he spends inordinate time
be a normal person. He was a hardworking fantasizing about female shoes.
student whose main interests were chemistry
and physics. What is the MOST likely He stole shoes to satisfy his
diagnosis? fetishism and this is not a case of
kleptomania.
A. Adjustment disorder
B. Fetishism Year: 2013.
C. Kleptomania
D. Sadomasochism
E. Voyeurism.

16. Which of the following psychiatric disorders is The answer is B.


MOST common among patients with bulimia
nervosa? Explanation: Major depressive
disorder is the most common
A. Alcohol use disorder psychiatric comorbidity in bulimia
B. Major depressive disorder nervosa (around 50- 85%)
C. Obsessive compulsive disorder
D. Panic disorder Year: 2013.
E. Intermittent explosive disorder.

Old age psychiatry


17. Manic episodes in old people are associated The answer is A.
with:
Explanation: Mania in old people is
A. Less euphoria associated with less euphoria, less
B. Less ‘mixed’ presentation with depression hyperactivity, less flight of ideas
C. Less paranoid delusions but more mixed presentation and
D. More hyperactivity more paranoid ideation.
E. More flight of ideas.
Year: 2013.

This is an advanced level question.


The answer is A.
18. All of the following statements are true
regarding post-operative delirium EXCEPT: Explanation: 30% of patients
suffering from delirium may have
A. Delirium cannot be diagnosed if all laboratory normal laboratory results and no
results are normal. abnormality was found. Delirium
B. It can occur in an 80-year-old man after can occur in young people who
gastrectomy. undergo a complicated operation
C. It can occur in a 16-year-old girl after renal with immunosuppressant.
transplant.
D. Haloperidol is one of the treatments of choice. Year: 2013.
E. Physical restraint should be avoided if possible.

19. A 70-year-old man with a history of drinking The answer is B.


Chinese wine, hepatitis, chronic renal failure
and hypertension was brought by his wife to Explanation:
the hospital for treatment of an acute cellulitis. This patient is already at a high
He was noted to have tachycardia in the risk for delirium based on his age
Accident and Emergency Department. You are and medical comorbidities,
the on-call medical resident and this man was including chronic renal failure.
admitted to the medical ward. What is the
MOST appropriate first approach? Without adequate collateral
history, providing a high dose
A. Assess short term recall to rule out regimen of potentially unnecessary
dementia on delirium. benzodiazepine puts the patient at
B. Obtain history from collaterals whether he risk of worsened delirium. His
has in fact been drinking Chinese wine elevated heart rate may reflect
recently. agitation or pain due to acute
C. Put him on propranolol 10mg three times cellulitis.
per day to control tachycardia.
D. Start diazepam 5mg three times per day to Beta blockers most often mask the
prevent delirium tremens. sympathetic outflow signs of
E. Start lorazepam 1mg three times per day to withdrawal of alcohol and the
prevent delirium tremens. prescription of propranolol is not
necessary.

Year: 2013.
20. A 70-year-old woman with severe depressive The answer is B.
symptoms and strong suicidal ideation which
is not responded to amitriptyline 150mg daily Explanation: This elderly woman
and fluvoxamine 200mg daily. Each suffers from treatment resistant
medication was tried for 6 months with good depression and electroconvulsive
adherence. She has good past health. Her therapy is the treatment of choice
children consult you for further management. to treat suicidal ideation.
The MOST appropriate treatment which you
would recommend is: Old age is not a contraindication
for electroconvulsive therapy.
A. Cognitive behaviour therapy
B. Electroconvulsive therapy Year: 2013.
C. Donepezil
D. Olanzapine
E. Repetitive transcranial magnetic stimulation.

Child and adolescent psychiatry and intellectual


disability
21. You are helping a mother to develop The answer is B.
behavioural program to deal with her son who
has oppositional defiant disorder. Which of Explanation: Positive
the following is the MOST significant reinforcement of promoting
component of this program? desirable behaviours through
rewards is the most significant
A. Extinction component of the program.
B. Positive reinforcement
C. Punishment Year: 2013.
D. School suspension
E. Strict parenting.

22. Which of the following factors are LEAST The answer is D.


likely to be the aetiological factors in attention
deficit and hyperactivity disorder (ADHD)? Explanation: Low birth weight, not
high birth weight is associated with
A. Antenatal exposure to alcohol the risk of developing ADHD.
B. Antenatal exposure to nicotine
C. Delivery complications Year: 2013.
D. High birth weight
E. Traumatic brain injury during infancy
and early childhood.
23. You are a general practitioner. A 40-year-old The answer is A.
woman consults you as a result of high blood
pressure. General inspection shows ptosis, a broad Explanation: She suffers from Turner’s
neck and indistinct hairline. Her chest appears to be syndrome (XO syndrome) as
broad. Auscultation reveals cardiac murmurs. She is evidenced by webbed neck (a broad
single and stays with her parents. She completed her neck and a low or indistinct hairline),
education in the Institute of Technological Education coarctation of aorta (cardiac
(ITE) and works as a store room officer. If murmurs, hypertension) and normal
chromosome analysis is performed, the MOST likely /borderline intelligence. She may
finding is: have infertility. Her single status
cannot confirm the infertility but it
A. XO syndrome remains a possibility.
B. XXY syndrome
C. XXXY syndrome Year: 2013.
D. XYY syndrome
E. YO syndrome. This is an advanced-level
question.
24. You are a paediatric resident. A 7-year-old boy The answer is B.
is suspected to suffer from autism. You
referred this boy to see an education Explanation: In autism,
psychologist for assessment. The education performance IQ (Block design) is
psychologist sends a report to you and you higher than verbal IQ.
need to explain the findings to her mother. His
strength is MOST likely found in which of the Year: 2013.
following areas?

A. Abstract thinking
B. Block design
C. Explain similarities
D. Oral presentation skills
E. Verbal concept formation

Substance abuse
25. Which of the following neuroanatomical The answer is C.
structures is MOST implicated in substance
misuse and dependence? Explanation: Substance abuse and
dependence produce additive
A. Amygdala effects on dopamine release in the
B. Basal nucleus of Meynert nucleus accumbens.
C. Nucleus accumbens
D. Hippocampus Year: 2013
E. Raphe nucleus.

26. A 50-year-old man is admitted to the The answer is B.


hepatobillary ward as a result of pancreatitis.
You are the resident on call. His wife tells you Explanation: GGT is an early
that she has seen him intoxicated with alcohol indicator of alcohol relapse,
in the past few days. The patient denies. SENSITIVITY is 40-60% and
Which of the following is the BEST method SPECIFICITY is 80%. 4 drinks per
available in a general hospital in Singapore to day for the past 2 weeks will
confirm that this patient has been drinking increase GGT. CDT is a very
recently? sensitive and specific test but it is
not readily available in a general
A. Carbohydrate-deficient transferrin (CDT) hospital in Singapore. The cost
B. Gamma-glutamyl transferase (GGT) CDT is much higher than GGT.
C. Mean corpuscular volume (MCV)
D. Serum alcohol level Year: 2013
E. Serum amylase.

27. A common and safe pharmacological The answer is E.


treatment for reducing relapse in alcohol
dependence has direct actions upon which Explanation: This question refers
receptors? to naltrexone, a common and safe
pharmacological treatment for
A. Benzodiazapine reducing relapse in alcohol
B. Cannabinoid dependence.
C. Dopamine
D. γ-Aminobutyric acid (GABA) Year: 2013
E. Opioid.

28. Which of the following is LEAST consistent The answer is D.


with the objective of motivational
interviewing? Explanation: Motivational
interviewing emphasizes on
A. Allow the patient give their inputs without individual’s patient’s needs and
interruption. avoid empirical advice.
B. Establishing a collaborative patient-therapist
relationship. Year: 2013
C. Identifying appropriate reinforcements.
D. Providing empirical advice.
E. Usage of open-ended questions.

Psychopharmacology
29. Which of the following antidepressants is The answer is C.
LEAST likely to have sexual side-effects?
Explanation: Mirtazapine and
A. Amitriptyline bupoprion have relatively low risk
B. Moclobemide of causing sexual side effects as
C. Mirtazapine compared to other
D. Fluoxetine antidepressants.
E. Venlafaxine.
Year: 2013
30. The benzodiazepine with the LONGEST half- The answer is B.
life is:
A. Clonazepam Explanation: Diazepam has the
B. Diazepam longest half life which lasts for
C. Flurazepam more than one day.
D. Lorazepam
E. Triazolam. Year: 2013

31. Which of the following is NOT an The answer is A.


acetylcholinesterase inhibitor?
A. Buprenorphine Explanation: Buprenorphine is a
B. Donepezil partial opioid agonist which is used
C. Galantamine in the treatment of opioid
D. Rivastigmine dependence but this medication
E. Tacrine. was banned in Singapore due to
diversion and misuse.

Year: 2013.

32. Which of the following regarding side effects The answer is E.


of risperidone is FALSE?
Explanation: The risk for weight
A. Akathisia is common with high dose of gain and metabolic syndrome is
risperidone. lower than olanzapine.
B. Prolactin elevation, probably greater than that
seen with other second generation
antipsychotics. Year: 2013.
C. Insomnia, headache and nausea are common.
D. Rhinitis is a possible side effect.
E. The risk for weight gain and the metabolic
syndrome is higher than olanzapine.

33. Tricyclic antidepressants should be avoided The answer is A.


with all of the following EXCEPT
A. Patients hospitalized for severe melancholic Explanation: TCA may be helpful
depression in severe depression. The other
B. Recent myocardial infarction are relative contraindications.
C. Right bundle branch block
D. Untreated glaucoma Year: 2013.
E. Urinary retention.

34. Which of the following is CORRECT regarding The answer is C.


galactorrhoea induced by antipsychotic
drugs? Explanation: Galactorrhoea can be
treated with bromocriptine.
A. Antipsychotic drugs modulate the hypothalamic
function and lead to galactorrhoea. Galactorrhoea is due to a direct
B. Antipsychotic drugs cause pituitary adenoma effect on the anterior pituitary by
and galactorrhoea. dopamine secreting
C. Bromocriptine can treat galactorrhoea induced tuberoinfundibular neurons, which
by antipsychotic drugs. normally inhibit prolactin release.
D. Galactorrhea is caused by antipsychotics acting
directly on the breast tissue. Antipsychotic drugs do not cause
E. Galactorrhoea is caused by the concurrent use pituitary adenoma.
of anticholinergic medication.
Year: 2013

This is an advanced-level
question.
35. Which of the following drugs is LEAST likely The answer is D.
to increase lithium toxicity?
A. Angiotensin-converting enzyme (ACE) inhibitors Co-administration of valproate and
B. Calcium channel blockers lithium have no effect on the steady-
C. Non-steroidal anti-inflammatory drugs (NSAIDS) state kinetics of lithium.
D. Sodium valproate
E. Thiazide diuretics. Year: 2013

36. Which of the following is FALSE about The answer is B.


clozapine?
Explanation: It has low affinity for
A. It can cause agranulocytosis. D2 receptors.
B. It has high affinity for D2 receptors.
C. It has low risk of extrapyramidal side effects. Year: 2013
D. It is associated with metabolic syndrome.
E. It is good for treatment resistant schizophrenia.

37. A 30-year-old woman with 3 episodes of major The answer is A.


depression in the past 5 years responds to
fluoxetine 40mg every morning. Her last Explanation: This patient has 3
episode was 6 months ago. In order to episodes of major depression in
minimize the risk of relapse, which of the the past 5 years and the number of
following treatment strategies is MOST episodes is considered to be
effective? frequent.

A. Continue fluoxetine 40mg every morning for It is dangerous to reduce the dose
five years. or stop antidepressant soon.
B. Continue fluoxetine 40 mg for six months and
then stop. Year: 2013
C. Provide intermittent maintenance
electroconvulsive therapy.
D. Stop fluoxetine and restart it at the first sign of
relapse.
E. Stop fluoxetine and start cognitive behaviour
therapy.

Psychotherapy
38. Which of the following treatment strategies The answer is B.
has the MOST evidence for treating obsessive
compulsive disorder? Explanation: Exposure and
A. Fluoxetine and brief dynamic response prevention is part of the
psychotherapy cognitive behaviour therapy and
B. Fluoxetine and exposure and response has the most evidence for treating
prevention obsessive compulsive disorder.
C. Fluoxetine and eye movement
desensitization and reprocessing Year: 2013
D. Fluoxetine and hypnotherapy
E. Fluoxetine and interpersonal
psychotherapy.

39. You have to study anaesthesia, emergency The answer is E.


medicine, obstetrics and gynaecology,
otolaryngology, ophthalmology and Explanation: This student
psychiatry in less than two weeks. Your first demonstrate selective abstraction:
thought is, “I will never be able to study all six focus on current negative aspect
subjects in such a short time! The upcoming but ignore the positive aspects in
examination is very stressful.” You ignore the the past.
fact that you were successful in the previous
professional MBBS examinations and you Year: 2013
could handle multiple subjects in one
examination in the past. From a cognitive
therapy perspective, what is the BEST
description of this type of thinking?

A. Catastrophic thinking
B. Magnification
C. Minimization
D. Personalization
E. Selective abstraction.

Ethics and laws


40. In Singapore, a person who is disturbed and The answer is B.
aggressive, with poor insight into his illness,
refusing treatment and threatening family Explanation: Mental Health (Care
members can be admitted involuntarily under and Treatment) Act was passed in
the: 2008 and this is the most correct
answer.
A. Advanced Care Directive
B. Mental Health (Care and Treatment) Act Year: 2013.
C. Mental Capacity Act
D. Mental Health Act
E. Psychiatric Disorders and Treatment Act.

Liaison and neuropsychiatry


41. Which of the following medical conditions is The answer is B.
LEAST likely to present with panic attacks?
Explanation: Cushing syndrome is
A. Asthma more likely to be associated with
B. Cushing syndrome depression rather than panic
C. Insulinoma attacks.
D. Phaeochromocytoma
E. Thyrotoxicosis. Year: 2013.

42. A 50-year-old man suffering from The answer is A.


schizophrenia and he has been taken
haloperidol for the past 20 years. His QTc is Explanation: A prolonged QTc
550 ms. The medical resident wants to find the mainly affects the ventricles,
potential medical complication if he continues leading to ventricular fibrillation,
to take haloperidol. Which of the following torsade de pointes and sudden
complications is LEAST LIKELY: cardiac death. Palpitation is a
recognized complication.
A. Myocardial infarction Myocardial infarction is unlikely to
B. Palpitation be associated with prolonged QTc.
C. Ventricular fibrillation
D. Sudden cardiac death Year: 2013.
E. Torsade de pointes.
This is an advanced-level
question.
43. A 40-year-old schizophrenia patient presents The answer is C.
with high urine volume, low urine osmolality,
low serum sodium and low urine sodium. Explanation: He suffers from
Which of the following is the MOST likely psychogenic polydipsia because
diagnosis? there is a dilution in urine volume,
urine osmolality, serum sodium
A. Diabetes mellitus and urine sodium due to fluid
B. Nephrogenic diabetes insipidus overload and a result of excessive
C. Psychogenic polydipsia drinking.
D. Simple hyponatremia
E. Syndrome of inappropriate antidiuretic hormone Year: 2013.
secretion.

44. You are a resident working in the gynaecology The answer is B.


ward. A 35-year-old woman was admitted for
ovarian cystectomy. She suffers from major Explanation: It is reasonable that
depressive disorder and insomnia. She patient may not be able to see her
consults a psychiatrist at the Institute of psychiatrist shortly after operation.
Mental Health (IMH). She is prescribed with Her suicide risk may not be
venlafaxine, zolpidem, lorazepam and genuine in this case because it is
hydroxyzine. She is due for discharge today after a condition (if no medication
and she wants to go home. She expresses is given).
concern that she cannot see her psychiatrist
in the coming weeks due to pain associated In real situation, the best answer is
with the operation. She requests to obtain 3- to assess her suicide risk by
month supply of her psychiatric medications yourself or your hospital
from you. She also requests a pain killer psychiatrist. This option is not
called tramadol. She threatens that if you do available and you need to choose
not give her medication, she will commit the next best answer.
suicide. What is the BEST approach to handle
this situation? Option A may cause a relapse of
her psychiatric illness by not giving
A. Do not give her any psychotropic medication her any psychotropic medications.
and tramadol. Discharge her the same day.
B. Negotiate with her for shorter duration of Option C will intensify her
psychotropic medications and tramadol. disappointment and postpone
Obtain earlier appointment at IMH. Discharge discharge does not offer solution.
her today.
C. Postpone discharge and use this to motivate Option D: 3-month supply
her not to request for more psychotropic medication is too long and it may
medications. pose risk to patients.
D. Give her 3-month supply of psychotropic
medications and tramadol. Discharge her Option E will affect therapeutic
today. alliance and it is important to
E. Send her to IMH for assessment because she further assess her suicide risk. She
threatens that she will commit suicide. mentions about suicidal thought as
a sign of anger.
Year: 2013

This is an advanced-level
question.
MCQ

History, mental state exam and psychopathology


1. A 50-year-old woman presents with severe The answer is D.
depressive episode with psychotic features. Which
of the following delusions is LEAST likely to occur Explanation: Delusion of jealousy is
in this patient? the least mood congruent as
compared to other options.
A. Deserved punishment
B. Guilt Year: 2014
C. Incurable illnesses
D. Jealousy
E. Self-depreciation.

2. A 30-year-old prisoner gives approximate answers The answer is D.


to questions. For example, if it is Thursday, he will
say it is Friday and claims a dog has three legs. Explanation: The patient provides an
This patient is MOST likely suffering from: approximate answer which is a clinical
feature of Ganser’s syndrome.
A. Capgras’ syndrome
B. Ekbom’s syndrome
Option A and C are misidentification
C. Fregoli’s syndrome
D. Ganser’s syndrome syndrome. Option B refers to delusion
E. Othello’s syndrome. of infestation. Option E refers to
morbid jealousy.

Year:2014
3. A psychiatric patient suddenly realises that he is The answer is B.
an interpreter for deaf people and tries to
interpret others’ speech in sign language. He
Explanation: Mannerism is a goal –
demonstrates strange repetitive movements.
His signs appear to come in threes or fours, directed repetitive movement (e.g. a
occasionally swinging his shoulders, as if he is speaker tries to move his hands
conveying a message. Which of the following is repetitively to convey his messages).
the BEST term to describe his movements?
This man’s movements seemed to be
A. Ambitendency goal directed because he attempted
B. Mannerism to show sign language but not
C. Negativism understandable to deaf people.
D. Stereotypies
E. Waxy flexibility.
Year 2014

Cognitive assessment
4. A 33-year-old man with childhood developmental Answer: C
delay and epilepsy received special education
and now works as a cleaner. He has worked in a
Explanation: This case is more severe
tofu factory for three years. He can only perform
simple routine work in the factory. His than mild retardation which
supervisor reports his work performance is slow demonstrates more competency in
and poor. His family says he cannot live self-care and independence.
independently and demonstrated delay in People with severe or profound
achievement in self-care since young. How mental retardation are not able to
would you rate the level of mental retardation?
work in the factory.
A. Very mild mental retardation
B. Mild mental retardation
C. Moderate mental retardation Year 2014
D. Severe mental retardation
E. Profound mental retardation.

Psychiatric epidemiology
5. A 65-year-old retired teacher is concerned about The answer is D.
dementia and has assessed online resources for
information. She asks which type of dementia is
Explanation: Vascular dementia (VaD)
more common in Asians as compared to
Caucasians. Your answer is: is found to be more common than
Alzheimer’s disease (AD) in some
A. Alzheimer’s disease Asian countries. In a study conducted
B. Lewy body dementia by National Neuroscience Institute
C. Pseudodementia (Singapore), 53% suffered from VaD
D. Vascular dementia
versus 47% from AD. The ethnic
E. Fronto-temporal lobe dementia.
variation is due to genetic variation,
differences in vascular risk factors and
lifestyles.

Year: 2014
Psychiatric aetiology, diagnosis and classification
6. A 20-year-old national serviceman was The answer is B.
diagnosed to suffer from adjustment
disorder with depressed mood. He read the Explanation:
internet and found another disorder known Option A is correct and relevant for
as dysthymic disorder. He wants to know this case.
the differences between these two Option B is incorrect because
disorders. The following statements about dysthymic disorder is more likely
differences between adjustment disorder with to show a genetic pattern.
depressed mood and dysthymia are true Option C is true because
EXCEPT: adjustment disorder begins within
three months of a stressor, and
A. Adjustment disorder with depressed mood ends within six months of the
results from a known stressor, whereas stressor ending. Dysthymic
dysthymic disorder does not. disorder has a duration of two
B. Adjustment disorder with depressed mood years in adults and one year in
shows a genetic pattern, whereas children.
dysthymic disorder does not. Option D is correct
C. The duration of symptoms is shorter for Option E is correct.
adjustment disorder with depressed mood
than it is for dysthymic disorder. Year: 2014
D. The co-existence of a major depressive
disorder with dysthymic disorder is called
double depression. There is no term for a
major depressive disorder complicating
adjustment disorder with depressed mood.
E. Treatment is usually shorter and does not
require the use of medication for adjustment
disorder with depressed mood as compared
to dysthymic disorder.

7. You are a family doctor. A male and a female The correct answer is D.
schizophrenia patient who stay in the
community have decided to get married and Prevalence of Schizophrenia in
have a child. What is the risk for their child to Specific Populations
develop schizophrenia in the future?
Population Prevalence
A. 16% (%)
B. 26% General Population 1
C. 36% Non-twin sibling of a 9
D. 46% schizophrenic patient
E. 56%. Child with one 13
schizophrenic parent
Dizygotic twin of a 14
schizophrenic patient
Child of two 46
schizophrenic parents
Monozygotic twin of a 46
schizophrenic patient

Year: 2014
8. You are an oncology resident. A 60-year-old oncology The answer is B.
patient developed severe nausea as a side-effect during Explanation: This phenomenon is
chemotherapy. However, after completing treatment known as classical conditioning.
she continues to experience nausea whenever she Chemotherapy is an unconditional
returns to the hospital for follow-up. Which of the
stimulus (UCS) and the hospital
following psychological theories BEST describes her
experience?
building is conditional stimulus (CS).
The conditional (CR) and
A. Biofeedback unconditional response (UCR) include
B. Classical conditioning nausea. The pairing of CS and UCR will
C. Cognitive learning persist even UCS (i.e. chemotherapy)
D. Operant conditioning disappears.
E. Social learning.
Year: 2014
General adult psychiatry
9. A 25-year-old woman experiences episodes of The answer is C.
depersonalization, hyperventilation, dizziness,
and diaphoresis lasting for 15 minutes whenever
Explanation: the history and
she arrives at work. She has been overwhelmed
by work and conflicts with her supervisor for the symptoms suggest the diagnosis of
past 3 months. There are no such episodes at panic disorder because the anxiety
home or outside the office. What is the MOST symptoms are episodic with a clear
likely diagnosis? trigger.

A. Acute stress disorder Year: 2014.


B. Generalized anxiety disorder
C. Panic disorder
D. Post-traumatic stress disorder
E. Somatization disorder.

10. You are an orthopaedic resident managing a 24- The answer is A.


year-old motorcyclist who was involved in a
road traffic accident 3 days ago in which he
Explanation: The duration of
sustained multiple fractures and his pillion rider
died. He is irritable, sleeps poorly and has symptoms is only 2 days. It is too
nightmares and refuses to talk about the early to conclude whether he will
accident. The MOST likely psychiatric diagnosis develop Post-Traumatic Stress
is: Disorder (PTSD).

A. Acute stress disorder Year: 2014


B. Adjustment disorder with anxiety
C. Depressive disorder
D. Generalized anxiety disorder
E. Post-traumatic stress disorder.

11. Based on the current research findings, which of Answer is C


the following statements is CORRECT regarding
the prognosis for a 35-year-old Indonesian man Explanation: Schizophrenia patients
with schizophrenia who lives in a rural village residing in the rural area have a more
and treated by risperidone?
favourable prognosis as compared to
patients staying in urban or industrialized
A. He has a worse prognosis as compared to
patients staying in an urban city. areas. In rural areas, there is less
B. His prognosis is poor because he will seek likelihood of social isolation and a greater
treatment from traditional healer in the village. degree of social support. There is more
C. He has a better prognosis as compared to patients emphasis on social and familial bonds
staying in an urban city. that protect patients from social isolation.
D. His prognosis is the same as patients staying in in Furthermore, it is easier for patients to
an urban city cope with work-related tasks in
E. His prognosis is not related to the urban agricultural or village settings. There is
development of the society where he resides. less stigma in rural areas.

Year: 2014.

12. You are a resident working in the Accident and The answer is E.
Emergency Department. A 39-year-old English man was
married to a Chinese Singaporean is brought to the Explanation: Hanging is considered to
Emergency Department after he attempted to hang be a dangerous suicide method and
himself. He is actively suicidal and has marital
this patient has high suicide risk. As
problems. He has not spoken to his wife for three days.
The hospital does not have a psychiatric ward nor a
he is not forthcoming and does not
stay-in psychiatric team. What is the MOST appropriate suffer from any medical or surgical
management plan? complication, sending him to IMH for
A. Admit Peter to the medical ward of your hospital. assessment under the Mental Health
B. Discharge Peter from Emergency Department with Act is the safest option.
follow-up in the polyclinic to reduce the stigma.
C. Discharge Peter from Emergency Department with Year: 2014
an early follow-up in the psychiatric department.
D. Refer the couple for marital counselling by on-call
medical social worker.
E. Transfer Peter to IMH for assessment and consider
admission under Mental Health (Care and
Treatment) Act.
13. A 40-year-old single man consulting you in a The answer is E.
general practice clinic reveals that since
adolescence he enjoys observing naked people
Explanation: Voyeurism refers to the
and those involved in sexual activities as it
helps him achieve sexual arousal. The presence of recurrent and intense
psychiatric condition associated with this sexual arousal from observing an
behaviour is: unsuspecting person who is naked, in
the process of disrobing or engaging
A. Exhibitionism in sexual activity over a total duration
B. Fetishism of 6 months.
C. Sadomasochism
D. Transvestism
E. Voyeurism. Year: 2014
14. A 65-year-old retired man consulting you in a The answer is E.
general practice clinic reveals that he has
marital problems. His wife complains that he is
Explanation: He exhibits narcissistic
very fuzzy about cooking. He does not eat
outside nowadays. If he goes to a high class personality trait. He demonstrates of
restaurant, he expects that the steak must be sense of entitlement and holds belief
properly done and he is entitled to change the that he is better than the others (e.g.
steak if it is poorly done. Now he finds most chef from all steak restaurants). He
steak restaurants in Singapore are not up to his preoccupies with unlimited success.
standard. He claims it is better to cook by
himself. If the food is not up to his standard, he
has no one to blame. He likes to go to high- Year: 2014
class club in town. He claims to be a special
member in the club due to his unlimited success
and donation. Which of the following
personality traits BEST describes this man?

A. Antisocial
B. Avoidant
C. Borderline
D. Histrionic
E. Narcissistic.

15. A 30-year-old executive complains that he cannot fall Answer is B.


asleep at night. Two months ago, his life was
interrupted by a work trip to the United States. After Explanation: This patient exhibits
returning to Singapore, he can only falls asleep at 2 to 3 circadian rhythm sleep disorder -
a.m. and he cannot wake up on time the next morning. delayed sleep phase because he
This leads to negative impact on his work. What is the can only sleep after 2 to 3 a.m.
MOST likely diagnosis? The advanced phase sleep
A. Circadian rhythm sleep disorder – advanced sleep disorder is associated with early
phase
onset of sleep and early morning
B. Circadian rhythm sleep disorder – delayed sleep
phase
wakening.
C. Excessive daytime sleepiness
D. Kleine-Levin syndrome Year: 2014
E. Random eye movement sleep disorder.
Old age psychiatry
16. A 65-year-old lady with bipolar disorder and severe The answer is E.
functional decline presents with three-day history of
acute agitation, suicidal ideation and refusing to eat Explanation: ECT seems to be the best
and drink. Her Mini-Mental Status Examination (MMSE) treatment because she is severely
score was 29/30. auditory hallucinations. She has not depressed with psychotic features
responded to sodium valproate and olanzapine. The
and suicidal ideation. Her MMSE is
MOST appropriate treatment at this stage is:
close to full score and potential
A. Acetylcholinesterase inhibitor
cognitive impairment should not be a
B. Augmentation with antidepressant
relative contraindication for ECT. On
C. Augmentation with anxiolytics
the other hand, her renal impairment
D. Change valproate to lithium
is a contraindication for lithium.
E. Electroconvulsive therapy.
Year: 2014
17. You are public health doctor and work in the The answer is C.
Ministry of Health. The Ministry wants to identify
protective factors against the development of Explanation:
Alzheimer’s disease. Which of the following is least Risk factors for Alzheimer’s disease
associated with the development of Alzheimer’s include old age, family history, ApoE4
disease? genotype, female gender,
A. Consuming red meat on a frequent basis hypertension, diabetes, never
B. Female gender married, low education level, head
C. High education level trauma, high homocysteine levels.
D. High homocysteine level
E. Never married. Protective factors include high levels
of education, APOE2, consuming fish,
anti-inflammatories, statins ,
bilingualism.

Year: 2014
18. You are a member of the Hospital Quality Improvement The answer is D.
Committee. The hospital wants to improve the
management of delirium. Which of the following Explanation: The typical course of
statements regarding delirium is FALSE? delirium last for 10-12 days. It is too early
to expect the majority of delirious
A. Around 30% of the elderly admitted to the medical ward patients recover by Day 3.
will develop delirium during hospitalisation.
B. Delirium is often missed or overlooked by health care Year: 2014
professionals in the elderly.
C. Delirium occurs in 10% of children undergoing day
surgery.
D. Delirium usually clears within 3 days of correcting
underlying medical cause.
E. The incidence of delirium increases with age.

19. You are a geriatric doctor. The manager of a The answer is A.


nursing home wants to consult you the behavior
which is most likely correlated with delusions in
Explanation: Patients with severe
Alzheimer’s disease. Your answer is:
Alzheimer’s disease often suffer from
A. Aggression and agitation delusion of theft and this correlates
B. Disrobing (i.e.state of wearing of no clothes) with aggression and agitation to
C. Shadowing family members and domestic
D. Suicide helpers.
E. Wandering.
Year: 2014

20. You are a paediatric resident. You are concerned The answer is B.
about the mental health of a 50-year-old mother
whose 12-year-old daughter died of leukaemia. Explanation: Recurrent suicidal
You are not certain whether she suffers from thoughts are considered to be
normal grief or pathological grief. Normal grief is pathological grief.
NOT characterized by:
Year: 2014
A. Denial
B. Recurrent suicidal thoughts
C. Searching for the deceased
D. Sensations of seeing or hearing the deceased
E. Transient guilt.

21. You are a geriatric resident reviewing a 75-year- The answer is B.


old woman with Alzheimer’s disease. Her family
is keen for her to take acetylcholinesterase Explanation: Acetylcholinesterase
inhibitor. Which of the following medical inhibitors increase cholinergic
conditions is MOST contraindicated?
functions and may cause spasm in
A. Anaemia smooth muscle or bronchospasm. As
B. Asthma a result, it may worsen asthma.
C. Diabetes
D. Hyperthyroidism Year: 2014
E. Rheumatoid arthritis.

Child and adolescent psychiatry


22. Which of the following is the COMMONEST The answer is C.
cardiac complication in a 14-year-old
adolescent suffering from anorexia nervosa Explanation: Bradycardia and
on admission? hypotension are the most common
cardiac complications when patients
A. Atrial fibrillation and cardiac arrhythmia. suffering from anorexia nervosa are
B. Bradycardia and hypertension admitted to the ward.
C. Bradycardia and hypotension
D. Tachycardia and hypotension Year: 2014
E. Tachycardia and hypertension.

Substance abuse
23. A 30-year-old man admitted to the medical ward The answer is E.
has no access to the substance which he used prior
to admission. He complains of nausea and muscle Explanation: Opiate withdrawal leads
pain. Physical examination reveals pupillary to mood changes, gastrointestinal
dilation. Which of the following substances is disturbances (e.g. nausea, vomiting),
MOST likely to cause the above withdrawal muscular aches, lacrimation and
symptoms? pupillary dilation.

A. Alcohol Year: 2014


B. Cocaine
C. Midazolam
D. Nictoine
E. Opiate.

24. A female student has returned from Amsterdam The answer is C.


after an exchange programme and admits using
cocaine for 1 year. She developed psychotic
Explanation: The risk factors for
symptoms. Which of the following is the MOST
IMPORTANT risk factor for the development of psychosis in cocaine users include:
psychosis while using cocaine? male gender, intravenous users, first
time users, longer duration and
A. An elevated body mass index greater amount of cocaine use, low
B. Combination with benzodiazepine BMI.
C. Being a first time user of cocaine
D. Being female Year: 2014
E. Non-intravenous use.
This is an advanced level question.
25. A 30-year-old man who is dependent on alcohol The answer is D.
enquires of his family physician if disulfiram will help
him to reduce his alcohol intake while he still drinks in a Explanation: This patient does not
controlled manner. Which of the following
recommendations is CORRECT? express motivation to quit drinking or
remain abstinent with alcohol. He
A. Disulfiram is recommended because it has more wants to reduce alcohol intake by
evidence than other medications in helping patients controlled drinking. It is a dangerous
to quit alcohol. situation if he drinks and takes
B. Disulfiram is recommended because it will disulfiram at the same time.
discourage him from drinking alcohol.
C. Disulfiram is recommended because he has
motivation to reduce his drinking. Year: 2014
D. Disulfiram is not recommended because it may
cause severe side effects when it mixes with
alcohol.
E. Disulfiram is not recommended because it is
indicated for opioid dependence.

Psychopharmacology
26. A 50-year-old man with a major depressive disorder did The answer is B.
not respond well to an adequate trial of sertraline
200mg (a selective serotonin reuptake inhibitor). Explanation: Option B may lead to
Neither did he fully respond to a subsequent 8-week serotonin syndrome.
trial of venlafaxine 150mg (serotonin noradrenaline
reuptake inhibitor). Which of the following is NOT
Year: 2014.
RECOMMENDED for this man?

A. Augment SSRI with lithium (a mood stabilizer)


B. Augment SSRI with phenelzine (a monoaxmine
oxidase inhibitor)
C. Consider electroconvulsive therapy
D. Consider psychotherapy
E. Increase the dose of venlafaxine.

The answer is B.
27. Bupoprion differs from SSRI in which of the
following ways? Explanation: Bupoprion has a lower
incidence of sexual side effects (not
A. Bupoprion has a higher incidence of sexual working on serotonin receptors) and a
side effects and a lower incidence of lower incidence of sedation and
sedation and weight gain. weight gain (as an energetic
B. Bupoprion has a lower incidence of sexual antidepressant).
side effects and a lower incidence of
sedation and weight gain.
Year: 2014.
C. Bupoprion has a lower incidence of sexual
side effects and higher incidence of
sedation and weight gain
D. Bupoprion has lower incidence of sexual
side effects and weight gain and a higher
incidence of sedation.
E. Bupoprion has a higher incidence of sexual
side effects and sedation and a lower
incidence of weight gain.
28. Besides treatment resistant schizophrenia, The answer is E.
another appropriate indication for clozapine is?
Explanation: Severe tardive dyskinesia
A. Severe agranulocytosis
B. Severe embolism is an indication for clozapine. Option
C. Severe hypersalivation A, B, C and D are side effects
D. Severe metabolic syndrome associated with clozapine.
E. Severe tardive dyskinesia.
. Year: 2014.
29. A 50-year-old cardiac patient suffers from The answer is C.
depression and the psychiatrist prescribed
fluoxetine. Which of the following statements Explanation: Fluoxetine has been
about fluoxetine is TRUE? reported to produce bleeding and
bruising in some individuals. SSRIs as
A. Fluoxetine decreases warfarin levels. a class are believed to inhibit platelet
B. Fluoxetine increases metabolism of warfarin. aggregation, which may underlie this
C. Fluoxetine increases bruising while patient is taking effect. The ability of SSRIs to reduce
warfarin. platelet aggregation may be an
D. He has no increased risk of gastrointestinal bleeding important intervention for patients
with fluoxetine. with occlusive coronary and
E. He should not change to paroxetine because cerebrovascular artery disease and
paroxetine decreases anticoagulant effect of deserves study.
warfarin.
In addition, warfarin is metabolized
via CYP 3A4 and fluoxetine inhibits
CYP 3A4 and increases the levels of
warfarin.

Year: 2014.

This is an advanced level question.


30. Which of the following hypnotic agents causes The answer is E.
the LEAST daytime sedation is:
Explanation: The hypnotic drug with
A. Diphenhydramine
B. Diazepam the least half life have least day time
C. Lorazepam sedation.
D. Zopiclone
E. Zolpidem. Diphenhydramine (half-life): 9-12 hours
Diazepam: 20-100 hours
Lorazepam: 9-16 hours
Zopiclone: 6 hours
Zolpidem 2-3 hours.

Year: 2014.

This is an advanced level question.


31. Which of the following medications has the most The answer is C.
convincing evidence for reducing suicidal
behavior in bipolar patients?
Explanation: Lithium reduces suicidal
A. Clozapine ideation in bipolar patients.
B. Lamotrigine
C. Lithium Year: 2014.
D. Olanzapine
E. Sodium valproate.

32. An otherwise healthy 50-year-old male patient The answer is A.


receiving clozapine 300mg at night for treatment
resistant schizophrenia suddenly dies. The
Explanation: Myocarditis occur in 1 in
MOST likely cause of death is?
1300 patients and causes death by
A. Myocarditis loss of right ventricular function.
B. Neuroleptic malignant syndrome Year: 2014.
C. Pneumonia
D. Serotonin syndrome
E. Tardive dyskinesia.

33. A 28-year-old woman has a borderline The answer is C.


personality disorder, primarily manifested by an
inability to control her urges to hurt herself. She
Explanation: Fluoxetine is the first line
frequently cuts her wrists when frustrated.
Which of the following medications has been treatment for impulsivity, mood
found to be MOST helpful and safest in curbing lability and rejection sensitivity in
her self-harm behaviour? borderline personality disorder.

A. Aripiprazole Year: 2014.


B. Clonazepam
C. Fluoxetine
D. Lithium
E. Olanzapine.

Psychotherapy
34. A 30-year-old woman suffers from obsessive The answer is B.
compulsive disorder and washes her hand 50
times a day. Which of the following techniques
Explanation: Exposure and response
is the MOST essential in the psychological
treatment of her compulsive hand washing prevention is most important
behaviour? behavioural treatment for OCD.
Systemic desensitization is indicated
A. Cognitive restructuring for phobia.
B. Exposure and response prevention
C. Social skill training
Year: 2014.
D. Systemic desensitization
E. Thought stopping.

35. A 25-year-old woman with a panic disorder seeks an The answer is E.


opinion whether pharmacotherapy or psychotherapy is
the best treatment option for her. Which of the Explanation: Short-term
following statements is CORRECT? benzodiazepine is useful and does not
A. Adding alprazolam on p.r.n. basis will make
cause dependence. SSRI is useful to
exposure therapy ineffective.
B. Adding alprazolam in the first week will lead to treat panic disorder. Relaxation
dependence and does not improve patients’ therapy can be part of the behaviour
outcome. therapy of CBT.
C. Antidepressants such as selective serotonin
reuptake inhibitors are not useful to treat panic Psychotherapy requires more
disorder.
D. Relaxation therapy is more effective than cognitive
motivation from patient as patient
behaviour therapy in treating panic disorder. needs to cooperate with psychologist
E. Psychotherapy such as cognitive behaviour therapy and practise various behavioural
requires more motivation than taking medication. techniques and challenge cognitive
errors. It requires 12-16 weekly or
fortnightly sessions.

Year: 2014.

36. A 70-year-old man lodged a complaint against the The answer is B.


hospital. His wife died suddenly one year ago. She
suffered from diabetes and had frequent episodes Explanation:
hypoglycaemia. The husband gave her higher than Displacement refers to transferring
prescribed dose of hypoglycaemic agent because he the emotional response to a
thought this could control her diabetes. His children particular person which carries less
disagreed with his management but he refused to
emotional risk.
listen. The coroner’s report stated that his wife died of
hypoglycaemia. The 70-year-old man is angry with the
endocrinologist and thinks that it was his fault to In this case, the endocrinologist bear
prescribe the wrong hypoglycamic agent leading to his less emotional risk than his children
wife’s death. He demanded a written apology from the who are angry with him. He does not
endocrinologist. Which of the following defence want to blame himself. He displaced
mechanisms is demonstrated by this man? his guilt of causing his wife death to
A. Altruism the endocrinologist.
B. Displacement
C. Reaction formation
D. Splitting
Year: 2014.
E. Undoing.
This is an advanced level question.
37. For post-traumatic stress disorder, which of the The answer is C.
following interventions is LEAST helpful to
patients?
Explanation: Group debriefing may
be harmful based on recent research
A. Cognitive behaviour therapy
B. Eye movement desensitization and reprocessing findings.
C. Immediate group debriefing
D. Relaxation training Year: 2014.
E. Supportive psychotherapy

38. You are a resident in plastic surgery. A 24- The answer is A.


year-old woman believes her nose is
malformed and always checks her nose in the Explanation: The current guidelines
recommend offering cognitive
mirror. She was seen by multiple doctors and
behaviour therapy (including
they confirmed that her nose is normal. exposure response prevention) to
Which of the following psychological people suffering from body
treatment is the treatment of choice for this dysmorphic disorder. Exposure and
patient? response prevention is part of the
cognitive behaviour therapy.
A. Exposure and response prevention
B. Family therapy Year: 2014
C. Problem solving therapy
D. Psychodynamic psychotherapy
E. Supportive psychotherapy.

Ethics and laws


39. A 40-year-old woman complained of headache, The answer is C
backache, nausea, numbness and visual problems. She
consulted a neurologist whom she trusted to be a good Explanation: Fidiciary duty refers to
doctor. The neurologist diagnosed the patient suffering the duty that a doctor (i.e.
from migraine and somatisation disorder. When the neurologist) must act in the patient’s
patient requested for Magnetic Resonance Imaging
best interest. The neurologist is in a
(MRI), the neurologist claimed that the patient had
exaggerated her somatic symptoms and insisted that legal contract with the patient to
the MRI scan should be reserved for more severe provide best care. A fiduciary duty
patients. Six months later, the patient died of brain exists when the late patient places
tumour. In addition to negligence, which of the confidence in the doctor and relied
following ethical principles was violated by the upon him to exercise his expertise in
neurologist? diagnosing and managing her illness.
In this case, the neurologist failed to
A. Autonomy offer earlier MRI scan despite
B. Confidentiality patient’s request. He wrongly
C. Fiduciary duty attributed the neurological symptoms
D. Non-maleficience to somatisation disorder without
E. Justice. thorough investigation.

Year: 2014.

This is an advanced level question.


40. 40-year-old man arrested for shoplifting is The answer is C.
referred for a psychiatric assessment after he
tells the judge he suffers from kleptomania. Explanation:
Which of the following criteria is MOST
important in establishing the diagnosis of Option Act reus refers to a criminal act
kleptomania? which involves bodily movement
whether voluntary or involuntary. In
A. Acts reus. this case, it refers to the act of stealing
B. Patient informs you that he was diagnosed with and its presence suggests that he
kleptomania by a private psychiatrist before but indeed committed shoplifting.
cannot provide further clinical details. Similarly, Option D and E suggest that
C. Patient informs you that he cannot control his he had mostly likely committed a
impulse to steal and he does not want to steal. crime as he has tendency to steal and
D. Patient informs you that he has started stealing is not a honest person.
since Primary 6.
E. Past history of criminal record of stealing. Option B is a common reason
provided by patients who committed
shopkifting but we need further clinical
details to establish the diagnosis of
kleptomania.

Year: 2014.

41. A medical student has applied for a 3-month The answer is A.


research elective and he follows a professor to
conduct research. The professor asks him to
Explanation: Autonomy refers to the
conduct a cross-sectional questionnaire study
to assess mood symptoms of cancer patients. obligation of a doctor to respect a
He is expected to recruit 300 patients in 3 patient’s rights to make his or her
months. This project was approved by the own choice in treatment or
ethics committee. He attempts to obtain consent participation in research in
from an inpatient who has received accordance with his or her beliefs or
chemotherapy to participate in this study.
preferences. The patient has refused
Patient refuses because he feels very lethargic.
Thirty minutes later, he asks the patient to re- to participate and the decision should
consider participating in this study again. The be respected without further asking.
patient lodges a complaint to the professor
about this medical student. Which ethical Year: 2014.
principles has the medical student violated?

A. Autonomy
B. Beneficence
C. Confidentiality
D. Non-maleficence
E. Justice.

Liaison and neuropsychiatry


42. A 50-year-old man who is dependent on alcohol is The answer is A.
admitted for observation after a fall when intoxicated.
The ward team is concerned about complications Explanation: Delirium tremens usually
associated with alcohol withdrawal. Which of the
following statements is INCORRECT? occur 3-4 days after the last drink.
A. Delirium tremens usually occur 6-12 hours after
the last drink. Year: 2014.
B. Tremors usually occur 6-12 hours after the last
drink.
C. Anxiety usually occurs 12-18 hours after the last
drink.
D. Convulsion usually occurs 12-18 hours after the last
drink.
E. Sleep disturbance usually occurs 12-18 hours after
the last drink.

43. You are a resident in general surgery. A 50- The answer is D.


year-old man was admitted last night after he
Explanation: This patient suffers from
cut his head, arms and hands during a
hypochondriasis or illness anxiety
suicide attempt. He is married with a 10- disorder because he worries (not
year-old child. He works in the field of firmly believes as in delusion) that he
informational technology. According to his suffers from gastric cancer but not
wife, his elder brother died of gastric cancer confirmed by multiple investigations.
6 months ago. He believed that he is
genetically vulnerable for gastric cancer. He There are not enough symptoms to
saw multiple gastroenterologists and gastric support the diagnosis of
schizophrenia and abnormal grief.
surgeons. He went through multiple
endoscopies and biopsies with normal Year: 2014.
findings. Despite normal findings, he still
worries that he has undiagnosed gastric
cancer. He argued with his wife tonight and
wanted to commit suicide. What of the
following is the MOST appropriate
psychiatric diagnosis?

A. Acute stress reaction/marital problems


B. Depression/abnormal grief
C. Generalised anxiety disorder / phobia
D. Hypochondriasis/illness anxiety
E. Schizophrenia/psychosis.

44. You have become a consultant physician. A The answer is C.


resident called you and informed you that the
parameter of a 20-year-old male psychiatric Explanation: This young man suffers
patient over the phone, “His temperature is from neuroleptic malignant syndrome
41 degree Celsius. His heart rate is 105 per as evidenced by high fever, muscle
minutes and blood pressure fluctuates from rigidity, tachycardia, labile blood
120/70 to 180/100. His breathing is normal pressure, leucocytosis and laboratory
and chest is clear. His abdomen is soft. His evidence of muscle injury (i.e.
creatinine kinase is 800 IU/L (normal <100 increase in creatinine kinase levels)
IU/L) and leukocyte count is 10 x109/L after intravenous antipsychotic
(normal range: 1.5-5 x109/L). His limbs are injection three days ago.
very rigid and he demonstrates difficulty in
swallowing. He was given intravenous
injection of haloperidol 20mg three days Year: 2014
ago.” What is the MOST likely diagnosis?

A. Catatonia
B. Meningitis
C. Neuroleptic malignant syndrome
D. Serotonin syndrome
E. Status epilepticus.

45. You are a doctor working in the polyclinic. A The answer is D.


40-year-old man suffers from bipolar
disorder and he takes lithium on a daily Explanation: Thiazide diuretics, ACE
basis. His blood pressure is 170/100mmHg. inhibitors and NSAIDS increase
Which of the following medications is the lithium level and may cause lithium
BEST to lower his blood pressure? toxicity.

A. Angiotensin-converting-enzyme inhibitor Option B and C do not reduce blood


B. Alpha agonist pressure.
C. Benzodiazapine
D. Beta blocker The best answer is Option D.
E. Thiazide diuretic.
Year 2014.

This is an advanced level question


46. In medically ill patients with psychiatric illness, The answer is C.
which pharmacological properties of a
psychotropic drug is MOST appropriate? Explanation: Narrow therapeutic
index like lithium is dangerous to
A. Long half-life psychotropic drug.
medically ill patients.
B. Psychotropic drugs with active metabolites.
C. Psychotropic drug with wide therapeutic index.
D. Psychotropic drug which inhibits metabolic
enzymes. Year 2014
E. Psychotropic drug which induces metabolic
enzymes.
MCQs

Psychopathology
1. A 50-year-old woman claims that unfamiliar The answer is D.
people whom she met on the street were her
husband. She believes that her husband put Explanation: Fregoli syndrome is a
on a disguise. What is the diagnosis? delusional mis-indentification syndrome
when unfamiliar people are identified as
A. Capgras syndrome a familiar person.
B. Charles de Bonnet syndrome
C. DeClerambault’s syndrome Year: 2014
D. Fregoli syndrome
E. Ganser syndrome.

2. A 50-year-old woman hears her late mother’s The answer is C.


voice when she falls asleep. She cannot recall
the content of her mother’s speech. This Explanation: Hypnogogic hallucination
phenomenon is known as: occurs when a person falls asleep and
conscious level fluctuates considerably in
A. Auditory illusion different stages of sleep.
B. Hypnopompic hallucination
C. Hypnagogic hallucination Year: 2014.
D. Pseudohallucination
E. Third-person auditory hallucination.

3. You are seeing a 70-year-old man who suffers The answer is D.


from Parkinson’s disease. He demonstrates a
dulled emotional tone and seems to be Explanation: Apathy refers to detachment
indifferent to jokes. His family describes him from feeling or a dulled emotional tone.
as being emotionally detached. Which of the The person conveys a sense of
following terms BEST describes his indolence and indifference to what
psychopathology? normally causes excitement or interest.
Detachment is one of the aspects of
A. Alexithymia apathy.
B. Alogia
C. Anhedonia Anhedonia is defined as the inability to
D. Apathy experience pleasure from activities
E. Attentional deficit. usually found enjoyable which is less
evident in this case.

Year: 2014.
4. A 50-year-old Singaporean Chinese man The answer is E.
firmly believes that he needs to flee from
Singapore to Russia for his personal safety. Explanation: This man firmly believes
He has developed this belief after reading a that the newspaper article referred to him
newspaper article about Edward Snowden and this is impossible because the
who released national security materials of person involved is a Caucasian with US
the United States (US). This man firmly citizenship. This is known as delusion of
believes that the newspaper mentions his reference.
name and describes about his role in the leak
of US national secrets. Which of the following Year: 2014.
terms BEST describe his experience?

A. Delusion of erotomania
B. Delusion of grandiosity
C. Delusion of guilt
D. Delusion of jealousy
E. Delusion of reference.

Cognitive assessment
5. You are administering the Mini-Mental State The answer is B.
Examination (MMSE) to a 70-year-old woman
who did not receive any formal education. She Explanation: Option B is the most useful
has difficulty to complete the full version of task to assess her attention because she
MMSE. Which of the following tasks is MOST may not be able to perform serial 7 due
useful to assess her attention? to low level of education. Option A
assesses constructional apraxia. Option
C assesses orientation. Option D
A. Constructing a double pentagon assesses registration and short term
B. Naming the months from December in backward memory. Option E assess working
direction memory.
C. Orientation to time and place
D. Three-item registration and recall Year: 2014.
E. Three-stage command.

6. The Montreal Cognitive Assessment (MOCA) The answer is A.


is different from the Mini-Mental State
Examination. Which of the following Explanation: MOCA tests the frontal lobe
neuroanatomical structures is specifically function by the trail making task. This
assessed in the MOCA but not the MMSE? task assesses alternating sequencing
(number and alphabet) and rule out
A. Frontal lobe perservation.
B. Hippocampus
C. Occipital lobe Year: 2014.
D. Parietal lobe
E. Temporal lobe. This is an advanced-level question.

Psychiatric epidemiology / Aetiology


7. After 65 years of age, the rate of Alzheimer’s The answer is C.
disease changes approximately by how many
times every 5 years increase in age? Explanation: the rate of Alzheimer’s
A. 0.5 time disease doubles approximately every 5
B. 1 time years of age.
C. 2 times
D. 3 times Year: 2014
E. 4 times.

8. Lesions in which of the following The answer is C.


neuroanatomical areas is the MOST
vulnerable to amnestic syndrome? Explanation: Damage to hippocampus is
most vulnerable to amnestic syndrome.
A. Basal ganglia
B. Cerebellum
C. Hippocampus
D. Nucleus accumbens Year: 2014
E. Raphe nucleus.

General adult psychiatry / Psychiatric diagnosis


9. Which of the following psychiatric diagnoses The answer is B.
is MOST likely to contribute to inpatient
suicide? Explanation: Major depressive disorder is
most likely to contribute to inpatient
A. Borderline personality disorder suicide among all the options.
B. Major depressive disorder
C. Generalised anxiety disorder Year: 2014.
D. Hypochondriasis/ illness anxiety disorder
E. Schizophrenia.

10. Which of the following features is MOST The answer is E.


characteristic of Cotard’s syndrome?
Explanation: Cotard’s syndrome is a
A. Delusional belief that familiar people have been nihilistic delusion in which a person
replaced by their imposters believes that their possessions, friends,
B. Delusional belief that the spouse is unfaithful or parts of their own body do not exist or
C. Delusional belief that strangers have taken on the are about to not exist.
psychological identity of a familiar person
D. Delusional belief that a person of higher status Year: 2014.
falls in love with the patient
E. Nihilistic and hypochondriacal delusions found in
psychotic depression.

11. A 23-year-old man, recently promoted to a The answer is E.


middle management position, presents to
your clinic with complaints of acute episodes Explanation: The diagnosis is social
of anxiety associated with heart palpitations, phobia but not panic disorder because he
dyspnoea, shaking, detachment from his demonstrates those symptoms during the
body, and fear of losing control. These presentation in front of the management.
experiences only occur during oral
presentation in front of the upper Year: 2014
management. Which of the following
diagnoses is MOST likely?

A. Generalized anxiety disorder


B. Panic disorder
C. Panic disorder with agoraphobia
D. Post-traumatic stress disorder
E. Social phobia.

12. A 40-year-old woman, Ms. Tan has witnessed The answer is C.


a road traffic accident. Four hours ago, an
elderly man was knocked down by a taxi and Explanation: This woman suffers from
suffered from severe head injury. Ms. Tan was acute stress reaction after witnessing the
walking along the street and witnessed the road traffic accident. She develops
whole accident. She did not suffer from any dissociative amnesia and not able to
injury. The police officer wants to interview provide details related to the accident.
Ms. Tan to obtain more information about the
accident. She cannot recall any details related Year: 2014
to the accident. Which of the following is the
MOST appropriate explanation for her
amnesia?

A. Acute and transient psychosis


B. Conversion disorder
C. Dissociative state
D. Factitious disorder
E. Underlying dementia.

13. You are a medical officer working in the Army The answer is D.
Camp. A 19-year-old man is recently enlisted
for National Service and his parents inform Explanation: Stimulant treatment (e.g.
you that he walks around his flat during sleep. modanfinil) is used to treat narcolepsy
Which of the following recommendations is but not sleep-walking.
LEAST appropriate?
Year: 2014
A. Safety measures
B. Sleep hygiene
C. Stay out of camp at night
D. Stimulant
E. Supportive psychotherapy.
14. A 25-year-old man believes that he suffers The answer is B.
from adult ADHD. Which of the following
criteria is compulsory to establish such Explanation: History of ADHD symptoms
diagnosis? in childhood is a compulsory diagnostic
criteria for diagnosing adult ADHD
A. Absence of inattention symptoms in adulthood symptoms.
B. History of ADHD symptoms before the age of 12
C. History of conduct disorder Year: 2014
D. Presence of hyperactivity symptoms in adulthood
E. History of substance abuse.

15. All of the following findings are found in The answer is B.


anorexia nervosa EXCEPT:
Explanation: Growth hormone is
A. Decreased bone mineral density increased but not reduced in anorexia
B. Decreased growth hormone nervosa.
C. Decreased gonadotropins
D. Decreased potassium Year: 2014
E. Decreased heart rate.

Old age psychiatry


16. You are a family doctor and a 70-year-old man The answer is E.
consults you for annual medical assessment
to certify that he is fit to drive. Which of the Explanation: Option A, B, C and D
following is LEAST relevant to driving? assess driving and body systems that are
directly involved in driving. Blood
A. Assess his visual acuity by Snellen’s Chart pressure is not directly involved in driving
B. Assess his hearing by whispering to his ears and less likely to influence driving
C. Inquire his driving practice and road safety performance.
records
D. Examine his elbow and knee joints Year: 2014
E. Measure his blood pressure.

17. Which of the following is TRUE about suicide The answer is E.


in old people?
A. Old people with terminal illness have low suicide Explanation: Suicide in elderly is
risk because they foresee that they will die soon. common and they are determined to die.
B. Old people often attempt suicide as a means to Doctors should take this seriously and
manipulate other people. advise for hospitalization.
C. Old people frequently communicate their intent
prior to committing suicide. Year: 2014
D. Old people generally attempt self-harm rather
than suicide.
E. Suicide attempts are more lethal in old people as
compared to young people.

18. Which of the following statements is FALSE The answer is D.


about delirium in old people?
Explanation: Old people who take
A. Could be an initial presentation of underlying multiple medications (or polypharmacy)
dementia are at higher risk to develop delirium.
B. Onset could be insidious
C. Rarely results in full resolution of symptoms in a Year: 2014
short period of time
D. Un-medicated old people are at higher risk to
develop delirium as compared to old people
treated by multiple medications.
E. Usually persists for weeks or months in old
people hospitalized for medical or surgical
reasons.

19. The daughter of an 80-year-old woman with The answer is A.


history of bipolar disorder calls you for
advice. Her mother assaulted one helper and Explanation: As part of the risk
the other helper is terrified. They tried to stop management, this elderly lady should be
her from leaving the house to buy things. admitted to the psychiatry ward for
Which of the following is MOST appropriate assessment. Besides further assessment
management strategy? of bipolar disorder, the doctors need to
assess other medical causes of violent
A. Admission to the psychiatry ward behaviour (e.g. delirium or alcohol
B. Employ a part-time nurse to look after her at withdrawal)
home
C. Increase the dose of antipsychotics and review in Year: 2014
1 week
D. Increase the dose of benzodiazepine and review
in1 week
E. Increase the dose of mood stabilizer and review
in 1 week.

Child and adolescent psychiatry


20. A 4-year-old boy’s grandmother died four The answer is E.
months ago. The grandmother was one of the
child’s primary caregivers. The child still Explanation: This is a normal grief
speaks to the grandmother. When he walks, reaction. Child at this age may not
he still holds her hands up as though holding understand the concept of death. He may
hands with his grandmother. Which of the believe that his grandmother will return
following is CORRECT? one day.

A. This child fully understands the concept of death. Year: 2014


B. This child suffers from major depression.
C. This child is psychotic. This is an advanced-level question
D. The child suffers from adjustment disorder with
depressed mood.
E. This reaction is within normal limits for a child of
this age.

21. Which of the following is the MOST The answer is B.


established psychopharmacological treatment
for children and adolescents suffering from Explanation: Antipsychotic drug e.g.
autism? risperidone is the most established
psychopharmacological drug to treat
A. Actetylcholinesterase inhibitor young people with autism.
B. Antipsychotic drug
C. Benzodiazepine Year: 2014.
D. Omega-3 supplement
E. Stimulant.

Substance abuse
22. You are a resident working at the Accident The answer is D.
and Emergency Department. A 21-year-old
man is brought in by police and he is Explanation: Ketamine-induced
disorientated to time, place and person. He ulcerative cystitis which includes urge
needs to urinate very often and seems to have incontinence, decreased bladder
urinary incontinence. A package of capsules compliance, decreased bladder volume,
is found in his pocket and suspected to be detrusor overactivity, and painful
illicit drugs. Which of the following drugs is haematuria.
MOST likely to cause the above symptoms?
Year: 2014.
A. Amphetamine
B. Cannabis
C. Cocaine
D. Ketamine
E. Phencyclidine.

23. Hypertension associated with cocaine misuse The answer is C.


is BEST treated by:
Explanation: Benzodiazapine is the
A. Angiotensin-converting-enzyme inhibitors safest and most appropriate option here.
B. Beta-blockers The other option is alpha blocker which is
C. Benzodiazapine not available. Beta-blockers will worsen
D. Loop diuretics hypertension by leaving alpha receptors
E. Thiazide diuretics. unopposed. As cocaine increases
noradrenaline levels, diuretics are less
relevant.

Year: 2014.

This is an advanced level question.


24. Which of the following addiction has the The answer is A.
HIGHEST suicide risk?
Explanation: The risk for suicide in
A, Gambling gambling is twice the risk compared to
B. Internet other forms of addiction as a result of
C. Sex huge financial consequences and
D. Steroid addiction in young athlete unsettled debts. Other tell-tales signs of
E. Stimulant. suicide amog gamblers include insomnia,
poor appetite loss of interest to work and
somatic complaints.

Year: 2014.

25. You are a general practitioner (GP). A 20-year-old The answer is B.


university student wants you to prescribe
methylphenidate (ritalin). The student believes Explanation: In Singapore, patients are
that ritalin can enhance his examination more likely to misuse prescribed
performance and he has been obtaining Ritalin medication than illicit substances due to
prescribed from another GP. He does not have tight control of illicit drugs. This student
history of attention deficit and hyperactivity may have misused methylphenidate as
disorder (ADHD). Which of the following advices stimulants and see different doctors to
is MOST APPROPRIATE? obtain methylphenidate. He is
recommended to stop taking
A. Advise him to take methylphenidate on the days methylphenidate and there is no
of examination and stops immediately after the evidence to suggest that this will help his
examination. examination performance.
B. Advise him to stop taking methylphenidate
because it is a controlled drug and not indicated Year: 2014.
in his situation.
C. Methylphenidate can enhance his attention if he
has family history of ADHD.
D. Methylphenidate can enhance his attention if he
has past history of conduct disorder.
E. There is evidence to suggest that
methylphenidate can enhance his attention and
improve his examination performance.

Psychopharmacology
26. A 30-year-old man suffers from autism and The answer is A.
epilepsy. He has been very aggressive. The
psychiatrist prescribes sodium valproate Explanation: The patient is on a relatively
1300mg nocte to control his aggression and high dose of sodium valproate. The first
epilepsy. His mother concludes the sodium step should check serum level of sodium
valproate is not effective. The first step of valproate because patient may not non-
management should be: compliant to the medication.

A. Check the serum sodium valproate level Year: 2014.


B. Increase the dose of sodium valproate
C. Increase the dose of sodium valproate and add This is an advanced level question.
an antipsychotic drug
D. Refer the patient to see a psychologist
E. Switch to another antipsychotic drug.
27. Which of the following side effects is more The answer is E.
common in tadalafil as compared to other
phosphodiesterase type 5 inhibitors such as Explanation: Tadalafil has the longest
vardenafil and sildenafil? half-life, 17.5 hours as compared to
vardenafil (4-5 hours) and sildenafil (3-4
A. Dyspepsia hours).
B. Flushing
C. Headache Year: 2014.
D. Impaired vision
E. Priapism. This is an advanced level question.
28. A 30-year-old man suffers from schizophrenia. The answer is B.
He presents a list of drugs which he took in
the past. Which of the following is NOT a Explanation: Option B is a first-
second generation antipsychotic drug? generation antipsychotic drug.
a. Clozapine
b. Flupenthixol Year: 2014
c. Olanzapine
d. Quetiapine
e. Risperidone.

29. A 30-year-old man suffers from insomnia and The answer is C.


he sleeps well after taking diazepam. He
worries that he lacks a particular Explanation: Benzodiazapine drugs bind
neurotransmitter in the brain and diazepam to benzodiazepine receptors and this
can enhance the actions of this enhances the binding of γ-Aminobutyric
neurotransmitter. Which of the following acid (GABA) to GABA receptors. This will
neurotransmitters is MOST relevant In this lead to influx of chloride ions and cause
case? sedation.

A. Acetylcholine Year: 2014


B. Dopamine
C. γ-Aminobutyric acid
D. Noradrenaline
E. Serotonin.
30. Mrs. Tan is a 65-year-old woman who lost her The answer is C.
husband three years ago. She is still angry
with him for not taking better care of his Explanation: increase fluoxetine to 80mg
health. He was cremated and she keeps his may worsen anxiety and restlessness.
ashes in the urn. She is unsure what to do
with the ashes. She coped well after her Year: 2014
husband’s death, until her son moved to
Australia a few months ago. She has long This is an advanced level question.
standing headaches, but these have become
worse over the past six months. She also has
new onset of somatic symptoms as well as
anxiety symptoms which are overwhelming
and include a fear of dying. Her depression
has been partially treated with fluoxetine
40mg per day, but her anxiety symptoms
persist and she feels restless. Which of the
following actions is LEAST appropriate?

A. Add alprazolam to her treatment for 2 weeks


B. Discontinue fluoxetine and start other
antidepressant drug such as paroxetine
C. Increase the dose of fluoxetine to 80mg per day
D. Order a thyroid function test
E. Start psychotherapy with a focus on grief.

31. Which of the following antidepressants cause The answer is D.


MOST SEDATION after potentiation with
benzodiazepine? Explanation: Mirtazapine is the most
sedative antidepressant. The sedation
A. Agomelatine effect will be worsen after potentiation
B. Escitalopram with benzodiazepine.
C. Fluvoxamine
D. Mirtazapine
E. Venlafaxine. Year: 2014.
32. Which of the following factors limits the use The answer is E.
of clozapine in treatment-resistant
schizophrenia? Explanation: 1 to 2% of patients using
clozapine will develop agranulocytosis,
A. A low incidence of seizures and this is the main factor limiting its use;
B. The lack of efficacy against positive symptoms risk increases with age, and is higher
C. The lack of efficacy against negative symptoms among women; the risk is the highest in
D. The lack of efficacy against cognitive symptoms the first six months of treatment (hence
E. The occurrence of agranulocytosis. the need for weekly blood tests in the
first six months, and then every two
weeks after that).

Year: 2014.
33. Which of the following medications is MOST The answer is C
likely to exacerbate psoriasis?
Explanation: Lithium has higher risk to
A. Amitriptyline worsen psoriasis as compared to other
B. Haloperidol psychotropic medications.
C. Lithium
D. Risperidone
E. Sodium valproate. Year: 2014.

This is an advanced level question.


34. Which of the following is CORRECT regarding The answer is E.
repetitive transcranial magnetic stimulation
(rTMS)? Explanation: rTMS does not require
general anaesthesia. rTMS is an
A. rTMS is administered under general anaesthesia. alternative to ECT if patient has medical
B. rTMS is more efficacious than electroconvulsive or surgical contraindicates to ECT. rTMS
therapy (ECT). is not intended to induce seizures and it
C. rTMS induces seizures with less energy as is not related to imaging.
compared to ECT.
D. rTMS is a new imaging modality in psychiatry.
E. rTMS may be useful in the treatment of Year: 2014.
depression.

35. The MOST robust effect of lamotrigine is The answer is A.


found in which of the following?
Explanation: Lamotrigine is effective in
a. Bipolar depression treatment of bipolar depression with
b. Manic episode 200mg per day.
c. Mixed episodes
d. Hypomanic episode
e. Cyclothymia. Year: 2014.

36. A 30-year-old woman complains of lack of The answer is C.


sexual drive after taking paroxetine. She
wants to change her antidepressant. Which of Explanation: The 5HT2A antagonism of
the following antidepressant is the BEST mirtazapine is associated with lesser
option to avoid sexual dysfunction? sexual side effects.

A. Amitriptyline
B. Reboxetine Year: 2014.
C. Mirtazapine
D. Duloxetine
E. Venlafaxine.
37.Agomelatine, a new antidepressant is BEST The answer is D.
given in which of the following parts of the day?
Explanation: Early evening is the best
A. Early morning time for circadian rhythms
B. Late morning resynchonizing (1-2h before dark phase).
C. Early afternoon
D. Early evening
E. Past mid-night.
Year: 2014.

Psychotherapy
38. A 24-year-old depressed woman said, “People The answer is C.
in my office try to make my life difficult and
then deem themselves as ultimate smart to Explanation: Interpersonal therapy (IPT)
know about me. I have resigned as I am is indicated for depressive disorder. It
uncomfortable with the office settings. I am helps the patient to understand the
aware that I am getting verbally aggressive communication difficulties (verbal
towards other people whenever they provoke aggression). Psychotherapist can
me. I want to see a psychotherapist because I perform role-play to help patient to
want to deal with these people before I lose my improve communcation. IPT also helps
self-identity.” Which of the following patient to deal with the loss associated
psychotherapies is MOST appropriate? with her self-identity and resignation.

A. Cognitive behaviour therapy


B. Grief therapy Year: 2014.
C. Interpersonal therapy
D. Relaxation exercise This is an advanced level question.
E. Supportive psychotherapy
39. You are a family doctor. A 30-year-old woman The answer is E.
suffers from schizophrenia. Although she is
free from first rank symptoms, she is affected Explanation: Sensate focus therapy is a
by weight gain, negative symptoms of
schizophrenia and no interest in sex. Her
sex therapy. The couple focus on non-
husband has high sexual drive and he is not vaginal intercourse and master other
happy with their sexual life. He requests for techniques such as communication and
anti-androgen to reduce his sexual drive. mutual stimulation. Prescription of anti-
What is BEST management strategy? androgen is associated with side effects
such as osteoporosis. The other options
A. Cognitive behaviour therapy
are not specific for psychosexual
B. Eye movement desensitization and
processing problems.
C. Hypnotic therapy
D. Prescribe anti-androgen Year: 2014
E. Sensate focus therapy.

40. A 40-year-old man suffers from post-traumatic The answer is C.


stress disorder (PTSD) and has history of
misusing cannabis. He claims that the
Explanation: This man justifies cannabis
government should legalise the sales of
cannabis because cannabis can reduce the misuse with an explanation but totally
PTSD symptoms and helps him to relax. What forgets the potential harms associated
is his defence mechanism? with cannabis misuse.

A. Denial Year: 2014


B. Projection
C. Rationalisation
D. Repression
E. Sublimation.

Ethics and laws


41. The Mental Health (Care and Treatment) Act in The answer is B.
Singapore can be applied at which of the
following hospital(s) for involuntary
Explanation: The Mental Health Act can
admission?
only be applied at IMH for involuntary
a. Changi General Hospital (CGH) only admission.
b. Institute of Mental Health (IMH) only
c. National University Hospital (NUH) only Year: 2014.
d. IMH, NUH and CGH
e. IMH, NUH, CGH, Singapore General
Hospital (SGH) and Tan Tock Seng
Hospital (TTSH).

42. A 21-year-old woman suffering from body The answer is D.


dysmorphic disorder consulted a doctor for
Botulinum toxin (botox) treatment. She was
Explanation: Non-maleficence refers to
concerned of the smoothness of her face and
spent $10,000 for multiple botox injection. The the obligation of a doctor to avoid harm,
botox injection caused paralysis of facial causing muscular atrophy by botox
muscles. Botox injection is not indicated for injection in this case.
women younger than 30 years and body
dysmorphic disorder. Which of the following Year: 2014.
ethical principles was violated by this doctor?

A. Autonomy
B. Capacity
C. Confidentiality
D. Non-maleficence
E. Justice.
43. A 25-year-old man was arrested for murder. The answer is C.
According to the police, he shows no remorse
towards his act. He firmly believes that killing
Explanation: In forensic psychiatry,
one person will not lead to death sentence. He
needs to kill several people to get death psychopath represents the most severe
sentence. He always carries a chopper and form of antisocial personality disorder.
wants to attack innocent people. If he is People with psychopathy demonstrate
released from the prison, he will kill more extremely low level of empathy and
people, including his parents. Which of the remorse. They have high chance of
following diagnoses is MOST appropriate?
recidivism.
A. Biopathy
B. Egopathy Year: 2014.
C. Psychopathy
D. Sociopathy
E. Superegopathy.
Consultation liaison psychiatry
44. You are a resident working in the Children The answer is D.
Emergency Department. A 5-year-old boy is
presented for recurrent urinary tract infection
Explanation: Münchausen syndrome by
and confirmed by raised white blood cell
proxy occurs when an individual (the
counts in urine. According to his mother, the
mother) must have presented another
boy likes to insert foreign body into his penis
individual (the boy) to others (AED
via the urethra meatus. After taking a
doctor) as medically ill (recurrent urinary
thorough history and conducting physical
tract infection) without the intention of
examination, you cannot gather any evidence
obvious external rewards but invasive
the boy could insert foreign body into his
investigation or hospitalizations.
penis. It is not anatomically and
Münchausen syndrome by proxy is
physiologically feasible for the boy to do so.
considered to be a form of child abuse.
You cannot exclude the possibility that it
She is displacing her anger (husband
could have been done by his mother. History
visiting commercial sex workers) towards
also reveals that the parents have marital
her son.
problems and his father is suspected to visit
commercial sex workers frequently. She is
very keen to admit her son for further invasive Year: 2014.
investigations. What is the MOST likely
diagnosis?

A. Conversion disorder induced by the mother


B. Hypochondriasis (or illness anxiety) induced by
the mother
C. Malingering induced by the mother
D. Münchausen syndrome by proxy induced by the
mother
E. Somatisation disorder induced by the mother.
45. Which of the following questionnaires would be The answer is D.
MOST useful in identifying symptoms of
depression and anxiety in patients suffering Explanation: The Hospital Anxiety and
from chronic medical illness? Depression Scale is the best scale
because the items are specially
A. The Beck Anxiety and Depression Inventory constructed to enhance sensitivity in
B. The General Health Questionnaire detecting depression and anxiety
C. The Hamilton Anxiety and Depression Rating symptoms in medical patients. The Beck
Scale depression and anxiety inventory and
D. The Hospital Anxiety and Depression Scale Hamilton Anxiety and Depression Rating
E. The State-Trait Anxiety Inventory. Scale are more suitable for psychiatric
patients.

Year: 2014.
46. You are a resident working in the Accident The answer is A.
and Emergency Department (AED). A 26-year-
old woman gave birth one month ago. She is a
Explanation: In this situation, both
single mother and broke up with her
boyfriend. She has poor social support and mother and baby should be admitted to
no one helps her to look after the baby. She the hospital because no one looks after
has been depressed for 1 month and wanted the baby at home. IMH does not have
to jump down from her flat two weeks ago. facility to look after her baby.
She was referred to the woman’s mental
health service. She was referred to a case The psychiatrist in the hospital should
manager but no medication was prescribed.
consider starting antidepressant because
She breastfeeds her baby. Today, she wants
to jump again. The mother and her baby are she is highly suicidal and SSRI like
bought to the AED by the case manager. sertraline is safe for breastfeeding.
Which of the following is the BEST
management? Year: 2014.
A. Admit the mother and her baby to the hospital.
The mother will go to the psychiatric ward and the
baby will go to paediatric ward. She will benefit
from antidepressant and psychological
intervention.
B. Admit the mother and her baby to the hospital.
The mother will go to the psychiatric ward and the
baby will go to paediatric ward. She should
receive psychological intervention only because
antidepressants are dangerous for breastfeeding.
C. Discharge the mother and her baby from the AED
because she suffers from adjustment disorder.
Her risk is low.
D. Discharge the mother and her baby from the AED
because she receives good care from case
manager.
E. Send both mother and her baby to the Institute of
Mental Health for compulsory admission.
47. What of the following psychiatric side effects The answer is D.
is MOST common in Hepatitis C patients
receiving interferon treatment? Explanation: Interferon treatment is
well known to be associated with
A. Anxiety
depression.
B. Cognitive impairment
C. Confusion
D. Depression Year: 2014.
E. Hallucination.

48. The hospital wants to develop the best The answer is A.


method to detect and identify delirium in
medically ill patients. Which of the following Explanation: Option A is the most cost-
is the BEST strategy to detect and identify effective strategy to detect and identify
delirium? delirium. Option D is not the best strategy
because there are elderly patients who
A. Adm inis ter Delir ium Rating Sc ale t o al l develop delirium without any laboratory
patie nts old er tha n 6 5 ye ars to s c reen abnormalities. Option E is not the best
f or delir ium . strategy because history of delirium may
B. Look ing f or abnor m alities on not predict future episodes of delirium.
c om puter i zed to pogr aph y (CT ) brain
s c ans f or all pat ients . Year: 2014.
C. Look ing f or abnorm alities on m agnetic
res onanc e im aging ( M RI) brain s c ans f or
s us pec ted pati ents .
D. Monit or s igns of delir ium on thos e
patie nts with labor at or y abn orm alities .
E. Sc reen f or his tor y of delirium and
m onitor thos e pat ient s with his tor y of
delir i um onl y.
Question 51 – 53

Options:
A. Cerebral autosomal dominant arteriopathy with subcortical infarcts and leukoencephalopathy
B. Korsakoff syndrome
C. Dissociative fugue
D. Vascular dementia
E. Lewy body dementia

Lead-in: Match the appropriate diagnosis to each of the following clinical scenarios.

51. A 60-year-old woman is brought in by the police for psychiatric assessment as she was found
missing for a few days. She is unable to provide any personal details about herself and appears to be in
a daze. She cannot recall where she has been in the past 3 days. Her son, with whom she is very close,
died in a road traffic accident few days ago. She appears to be tidy and well-groomed. All
investigations done for her are normal. (Choose 1 option)
52. A 65-year-old man presents with fluctuating memory loss with mood swings. He has slurred speech
and left-sided muscle weakness. He has multiple medical comorbidities, including poorly controlled
hypertension and diabetes. (Choose 1 option)

53. A 60-year-old man is brought in by the police to the hospital for assessment as he was found
wandering the streets. When the doctor speaks to him, he says that he is here to meet a friend who was
hospitalized and that he has seen the doctor before. Medical records show that he has been a chronic
alcohol drinker. (Choose 1 option)

Answers:

51: C

52: D

53: B

Question 54-55

Options:

A. 5-HT2 antagonist

B. Buytrylcholinesterase inhibitor

C. D2 receptor partial agonist

D. GABAA agonist

E. NMDA receptor antagonist

Lead-in: select the mechanism of actions for the following drugs.

54. Diazepam (Choose 1 option)


55. Aripiprazole (Choose 1 option)

Answers:

54: D

55: C

Question 56-57:

Options:

A. Action

B. Contemplation

C. Pre-contemplation

D. Maintenance

E. Relapse

Lead-in: identify the stages of behaviour change based on the model established by Prochaska and
DiClemente.

56. Begins to realize that his/her behaviour has both advantages and disadvantages and start to feel

conflicted about his/her behaviour. (Choose 1 option)

57. Makes an initial attempt to rectify the problematic behaviour. (Choose 1 option)
Answers:

56: B

57: A

Question 58-60

A. Antisocial personality disorder

B. Conduct disorder

C. Enuresis

D. Oppositional defiant disorder

E. Separation anxiety

Lead –in: Match the appropriate diagnosis to each clinical scenario.

58. A 8 year-old boy regularly wets his bed at night ever since his parents were divorced. He has also

been noted by his teacher to be more quiet and irritable than usual since the parents’ separation.

(Choose 1 option)

59. A 15-year-old boy regularly bullies and gets into fights with fellow students in school. He has on

several occasions set fire to the grocery shops and vehicles. He also has a history of stealing from

others and cruelty to animals. (Choose 1 option)

60. A 8-year-old boy is frequently rude and argumentative towards his teachers. He gets irritated
easily

by others and would often disobey the school rules. Nevertheless, he is not aggressive towards
others and has no forensic history. He does not appear to have any problems at home.
(Choose 1 option)

Answers:
58: C

59: B

60: D
MCQs

History, mental state exam, psychopathology and risk


assessment
1. You are a resident working in the accident and The answer is B.
emergency department (AED). A 30-year-old
schizophrenia patient is brought in by his mother.
He suddenly heard a voice asking him to use a sword Explanation: This patient is
to stab himself and open up his abdomen. His at high risk due to two
mother stopped him and bought him to the hospital. reasons: 1) command
He does not have antisocial behaviour. Which of the hallucination 2) the
following BEST describes his current risk at the possibility of damaging the
AED? aorta and liver. He may die
in 30 seconds if aorta is
A. High risk due to the possibility that he may attack other damaged.
people.
B. High risk due to the possibility of damaging internal Year: 2014
organs such as aorta and liver if he stabs himself.
C. Low risk due to the fact that he has informed his mother 。
and the chance to carry out action is low.
D. Low risk due to the unlikeliness for him to harm
himself as a result of disorganized behaviour seen in
schizophrenia patients.
E. Moderate risk due to possibility of superficial cut on
abdomen.

2. You are a resident working in the accident and The answer is C.


emergency department. A 30-year-old bank officer is
brought in by her supervisors after she has a failed
Explanation: The patient
suicide attempt in the bank. She was recently
transferred out from the head office to another branch. suffers from delusion of love
According to her relatives, the patient claims that the for a person with high status
Chief Executive Officer (CEO) fell in love with her. She but has no contact with the
spent hours waiting for the CEO after work. She tried to patient.
stop his car from leaving the car park. The CEO has
never worked with her and does not know her
personally. Which of the following is the correct
Year: 2014
diagnosis?

A. Capgras’ syndrome
B. Charles de Bonnet’s syndrome
C. De Clérambault's syndrome
D. Ganser’s syndrome
E. Othello’s syndrome

3 Which of the following is NOT a thought disorder found in The answer is B.


patients suffering from schizophrenia?
Echopraxia is the motor symptom
A. Derailment analogous to echolalia.
B. Echopraxia Echopraxia refers to the imitation
C. Loosening of association of movements and gestures of the
D. Knight’s move thinking person the patient is observing.
E. Word salad.
Year: 2013.

1
Cognitive assessment
4 Which of the following is NOT a prominent cognitive The answer is A.
feature of cortical dementia?
Explanation: Articulation
A. Problems with articulation
B. Problems with calculation difficulty is associated with
C. Problems with drawing a clock face subcortical dementia which is
D. Problems with expression of emotion characterised by neurological
E. Problems with recognition sign.

Year: 2014

5 A 20-year-old woman complains of low mood because The answer is B.


she is bullied in her workplace. She is a factory worker
and her supervisor complains that she is very slow at
Explanation: She has mild
work. She stays with her family and is able to look after
herself. Her mother describes the patient as simple- intellectual disability which is
minded. When she was young, she exhibited delay in characterised by ability to
understanding and learning language. She kept failing work in practical occupations
the “N” level examination despite multiple attempts. and ability to look after
Which of the following BEST describes her current level oneself.
of intelligence?

A. Normal intelligence Her slowness at work,


B. Mild intellectual disability repeated failing of the “N”
C. Moderate intellectual disability level examination and delayed
D. Severe intellectual disability in language development do
E. Profound intellectual disability. not support normal
intelligence.

Year: 2014

Psychiatric epidemiology
6 By 2020, which of the following illnesses is The answer is D.
predicted to have higher global disease burden
than depressive disorder? Explanation: Based on the
prediction of WHO, ischaemic
A. Anxiety disorder heart disease will rank No.1
B. Dementia while depressive disorder will
C. Diabetes rank No.2 in global disease
D. Ischaemic heart disease
burden.
E. Schizophrenia.
Year: 2014

7 Which of the following mood disorders has the HIGHEST Answer is D.


rate of co-occurrence with a substance abuse disorder?
Explanation: Bipolar disorder
A. Adjustment disorder
B. Seasonal affective disorder is the commonest mood
C. Cyclothymia disorder and demonstrates
D. Bipolar disorder the highest rate of co-
E. Dysthymia. occurrence with a substance
abuse disorder among all the
choices.

2
Year: 2014

Psychiatric aetiology, diagnosis and classification


8 Which of the following neuroanatomical areas The answer is E.
is MOST implicated in obsessive compulsive
Explanation: Orbito-frontal
disorder? cortex and basal ganglia are
most implicated in compulsive
A. Amygdala and prefrontal cortex behaviours associated with
B. Frontal lobe and parietal lobe OCD.
C. Hippocampus and locus coeruleus
D. Nucleus accumbens and prefrontal cortex Year: 2014
E. Orbito-frontal cortex and basal ganglia
9 Which of the following is the MOST common cause of Answer is D.
secondary hypersomnia?
Explanation: Obstructive sleep
A. Consumption of hydroxyzine
B. Dementia apnoea is the most common
C. Mixed anxiety and depression cause of secondary
D. Obstructive sleep apnea hypersomnia
E. Shift work.
Year: 2014

10 You are a general practitioner. A 33-year-old man The answer is D.


complains of stress due to interpersonal problems and
long term unemployment. He claims that he suffers from
Explanation: This patient is
depression but does not require antidepressant
treatment. He has no history of self-harm or criminal characterised by being
record. He has long-standing problems with his wife. His perfectionistic, rigid, frugal
wife mentions that he knows every detail about cooking and preoccupation with minor
and household chores. She finds the 3- year marriage details.
miserable and has gone through a very difficult period. His personality disorder
His wife realises that she will never satisfy his
resulted in long term
expectations. She describes the patient as being too
idealistic and extremely careful. When she cooks, the unemployment, interpersonal
patient tries to belittle her by watching her every single problems and marital
movement and interjecting a lot of advice to make a problems.
perfect meal. He often scolds her for spending money
and she is afraid of the patient. She finds him very rigid Year: 2014
with no sense of humour. Which of the following is the
correct diagnosis?

A. Avoidant personality disorder


B. Borderline personality disorder
C. Narcissistic personality disorder
D. Obsessive compulsive personality disorder
E. Paranoid personality disorder.

11 The mother of a schizophrenia patient wants to consult The answer is B.


you the risk of his dizygotic twin brother in developing
schizophrenia. Your answer is:
Explanation: The risk for
A. 4% another dizygotic twin to
B. 14% develop schizophrenia is 14%.
C. 24%
D. 34% Year: 2014
E. 44%.

3
General adult psychiatry
12 You are a general practitioner. A 22-year-old The answer is A.
patient comes to consult you because she is
stressed. Her father boarded an aeroplane Explanation: As the plane has
which was found missing and probably has crashed, the chance of survival
crashed. She is not certain about the status of for her father is low. She will
her father and he is not contactable. At this experience anticipatory grief
moment, she is MOST likely to develop which during the period of
of the following psychiatric conditions? uncertainty.

Year: 2014
A. Anticipatory grief
B. Compensation neurosis
C. Dissociative state
D. Psychosis
E. Hypochondriasis.

13 A 30-year-old schizophrenia patient drops out The answer is A.


from rehabilitation. The family member wants
to find out the underlying reason. Which of the Explanation: Auditory
following factors is LEAST likely to restrict hallucination is positive
rehabilitation potential? symptom and responds to
antipsychotic drug. Other
A. Residual auditory hallucination options are negative
B. Intellectual disability symptoms of schizophrenia
C. Lack of pleasure from social intervention and and intellectual disability.
physical activity These factors are less likely to
D. Poor motivation respond to antipsychotic
E. Restricted affect. treatment.

Year: 2014
14 Which of the following statements concerning generalized The answer is C.
anxiety disorder is FALSE?
Explanation: Option C refers to
A. Alcohol misuse is a common comorbidity. panic disorder.
B. Cognitive behaviour therapy is generally helpful.
C. Excessive worries and anxiety symptoms are episodic. Year: 2014
D. Short-term benzodiazepines can be an effective treatment
E. There is a 2 to 1 ratio of women to men suffering from this
disorder.

15 Which of the following is TRUE regarding delusional The answer is E.


disorder?
Explanation: Delusion disorder
A. Infrequent hallucinations totally rule out the diagnosis of is quite rare, from 0.025 to
delusional disorder. 0.03%. The mean age of onset
B. Onset is in adolescence. is 40 years. Delusional
C. Point prevalence is 3%. disorder does not cross over in
D. From genetic point of view, there is frequent crossover to family studies with
other psychotic disorders such as schizophrenia. schizophrenia or mood
E. The persecutory subtype is the most common. disorder. Olfactory
hallucinations in a somatic
delusion of body odour, may

4
occur.

Year: 2014.
16 A 45-year-old man consults you because he is stressed The answer is A.
over his daughter’s eating disorder. His daughter suffers
from anorexia nervosa. He is very concerned about her
Explanation: Based on eating
condition. He wants to seek your reassurance. Which of
the following indicates a GOOD prognostic factor for his disorder research, early age of
daughter? onset indicates good
prognostic factor. The other
A. Early age of onset factors are poor prognostic
B. Past hospitalisation factors.
C. Frequent self-induced vomiting rather than food restriction
D. Very low BMI
E. Perfectionistic personality. Year: 2014.

17 You are a general practitioner. A 25-year-old man with The answer is D.


history of Asperger’s syndrome. He has history of
violent and compulsive behaviour. After recent increase
Explanation: Sertraline is A
in dose of psychotropic medications, his father
discovers that he masturbates less often and seems to SSRI and most likely to cause
develop erectile dysfunction. Which of the following sexual dysfunction among all
medications is MOST likely to cause erectile the choices.
dysfunction?
Year: 2014.
A. Alprazolam
B. Bupropion
C. Mirtazapine
D. Sertraline
E. Hydroxyzine
18 You are a general practitioner. A 30-year-old man comes The answer is B.
to the clinic and claims that he received a diagnosis of
bipolar II disorder in the United States. Bipolar II Explanation: Bipolar I
disorder is best described as: disorder is characterised
by at least one manic
A. Dysthymia and cyclothymia
B. Hypomanic episode and depressive episode episode. Bipolar II disorder
C. Manic episode only is characterised by
D. Manic episode and depressive episode hypomanic episode and
E. Seasonal affective disorder depressive episode.

Year: 2013.
Old age psychiatry
19 Which of the following statements about depressive The answer is D.
disorder in old people is TRUE?
Explanation: Depression in
A. Depressive disorder in elderly is accompanied by a much
lower suicide risk than in younger adults. elderly is accompanied by
B. Depressive disorder in elderly is less likely to be associated higher suicide risk and more
with paranoia as compared to younger adults. paranoia. Elderly takes longer
C. Elderly suffering from depressive disorder takes shorter time time to respond as they cannot
to respond as compared to younger adults tolerate medications as good as
D. Mild depressive disorder is more prevalent in elderly women
young adults. Prevalence of
than men.
E. Prevalence of depressive disorder rises sharply with age in depression in elderly is around
the community. 3% (10-15% if depressive
symptoms are included). Only
10% of depression emerges in

5
elderly and there is no sharp
rise in prevalence.

Year: 2014.

20 Which of the following is MOST commonly seen as a The answer is C.


normal change associated with aging?
Explanation: Option A, D and E
A. Increase in acetylcholine should be reduced. Increase in
B. Increase in paranoia paranoia occurs in late onset
C. Increase in brain ventricle size depression, bipolar disorder and
D. Increased electroencephalogram (EEG) frequency schizophrenia but not part of
E. Increased slow-wave sleep. normal aging.

Year: 2014
21 What is the MOST likely diagnosis in a 75- year-old man The answer is B.
with declining cognition, visual hallucinations, and
parkinsonism symptoms? Explanation: The classical triad
of dementia with Lewy Bodies is
A. Alzheimer’s disease memory loss, visual
B. Dementia with Lewy Bodies hallucination and parkinsonism.
C. Late-onset schizophrenia
D. Fronto-temporal lobe dementia Year: 2014
E. Vascular dementia.

22 Which of the following statements is false with The answer is D


regards to delirium in the elderly?
Explanation: Delirium will take
A. 30% or more of the elderly admitted to medical long time to resolve and not
ward will develop delirium during hospitalisation. typically resolve in 3 days.
B. Antipsychotic drug is the first-line psychiatric
treatment for delirium. Year: 2014
C. Delirium is often overlooked by health care
professionals in the elderly.
D. Delirium typically resolves within 3 days after
correcting the underlying cause.
E. Incidence of delirium increases with age.
Child and adolescent psychiatry
23 If a young boy presents with intellectual disability, The answer is D.
autistic features and multiple maternal relatives are
carriers of a genetic condition. Which of the following
Explanation: Males with
diagnosis is MOST likely?
intellectual disability, autistic
A. Angleman syndrome features and have maternal
B. Down syndrome relatives being carrier highly
C. Prader Willi syndrome suggestive of fragile X
D. Fragile X syndrome syndrome. Option A, C, and E
E. Williams syndrome
are microdeletion syndromes
and less likely to run in families.

6
Year: 2014

Addiction and Substance abuse


24 You are an orthopaedic resident. A 22-year-old man The answer is B.
suffers from cervical spondylosis and results in severe
neck pain. His pain does not respond to analgesics and
Explanation: This patient has
becomes very depressed and suicidal. His mother
describes him as being obsessed with online games. He psychological withdrawal (i.e.
is an active member of several online game groups. He agitation) when he is not
spends more around 20 hours per day in the internet allowed to play online game.
café and becomes very agitated when he is not allowed This is common in other forms
to play online games. He has history of alcohol of substances abuses (e.g.
dependence. Which of the following symptoms found in
opioid dependence) when the
this patient is MOST important in establishing the
diagnosis of internet addiction? subject is denied access to the
substance.
A. Active member of the online group
B. Agitation when not allowed to play online game Year: 2014
C. Cervical spondylosis
D. Depression and suicidal thought
E. History of alcohol abuse.
25 Which of the following is NOT a factor which increases the risk The answer is C.
of relapse to cocaine use in a 30-year-old man who recently
became abstinent from cocaine? Explanation: Seeing a counsellor
A. Feeling depressed, tired and the need to overeat. to talk about the previous use
B. Going to pubs where “ice” and “special K” are available. does not increase the risk of
C. Seeing a counsellor to talk about the previous use of cocaine. relapse. Option A refers to crash
D. Seeing a psychiatrist to complain about attention deficit and try to from stimulation. Option B, C,
obtain methylphenidate. and E will increase the exposure
E. Spending time with colleagues who use “coke”.
to stimulants.

Year: 2014

26 A 27-year-old woman presents with unplanned The answer is D.


pregnancy. She smokes cannabis 3-4 times per
day. Her husband is concerned about the effects of Explanation: She has no
cannabis on her baby. During the interview, she motivation to change with
avoids eye contact and appears to be very anxious. impaired insight on the effect of
She claims that she eats and sleeps well. She cannabis on the foetus. This
thinks cannabis is safe for her foetus because she stage is known as pre-
has a friend who delivered a healthy baby despite
contemplation.
continued use of cannabis during pregnancy.
Which Prochaska’s and Diclemente’s stage of
change BEST describes her current status? Year: 2014

A. Action
B. Contemplation
C. Denial
D. Precontemplation
E. Maintenance.

Psychopharmacology
27 Which of the following neurochemicals is metabolized by The answer is E.
monoamine oxidase?

7
A. Acetylcholine Explanation: This refers to
B. Brain-derived neutrophic factor pharmacodynamic action of
C. Histamine MAOI.
D. Glutamate
E. Serotonin. Year: 2014

28 A 30-year-old man takes mirtazapine every night and The answer is D.


experiences significant sedation at 7.5mg/day. Which of
the following neurotransmitter receptors are MOST likely Explanation: Besides the action
to be responsible for causing this side effect? of mirtazapine on the serotonin
receptors, its action on the
A. Adrenergic receptors histamine receptors cause
B. Dopamine receptors sedation and weght gain.
C. Glutamate receptors
D. Histamine receptors Year: 2014
E. Nicotinic receptors.

29 Which of the following psychotropic The answer is E.


medications is the MOST likely to raise the
prolactin levels in female psychiatric patients? Explanation: Risperidone is
most potent D2 receptor
A. Aripiprazole blockage among all second
B. Clozapine generation antipsychotic
C. Lithium drugs.
D. Olanzapine
E. Risperidone. Year: 2014

30 Which of the following statements regarding The answer is D.


quetiapine is LEAST CORRECT?
Explanation: Unlike the other
a. Quetiapine has anxiolytic effects. second generation
b. Quetiapine has antidepressant effects antipsychotic drug, the
c. Quetiapine has antipsychotic effects. incidence of extrapyramidal
d. Quetiapine has extrapyramidal effects effects with quetiapine is the
e. Quetiapine has sedative effects. same as with placebo.

Year: 2014

31 Which of the following is NOT a well-established side The answer is A.


effect of lithium?
Explanation: Lithium is excreted
A. Hepatotoxicity
B. Hypothyroidism by kidney. It has minimal effects
C. Nephrotoxicity on the liver.
D. Tremor
E. Weight Gain Year: 2014

32 A 30-year-old man suffers from bipolar disorder. He The answer is B.


requests to be treated by carbamazepine because it
works on his friend. Which of the following genetic tests
Explanation: The MOH and
is required by the Ministry of Health (MOH) before
prescription of carbamazepine? Health Science Authority of
Singapore recommends doctor
A. Dopamine transporter gene - D*1502 to check HLAB*=1502 genotype
B. Human leukocyte antigen gene - B*1502 before prescribing
C. Integrin alpha M gene – I*1502

8
D. Mannan-binding lectin gene – M*1502 carbamazepine. Those who are
E. Serotonin transporter gene – S*1502. positive for this genotype may
have high risk of developing
Steven Johnson syndrome.

Year: 2014

33 A 25-year-old woman is treated by bupropion for The answer is E.


depressive disorder. She gives you a list of symptoms
and attributes to the side effects of bupropion. Which of
Explanation: Bupropion is
the following symptoms is LEAST likely to be side effect
of bupropion? associated no weight gain and
mild degree of weight loss.
A. Anxiety
B. Nausea Year: 2014
C. Seizure
D. Transient hallucination
E. Weight gain.

34 A 30-year-old schizophrenia patient receives olanzapine The answer is C.


20mg per day. He is a chronic smoker and agrees to
cease smoking in the past one month. After he quitted
Explanation: Nicotine induces
smoking, he experiences more side effects associated
with 20mg of olanzapine which include significant the metabolism of olanzapine
sedation and weight gain. Which of the following is via cytochrome P450 1A2 and
MOST likely explanation? reduces its serum
concentration. After he quits
A. He is not complaint to olanzapine. smoking, the serum
B. He took an overdose of olanzapine.
concentration of olanzapine has
C. After he stopped smoking, the hepatic metabolism of
olanzapine reduces. increased and resulted in more
D. Nicotine reduces the risk of metabolic syndrome. side effects.
E. This is due to the effect of nicotine patch.
Year: 2014.

35 Which of the following is the mechanism of action of The answer is D.


memantine, a medication used to slow cognitive decline in
Alzheimer’s dementia? Explanation: Memantine works
on the NMDA receptors and
A. Cholinesterase inhibitor reduces the neurotoxicity
B. Dopamine receptor blocker caused by glutamate.
C. Gamma-aminobutyric acid receptor agonist
D. N-methyl-D-aspartate receptor antagonist Year: 2014.
E. Serotonergic reuptake inhibitor.

Psychotherapy
36 A 30-year-old woman with panic disorder does not The answer is B.
respond to an initial treatment with a selective serotonin
reuptake inhibitor (SSRI). Which of the following
Explanation: The effect of CBT is
treatment is considered the best approach?
as efficacious as SSRI.
A. Benzodiazepine
B. Cognitive behaviour therapy
C. Hypnotherapy Year: 2014.
D. Olanzapine
E. Psychodynamic psychotherapy.

9
37 You are a medical officer working in the army. A 22-year- The answer is C
old national serviceman comes to see you because he is
very angry with his male supervisor and wants to see a
Explanation: Psychodynamic
psychologist. He also informs you that he is very angry
with his own father. He passed you an emotional 500- psychotherapy is most
word open letter posted on his Facebook account. In his appropriate in this case because
letter, the patient mentions that his father has been this patient needs to analyse the
abusive towards him. He finds his male supervisor is as pattern of history repeating
abusive as his father and the pattern keeps repeating itself. He may learn not to
itself. Which of the following psychotherapy is MOST
project his father onto the male
appropriate in this case?
supervisor.
A. Cognitive behaviour therapy
B. Problem solving therapy Year: 2014.
C. Psychodynamic psychotherapy
D. Reminiscence therapy
E. Validation therapy.
38 A 20-year-old woman suffers from borderline The answer is E.
personality disorder. She works as a clerk. During
psychotherapy session, she discloses that she really Explanation: The patient
hates the psychologist and has thoughts of killing the exhibits intense negative
transference towards the
psychologist. Her conviction to kill the psychologist is
psychologist. She does not have
1 out of 10 (1 =very unlikely, 10 = very likely).
the real intention to kill the
Furthermore, she does not have a plan or not sure psychologist but she mentions
when to kill the psychologist. She has no forensic this due to intense negative
history and no history of violence. She admits the transference.
idea of killing the psychologist is her own thought.
She has no command hallucination. Which of the Year: 2014.
following BEST describes this phenomenon?

A. Chronic felling of emptiness


B. Displacement
C. Projective identification
D. Splitting
E. Transference

39 A 30-year-old man suffers from disseminated The answer is A.


gonococcal infection. The medical team asks him about
unprotected sexual activities. He claims he has one
Explanation: He denies the
stable sexual partner and practises safe sex all the time.
The medical team thinks that his condition could due to threatening fact that he has
an underlying autoimmune cause because he has no been practising unprotected sex
risky behaviour. Several days later, he finally admits that with commercial sex workers
he has sexual intercourse with commercial sex worker and result in gonorrhoea
once per month. Sometimes, he does not use condom to infection.
protective himself. What is the initial defence
mechanism used by this patient?

A. Denial
B. Displacement Year: 2014
C. Projection
D. Projective identification
E. Undoing

Ethics and laws


40 You are a general practitioner. A 30-year-old female The answer is A.

10
secretary complains of low mood, poor appetite, poor
sleep and suicidal thought. She is very depressed over Explanation: Most depressed
the death of her son who passed away 26 hours after
patients have the capacity to
birth. She has history of depression without any
psychotic feature. She underwent a traumatic delivery give consent. Based on the
because an obstetrician in private practice performed history, there is no evidence to
amniocentesis, induction of labour and administered suggest that she lacks the
syntocinon 3 weeks before the expected date of capacity to make decision (e.g.
delivery. The patient later found out that the obstetrician no psychotic features, working
induced labour because the doctor planned to go for a
as a secretary). The obstetrician
holiday. Patient did not fully understand about the
procedure and did not agree with induction of labour did not respect patient’s
which resulted in premature birth of her son. The autonomy before carrying out
obstetrician claims that she does not have capacity to aminocentesis, induction of
make decision due to history of depression. She felt that labour and administering
the doctor performed the procedure against her will and syntocinon.
the death of her son was avoidable. Which ethical
principle was violated by the obstetrician? Year: 2014.
A. Autonomy
B. Beneficence
C. Confidentiality
D. Involuntary treatment
E. Justice.

41 You are a resident posted to the Accident and The answer is C.


Emergency Department of Institute of Mental
Health (IMH). A 55-year-old man attended a party in Explanation: Automatism is a
a buffet restaurant. He seldom drinks alcohol. In legal term and the person
the party, he drank a few glasses of hard liquor commits a crime without his or
and could not find his way to the table. When a her own personal control or
female waitress tried to help him, he was irritable awareness due to an underlying
and physically assaulted her. The waitress
medical condition or substances.
suffered from minor injury but the restaurant
Automatism can be caused by
manager insisted to report to the police.
Subsequently, he was arrested and stayed in the alcohol intoxication, sleep
police station for one night. On the next day, he walking, epilepsy and
was sent to IMH for assessment. When you hypoglycaemia.
interview him, he says, “Oh gosh, I can’t recall the
event which occurred last night.” He does not have Year: 2014.
past psychiatric illness. Which of the following
terms BEST describes his behaviour leading to the
arrest?

A. Actus reus
B. Jealousy
C. Automatism
D. Diminished responsibility
E. Mens rea.

Liaison Psychiatry
42 A 20-year-old man was given a high dose of The answer is C.
intravenous haloperidol. He develops high fever
and you suspect that he may develop neuroleptic Explanation: Patients
malignant syndrome (NMS). All of the following suffering from NMS usually
clinical features suggest NMS EXCEPT: present with elevated CK,
ALT, AST, LDH in the serum

11
A. Autonomic instability and high levels of myoglobin
B. Diaphoresis and protein in the urine
C. Decreased catecholamines in urine
D. Myoglobinuria Year: 2014.
E. Rigidity.

43 An obstetrician refers a 30-year-old woman The answer is A.


who presents with psychosis after delivery to
you. She wants to know whether this woman Explanation: The onset is usually
suffers from postpartum psychosis. You want between 1 to 6 weeks because
to find out from the obstetrician the onset of the progresterone levels fall but
psychotic symptoms after delivery. Which of dopamine levels rise in the days
the following days of onset is MOST congruent after child birth.
with the diagnosis of postpartum psychosis?
Year: 2014.
A. 1- 6 weeks after delivery
B. 7-12 weeks after delivery
C. 13 -18 weeks after delivery
D. 19 - 24 weeks after delivery
E. 25 – 31 weeks after delivery.

44 A 40-year-old man went to South Korea for skiing The answer is D.


and suffered from head injury. All of the following
symptoms are commonly associated with sequelae of Explanation: Obsession is not a
head injury EXCEPT: common symptom of post-
concussion syndrome.
A. Light and noise sensitivity
B. Headache
C. Insomnia Year: 2014.
D. Obsession
E. Word finding difficulty.
45 Which of the following is LEAST likely to be an area of The answer is C.
concern in a 30-year-old woman suffering from body
dysmorphic disorder?
Explanation: Genitalia is least
A. Breast likely to be an area of concern
B. Nose among the other options. The
C. Genitalia common areas are listed as
D. Hair follows: skin (73%), hair (56%),
E. Skin. nose (37%), weight (22%),
stomach (22%),
breasts/chest/nipples (21%)
eyes (20%) thighs (20%) teeth
(20%).

Year: 2014.
46 You are a medical resident. A 40-year-old man is The answer is A.
admitted to the medical ward due to altered mental state
and requires further medical investigations. Past records Explanation: The patient suffers
show a history of substance abuse. He sleeps poorly and from delirium tremens, resulted
paces around the unit, restless and grumpy on the next from alcohol withdrawal.
day. He appears to be confused and exhibits tachycardia

12
at 106 beats per minutes. His palms and forehead are Year: 2014.
sweaty and his tongue is showing a course tremor. When
asked, the patient says he feels anxious. Which of the
following is the CORRECT diagnosis?

A. Alcohol withdrawal
B. Amphetamine withdrawal
C. Cannabis withdrawal
D. Nicotine withdrawal
E. Opioid withdrawal.

Question 51 – 53

Options:
A. Acting out
B. Splitting
C. Projection
D. Rationalization

13
E. Regression

Lead-in: A 23-year-old woman with a psychiatric diagnosis of borderline personality disorder is admitted
to the ward for deliberate self-harm. She has a past history of physical abuse by her parents when she was
young.

Select the most appropriate defense mechanism for each of the following situations.

51. A staff nurse asks her to change out of her own clothing and wear hospital attire. She
becomes
upset and refuses to follow the instruction of the nurse. She purposely tries to upset the nurse
and tests
her reactions. During a psychotherapy session, she finds the nurse is as hostile and abusive as
her
mother. (C)

52. She says that she cut herself on the arm with a penknife to externalize her pain and ease
her stress
in life. She feels that self-cutting is an acceptable way of coping and that it has benefits in
making her
mind clearer and stabilizing her emotions. (D)

53. She is noted to be lying flat on the floor, clutching a teddy bear and crying loudly. When
the nurse
asks what happened, she replies, in a child-like manner, that the other patients bullied her.
(E)

Question 54 – 56

Options:

A. Confabulation

B. Déjà vu

C. Jamais vu

D. Delusional memory

E. Factitious disorder

14
Lead-in: Select the most appropriate item from above that fits the following clinical description.

54. A 45-year-old man with temporal lobe epilepsy returns home after being hospitalized for 4 days

due to breakthrough seizure. He says that his house appears to be unfamiliar as if it is a stranger’s

house, but he soon quickly recognizes it as his own. (C)

55. A 32-year-old woman with schizophrenia suddenly breaks into tears during the ward round and

shouts that alien performed an operation to insert a device in her abdomen when she was young.

Corroborative history from her mother reveals that she had an appendectomy at the Singapore

General Hospital when she was young. (D)

56. A 28-year-old man presents to the hospital with abdominal pain and bilateral upper limb numbness.

He demands to be admitted and thoroughly investigated for his symptoms. On checking of his medical

records, it is revealed that this is his 25th visit to the hospital complaining of unusual presentation of

symptoms for which extensive investigations were done each time, all results being unremarkable. He

always requests for admission and feels very happy to stay in the ward as a patient. (E)

Question 57 – 58

Options:

A. Concentration

B. Constructional praxis

C. Delayed recall

D. Language – naming

E. Luria’s test

15
Lead-in: Match the above with the following:

57. Alternating hand sequence (E)

58 . Copying a diagram of interlocking pentagon (B)

Question 59 – 60

Options:

A. Delusion of control

B. Delusion of reference

C. Delusion of thought interference

D. Nihilistic delusion

E. Persecutory delusion

Lead-in: The following scenarios refer to a patient with delusional ideas. Select the most appropriate item
from above that best match the type of delusion described below.

59. A 64-year-old woman believes that her life and the world are coming to the end after her son

passed away from cancer. She has stopped eating and bathing for the past 4 days, as she believes firmly

that her organs are decaying. (D)

60. A 21-year-old man feels that his feelings and actions are no longer his own. He firmly believes that

someone has planted a device in his brain to make him think and react strangely as if he is a robot. (A)

16
MCQ

History, mental state exam, psychopathology,


diagnosis
1. You are a resident doctor working at the Institute The answer is B.
of Mental Health (IMH). The police officers
bought a 39-year-old man charged with Explanation: Loosening of
possessing a scheduled weapon and was sent to association is defined as a
the IMH for a psychiatric evaluation. Although he speech where topics seem to be
speaks English, you cannot make out what he is disconnected and it is hard for
saying because his speech becomes diffuse and others to establish a logical link
unfocused. Which of the following best describes between topics. The speech
his psychopathology? becomes diffuse and unfocused.

A. Circumstantiality Year: 2014.


B. Loosening of association
C. Thought insertion
D. Thought withdrawal
E. Tangentiality.

2. You are a resident working at the Accident and The answer is B


Emergency Department (AED). A 35-year-old
man was sent to the AED for psychiatric Explanation: This man suffers
assessment by an ambulance. He was involved from bipolar disorder, manic
in a road traffic accident. He was a passenger in episode as evidence based by
a taxi. He told the taxi driver that he was God and grandiose delusion, irritability,
threatened the driver to believe that he was the reckless and dangerous
saviour of the world. The driver found him very behaviour.
irritable and he kept hitting the head of the driver.
The driver stopped the taxi and tried to call the Year: 2015.
police. The patient drove the taxi at high speed
and crashed the taxi into a lamp post. He was not
under influence of alcohol at the time of accident.
You are required to enter a diagnosis into the
AED computerised system. Based on the
information provided, which of the following
psychiatric diagnosis is most relevant?

A. Antisocial personality disorder


B. Bipolar disorder
C. Conversion disorder
D. Dissociative disorder
E. Schizophrenia.

Cognitive assessment
3. A 30-year-old woman has received 6 sessions of The answer is B.
electroconvulsive therapy (ECT). After the ECT,
she complains of cognitive impairment and Explanation: Young patients
attributes the following symptoms as side effects usually develop retrograde
of ECT. Which of the following is most likely amnesia of recent events but not
caused by ECT? anterograde amnesia (option A)
or life events (option C). ECT
A. Cannot follow the instructions given by her does not affect multi-tasking and

1
supervisor after she returns to work calculation.
B. Cannot recall the password of her email account
C. Cannot remember details of her ex-marriage Year: 2015.
D. Difficulty to perform two tasks at one time
E. Unable to calculate.

4. You are administering the “Serial 7” test to a patient. The answer is C.


His answers are listed as follows: 100 – 7 = 90; 90 – 7
= 77; 77 – 7 = 70; 70 – 7 = 63; 63 – 7 = 50. What is his
Explanation: 100 – 7 = 90 (0 point)
score?
90-7 = 77 (0 point)
A. 0 out of 5 77-7 = 70 (1 point)
B. 1 out of 5 70-7 =63 (1 point)
C. 2 out of 5 63-7 = 50 (0 point)
D. 3 out of 5 Total: 2 points out of 5
E. 4 out of 5.
Year: 2014.
Psychiatric epidemiology
5. Which of the following is the most common The answer is B.
psychiatric co-morbidity of panic disorder?
Explanation: Agoraphobia (40%)
A. Alcohol misuse > alcohol dependence (30%).
B. Agoraphobia The other options are less likely
C. Avoidant personality disorder to be associated with panic
D. Benzodiazapine misuse disorder.
E. Generalised anxiety disorder.
Year: 2014.

Psychiatric aetiology, diagnosis and classification


The answer is C.
6. The prevalence of borderline personality
disorder in Singapore and worldwide has Explanation: increase in divorce
increased in the past three decades. The most rate is associated with trauma,
likely explanation for this observation is: neglect and splitting in family.
These are important
A. Changes in diagnostic criteria predisposing factors for
B. Increase in competition in schools borderline personality disorder.
C. Increase in divorce rate There is no major change in
D. Increase in immigration and migration diagnostic criteria. Some of the
E. Increasing urbanization of the population. factors may occur too late to
affect personality development
(e.g. migration). As Singapore is
a city state, the effect of
urbanization is minimal.

Year: 2014
7. Which of the following is least likely to be a risk The answer is A.
factor for delusional disorder?
Explanation: Having a religion is
A. Having a religion not a risk factor for developing
B. Increased age delusional disorder. The risk of
C. Immigration delusional disorder increases
D. Sensory impairment with age. Hearing loss is a risk
E. Social isolation. factor. Low socioeconomic status
and severe stress are also risk

2
factors.

Year: 2014

8. The genetic polymorphisms of which of the The answer is E.


following neurotransmitter transporter genes
affect the risk of developing depressive disorder? Explanation: Individuals with
homozygous short alleles at the
A. Dopamine serotonin transporter gene are
B. γ-Aminobutyric acid (GABA) more likely to develop
C. Glutamate depression.
D. Norephinephrine
E. Serotonin. Year: 2014

9. A 40-year-old patient with schizophrenia presents The answer is B.


to the emergency department with confusion and
agitation following an overdose of haloperidol. Explanation: This patient
The patient has a temperature of 41 degree develops neuroleptic malignant
Celsius, blood pressure of 160/100 mm Hg, and syndrome due to blockage of
pulse of 120 beats per minute. Physical dopamine receptors by
examination reveals rigidity, hot, wet skin, and haloperidol.
decreased bowel sounds. This patient‟s
symptoms, are most likely caused by blocking of Year: 2014
which of the following receptors?

A. Alpha1-adrenergic
B. Dopaminergic
C. Histaminergic
D. Muscurinic cholinergic
E. Nicotinic cholinergic.
General adult psychiatry
10. A 30-year-old man suffers from schizophrenia. The answer is C.
He first exhibited first rank symptoms at the age
of 16 and the onset was insidious. He exhibits Explanation: Poor prognostic
anhedonia although his hallucinations are under factors include young onset, no
control by antipsychotic drug. His uncle suffers precipitating factors, insidious
from depressive disorder. All of the following are onset, poor premorbid
poor prognostic factors except? functioning, family history of
schizophrenia, negative
A. Anhedonia symptoms, poor support,
B. Exhibition of first rank symptoms at the age of 16 neurological symptoms and poor
C. Family history of depressive disorder compliance.
D. Male gender
E. Insidious onset. Year: 2014

11. You are the resident working in the Accident and The answer is B.
Emergency Department. A schizophrenia patient
informs you that he has homicidal thought. Which Explanation: Delusion of
of the following sign or symptom is least reference is not directly related
important in predicting homicide? to violence and carries lower risk
of homicide as compared to
A. Command hallucinations other signs and symptoms.
B. Delusion of reference
C. History of previous violence Year: 2014

3
D. Irresistible urge to attack
E. Need to defend oneself as are result of
persecutory delusion.

The answer is C.
12. All of the following are poor prognostic signs for
obsessive compulsive disorder except: Explanation:
Poor prognosis:
A. Bizarre compulsions 1. yielding to compulsions
B. Childhood onset (not resisting)
C. Episodic course 2. childhood onset
D. Comorbid major depression 3. bizarre compulsions
E. Succumbing to obsessions and compulsions. 4. need for hospitalization
5. presence of overvalued
ideas (i.e. acceptance)
6. personality disorders
(especially schizotypal)
7. co-existing delusional
beliefs
8. co-existent major
depressive disorder

Year: 2014
13. You are a general practitioner. A 35-year-old The answer is E.
hotel butler came to see you because his lawyer
wants him to be seen by a psychiatrist. He was Explanation: Voyeurism is
recently arrested for installing pinhole camera in defined as recurrent and intense
the hotel guestrooms. He secretly filmed couples sexual arousal from observing
having sex in the hotel guestrooms. He has other people being naked or
filmed more than 100 videos and enjoyed engaging sexual activity.
watching them over and over again. He claims
that he has the compulsion to collect those Year: 2014
videos and he feels relieved after the arrest.
Which of the following is the most likely
diagnosis?

A. Erotomania
B. Kleptomania
C. Obsessive compulsive disorder
D. Sadomasochism
E. Voyeurism.

14. A 30-year-old woman always believes that she is The answer is A.


socially inept and fears of negative evaluation by
other people. Her sisters describe her as timid Explanation: She suffers from
and insecure. She is single and stays with two avoidance personality disorder
elder sisters. She was admitted to the psychiatric as evidenced by pervasive
ward due to „nervous breakdown‟ after meeting tension (e.g. nervous
new colleagues in her workplace. She refuses to breakdown), unwilling to be
attend family meeting with her sisters because involved unless being liked (e.g.
she feels that her sisters do not like her although family meeting with her sister),
they appear to be very concerned about her fear of criticism, showing
condition. She is isolated and has one best friend restraint in intimate relationship
in her work place. Which of the following (e.g. being single) and inhibition

4
personality best describes this patient? of new interpersonal situations
(e.g. meeting new colleagues)
A. Avoidant personality disorder and belief of socially inept.
B. Borderline personality disorder
C. Histrionic personality disorder
D. Paranoid personality disorder Year: 2014
E. Schizoid personality disorder
15. Which of the following is not a common feature of Answer: D
serotonin syndrome?
Explanation: Serotonin syndrome
A. Acidosis actually causes fever. Metabolic
B. Diaphoresis acidosis in seen in 9% of
C. Hyperreflexia patients.
D. Hypothermia
E. Myoclonus. Year: 2014

16. You are a resident working in the accident The answer is E.


and emergency department (AED). A single
mother was sent to the AED with her 2-year- Explanation: The immediate
old daughter. She was arrested by police diagnosis is PCP intoxication
because she tried to assault her daughter as evidenced by
and stopped by passer-by. According to the unpredictability (staying in
informant, her partner refuses to marry her AED versus walking out of
due to her drug habit. She does not have AED), assaultativeness on her
past forensic history. She appears to be daughter, agitation and
agitated and restless. At one moment, she nystagmus. Antisocial
wants to stay in the AED. At another personality disorder requires
moment, she wants to walk out of the AED. further evidence as forensic
Physical examination reveals nystagmus. history is negative and cannot
Which of the following diagnosis is most explain nystagmus.
relevant?
A. Antisocial personality disorder
B. Cannabis intoxication
C. Delusion of jealousy Year: 2014
D. Paranoid schizophrenia
E. Phencyclidine intoxication
Old age psychiatry
17. You are about to start a selective serotonin The answer is A.
reuptake inhibitor (SSRI) for an 80-year-old man
who suffers from depression. Which of the Explanation: There is no
following pharmacokinetic changes has the least consistent age effect on
effect on drug therapy in the elderly? absorption of antidepressant in
elderly. For distribution, the
A. Absorption elderly have more fat stores;
B. Distribution therefore they will have lower
C. Excretion plasma concentrations of lipid
D. Metabolism soluble drugs, but will have
E. Protein binding. larger distribution volumes of
these drugs and therefore longer
half-lives. This may cause
problems for elderly taking
fluoxetine.

5
For metabolism, all psychotropic
medications metabolized in the
liver have their elimination half-
life increased two or three-fold in
the elderly.

For excretion, decreased GFR


and renal blood flow lead to
decreased clearance of lithium
and antidepressant metabolites.

For protein binding, the elderly


have lower levels of plasma
albumin due to malnutrition, and
therefore have an increased free
fraction of protein bound drugs.

Year: 2014

This is an advanced level


question.

18. A 79-year-old man complains of extreme The answer is B.


drowsiness when he goes to toilet at midnight
and he almost fell down on several occasions. He Explanation: Mirtazapine is the
wants to avoid fall at night. The following is the most sedative medication among
list of his medications. Which of the following all the options.
medications should be stopped?
Year: 2014
A. Melatonin
B. Mirtazapine
C. Risperidone
D. Rivastigmine
E. Sodium valproate.
The answer is D.
19. A 70-year-old woman suffering from bipolar
disorder and her daughter came to see you Explanation: Elderly with bipolar
today. Her daughter read about the symptoms of disorder are more likely to be
bipolar disorder and found her mother different paranoid and present with mixed
from younger adults suffering from the same clinical presentations. Elderly are
condition. Which of the following clinical features less likely to develop flight of
is more likely to be found in this 70-year-old ideas, hyperactivity and
woman? overspending.

A. Demonstrates less paranoid ideas as compared Year: 2014


to adult patients
B. Demonstrates more flight of ideas as compared
to adult patients
C. More likely to be hyperactive as compared to
adult patients
D. More likely to demonstrate mixed
(depressive/manic) clinical presentations as
compared to adult patients
E. More spending as compared to adult patients.

6
20. Which of the following is most important risk The answer is C.
factor for vascular dementia?
Explanation: Among all the
A. History of depression options, hypertension is the most
B. History of poor academic performance significant risk factor for vascular
C. History of hypertension dementia.
D. High level of physical activity
E. High level of high density lipoproteins. Year: 2014

21. You are the visiting physician of a nursing home. The answer is A.
The nursing home staffs feel very helpless when
handling aggression in dementia patients and Explanation: Antipsychotic drug
they need to seek your advice. Which of the (e.g. risperidone) has the most
following psychotropic medication have the most evidence in improving
evidence in managing aggression in patients aggression in dementia patients
suffering from dementia? among all options.

A. Antipsychotic drug Year: 2014


B. Beta-blocker
C. Benzodiazapine
D. Lithium
E. Stimulant.

Child and adolescent psychiatry


22. In children and adolescents, high stimulation- The answer is C.
seeking behaviour is associated with which of the
following conditions? Explanation: Young people with
conduct disorder have to be
A. Adolescent-onset schizophrenia involved in delinquent behaviour
B. Autism to get high stimulus.
C. Conduct disorder
D. Obsessive-compulsive disorder Year: 2014
E. School refusal.

23. Children with autism are most comfortable in The answer is D.


which of the following situations?
Explanation: Travelling in a bus
A. Attending a party where there are a lot of because it has a predictable
surprises and have a chance to meet different route (i.e. the bus stops at every
people. bus stop and there is
B. Attending drama classes and imagines oneself uncertainty).
playing different roles in the drama.
C. Looking for toys in a multi-storey department Year: 2014
store from one level to another level.
D. Travelling in a bus when it stops at every bus
stop until it reaches the bus terminal.
E. Going to school where there are multiple classes
to attend in a day and teaching content varies
from subject to subject.
24. Which of the following statements is most correct The answer is E.
regarding the impact of divorce on children?
Explanation: Options A and B

7
A. Boys aged two to four are not significantly have no evidence base. For
affected. option C, most children take 3-5
B. Children do best if they have no contact with the years to recover. Socio-
father for two years. economic status affects the
C. Most children have no significant problems after outcome.
three to six months.
D. Socio-economic status is not associated with the Year: 2014
degree of impairment.
E. To evaluate the impact, one must take into
consideration the age and the developmental
level of the child.

25. A 15-year-old obese female is brought by her The answer is E.


parents for an evaluation as part of the admission
process for a weight management program. The Explanation: The patient must
adolescent agrees with the fact that she is agree to use conservative
overweight and she is keen to lose weight. methods including changing diet
Which of the following is most indicative that she habits, seeing a dietician and
will be successful in the program? going for exercise to lose weight.

A. Her current BMI is between 26 and 30. Year: 2014


B. Mother suffered from obesity and successfully
lost weight
C. No family history of eating disorders
D. Motivation to take antidepressant
E. Willingness to change her eating habits.
Substance abuse
26. Methylene dimethylamphteamine (street name: The answer is B.
ecstasy) usually cause people with which of the
following symptom? Explanation: Ecstasy is also
a. Amotivation known as an empathogen and
b. Closeness to others makes the patient to feel close to
c. Depression others.
d. Introversion
e. Phobia. Year: 2014

27. A 30-year-old adult claims to suffer from adult The answer is D.


onset attention deficit and hyperactivity disorder
(ADHD). He has been seeing different doctors to Explanation: This patient is
obtain methylphenidate. He claims that he lost all doctor shopping and tries to
methylphenidate in the bus few days ago and his obtain methylphenidate because
wife has noticed that he looks very tired. Which he is dependent on this
of the following is most appropriate explanation medication. He lost the
for his tiredness? medication few days ago and
developed psychological
A. He develops depression and psychomotor withdrawal symptoms including
retardation depression, tiredness,
B. He exhibits secondary gain and seeking oversleeping and overeating.
attention from his wife
C. He suffers from comorbid somatisation Year: 2014
disorder in addition to ADHD
D. He suffers from psychological withdrawal
symptoms from stimulant misuse

8
E. He suffers from stimulant intoxication.
28. The most important objective of The answer is D.
psychological intervention in treating people
suffering from alcohol misuse is: Explanation: Achieving
abstinence or reduction of
A. To conduct in a group therapy format alcohol use is most important
B. To apply cognitive behaviour therapy (CBT) as psychotherapeutic objective.
the first approach
C. To combine psychological intervention and Year: 2014.
pharmacotherapy (e.g. disulfiram)
D. To focus on achieving and maintaining
abstinence or reducing alcohol use
E. Explore unresolved psychodynamic conflicts.

Psychopharmacology
29. Which of the following medications is not The answer is E.
associated with weight gain?
Explanation: Topiramate is
A. Clozapine associated with weight loss and
B. Mirtazapine not weight gain.
C. Olanzapine
D. Sodium valproate Year: 2014
E. Topiramate.

30. A 60-year-old man is started on an The answer is D.


antidepressant for treatment of major depressive
disorder. Three weeks later, he complains of Explanation: Imipramine is a
blurred vision, dry mouth, constipation, urinary tricyclic antidepressant and
retention and tachycardia. Which of the following associated with anticholinergic
medications is most likely to have caused this side effect.
presentation?

A. Agomelatine Year: 2014


B. Duloxetine
C. Fluoxetine
D. Imipramine
E. Mirtazapine

31. Which of the following medications is most likely The answer is D.


to be associated with polycystic ovarian
syndrome in female patients? Explanation: Sodium valproate is
associated with polycystic ovary
A. Carbamazapine syndrome in female patients.
B. Lamotrigine
C. Lithium Year: 2014
D. Sodium valproate
E. Topiramate.

32. Regarding an antidepressant called bupropion, The answer is C.


which of the following statements is false?
Explanation: bupoprion works on
A. It causes insomnia. noradrenaline and dopamine but
B. It has a half-life of approximately 12 hours. not serotonin. It is not
C. It is safe for patients suffering from bulimia recommended for eating disorder

9
nervosa. because of the risk of seizure.
D. It is an antidepressant used for depressed
patients with cardiovascular disease and habit of Year: 2014
smoking.
E. It does not cause weight gain.

33. Mirtazapine is associated with lesser sexual side The answer is B.


effects. Which of the following
pharmacodynamics actions explains this Explanation: Mirtazapine
phenomenon? antagonises 5HT2A receptors and
leads to anxiolytic effects,
A. Serotonin receptor 5HT1A agonism restores sleep and less sexual
B. Serotonin receptor 5HT2A antagonism side effects.
C. Serotonin receptor 5HT2C antagonism
D. Serotonin receptor 5HT3 antagonism Year: 2014
E. Histamine receptor antagonism.

34. Which of the following side effects is least likely The answer is D.
to occur in patients taking quetiapine?
Explanation: Quetiapine is
A. Antihistamine effects associated with antihistamine
B. Orthostatic hypotension effects, orthostatic hypotension,
C. Increase in liver transaminase increase in liver transaminase
D. Increase in prolactin and weight gain.
E. Weight gain.
Year: 2014
35. Which of the following is the best treatment Answer is E.
option for severe depressive episode with
psychotic features? Explanation: ECT has the most
evidence in treating psychotic
A. Amitriptyline depression as compared to other
B. Cognitive behaviour therapy treatment options.
C. Lamotrigine
D. Methylphenidate Year: 2014
E. Electroconvulsive therapy.

36. A 35-year-old man suffers from schizophrenia The answer is B.


symptoms and his GP has started him a
Explanation: Chlorpromazine is
medication to treat his symptoms. He develops associated with cholestatic
jaundices after taking the medication. Which of jaundice and is most likely to
the following medications is most likely to cause cause jaundice among all
jaundice? options.
Year: 2014
A. Clozapine
B. Chlorpromazine
C. Haloperidol
D. Risperidone
E. Sulpiride.

37. A 60-year-old man suffers from depressive The answer is E.


disorder and you have increased the dose of
antidepressant. He is seeing you today. His blood Explanation: Venlafaxine is

10
pressure is 180/90. Which of the following associated with dose-dependent
medications should be stopped? hypertension.

A. Diazepam Year: 2014


B. Quetiapine
C. Fluvoxamine
D. Hydroxyzine
E. Venlafaxine.

38. A 25-year-old woman with bipolar disorder is The answer is C


pregnant for the first time. Throughout the
pregnancy, she has been maintained on a mood Explanation: Ebstein‟s anomaly
stabilizer. At the time of birth, the baby is noted is associated with lithium.
to have Ebstein‟s anamoly. Which of the
following medications is most responsible for this Year: 2014
teratogenic effect?

A. Carbamazepine
B. Lamotrigine
C. Lithium
D. Sodium valproate
E. Topiramate
Psychotherapy and defence mechanisms
39. A 28-year-old woman comes to consult you The answer is C.
because she is very upset after being scolded by
her manager. The chief executive officer (CEO) Explanation: The manager
of her company has raised the standard for transferred his anger to his
annual performance. Her manager is very upset subordinate which carries less
this arrangement and scolds her for no reason. emotional risk as compared to the
Which of the following is the defence mechanism CEO. This is known as displacement.
exhibits by the manager?

A. Altruism Year: 2014


B. Denial
C. Displacement
D. Repression
E. Reaction formation.

40. You are a doctor working in polyclinic. A 25-year- The answer is E.


old man bought his 55-year-old mother to see
you. He is concerned that his mother suffers from Explanation: Family therapy such
depression but she refuses to take as structural family therapy
antidepressant. You realise that the mother and focusing on boundary is helpful
son are enmeshed as the son is unmarried and in this situation.
not able to work. He claims that he needs to look
after his mother although she has good past Year: 2014
health. The mother and son hold very negative
view against the father who is the sole
breadwinner of family. There is a psychologist
working in your polyclinic. Which of the following
forms of psychological treatment is most
appropriate?

A. Couple therapy
B. Dialectical behaviour therapy

11
C. Problem solving psychotherapy
D. Supportive psychotherapy
E. Family therapy.

41. A national serviceman is seeing a male The answer is E.


psychologist because he suffers from adjustment
disorder and he finds his army supervisor Explanation: Interpretation is a
uncaring. Furthermore, he is very angry with his statement made by the
own father. He recalls an incident in camp about psychologist to enhance patient‟s
getting angry with his male supervisor. The understanding of his or her own
psychologist asks a factual question about the thoughts or behaviours.
circumstance and rage in the patient. Then the Affirmation refers to confirmation
patient complains that the psychologist is of validity of previous judgement
uncaring and trying to find fault in him. The or behaviour in a patient.
psychologist says, „I wonder if what you‟ve Empathetic validation refers to
feeling right now is just like the feeling you have the situation when the therapist
in camp, when you attributed the same uncaring tries to put himself into the
attitude toward your father and supervisor in patients‟ shoes and tries to
camp. These repetitive patterns of understand patient‟s inner state.
misunderstanding seem to make you very upset
in different situations and affect your life.‟ Which Year: 2014
of the following most accurately describes the
psychotherapeutic technique?

A. Affirmation
B. Clarification
C. Empathic validation
D. Observation
E. Interpretation.

42. A medical student used to be very keen to The answer is E.


become an orthopaedic surgeon. He has
assisted an orthopaedic surgeon to conduct Explanation: His external attitude
research since the second year of medical and behaviours (not caring about
school. During the residency application, he tells orthopaedic surgery) is the
other classmates that he does not bother whether opposite of the internal intention
he is accepted by the orthopaedic residency. He (wanted to join orthopaedic
also encourages other classmates not to apply surgery).
for orthopaedic surgery as this specialty is not a
good career choice. On the other hand, he is Year: 2014
secretly asking his orthopaedic mentor to prepare
him for the orthopaedic residency intake
interview. What is his defence mechanism?

A. Altruism
B. Denial
C. Displacement
D. Repression
E. Reaction formation.
43. Which of the following is the most effective The answer is C.
approach for a 30-year-old woman with
obsession about contamination and washes her Explanation: Option C is known
hands every time she touches something which as exposure and response
she considers dirty? prevention (ERP) and is the most

12
effective approach among all
A. Having the patient place her hands in a container options. Option D is not good
of worms to create extreme phobia and ability to enough because the
overcome obsession psychologist should guide the
B. Having the patient snap her wrist with a rubber patient to further challenge the
band when she thinks about contamination and obsessions rather than just
about to wash her hands highlighting the obsessions.
C. Having the patient touch a dirty object, then not Option B is thought stopped and
allowing her to wash her hands for several hours is not as effective as ERP.
D. Highlighting to the patient that her obsession is a
cognitive error. Year: 2014
E. Providing the patient with message cards to
remind herself that the hand washing is
unnecessary.
44. A 30-year-old army officer presents for treatment The answer is C.
of post-traumatic stress disorder after surviving
an accident which his tank fell into the river from Explanation: The psychologist is
a bridge. He was driving a tank on a bridge when providing in-vivo exposure by
it suddenly collapsed. Several army officers were imaging the accident scene to
seriously injured in this accident. As part of the overcome fear and anxiety
treatment, the psychologist asks the patient to associated with flashback.
imagine that he is safely driving his tank over a
bridge. Which of the following best describes this Year: 2014
therapeutic intervention?

A. Aversion therapy
B. Cognitive therapy
C. Exposure therapy
D. Interpersonal therapy
E. Supportive therapy.

Ethics, management dilemma and laws


45. You are a paediatric resident. A 14-year-old girl The answer is E.
has stayed in the paediatric ward for 1 week after
a suicide attempt by drug overdose. She is due Explanation: As this female
for discharge tomorrow. At 6 pm, she suddenly adolescent suffers from conduct
claims that her elder sister had physically abused disorder, she has the tendency
her prior to admission. Her medical record to tell lies to avoid discharge. As
reveals that she suffers from conduct disorder. the patient is currently in the
She also made similar reports of physical abuse ward and she is safe, there is no
in the past. The police officer and social worker need to inform the police
found her claims inconclusive and did not immediately without further
investigate further. She is ambivalent to make a clarification from the treatment
report about the alleged abuse and she is not team. Furthermore, the patient is
keen for discharge. You try to contact the ambivalent to make a report. The
consultant in charge of this case but the best approach is to discuss with
consultant is not contactable. Which of the the treatment team and they can
following is most appropriate action? clarify with her parents whether
the elder sister did physically
A. Banning her parents from visiting her. abuse her.
B. Call 995 to inform the police immediately
C. Inform the on-call team and request the on-call Year: 2014
doctors to inform the police before midnight
D. Refer the patient to gynaecologist for further

13
assessment
E. Discuss with the consultant-in-charge and the
social worker on the next day.

46. A 35-year-old man was hospitalised for treatment The answer is D.


of severe depressive episode in September. The
inpatient team prescribed two antidepressants for Explanation: The polyclinic
the patient and then referred the patient to doctors forgot to prescribe
polyclinic. The patient came to the polyclinic for antidepressants and resulted in
review in October and November respectively. relapse and suicide. Their act is
The patient had a relapse of depression and considered to be negligence /
committed suicide in December. The unintentional torts. Option A is
computerised record shows that the polyclinic intentional torts and not relevant
doctors had forgotten to prescribe for this case. Diminished
antidepressants since the first visit. Which of the responsibility only applies in
following best describes the above situation murder cases. Tarasoff‟s rule
based on the Common Law? involves duty to warn when a
patient expresses his/her wish to
A. Battery harm another person.
B. Diminished responsibility
C. Intentional torts Year: 2014
D. Negligence
E. Tarasoff‟s rule

Liaison psychiatry and neuropsychiatry


47. You are a resident in obstetrics. Which of the The answer is B.
following is the safest medication to be given to a
pregnant woman who develops psychosis during Explanation: Haloperidol is the
pregnancy? oldest medication and is the
safest among all options.
A. Aripiprazole
B. Haloperidol Year: 2014
C. Olanzapine
D. Quetiapine
E. Risperidone
48. A 25-year-old man expresses concerns that he The answer is B.
got infected with HIV after visiting commercial sex
workers last week. According to him, he only Explanation: It is common in
received hand-genital stimulation and there was clinical practice for patients
no vaginal intercourse. He also mentions that the asking for urgent HIV test after
commercial sex worker kissed his body. He has they engaged in activities which
sought his girl-friend‟s forgiveness. Today, he did not transmit HIV. The
requests HIV test urgently. He consulted you one patients request for such test as
year ago for similar concern after he kissed a result of guilt. It is unlikely to be
another lady without informing his girl-friend. caused by obsession as the
What is the most likely explanation of his patient requests for the HIV test
behaviour? after sexual activity but not a
thought. The patient does not
A. Underlying depression have any somatic concern.
B. Underlying guilt
C. Underlying obsession Year: 2014
D. Underlying psychosis
E. Underlying somatic concern
49. You are orthopaedic resident. A 25-year-old The answer is D.

14
construction worker had a fall, fractured his spine
and injured the brain. He develops pain in Explanation: Duloxetine is an
multiple sites. He has one episode of epilepsy antidepressant indicated for both
after the injury. He is depressed but does not depression and pain. Bupropion,
have suicidal thought. Electrocardiogram is amitriptyline and chlorpromazine
normal. Which of the following medication is most increases the risk of seizure.
helpful in this situation? There is no indication to
prescribe moclobemide in this
A. Amitriptyline case.
B. Bupropion
C. Chlorpromazine Year: 2014
D. Duloxetine
D. Moclobemide

50. You are a medical resident and rotated to The answer is B.


rheumatology. A 24-year-old woman suffers
from systemic lupus erythematosus (SLE). Explanation: Binge eating is not a
She suffers from lupus nephritis and requires recognised neuropsychiaric
dialysis three times a week. She was very symptom of SLE.
concerned about her illness and felt
hopeless. She took an overdose of her lupus Year: 2014
medications and admitted to the medical
ward. You need to take a history from her.
Which of the following is not part of the
neuropsychiatric symptoms of SLE
experienced by this patient?
A. Anxiety
B. Binge eating
C. Cognitive impairment
D. Epilepsy
E. Headache.

EMI

15
Question 51 – 53

Options:
A. Abstinent
B. Action
C. Contemplation
D. Pre-contemplation
E. Preparation

Lead-in: Choose the most appropriate stage based on Prochaska and DiClemente’s stages of change
model.

51. A 35-year-old man with cannabis misuse ignores the advice of family and friends, and
continues using the drug and causing amotivational syndrome.

Answer: D

52. A 40-year-old woman has been attending Narcotics Anonymous meetings and cessation to
use pain killer has been integrated into her life.

Answer: B

53. A 40-year-old man recognises the problem of alcohol misuse and is considering the pros and
cons of treatment.

Answer: C

Question 54 – 57

Options:

16
A. Deficiency of thiamine

B. Deficiency of vitamin B12

C. Presence of amyloid plaques and neurofibrillary tangles

D. Presence of eosinophilic intracytoplamic neuronal inclusions with abnormal phorphorylation

E. Raised growth hormone level with dysregulation of hypothalamic-pituitary-adrenal axis

Lead-in: Select the most appropriate item from the above that is most likely seen in the following
clinical scenario.

54. An 80-year-old man presents with increased forgetfulness and memory problems over the last 2
years. He often misplaces his things and unable to find his own way home when he goes out of the
house.

Answer: C

55. A 50-year-old man with long standing alcohol dependence presents with acute onset of confusion

and unsteady gait. On physical examination, he has opthalmoplegia and nystagmus.

Answer: A

56: A 22-year-old female presents with syncope and dizziness. She believes that she is overweight

and has been restricting her diet. Her BMI is17 and she has not had her menstruation for the past 3

months.

Answer: E

Question 58 – 59

Options:

17
A. Alcohol intoxication

B. Cannabis intoxication

C. Frontal lobe dementia

D. Manic episode

E. Pseudobulbar palsy

Lead-in: Select the most appropriate diagnosis from the above for the following clinical scenario.

57. A 54-year-old man is admitted following a stroke. He is noted to be crying and laughing in the
ward at time. His speech has become nasal “like a duck” and his tongue is spastic and stiff.
His score for Mini Mental State Examination is 28/30.

Answer: E

58. A 45-year-old man presents to the Emergency Department in a disinhibited manner. He tries to
hit the nurses when they attempt to draw blood from him and he shouts loudly on how badly
treated he is by his wife. However he soon becomes apologetic and tearful. He is unsteady in
his gait and speech is slurred. He subsequently falls asleep soundly in the cubicle while
waiting for the doctor.

Answer: A

Question 59 - 60

Options:

A. Acetaldehyde dehydrogenase inhibitor

B. Acetylcholine dehydrogenase inhibitor

18
C. Dopamine D2 receptor antagonist

D. NMDA receptor antagonist

E. Partial dopamine D2 agonist

Lead-in Select the most likely mechanism of drug action from above.

59. Disulfiram

Answer: A

60. Haloperidol

Answer: C

19
MCQ exam 11 (2015 Rotation 1)

History, mental state exam and psychopathology Answers


1. A 35-year-old woman complains of hearing The answer is A.
voices. She mentions that several male and
female voices try to repeat her thoughts outside Explanation: Echo de la pensée
her head. This psychopathology is known as: refer to thoughts spoken aloud. It is a
type of complex auditory hallucination
A. écho de la pensée experienced by schizophrenia patients.
B. folie à deux Folie à deux (shared delusion by 2
C. folie imposée people) and folie imposée (the
D. jamais vu dominant person who imposes delusion
E. la belle indifference. onto the others) are delusional disorder.
Jamais vu occurs when a person
momentarily does not recognise a word
or, less commonly, a person or place,
that she or he knows. La belle
indifference occurs in dissociative
amnesia when the person does not
show any reaction to drastic memory
loss.

Author: Roger Ho
Year: 2015
2. Which of the following is an example of Answer is D
obsession?
Explanation: Option A refers to
A. A man has a false belief which is non-bizarre in delusion (e.g. morbid jealousy which
content and the patient tries to convince others to can be non-bizarre in content). Option
believe it. B is not a psychopathology. Option C
B. A woman cannot get her ex-boyfriend out of her refers to binge eating. Option E refers
mind and feels compelled to know his to distortion of body image.
whereabouts.
C. A woman compulsively eats everything in front of Author: Roger Ho
her and feels guilty afterwards. Year: 2015
D. A man recognises the irrationality to open an
envelope multiple times to ensure no vulgarity was
written on the letter.
E. A woman has a distorted belief of being too fat and
counting calorie intake not to exceed 1000 per day.

3. A 20-year-old woman suffering from borderline The answer is D.


personality disorder mentioned that she saw the
curtain being transformed into flame when the Explanation: An illusion is an
wind blew through the window last night. This involuntary false perception in which a
psychopathology is known as: transformation of real object (i.e. the
curtain) takes place and being
A. Eidetic imagery transformed into flame.
B. Fantasy
C. Hallucination
D. Illusion Author: Roger Ho
E. Pseudohallucination Year: 2015
4. A65-year-old w oman is brought in by The answer is E.
her family members because she wanted
to end her lif e. She accuses her husband Explanation: Othello syndrome is
of being unfaithful and abusive towards another name of morbid jealous. The
her and meeting Chinese women word, “Othello” originated from
secretly. Her husband is a 75-year-old Shakespeare‟s play. Othello murdered
man who suffered a serious stroke his wife due to jealousy. In this
which left him with left hemiparesis and delusion disorder, the patient firmly
aphasia. He is currently wheelchair - believes that her spouse is unfaithful. It
bound and cannot leave the HDB flat on seems to be impossible in this case
his own. Her son claims that his mother because the patient is wheelchair
is abusive towards his f ather and he bound, with difficulty in speech and
suspects something is wrong with her . motor weakness. It often associates
Which of the following is the most likely with confrontation and violence.
diagnosis?

A. Capgras syndrome Author: Roger Ho


B. Charles de Bonnet syndrome Year: 2015
C. Fregoli syndrome
D. Ganser syndrome
E. Othello syndrome

5. Which of the following psychopathology is The answer is E.


most likely to be associated with
schizophrenia? Explanation: Thought board casting
is part of the first rank symptoms
A. Acute onset of psychotic symptoms and most likely to be associated
B. Auditory hallucination with schizophrenia . Auditory
C. Apathy hallucination may occur in severe
D. Persistent deterioration of personality depression, bipolar disorder,
E. Thought broadcasting. schizophrenia and borderline
personality disorder. Apathy occurs
in dementia and schizophrenia.
Acute onset of psychotic symptoms
occur in acute and transient
psychosis or substance abuse.
Persistent deterioration of
personality occur in dementia and
organic brain disorder.

Author: Roger Ho
Year: 2015.
Cognitive assessment
6. A 70-year-old man is receiving inpatient The answer is E.
treatment at the National University Hospital.
Which of the following is the BEST way to Explanation: The best way to assess
assess “the orientation to place” during the Mini location should be least controversial
Mental State Examination? among healthy subjects. Answer A will
create a lot of inconsistency because
A. Name of Hospital and Name of Street, Name of every person has a different home
Home Address, The distance between home and address. Direction is another
hospital, Name of Country controversial answer. Some healthy
B. Name of Hospital, Name of City, Name of subjects would say NUH is at the south
Country, Name of nearby country, Name of of Singapore. Some would argue it is
Continent. southwest and makes it hard to decide
C. Name of Hospital, Road, District, Direction on the score. Option B asks about
(North/South/East/West), Name of country location outside of Singapore and this
D. Ward Number, Floor, Name of Hospital, Direction is less relevant to the orientation to
(North/South/East/West), Name of country current place. Option E is the best way
E. Ward Number, Floor, Name of Hospital, Nearest to assess the orientation to place during
MRT station and Name of country. the MMSE and the subject will get 5
points if all answers are correct.

Author: Roger Ho
Year: 2015
Psychiatric epidemiology
7. A 30-year-old woman suffering from The answer is B.
schizophrenia is married to a 35-year-old man
without history of psychiatric illness. Her Explanation: If one parent suffers from
husband is concerned that their child will schizophrenia and the other parent does
develop schizophrenia. If the prevalence of not have mental illness, 13% of the
schizophrenia in the general population is 1%, children will develop schizophrenia. As
what is the risk of their child developing 1% of the general population suffering
schizophrenia as compared to the general from schizophrenia, the risk is 13
population? times.

A. 3 times increase in risk Author: Roger Ho


B. 13 times increase in risk Year: 2015
C. 23 times increase in risk
D. 33 times increase in risk
E. 43 times increase in risk
8. The male to female gender ratio for conduct Answer: B
disorder is:
Explanation: Based on previous
A. 2: 1 epidemiological research data, the male
B. 3: 1 to female gender ratio for conduct
C. 4: 1 disorder is approximately 3:1.
D. 5: 1
E. 6:1 Author: Roger Ho
Year: 2015
Psychiatric aetiology, diagnosis and classification
9. A 30-year-old woman developed panic attacks The answer is C.
after arguments with a fearful supervisor. The
arguments had occurred on a weekly basis for 6 Explanation: This example is based on
months. Despite the resignation of her classical conditioning. In this example,
supervisor 2 months ago, her panic attacks the unconditional stimulus (UCS) is her
persisted whenever she steps into the office. fearful supervisor. The response is
Which of the following learning theories best panic attack. The conditional stimulus
explains her condition? (CS) is her office and conditional
response is panic attack. After pairing
A. Aversion learning the UCS and CS for 6 months, she
B. Cognitive learning develops same response after the UCS
C. Classical conditioning disappears.
D. Operant conditioning
E. Observational learning. Author: Roger Ho
Year: 2015
This is an advanced level question.
10. Which of the neurotransmitters is most The answer is C.
implicated in the aetiology of narcolepsy?
Explanation: In narcolepsy, there is a
A. Dopamine lack of hypocretin. Hypocretin is a
B. Noradrenaline hypothalamic neuropeptide transmitter
C. Hypocretin which regulates the sleep-wake cycle.
D. Neuropeptide Y
E. Serotonin. Author: Roger Ho
Year: 2015

This is an advanced level question.


11. Which of the following psychiatric disorders The answer is C
confers the highest genetic relative risk for first
degree relatives? Explanation: Bipolar disorder has the
highest heritability (79-83%) among all
A. Alcoholism disorders. The low heritability of
B. Anorexia nervosa major depressive disorder (30%),
C. Bipolar disorder
panic disorder (30%), alcohol
D. Major depressive disorder
E. Panic disorder.
dependence and anorexia nervosa
suggest that environmental factors
are contributing more variations in
the phenotype of these disorders.

Author: Roger Ho
Year: 2015

12. Which of the following micro-organisms is The answer is D.


most likely to be associated with childhood
obsessive compulsive disorder? Explanation: Childhood OCD is
caused by Paediatric autoimmune
A. Cytomegalovirus neuropsychiatric disorders
B. Epstein-Barr virus associated with Streptococcal
C. Staphlococcus aureus infections.
D. Streptoococcus pyogenes
E. Rubella Author: Roger Ho
Year: 2015

General adult psychiatry


13. A 23-year-old American woman, Carol The answer is A.
claims that she has multiple personalities.
Currently, she is an exchange student at a Explanation: Dissociative identity
local university and the university disorder is characterised by the
counsellor is concerned about the presence of 2 or more distinct
discontinuity of her own self and absences personality states. There is a
from school. She claims that each discontinuity self and changes in
personality inside herself has his or her own affect, behaviour and functioning.
name, age and ethnicity. The three There are repetitive memory gaps
personalities are a 4-year-old girl British when one personality is dominant.
girl, Nancy; 19-year-old Japanese female The symptom is not part of a
singer, Yuko and 27-year-old American cultural or social norm.
man, Tom. Nancy is not aware of the Misidentification syndrome refers to
existence of Yuko and Tom and vice versa. Capgras or Fregoli syndrome.
She cannot recall what Nancy, Yuko and
Tom have done to her body. Which of the Author: Roger Ho
following is the most appropriate diagnosis? Year: 2015

A. Dissociative identity disorder


B. Ganser‟s syndrome
C. Misidentification syndrome
D. Schizoaffective disorder
E. Schizophrenia

14. You are a general practitioner. A 30-year- The answer is D.


old English teacher suffers from low mood,
poor motivation and lethargy. She was Explanation: This patient suffers
unable to complete her practicum. She was from hypomania. Her hypomanic
treated by her psychiatrist with two episode is precipitated by
antidepressants (bupoprion and fluoxetine) antidepressants. Her hypomanic
for 4 weeks. She comes to see you and episode is characterised in elated
requests a letter to certify that she is fit to mood, lack of sleep and
teach. She has bought you a box of overfamiliarity. Her symptoms do
chocolates and seems to be elated. She also not suggest alcohol withdrawal,
mentions that she likes to hug other teachers histrionic personality disorder or
in the school and feels excited. She does not psychosis. Erotomania is a delusion
sleep at night and she is writing a book on disorder which a female patient
poetry. Which of the following diagnosis is believes that a man of higher social
most likely? status is in love with her but there is
no evidence to suggest that.
A. Alcohol withdrawal
B. Erotomania Author: Roger Ho
C. Histrionic personality disorder Year: 2015
D. Hypomania
E. Psychosis

15. Which of the following features is MOST The answer is B.


characteristic of Capgras’ syndrome?
Explanation: Capgras syndrome is a
A. A delusion is transferred from a man with delusional misidentification
psychotic disorder to his wife and they share syndrome of a familiar person and
the same delusion. the person believes that the familiar
B. Delusional belief that familiar people have person is replaced by an imposter or
been replaced by their imposters. a double. Option A refers to Folie a
C. Delusional belief that strangers have taken on deux. Option C refers to Fregoli
the psychological identity of a familiar person. syndrome. Option D refers to
D. Delusional belief that a woman firmly believes DeClerambault‟s syndrome. Option
that a man of higher status falls in love with E refers to Cotard‟s syndrome.
her.
E. Nihilistic and hypochondriacal delusions found
in psychotic depression.

16. A 50-year-old Singaporean woman who The answer is A.


stayed in Canada for 30 years has returned
to Singapore. She divorced her Canadian Explanation: She suffers from
husband and decided move back to seasonal affective disorder. It
Singapore because she preferred the local occurred from November to
culture. In Canada, she would usually feel February every year in North
anxious in October before gradually America. The shortening of the
becoming depressed and hide in her house length of the day affects the
from November to February the following circadian rhythm and result in
year. She preferred to stay in the house features of atypical depression.
because the heater kept her warm. During
this period, she felt lethargic with increased Author: Roger Ho
appetite. From April onwards, these Year: 2015
symptoms disappeared. She did not see a
psychologist there due to long waiting list.
Which of the following explanations is most
correct?

A. Her condition was best explained by changes


in the length of the day.
B. Her condition was best explained by difference
in temperature inside and outside of her house.
C. Her condition was best explained by cultural
differences between Canada and Singapore.
D. Her condition was best explained by lack of
access to psychotherapy.
E. Her condition was best explained by marital
problems.

17. A 40-year-old man came to see you because The answer is D.


he has difficulty with his wife. They have
recently moved into a new HDB flat and Explanation: His wife has an
need to renovate the new flat. His wife takes obsessive compulsive (anankastic
a long time to make a decision due to her personality). It is characterised by
perfectionism. She carefully holds herself carefulness, perfectionism, rigidity,
back until she is sure that her decision is undue preoccupation with
perfect. His wife is upset with the contractor productivity and sacrificing
because she finds minor flaws in the timeliness to ensure correctness.
renovation. She is inflexible and never
accommodates his needs. Which of the Author: Roger Ho
following best describes the personality of Year: 2015
his wife?

A. Borderline personality
B. Histrionic personality
C. Narcissistic personality
D. Obsessive compulsive personality
E. Paranoid personality
18. A 30-year-old woman suffers from The answer is A.
depression due to marital problems.
Further inquiry reveals that she encounters Explanation: The stop-start
psychosexual problems in her marriage. technique is used in the treatment of
Her husband has premature ejaculation but premature ejaculation. The woman
refuses to see a doctor. Which of the uses her hands to stimulate the man
following is most appropriate advice? to erection. Then she stops the
stimulation. After he loses his
A. Advise her to apply the stop –start technique. erection, she can resume stimulation
B. Advise her to apply the suction device on her and then stop again. This will
husband. prolong the time before ejaculation.
C. Adviser her to change the method of
contraception. Author: Roger Ho
D. Advise her to consider psychodynamic Year: 2015
psychotherapy.
E. Advise her to persuade her husband to consider
sildenafil.

19. A 40-year-old man became suicidal because The answer is D.


of sleep difficulties: unrefreshed sleep at
night and drowsiness in the day. His wife Explanation: This man is most
complains about his loud snoring at night likely suffering from obstructive
and difficulty awakening in the morning. sleep apnoea (OSA). OSA affects
On inspection, he is an obese man. He is 4% of the male population. He has
most likely to suffer from which of the unrefreshed sleep because he wakes
following conditions? up at least 5 times per hour as a
result of difficulty in breathing
A. Circadian rhythm sleep disorders during his sleep. Obesity and middle
B. Melancholia age are risk factors for OSA.
C. Narcolepsy
D. Obstructive sleep apnoea Author: Roger Ho
E. Kleine-Levin syndrome. Year: 2015

20. You are seeing a 30-year-old woman who is The answer is B


12-week pregnant. She is highly suicidal and
wants to end her life with her foetus. Which Explanation: As the patient is
of the following is the safest and most pregnant, option A (CBT), option B
effective treatment? (ECT) and option E (rTMS) are the
safest options. As he is highly
A. Cognitive behaviour therapy suicidal, only ECT is the most
B. Electroconvulsive therapy effective treatment.
C. Paroxetine
D. Lithium Author: Roger Ho
E. Repetitive transcranial magnetic stimulation Year: 2015.

21. A father came on his son’s behalf because The answer is C.


his son refuses to consult any doctor. He is a
23-year-old man who usually isolates Explanation: This 23-year-old man
himself in a room and watches old 1950s suffers from schizoid personality
movies all time. He appears to be disorder which is characterised by
emotionally cold. He does not have friends emotional coldness, limited capacity
and quit university studies recently. His to express warmth and appearing
father was very upset with him but he was indifferent to criticisms. He is
indifferent to his comments. When his interested in solitary activities (i.e.
childhood friends visited him, he refused to watching 1950s movie). These
speak to them. He does not exhibit first rank symptoms do not occur in the
symptoms. He was exempt from national course of schizophrenia because he
service because of his behaviour. His father does not have first rank symptoms.
is helpless and wants to know what’s wrong He does not exhibit unusual belief
with his son. Which of the following is most as in schizotypal disorder.
correct diagnosis?
Author: Roger Ho
A. Delusional disorder Year: 2015.
B. Paranoid personality disorder
C. Schizoid personality disorder
D. Schizotypal disorder
E. Schizophrenia

22. You are seeing a 50-year-old woman who is The answer is C.


a part-time Chinese opera actress and
temple medium. She was found missing for Explanation: This woman works as
a few days. Her daughter tried to contact a temple medium. She is prone to
her but she did not bring her hand-phone. develop dissociative states. She
She does not recall where she went. There is went missing and undertook an
no injury and she seemed to be able to look unexpected and yet organised
after herself when she was away. She journey away from home but self-
accepted a few assignments during the care was maintained. There was
“hungry ghost” month. She is able to amnesia during the journey but not
perform during the opera. She remembers in other aspects of life. This most
her performance dates and scripts well. correct diagnosis is dissociative
Which of the following diagnosis is most fugue.
likely?
Author: Roger Ho
A. Alzheimer‟s disease Year: 2015.
B. Dissociative amnesia Used in 2016
C. Dissociative fugue
D. Mild cognitive impairment
E. Frontal lobe dementia.

23. Which one of the following medical Answer is D.


complications is least likely to be found in
anorexia nervosa? Explanation: Growth hormone is
increased in patients suffering from
A. Hypocalcaemia anorexia nervosa. Bone mineral
B. Leucopenia density and female sex hormones
C. Reduction in bone mineral density are reduced. Leucopenia and
D. Reduction in growth hormone osteopenia occur in anorexia
E. Reduction in luteinizing hormone nervosa.
2016 used

Author: Roger Ho
Year: 2015.
Old age psychiatry
24. The following are features of pseudo- The answer is C.
dementia with the exception of:
Explanation: The onset of pseudo-
a. Usually acute in onset dementia is usually acute. There is
b. Presence of lack of motivation and does not attempt the presence of the lack of
to answer the questions motivation and attempt to answer
c. Memory deficits are often reported by family questions. Individuals usually give
members the answer „I don‟t know‟ to the
d. Intact arithmetic skills questions. Memory deficits are
e. Intact visual-spatial organisation. usually reported by the patients
themselves, instead of their relatives
and friends. As compared to
dementia, there are intact arithmetic
skills, paired associate learning and
visual-spatial organization.

Author: Roger Ho
Year: 2015.

25. Which of the following functions is least likely to The answer is C.


be affected in normal aging?
Explanation: Recall of important life
A. Acquisition of new information event and not recent information is
B. Problem solving skills least likely to be affected in normal
C. Recall of important life event aging.
D. Verbal fluency
E. Sensorimotor function. Author: Roger Ho
Year: 2015

26. Which of the following is the best initial treatment The answer is E.
for agitation in dementia?

A. Antipsychotic drug
Explanation: Behavioural therapy
B. Antidepressant (e.g. relaxation training, rewarding
C. Aversion therapy calm behaviour and modulating
D. Benzodiazepine environment) is the best initial
E. Behavioural strategies treatment for dementia.

Author: Roger Ho
Year: 2015

27. A 70-year-old man is arrested by the police The answer is D.


for attacking his neighbour. He is a widower
living alone and does not have past Explanation: This elderly man
psychiatric history. The elderly man claims suffers from late onset psychosis.
to receive radar signals from his neighbour Late onset psychosis is associated
and believes this neighbour wants to harm with schizoid/paranoid personality,
him. As a result, he attacked the neighbour sensory impairment, affect
to protect himself. Which of the following is flattening, hallucinations and
the most important predisposing factor for cognitive impairment. Manic
his condition? symptoms, grief reaction and
morbid jealousy are irrelevant for
A. Avoidant personality this case.
B. History of manic symptoms
C. Grief reaction towards deceased wife Author: Roger Ho
D. Sensory impairment Year: 2015
E. Unemployment

28. Which of the following complaint is most The answer is C.


suggestive of Lewy body dementia?
Explanation: Visual hallucination is
A. A patient complains hearing two voices talking most likely associated with Lewy
about him. body dementia.
B. A patient complains of a male voice giving
him instructions. Author: Roger Ho
C. A patient complains of seeing an orange man Year: 2015
standing in the air outside the window.
D. A patient complains of foul smelling in the
ward but not smelled by other people.
E. A patient complains that his elbow joint is
moving from his right arm to his left leg.

Child and adolescent psychiatry


29. Which of the following is FALSE about The answer is B.
factitious disorder by proxy or Munchausen
syndrome by proxy? Explanation: Munchausen syndrome by
proxy is considered to be a form of
A. This condition may put the child at risk. child abuse because the parent inflicts
B. This condition is not considered to be child abuse physical symptoms on the child to get
because the parents are very concerned about admitted to the hospital.
their child.
C. This condition is most likely to involve pre-school Author: Roger Ho
children. Year: 2015
D. The parents are often health care professionals.
E. The parent involved is most likely to be the
mother.

30. A 17-year-old male adolescent was admitted The answer is E.


to the psychiatric ward. It is unclear
whether he suffers from autism or Explanation: People with autism may
schizophrenia. Which of the following have less than 1- year of normal
features is the STRONGEST evidence development.
which suggests the diagnosis of autism?
Author: Roger Ho
A. Negative symptoms Year: 2015.
B. Poor peer relationships
C. Self-injurious behaviour
D. Unusual content of speech
E. Very short period of normal development.

31. Regarding the pharmacological treatment The answer is C.


for attention deficit and hyperactivity
disorder, which of the following medications Explanation: Methylphenidate is the
is commonly used as the first-line treatment first-line treatment. It is a stimulant,
option? which works by inhibiting dopamine
reuptake and hence causes the direct
a. Bupropion release of dopamine. The usage of
b. Clonidine stimulant is indicated for ADHD or
c. Methylphenidate ADHD with comorbid conduct disorder.
d. Nortriptyline Beneficial effects of methylphenidate
e. Omega-3 fatty acids include it effects of helping to improve
attention span and hyperactivity for a
certain number of hours in a school
based setting.

Author: Roger Ho
Year: 2015.
Substance abuse
32. A 24-year-old man has been using an illicit The answer is E.
drug on a regular basis. He presents with
urinary problems. In the past, he could Explanation: Regular use of
drink two cans of soft drink (250ml per can) ketamine will lead to the epithelial
without the need to rush to toilet. inflammation in urinary tract and
Nowadays, he needs to rush to toilet after cause urinary incontinence. As a
drinking a small bottle of Yakult (100 ml). result, his urinary retention capacity
Which of the following drugs is most likely has reduced significantly to one-
to be associated with the above fifth of his original capacity.
phenomenon?
Author: Roger Ho
A. 3-4 methylenedioxymethamphetamine Year: 2015.
B. Buprenorphine
C. Cocaine
D. Heroin
E. Ketamine

33. An 18-year-old female is using an illicit The answer is B.


drug. She smuggles the drug in and out of
disco easily because it is in blotter paper Explanation: Blotting paper is used to
and hard to be noticed by the police. The absorb the lysergic acid
drug is absorbed from the oral cavity. diethylamide (LSD) and it is a mean to
Which of the following drugs best match the transport LSD.
above description?
A. Cannabis Author: Roger Ho
B. Lysergic acid diethylamide Year: 2015.
C. Magic mushroom
D. Nitrazapam
E. Phencyclidine.
Psychopharmacology
34. A 20-year –old man suffering from autism The answer is D.
harms himself many times a day. Which of
the following medication is most effective in Explanation: Endorphins are
reducing self-harm? endogenous opiate-like substances
in the brain, and self-injury may
A. Buprenorphine increase the production of
B. Methadone
C. Morphine
endorphins. As a result, the
D. Naltrexone individual experiences an
E. Oxycodin. anaesthesia-like effect and he does
not feel any pain while engaging in
self-harm. Furthermore, the release
of endorphins may provide the
individual with a euphoric-like
feeling. Drugs that block the
binding at opiate receptor sites such
as naltrexone can successfully
reduce self-injury.

This is an advanced-level question.


Author: Roger Ho
Year: 2015.
35. A 30-year-old woman suffers from The answer is E.
depression. She complains of frequent
nightmare. She takes fluvoxamine every Explanation: Fluvoxamine can
night and wants to know the effect of reduce the frequency of nightmares
fluvoxamine on nightmare. Which of the by reducing the duration of REM
following statements is correct? sleep. Dreams and nightmares
occurring during REM sleep.
A. Fluvoxamine does not have any effect on
nightmare. Author: Roger Ho
B. Fluvoxamine has unpredictable effects on Year: 2015.
nightmare.
C. Fluvoxamine increases the occurrence of
nightmares throughout the night.
D. Fluvoxamine increases the occurrence of
nightmares towards the later part of the night.
E. Fluvoxamine reduces nightmares by reducing
the duration of rapid eye movement (REM)
sleep.

36. The first rank symptoms of a schizophrenia The answer is C.


patient did not respond to combination of
risperidone, olanzapine and aripiprazole Explanation: The first rank
with adequate dose for 6 months. The symptoms do not respond to the
patient is compliant to medication. Which of second generation antipsychotics.
the following actions is MOST appropriate? He/she should consider the first
generation antipsychotic drug such
A. Consider clozapine as haloperidol before clozapine.
B. Consider electroconvulsive therapy
C. Consider haloperidol Author: Roger Ho
D. Consider lithium Year: 2015.
E. Consider quetiapine

37. A 33-year-old man suffering from bipolar The answer is D.


disorder presents to the emergency department
with severe right upper quadrant pain radiating Explanation: The patient presents with
to his back, nausea and vomiting. Recently, the drug-induced hepatitis. Valproate is
psychiatrist has prescribed a new medication to most likely to cause drug – induced
treat his bipolar condition. Which of the hepatitis among all options.
following is most likely contributing to these
symptoms?
Author: Roger Ho
Year: 2015.
A. Carbamazapine
B. Lithium
C. Olanzapine
D. Valproate
E. Topiramate.
38. Based on the report published by the Health The answer is A.
Science Authority (2013), which of the
following drugs have the highest risk of Explanation: It is currently compulsory
causing the Steven Johnson Syndrome? in Singapore to test for genotype HLA –
B*1502 to identify patients who are at
A. Carbamazapine risk to develop the Steven Johnson
B. Lithium Syndrome.
C. Olanzapine
D. Valproate Author: Roger Ho
E. Topiramate. Year: 2015.
39. A 30-year-old man suffers from The answer is B.
schizophrenia. He has history of
amphetamine abuse. Three months ago, he Explanation: Clozapine is most likely to
was admitted to the psychiatric ward. On cause diabetes among the five options.
admission, his body mass index (BMI) was This patient is suffering from diabetic
25. The psychiatrists prescribed the ketoacidosis.
following medications: haloperidol,
fluoxetine, lorazepam and clozapine. During Author: Roger Ho
the past three months, his BMI has Year: 2015.
increased to 30. He has complained of
blurred vision and polydipsia. Today, he
suddenly becomes unconscious. His blood
sugar level is 39 mmol/L (normal: 3- 6
mmol/L). Which of the following drugs is
most likely to be responsible for the sudden
deterioration?

A. Amphetamine
B. Clozapine
C. Fluoxetine
D. Lorazepam
E. Haloperidol
40. A patient complains of nausea after taking The answer is D
escitalopram. Which of the following
antidepressants is least likely to cause Explanation: Mirtazapine works on the
nausea as compared to escitalopram? 5-HT3 receptors and avoid nausea.

A. Bupropion Author: Roger Ho


B. Fluoxetine Year: 2015
C. Fluvoxamine
D. Mirtazapine
E. Sertraline

41. A 40-year-old man with bipolar disorder The answer is A.


and on lithium 800mg at night, had a serum
lithium level of 0.6 mEq/L (normal level: 0.6 Explanation: The serum lithium
– 1 mEq/L).The dose of lithium was level can identify whether the dose
increased to 1000 mg at night and five days of lithium is too high or the patient
later, the serum lithium level was 0.3 is not compliant to lithium. It is not
mEq/L. Which of the following is the MOST possible that the dose was increased
appropriate action? but the patient has lower serum
level as compared to previous
A. Ask his family member to check whether he readings. This observation is due to
swallows the lithium tablet. non-compliance and the doctor
B. Change his medication to sodium valproate. should ask family member to check
C. Check the renal function tests. whether he actually swallows the
D. Look for drug interaction and reduce the dose tablets.
of other psychotropic medications.
E. Refer him for dialysis immediately. Author: Roger Ho
Year: 2015
42. A 30-year-old woman complains that she The answer is D.
starts to feel sad and emotionally
overwhelmed 3 days before her menses. Her Explanation: This patient suffers from
appetite is haywired and wants to eat junk premenstrual syndrome (PMS).
food before her period. She also complains Sertraline is a selective serotonin
of lower right abdominal pain and muscle reuptake inhibitor (SSRI) and this is the
tension before menses. She wants to know first-line treatment of PMS.
which of the following medications benefits
her condition. Your answer is: Author: Roger Ho
Year: 2015
A. Midazolam
B. Mirtazapine
C. Risperidone
D. Sertraline
E. Sulpiride

43. Which of the following is the most The answer is B.


important contraindication for
electroconvulsive therapy? Explanation: Option B seems to be
most risky and haemorrhagic stoke
A. Mini Mental State Examination score of 25. may increase the intracranial
B. Recent cerebrovascular accident pressure. Other options are not
C. Recent tonic-clonic seizure contraindications for ECT.
D. Recent hospitalisation due to occurrence of
manic symptoms after starting antidepressants Author: Roger Ho
E. Recent osteoporosis Year: 2015

Psychotherapy
44. The psychiatrist has ordered intramuscular The answer is B.
haloperidol instead of intramuscular lorazepam.
The patient developed acute dystonia. He is very Explanation: This patient transfers
angry at the psychiatrist. The patient does not his emotional response (i.e. anger)
express the anger towards the psychiatrist, but to a particular person (i.e. healthcare
became verbally abusive towards the ward assistant) who was not involved in
attendant and healthcare assistants. This patient the medication error and carries less
exhibited which of the following defence emotional risk (i.e. affecting his
mechanisms? discharge plan made by the
doctors).
A. Denial
B. Displacement Author: Roger Ho
C. Projection Year: 2015
D. Reaction formation
E. Sublimation

45. A 25-year-old woman with depression The answer is C.


works in a company which hired a new
supervisor. She feels that the new supervisor Explanation: Interpersonal
a few months ago. She feels that the new psychotherapy (IPT) is most
supervisor looks down on her because she is appropriate in this case. IPT allows
young. She is hurt by his remarks and has the patient to understand her
thoughts of resigning from her job. She communication difficulties in her
finds that her supervisor is biased. She company and change maladaptive
cannot get along with other colleagues patterns (e.g. avoiding her
whom she tries to avoid. Which of the colleagues). IPT can help the patient
following is the MOST suitable to build a better social network.
psychotherapy?
Author: Roger Ho
A. Cognitive behaviour therapy Year: 2015
B. Dialectical behaviour therapy
C. Interpersonal psychotherapy
D. Mentalization based therapy
E. Psychodynamic psychotherapy

Ethics and laws


46. A 35-year-old pathologist with 3 – year post The answer is E.
specialisation experience was found to have
made multiple mistakes in the histological Explanation: Based on ethical principle,
diagnosis. She made mistakes in 118 doctors should do no harm to patients.
pathological reports (total 200 pathological As the mistakes made by this doctor
reports) in the past 6 months, leading to affect treatment of patients, she
wrong diagnosis and treatment. She admits should not work independently and be
that she suffers from depression as a result referred to a psychiatrist from another
hospital to provide an independent
of marital problems and cannot concentrate opinion which is unbiased.
at work. Which of the following actions is
MOST appropriate?
Author: Roger Ho
A. Allow her to work independently because she Year: 2015
made careless mistakes and such mistakes would
not be repeated.
B. Allow her to work independently because she has
passed the specialist exam.
C. Allow her to work independently by reducing her
workload.
D. Do not allow her to work independently and refer
her to see a psychiatrist in the same hospital for
assessment. The psychiatrist should know her
well.
E. Do not allow her to work independently and refer
to see a psychiatrist in another hospital for
assessment. The psychiatrist should not know her.

Liaison and neuropsychiatry


47. A 40-year-old woman suffered from head The answer is A.
injury in a road traffic accident in January,
2010. She was comatised for 6 weeks. After Explanation: The head injury leads to
she recovered, she could only recall the anterograde amnesia and she has
events in the 1990s and early 2000s. She difficulty to form new memory. As a
firmly believes that Mr. Goh Chok Tong is result, she cannot recall or remember
the current prime minister of Singapore. the current prime minister and new
She cannot identify and recall new buildings buildings in Singapore.
or new places of interest in Singapore.
Which of the following is the most Author: Roger Ho
appropriate term to describe her condition? Year: 2015

A. Anterograde amnesia
B. Dissociative amnesia
C. Misidentification syndrome
D. Retrograde amnesia
E. Selective amnesia

48. A 40-year-old architect suddenly became The answer is C.


aggressive and agitated a day after a
spinal operation. When you assessed him, Explanation: This man suffers from
he talked about selling toys in Chinatown. delirium and the best treatment is to
Few hours later, you visited him again and start low dose antipsychotic drug.
he could not remember you and your
Author: Roger Ho
earlier visit. He appears to be confused. Year: 2015
Which of the following is most
appropriate?

A. Start low dose acetylcholinesterase


inhibitor
B. Start low dose antidepressant
C. Start low dose antipsychotic
D. Start low dose benzodiazepine
E. Start low dose mood stabilizer.

49. Which one of the following statements about Answer: E The patient believes that
somatisation disorder is false? he suffers from a terminal illness.

Explanation: Option E is incorrect.


A. Complaints in the gastrointestinal system are There are usually multiple, recurrent
common. and frequently changing physical
B. Most patients have multiple contacts with symptoms. Gastrointestinal and
primary and specialist medical services. neurological symptoms are common.
C. Most of the patients tend to have negative There must be the presence of
investigation results. multiple and variable physical
D. Major depressive disorder is a common symptoms, with no physical
comorbidity. explanation found. There is
E. The patient believes that he suffers from a impairment of functioning attributable
terminal illness. to symptoms and resulting behaviour.

50. A 55-year-old woman suffers from breast The answer is A.


cancer and receives tamoxifen. She also
suffers from depression. Which of the Explanation: Fluvoxamine and
following antidepressant is safest? paroxetine are contraindicated
A. Escitalopram for breast cancer patients
B. Fluoxetine receiving tamoxifen as it will
C. Fluvoxamine inhibit metabolism of tamoxifen
D. Paroxetine and leads to cancer recurrence.
E. Sertraline Escitalopram has the least drug
interaction and most appropriate.

This is an advanced level


question.

Author: Roger Ho
Year: 2015
Extended matching items

Question 51 – 54

Options:

A. Attention deficit and hyperactivity disorder


B. Conduct disorder
C. Oppositional defiant disorder
D. Separation anxiety disorder
E. Tourette‟s syndrome

Lead-in: Select the most appropriate diagnosis from the above for the following clinical
scenario.

51. Tiffany is a ten-year-old girl brought to the clinic for an assessment. The mother
describes her having facial tics and muscle spasms in various parts of her body,
especially of her fingers and toes, eyes, excessive blinking and twitching around her
mouth. The muscle spasms affect her writing. She has been punished in school for
shouting vulgarities. (Answer: E)

52. Tim is a ten-year-old boy brought to the clinic for an assessment. The mother
describes him as an argumentative person. He is also spiteful and blames others for
his troubles. Following an argument with his father, his family called a church
counsellor for advice. Tim told the counsellor that he could scold his father as he has
the freedom to express himself. (Answer: C)

53. Tina is a ten-year old girl with difficulties attending school. She complains of gastric
discomfort in the mornings and she has persistent fears that her parents might have an
accident or be kidnapped on their way to work. Tina does not sleep well at night.
(Answer: D)

54. Tom is a ten-year-old boy brought to the clinic for an assessment. On entering the
examination room, Tom was so disruptive that the doctor and nurses could not handle
him. The mother described a 4-year history of aggressive and destructive behavior at
home and in school. He has been suspended by the school on a yearly basis since
Primary 1 due to fighting. He is often uncontrollable at home and has broken dishes
and furniture. Last year, Tom was playing with the gas stove and started a small fire
in the HDB flat after an argument with his father. Tom frequently pulls the family dog
around by its tail. (Answer: B)

Author: Roger Ho
Question 55 – 57

Options:

A. Agranulocytosis
B. Irritable bowel syndrome
C. Neuroleptic malignant syndrome
D. Psychogenic polydispsia
E. Pseudoseizure

Lead-in: Select the most appropriate diagnosis from the above for the following clinical
scenario.

55. A 25-year-old man who suffers from schizophrenia, has a tonic – clonic
seizure as a result of hyponatraemia. He believes that he must drink water
constantly to purify himself of all sin. (Answer: D)

56. A 30-year-old woman who suffers from schizophrenia is stressed about her
new job as a receptionist. She complains of bloating, abdominal distension,
diarrhoea and frequent abdominal pain that is relieved by defecation. (Answer:
B)

57. A 35-year-old man with acute psychosis develops high fever, difficulty in
swallowing and labile blood pressure 3 days after commencing intramuscular
haloperidol. (Answer: C)

Author: Roger Ho

Question 58 – 60

Options:

A. Adherence therapy
B. Cognitive remediation
C. Graded exposure and desenitisation
D. Grief therapy
E. Psychoeducation

Lead-in: Select the most appropriate psychological treatment from the above for the
following clinical scenario.

58. A 20-year-old nursing student has been diagnosed with social phobia. She is
currently on medical leave but needs to prepare to return to her studies. She
says that she does not know how to overcome anxiety when she performs a
procedure in front of her tutors and classmates. (Answer: C)
59. A 23-year-old woman newly diagnosed with a first manic episode and is
currently an inpatient. She insists on writing a will, marrying a taxi driver and
booking a hotel to view fireworks during the National Day. She says that she
does not understand why she has so many new ideas racing through her
mind. (Answer: E)

60. A 40-year-old woman lost her daughter who committed suicide 6 months ago.
She feels very guilty and finds it difficult to move on. She is bargaining with
God to let her daughter be alive again. (Answer: D)
MCQ exam 12

History, mental state exam and


psychopathology
1. You are an opthalmologist. You need to The answer is B.
perform an eye operation for a 64-year-
old woman who has an upper eyelid Explanation: In this example, her
conjunctival retention cyst. She speech takes a long time to reach
appears to be anxious. You find it the point (i.e. she finally agrees to
difficult to obtain consent from her. She go for eye operation). She includes
provided irrelevant information such as a great deal of unnecessary
the peeling skin of her eyes, doctors details. This is knowns as
being too rush and doctors being not circumstantiality.
happy with her. Towards the end of the
interview, she finally agrees to go for Author: Roger Ho
the eye operation. Which of the Year: 2015
following terms BEST describes the
above phenomenon?

A. Approximate answer
B. Circumstantiality
C. Flights of ideas
D. Lossening of associations
E. Tangentiality.

2. Which of the following is NOT a The answer is E.


classical obsession?
Explanation: Recurrent worries of
A. Recurrent doubts about the closure of one’s child not doing well in the
windows and doors at home PSLE is common among parents in
B. Recurrent sexual imagery Singapore and people with
C. Repeated thoughts about contamination generalized anxiety disorder. This
D. Recurrent thoughts of shouting vulgarities thought does not lead to
during a church service compulsions.
E. Recurrent worries that one’s child will not
do well in the Primary School Leaving Author: Roger Ho
Examination (PSLE) Year: 2015
Used in 2016
3. A 60-year-old Caucasian man has The answer is B.
stopped antidepressant for 6 months.
He cannot sleep at night. He has very Explanation: Patients suffering
poor appetite and lost 20 kg in 6 from Cotard's syndrome present
months. His wife complains that he is with nihilistic delusion. These
losing touch with reality. The patient people believe that they are dead
insists that he does not exist in this or no longer exist.
world and he is dead. He also claims
that he does not have any family Author: Roger Ho
member. Which of the following BEST Year: 2015
describes his psychopathology?
A. Capgras syndrome
B. Cotard syndrome
C. De Clérambault's syndrome
D. Fregoli syndrome
E. Ganser’s syndrome

4. A 23-year-old man suffers from the first The answer is B.


episode of schizophrenia. The psychiatrist
has prescribed risperidone 6mg. This Explanation: Akathisia is common side
morning, he suddenly wanted to jump effect of risperidone, especially at high
down from a building because he could not doses. It has two components: an
control his inner urge to move around. external state of restlessness (i.e. his legs
During the interview, you observed that his were moving all the time) and inner state
legs were moving all the time although he
of restlessness, which resulted in the urge
was sitting on a chair. Which of the
to jump down.
following terms BEST describes his motor
psychopathology?
Author: Roger Ho
A. Acute dystonia Year: 2015
B. Akathisia
C. Catatonia
D. Mitgehen
E. Pseudoparkinsonism
Cognitive assessment
5. Which of the following is NOT related to The answer is A.
impairment in the frontal lobe function?
Explanation: Constructional
A. Constructional apraxia apraxia is related to impairment in
B. Impaired cognitive estimation the parietal lobe function.
C. Failure in performing the Luria’s hand test
D. Perseveration Author: Roger Ho
E. Verbal fluency. Year: 2015

Psychiatric epidemiology and genetic risk


6. A 30-year-old woman suffers from The answer is B.
schizophrenia. Her husband does not
have any psychiatric illness. She wants Explanation: The chance for the
to know the chance of her son child of a schizophrenia patient
developing schizophrenia in the future. developing schizophrenia is 13%.
Your answer is:
Author: Roger Ho
A. 3% Year: 2015
B. 13%
C. 23%
D. 33%
E. 43%

Psychiatric aetiology, diagnosis and


classification
7. Which of the following neuroanatomical The answer is B.
areas is MOST implicated in the
pathology of obsessive–compulsive Explanation: Injury in the basal
disorder? ganglia is associated with brain
injury has been reported to be
A. Amygdala related to onset of OCD. Brain-
B. Basal ganglia imaging shows morphological
C. Nucleus accumbens changes in the basal ganglia in
D. Hippocampus OCD patients. Functional
E. Substantia nigra neuroimaging studies show
increased blood flow in the basal
ganglia.

Author: Roger Ho
Year: 2015

8. Which one of the following is the MOST The answer is E.


important predisposing factor for
borderline personality disorder? Explanation: Borderline personality
disorder results from early
A. Conduct disorder traumatic experiences occurring
B. Embarrassment within the context of sustained
C. Magical thinking neglect, physical or sexual abuse,
D. Psychosis resulting in enduring rage and self-
E. Sexual abuse hatred. It is the result of a lack of
stable attachment during early
development. This leads to an
inability to maintain a stable sense
of self or others without developing
trust with other people. The child
will have limited a capacity to
depict feelings and thoughts in self
and other people.

Author: Roger Ho
Year: 2015

General adult psychiatry


9. Which of the following is TRUE about The answer is B.
somnambulism?
Explanation: Somnambulism
A. This condition is common in old people. (Sleep-walking) improves with
B. This condition improves with SSRI (e.g. paroxetine) or TCA (e.g.
antidepressant treatment. imipramine). This condition is
C. This condition occurs in the late stage of common amongst children and
sleep, usually 15-20 minutes before the adults. This condition occurs in the
person wakes up from sleep. early stage of sleep, usually 15-20
D. The patient usually exhibits simple jerking minutes after an individual falls
movement. asleep. The patient usually exhibits
E. The patient can recall the exact incident complex movement and not able to
during somnambulism. recall the incident during
somnambulism.
Author: Roger Ho
Year: 2015
Used in 2016

10. A 40-year-old woman feels very The answer is A


nervous when she is in a crowded MRT
train. She avoids leaving her house Explanation: The patient develops
even with the support of her husband. anxiety symptoms when she is in
She does not experience palpitation crowded MRT train where she has
and hyperventilation. Which of the difficulty escaping. She tries to
following diagnoses is MOST likely? avoid leaving her house. She does
not have panic attacks. The best
A. Agoraphobia without panic disorder diagnosis is agoraphobia without
B. Agoraphobia with panic disorder panic disorder.
C. Generalised anxiety disorder
D. Panic disorder Author: Roger Ho
E. Social phobia. Year: 2015

11. A 40-year-old woman has suffered from The answer is E.


bipolar disorder for 20 years. She
consults a psychiatrist in a general Explanation: This patient is at risk
hospital on a regular basis. She to her sister (i.e. holding a weapon
requires taking sodium valproate for and her sister was injured).
long term. Her sister is the main Although the caregiver is not keen
caregiver. Recently, the patient has to send the patient to the IMH for
labile mood and refuses to take sodium assessment, the doctor needs to
valproate. Her sister worries that she is do so because this patient is
having a relapse. This morning, the dangerous and she may harm
patient refused to see the psychiatrist other people. She will be admitted
and used a knife to threaten her sister. to the IMH for observation under
Her sister suffered from superficial the Mental Health (Care and
lacerations on her hands. You are a Treatment) Act.
general practitioner. Her sister bought
her to your clinic for assessment but Author: Roger Ho
she does not want the patient to be Year: 2015
sent to the Institute of Mental Health
(IMH). Which of the following
management is MOST appropriate?

A. Arrange electroconvulsive therapy in the


general hospital.
B. Ask her sister to administer covert
antipsychotic (i.e. putting droplets of
haloperidol in her food without her
knowledge)
C. Offer depot antipsychotic injection and
arrange follow-up appointment in your
clinic
D. Offer sodium valproate and refer the
patient to community psychiatry team for
close monitoring
E. Sedate the patient and then send her to
the IMH for assessment by an ambulance.

12. A 23-year-old man came with his mother to The answer is C.


your clinic. He requested for a letter from you
to support him quitting his study from a private
university. He wants to quit because he failed
Explanation: This patient suffers from
his group project. He has difficulty getting paranoid personality disorder because
along with his group members. He does not of the following clinical features:
trust his classmates and accuses his excessive sensitivity to setbacks (e.g.
classmates of deceiving him although there is He wants to quit his study because
no such evidence. His mother mentions that he
often has mood swing from normal mood to he failed his group project);
hostility. Recently, he had a fight with a suspiciousness of others (e.g.
stranger in the shopping mall because he accuses his classmates of
felt that the other person posed as a threat deceiving him although there is no
to him. He is also stubborn and insists on such evidence); unjustified doubts
running an online business which does not of others (e.g. He does not trust his
generate any income. He has a strong classmates); angry/hostile
sense of autonomy over his failed online reactions to perceived threat (e.g.
business. He cannot accept any feedback He had a fight with a stranger in
from his parents because he believes that
there are hidden meanings in benign
the shopping mall because he felt
comments. Which of the following that person posed potential threat
diagnosis is MOST likely? to him); persistent self-
referential/self-important attitude
A. Antisocial personality disorder (He is also stubborn and insists to
B. Narcissistic personality disorder run an online business which does
C. Paranoid personality disorder
not generate any income) and
D. Schizoid personality disorder
E. Schizotypal personality disorder hidden meaning in benign remarks.

Author: Roger Ho
Year: 2015

13. A 30-year-old woman is bought to the The answer is B.


Accident and Emergency Department.
Her family members are very
concerned about her well-being Explanation: Catatonia is usually
because her boyfriend died after he fell caused by schizophrenia or
from height as a result of alcohol organic conditions. Among all
intoxication. Her family members want the options, catatonia is least
to know more about her prognosis.
Which of the following psychiatric likely to develop.
disorders is LEAST likely to develop?
Author: Roger Ho
A. Acute psychosis Year: 2015
B. Catatonia
C. Dissociative amnesia
D. Post-traumatic stress disorder
E. Prolonged grief
14. A 22-year-old woman is admitted to the The answer is B.
ward after she ingested 30 tablets of
paracetamol. She seems to be Answer: This patient suffers from
ambivalent about her studies. Initially, borderline personality disorder
she wanted to study economics. Then characterised by identity confusion,
she changed to study biomedical inability to trust others, unstable
science but she could not cope. Now emotion, impulsive behaviour (e.g.
she wants to study veterinary medicine. binge eating) and transient
Her boyfriend could not cope with her psychosis. Her condition is
mood swings which is characterised by predisposed by an invalidating
normal mood and anger. The environment.
relationship had lasted for 3 months.
She agrees to be an impulsive person Author: Roger Ho
and has history of binge eating. She Year: 2015
sees ghost in her room when she is
stressed. She cannot trust other people
due to her complicated family
environment. Her father has two
marriages. She was born from the
second marriage and feels rejected by
her step-siblings. She appears to be
guarded during the interview. Her
personality is best described as:

A. Avoidant personality disorder


B. Borderline personality disorder
C. Dependent personality disorder
D. Obsessive compulsive personality disorder
E. Paranoid personality disorder
15. A 50-year-old man works as a MRT train driver. The answer is D.
He was admitted to the psychiatric ward due to
relapse of bipolar disorder. He did not adhere
Explanation: The patient has just
to his mood stabiliser prior to admission. He is
ready for discharge and hopes to resume his recovered from schizophrenia. This
duty. Which of the following actions is the condition may put himself and his
MOST important? passengers at risk if he experiences first
rank symptoms. The doctor should advise
A. Consider depot antipsychotic drug. him to inform his company. If the patient
B. Issue medical certificate for one week after
refuses to inform the company, the
discharge
C. Issue medical certificate for one month after doctor may consider breaching
discharge confidentiality to inform the company to
D. Encourage the patient to inform his company about protect the public.
his condition.
E. Encourage patient to return to work as soon as Author: Roger Ho
possible.
Year: 2015
16. A 48-year-old man has been suffering from The answer is D.
schizophrenia for 25 years. His family kept him
from seeing any doctor and refuses to let him
Explanation: He has poor prognosis due
to take antipsychotic drug as they do not
believe he needs to take medication. What to long duration of untreated psychosis,
should you tell the family? male gender and over involvement of
family.
A. He has good prognosis because of the intensive
family involvement. Author: Roger Ho
B. He has good prognosis because of his male
gender. Year: 2015
C. He has good prognosis because he receives
treatment at old age.
D. He has poor prognosis because of long duration of
untreated psychosis
E. He has poor prognosis because his family has
protected him from external stress for 25 years.

17. Which one of the following statements regarding The answer is B.


schizotypal disorder is false?
Explanation: Schizotypal personality
A. It is more common among the first-degree relatives of disorder is more common among the
patients suffering from schizophrenia. first-degree relatives of schizophrenic
B. Individuals usually exhibit hallucinations and thought patients. People with this personality
disorders. disorder do not exhibit first-rank
C. Individuals have low capacity and motivation for symptoms and do not have hallucinations
relationships and thought disorders. Supportive
D. Most individuals rarely seek treatment because of poor psychotherapy is useful to establish
insight therapeutic alliance. As trust increases,
E. Supportive psychotherapy is useful the therapist may be able to access the
patient’s beliefs.

Author: Roger Ho
Year: 2015
18. A 40-year-old woman suffes from depression The answer is D.
and insomnia. She worries that she does not
get enough rapid eye movement (REM) sleep at
Explanation: The REM sleep is more
night. She has read about REM sleep and wants
to discuss with you. Which of the following frequent towards the second half of
statements is CORRECT about the REM sleep? sleep. Each episode of the REM sleep
lasts for 90 minutes. The complexity of
A. Each episode of the REM sleep lasts for 3-4 hours. dream and sympathetic activity increase
B. Reduction of the REM sleep will lead to increased during the REM sleep. There is an
complexity of dream.
increase in vaginal blood flow during the
C. Reduction of the REM sleep will increase
sympathetic activity throughout her sleep. REM sleep.
D. She wakes up at 2 am and maintains light sleep
from 2am onwards. As a result, there is more Author: Roger Ho
interruption to the REM sleep. Year: 2015
E. There is reduction in vaginal blood flow during the
REM sleep.

Old age psychiatry


19. Which of the following psychiatric complications The answer is D.
is MOST likely to occur in a 62-year-old man with
a 30-year history of alcohol dependence: Explanation: Alcohol dependence is
associated with alcohol dementia or
A. Late onset anxiety disorder Korsakoff syndrome. This is most likely to
B. Late onset bipolar disorder occur among all complications.
C. Late onset schizophrenia
D. Dementia Author: Roger Ho
E. Pathological grief Year: 2015
20. Which of the following is NOT related to Lewy The answer is A.
Body Dementia?
Explanation: Patients suffering from Lewy
A. A poor response to cholinesterase inhibitor body dementia respond to rivastigmine.
B. Fluctuations in cognition The other options are classical features
C. Parkinsonism of Lewy Body Dementia.
D. Visual hallucinations
E. Sensitivity to antipsychotics. Author: Roger Ho
Year: 2015
21. Which of the following medications is LEAST The answer is C.
likely to be misused by old people?
Explanation: Besides anticoagulants, the
A. Analgesics
B. Anxiolytics other drugs have abuse liability (e.g.
C. Anticoagulants analgesics) or related to relief of somatic
D. Anticholinergics complaints.
E. Anti-nausea agents.
Author: Roger Ho
Year: 2015

22. Which of the following psychotropic The answer is D


medications have the MOST evidence to
treat aggression in elderly who suffer from Explanation: Second generation
Alzheimer’s disease? antipsychotic drug (e.g. risperidone)
have the most evidence to treat
A. Antihistamine aggression in patients suffering from
B. Benzodiazepine
C. Melatonin
Alzheimer’s disease among all 5
D. Second generation antipsychotic drug options.
E. Selective serotonin reuptake inhibitor
Author: Roger Ho
Year: 2015

23. All the following clinical features are more The answer is D.
common in late onset depression as compared to
early onset depression EXCEPT: Answer: Late onset depressive disorder is
associated with absence of family history
A. Cognitive impairment of depressive disorder. The following
B. High suicide risk features are more common in late onset
C. Paranoia depressive disorder: psychomotor
D. Presence of family history of depressive disorder retardation/agitation, complaints of
E. White matter changes loneliness, depressive pseudodementia
(cognitive impairment), paranoia, anxiety,
high suicide risk, somatic complaints,
white matter changes due to
microvascular infarcts.

Author: Roger Ho
Year: 2015

Child and adolescent psychiatry


24. A 12-year-old male suffers from autism and The answer is A
stereotypies. His parents complain that the
stereotypies have caused a lot of problems in Explanation: SSRI is the best option to
his daily life. Which of the following
manage repetitive movement or
psychotropic medications is MOST effective in
treating stereotypies? stereotypies in patients suffering from
autism. Haloperidol or sodium valproate
A. Fluoxetine may cause tremor and cause more
B. Haloperidol movement symptoms.
C. Hydroxyzine
D. Lorazepam
Author: Roger Ho
E. Sodium valproate
Year: 2015

25. Which of the following antidepressants is the The answer is D.


SAFEST medication in treating a 15-year-old
adolescent with depression and risk of suicide? Explanation: Fluoxetine is the safest SSRI
in treating adolescents with depression
A. Amitriptyline and suicide risk.
B. Moclobemide
C. Mirtazapine Author: Roger Ho
D. Fluoxetine Year: 2015
E. Venlafaxine.

26. You are a paediatric resident. A 13-year-old The answer is C.


male was bought in by his parents to your
clinic. He is studying in Secondary 1. He was Explanation: This patient presents for
scolded by his teachers for poor handwriting. motor tics for several years but does
He wanted to jump down from a building
not present with vocal tics. The
because he was too slow during the
examination and could not write properly. His diagnosis is persistent motor tic
parents said that he has sudden, brief and disorder.
repetitive jerking movements of his right
hand and arm for several years. He does Author: Roger Ho
not have other symptom. He has been in Year: 2015
good past health in the past. Which of the
following is MOST likely diagnosis?

A. Acute stress disorder


B. Hyperkinetic disorder
C. Persistent motor tic disorder
D. Specific learning disorder
E. Tourette’s disorder

27. A 14-year-old female is treated with The answer is D


methylphenidate for Attention Deficit and
Hyperactivity Disorder (ADHD). All of the Explanation: The common side effects
following should be monitored continuously associated with the use of
methylphenidate include reduction in
throughout the course of the treatment
appetite, gastric discomfort, insomnia,
EXCEPT?
headache, elevation of blood pressure,
tics, dysphoria and irritability. With
A. Anxiety symptoms
regards to long-term usage, it is
B. Blood pressure
important to monitor for height and
C. Height and weight
weight (every 6 monthly), cardiovascular
D. Menstrual disturbances
status (e.g. blood pressure) (every 3
E. Tics
months), seizures, tics, psychotic
symptoms and anxiety symptoms.

Author: Roger Ho
Year: 2015
The answer is B.
28. Peter is a 11-year-old boy and his mother
brought him to see you because she does Explanation: The most appropriate
not have the energy to deal with Peter. Hediagnosis is conduct disorder. In order to
lives with his mother and three siblings. fulfil the diagnostic criteria of conduct
His father left them when he was 7-year- disorder, here must be repetitive and
old. He is aggressive towards his sibling persistent pattern of behaviour in which
and classmates. He has difficulty keeping either the basic rights of others or major
up with school work and has been age appropriate societal rules are
transferred to 3 different schools in the violated. The duration of the symptoms
past 3 years. He has always been known to must have lasted for at least a minimum
be a bully in school. He has destroyed of 6 months. Other symptoms might
properties in school and wanted to join include setting fire and stealing objects of
gangs. Which of the following is the MOST value within the house and also outside
appropriate diagnosis? of the house.

A. Asperger’s syndrome Author: Roger Ho


B. Conduct disorder Year: 2015
C. Hyperkinetic disorder
D. Learning disability
E. Oppositional defiant disorder

Substance abuse
29. A 20-year-old schizophrenia patient receives The answer is C.
olanzapine 20 mg per day. He has been
compliant to olanzapine. He started smoking
Explanation: Nicotine increases the
for the past one month. After he started
smoking, he experiences more psychotic metabolism of olanzapine and smokers
symptoms. Which of the following is the MOST requires higher dose of olanzapine in
likely explanation? order to be effective.

A. He is not complaint to olanzapine. Author: Roger Ho


B. He took an overdose of olanzapine.
Year: 2015
C. Nicotine increase the hepatic metabolism of
olanzapine
D. Nicotine reduces the risk of metabolic syndrome.
E. This is due to the effect of nicotine patch.

30. A 29-year-old woman suffers from mixed The answer is E.


anxiety and depression but she is concerned
about addiction. Which of the following
Explanation: Option A and C are
medications is the LEAST likely to cause
addiction or dependence when it is prescribed? benzodiazepine. Option B is
A. Alprazolam anticholinergic and may have euphoria
B. Benztropine effects. There are patients who are
C. Diazepam dependent on benztropine. Propranolol is
D. Methylphenidate least addictive among all options.
E. Propranolol
Author: Roger Ho
Year: 2015
31. The US FDA has approved a drug called The answer is D.
acamprosate to treat alcohol dependence. It
works on which of the following receptors?
Explanation: Acamprosate inactivates the
A. Cannaboid receptors NMDA receptors and avoid neurotoxicity
B. Dopamine receptors associated with glutamate which occurs
C. Opioid receptors during alcohol withdrawal.
D. N-Methyl-D-aspartic acid (NMDA) receptors
E. Serotonin receptors Author: Roger Ho
Year: 2015

The answer is D.
32. A 30-year-old man has been drinking 8
cups of coffee per day. Which of the Explanation: Excessive caffeine intake
following is NOT a side effect of excessive is associated with relaxation of lower
caffeine intake? oesophageal sphincter which is
associated with reflux.
A. Agitation
B. Diuretic effect Author: Roger Ho
C. Migraine Year: 2015
D. Tightened lower oesophageal sphincter
E. Sinus tachycardia.

Psychopharmacology
33. Which of the following medications has The answer is B.
recently obtained the FDA approval to treat low
sexual desire in women? Explanation: Filbanserin is a
norepinephrine-dopamine disinhibitor
A. Dapoxetine approved for the treatment of pre-
B. Flibanserin menopausal women with hypoactive
C. Sildenafil sexual desire disorder.
D. Tadalafil
E. Uprima
Author: Roger Ho
Year: 2015

34. A 25-year-old woman is 12-week pregnant. She The answer is A.


suffers from severe depressive disorder. Which of the
following psychotropic medications is MOST Explanation: Amitriptyline is the oldest
appropriate?
antidepressant and there is evidence that
A. Amitriptyline it does not cause harm to foetus.
B. Diazepam Paroxetine may cause pulmonary
C. Mirtazapine hypertension in foetus.
D. Paroxetine
E. Venlafaxine.
Author: Roger Ho
Year: 2015
35. A 40-year-old woman suffers from depression The answer is C.
and poor appetite. Her husband wants you to
prescribe an antidepressant to increase her
Explanation: Mirtazapine is most
appetite. Which of the following
antidepressants is MOST appropriate? appropriate because it works on the
5HT2C receptors.
A. Amitriptyline
B. Diazepam Author: Roger Ho
C. Mirtazapine Year: 2015
D. Sertraline
E. Venlafaxine.

36. A 40-year-old woman complains of frequent The answer is D.


palpitations and high blood pressure after
quarrelling with her husband. She has always Explanation: Propranolol can reduce
been good in health. She does not have a past palpitation and high blood pressure. She
psychiatric history. Which of the following is does not have history of asthma. The half-
MOST appropriate? life of diazepam is too long. As she has no
past psychiatric history, she does not
A. Chlorpromazine need to take chlorpromazine or
B. Diazepam olanzapine.
C. Mirtazapine
D. Propranolol Author: Roger Ho
E. Olanzapine Year: 2015

37. A 20-year-old patient took an overdose of 20 The answer is A.


tablets of selective serotonin reuptake inhibitor
(SSRI). Which of the following is MOST likely to Explanation: All SSRIs except escitalopram
be associated with prolonged QTc? have minimal or no effect on prolonged
QTc. Mirtazapine is not associated with
A. Escitalorpam prolonged QTc.
B. Fluoxetine
C. Fluvoxamine Author: Roger Ho
D. Mirtazapine Year: 2015
E. Sertraline.

38. A 30-year-old woman suffers from depressive The answer is C.


disorder. You prescribed the following
medications two days ago. She is seeing you
Explanation: 10% of patients treated with
today because she has developed generalized
rash on her body. Which of the following lamotrigine developed benign rash and
medications should be stopped? 0.1% developed severe rash.

A. Alprazolam Author: Roger Ho


B. Fluvoxamine Year: 2015
C. Lamotrigine
D. Quetiapine
E. Sodium valproate.

39. A 25-year-old American came to Singapore to The answer is B.


pursue a PhD degree. He claims that he was
prescribed with methylphenidate in the US
Explanation: Bupropion increases
although he has no history of attention deficit
and hyperactivity disorder. He feels depressed dopamine and improves energy of
due to adjustment problem in Singapore. He patients. It is a third line treatment for
feels tired and cannot concentrate on his ADHD and has ability to improve
studies. Which of the following medication is attention.
the MOST appropriate to substitute for
methylphenidate?
Author: Roger Ho
A. Agomelatine Year: 2015
B. Bupropion
C. Quetiapine
D. Risperidone
E. Sodium valproate
40. A 30-year-old man suffers from severe Answer: C
depression and he will receive
electronconvulsive therapy (ECT) tomorrow.
Explanation: Selective serotonin re-
Which of following medications do not need to
be held off to reduce interaction with ECT? uptake inhibitors (SSRIs) do not need to
be discontinued before the
A. Bupropion administration of ECT. Bupropion and
B. Clozapine clozapine may increase the risk of
C. Fluoxetine prolonged seizures. Lithium may increase
D. Lithium the risk of delirium or prolonged seizures.
E. Zopiclone Benzodiazepine has anticonvulsant
activity and may decrease the efficacy of
ECT.

Author: Roger Ho
Year: 2015

Psychotherapy
41. A 40-year-old man who is admitted to the The answer is E.
medical ward for upper gastrointestinal tract
bleeding, is known to be patient dependent on Explanation: This patient expresses his
alcohol. His gamma-glutamyl transferase (GGT) views to ban alcohol and claims that he
level is raised. During the interview, he hates drinking which is opposite to his
mentions that, ‘I hate drinking and alcohol. The unacceptable internal impulses to drink.
government should ban alcohol.’ What is his He drank prior to admission as evidenced
defence mechanism? by the raised GGT.

A. Acting out Author: Roger Ho


B. Projective identification Year: 2015
C. Projection
D. Rationalization
E. Reaction formation.

42. A 45-year-old old woman is very angry with The answer is D.


her in-laws. Instead of smashing all the
furniture, she expresses her emotion Explanation: Sublimation is a healthy
through calligraphy. Which of the following or mature defence mechanism. The
defence mechanisms BEST describe the external expression of unacceptable
above phenomenon? internal impulse is expressed in
socially acceptable way.
A. Displacement
B. Projection Author: Roger Ho
C. Repression Year: 2015
D. Sublimation
E. Reaction formation

Ethics and laws


43. A 50-year-old British tourist was arrested The answer is C.
by police and sent to the Institute of Mental
Health for assessment. He is supposed to Explanation: Automatism is a legal term
board a plane bound for London. After he and the person commits a crime without
checked in at the airport, he went to the bar his or her own personal control or
and drank for three hours. Then he went to awareness due to an underlying medical
the boarding gates and performed security condition or substances. Automatism can
screening on female passengers. He was
be caused by alcohol intoxication, sleep
stopped by other security officers who walking, epilepsy and hypoglycaemia.
called the police subsequently. Which of
the following is the BEST defence for his Author: Roger Ho
behaviour? Year: 2014.

A. Actus reus
B. Jealousy
C. Automatism
D. Diminished responsibility
E. Mens rea.
44. A 21-year-old male patient suffered from The answer is D.
psychosis and his mother worried that his
condition would deteriorate. She brought him
Explanation: Non-maleficience refers to
to see a general practitioner. The doctor told
them that he had high toxin level in his body the obligation to avoid harm. In this case,
without performing any biological investigation. the general practitioner did not refer to
The doctor suggested magnetic field therapy the patient to see a psychiatrist or start
(MFT) to help him to detoxify. The doctor with an antipsychotic drug. He applied
charged $5000 for the MFT. His parents agreed magnetic field therapy (MFT) which has
for the treatment and borrowed money to pay
no evidence in treating psychosis. As a
for the treatment. After the MFT, his psychosis
got worse. Which of the following ethical result, the condition of patient got worse
principles did the general practitioner violate? and caused harm to patient.

A. Autonomy Author: Roger Ho


B. Capacity Year: 2014.
C. Confidentiality
D. Non-maleficence
E. Justice

45. You are seeing a 21-year-old prisoner. He The answer is D.


mentions that he was fascinated with
murdering people at the age of 10. He Explanation: This man suffers from
started stealing when he was a teenager. psychopathy which is a more severe form
At age 15, he stole the first handbag in his of antisocial personality disorder and he
life. At age 17, he assaulted his first victim. has no remorse of previous offences and
He does not feel sorry for the victims has urge to commit more offences. He
because they are weak. After release from
has no victim empathy.
the prison, he will commit crime again due
to adrenaline rush. Which of the following
psychiatric diagnoses BEST describe his Author: Roger Ho
condition? Year: 2014.

A. Conduct disorder
B. Exhibitionism
C. Kleptomania
D. Psychopathy
E. Malignant narcissism

Liaison and neuropsychiatry


46. Which of the following is NOT an early The answer is A.
symptom of neuroleptic malignant
syndrome? Explanation: It should be tachycardia,
not bradycardia.
A. Bradycardia
B. Catatonia Author: Roger Ho
C. Dysphagia Year: 2014.
D. Hyperthermia
E. Rigidity

47. A 24-year-old housewife is brought in The answer is C.


by her husband. She delivered her first
born baby few weeks ago. She appears Explanation: She exhibits manic
to be disorientated to time and place. symptoms, disorientation and
She firmly believes that she is a doctor delusion one month after giving
graduated from the United Kingdom birth to her first born. This is the
and she can heal other patients. She classical trials of postpartum
firmly believes that her son is Jesus psychosis.
and devils are trying to harm him.
According to her husband, her highest Author: Roger Ho
education is diploma from the Institute Year: 2014.
of Technical Education (ITE). What is
the most likely psychiatric diagnosis?

A. Kosakoff’s syndrome
B. Postnatal depression
C. Postpartum psychosis
D. Schizoaffective disorder
E. Wernicke’s encephalopathy

48. A 50-year-old man suffering from atrial The answer is D.


fibrillation needs to take warfarin on a long-
term basis. He suffers from depression and Explanation: Mirtazapine has the least
hypertension. Which of the following drug interaction with warfarin and will
antidepressants is MOST appropriate? not enhance its anticoagulant effect.
Fluoxetine and fluvoxamine will have
A. Amitriptyline drug interaction with warfarin.
B. Fluoxetine Venlafaxine is associated with dose –
C. Fluvoxamine related hypertension. Amitriptyline
D. Mirtazapine can be cardiotoxic if patient takes an
E. Venlafaxine overdose.

Author: Roger Ho
Year: 2014.
49. A 60-year-old man develops abnormal The answer is B.
movements including paralysis and gait
disturbances. He was stressed after moving Explanation: He suffers from
into a new HDB flat. His daughter is very conversion disorder due to stress
concerned. The neurologist has assessed him associated with moving house.
but cannot identify any underlying organic Conversion disorder usually involves
cause for his abnormal movements. His motor or sensory functions.
abnormal movements are controlled by
alprazolam. What is the MOST appropriate Author: Roger Ho
Year: 2014.
psychiatric diagnosis?

A. Body dysmorphic disorder


B. Conversion disorder
C. Dissociative disorder
D. Hypochondriasis / Illness anxiety disorder
E. Somatisation disorder
50. A 30-year-old man claims to have The answer is D.
contracted Ebola infection and insists on
being admitted to the hospital. He claims to Explanation: This man suffers from
have returned from Nigeria 2 days ago. At Munchausen syndrome, also known
the Accident and Emergency Department, as hospital addiction syndrome. He
lies about his trip to Nigeria and
he does not have fever or presents with any pretended to suffer from Ebola to
sign that suggests Ebola infection. He is very make himself admitted to the hospital.
insistent on admission and requests to stay
in the hospital. He seems to be inconsistent Author: Roger Ho
in his travel history and is not able to Year: 2015
provide details on how he flew from
Singapore to Nigeria. You manage to
contact his family member. He does not
have a passport and has never left
Singapore. Past medical record reveals
multiple admissions. He likes to stay in the
hospital to enjoy free food and nursing care.
Which of the following is MOST likely
diagnosis?

A. Conversion disorder
B. Dissociative fugue
C. Ganser syndrome
D. Munchausen syndrome
E. Somatisation disorder
Extended matching items

Question 51 – 54

Options:

A. Command hallucination
B. Pareidolia
C. Hypnagogic hallucination
D. Synaesthesia
E. Tactile hallucination

Lead-in: Select the most appropriate psychopathology from the above for the following
clinical scenario.

51. A 10-year-old girl saw a vivid image of Santa Claus on the cloud without conscious
effort. (Choose 1 option)

(Answer: B)

52. A 20-year-old woman complains of hearing colour after taking hallucinogen. (Choose
1 option)

(Answer: D)

53. A 30-year-old man is disoriented and says there are insects crawling under his skin.
(Choose 1 option)

(Answer: E)

54. A 70-year-old man complains of hearing a female voice from the toilet when he is
falling asleep. (Choose 1 option)

(Answer: C)

Question 55 – 57

Options:

A. Agranulocytosis
B. Myocarditis
C. Neuroleptic malignant syndrome
D. Psychogenic polydipsia
E. Syndrome of inappropriate anti-diuretic hormone secretion (SIADH)

Lead-in: Select the most appropriate diagnosis from the above for the following clinical
scenario.
55. A 40-year-old man is admitted due to seizure and hyponatraemia. He suffers from
chronic schizophrenia and believes that he must drink water constantly to purify
himself of all sin. (Choose 1 option)

(Answer: D)

56. A 45-year-old woman is admitted due to depression, low back pain and
hyponatraemia. She has history of lung cancer and the cancer has spread to the bone.
(Choose 1 option)

(Answer: E)

57. A 50-year-old man is admitted due to chest pain and dizziness after commencing
clozapine 5 weeks ago. (Choose 1 option)

(Answer: B)

Question 58 – 60

Options:

A. Agomelatine
B. Carbamazepine
C. Lithium carbonate
D. Risperidone
E. Vortioxetine

Lead-in: Select the most appropriate psychotropic medication from the above for the
following clinical scenario.

58. A 20-year-old woman develops galactorrhoea after commencing a medication 6


months ago. (Choose 1 option)

(Answer: D)

59. A 30-year-old woman develops liver impairment after commencing a medication 2


weeks ago. (Choose 1 option)

(Answer: A)

60. A 40-year-old woman develops toxicity commencing thiazide and another medication
together. (Choose 1 option)
(Answer: C)
MCQ exam 13

History, mental state exam, psychopathology


1. A 20-year-old woman suffering from borderline The answer is C.
personality disorder went back to the HDB block
where she stayed during her childhood. She is a
Explanation: Jamais vu is a feeling of
victim of childhood abuse. She feels unfamiliar
with the HDB block and cannot recall her unfamiliarity of a familiar place. It
childhood memory. This phenomenon is BEST occurs in people under stress or
described as: exhaustion.

A. Anterograde amnesia Author: Roger Ho


B. Déjà vu Year: 2015
C. Jamais vu
D. Mild cognitive impairment
E. Perseveration.

2. Which one of the following is not part of the The answer is C.


Schneiderian first-rank symptoms?
Explanation: The Schneiderian first-
A. Auditory hallucinations in the form of running rank symptoms include: (1) auditory
commentary hallucinations in the form of
B. Delusional perception repeating the thoughts out loud,
C. Emotional blunting speaking to the person in the third
D. Somatic passivity person, in the form of running
E. Thought broadcasting commentary; (2) delusion of
passivity in the form of thought
insertions, withdrawal and
broadcasting; (3) somatic passivity;
and (4) delusional perception.
Emotional blunting is a negative
symptom and is not considered to be
first-rank symptom.

Author: Roger Ho
Year: 2015
3. A 30-year-old man suffers from brain tumour and The answer is C.
develops catatonia. Which of the following is NOT
a classical sign of catatonia?
Explanation: Lip smacking is a
A. Ambitendency classical feature of tardive dyskinesia
B. Echolalia but not catatonia. Option A, B, D and
C. Lip smacking E are classical features of catatonia.
D. Mutism
E. Waxy flexibility Author: Roger Ho
Year: 2015
4. A 65-year-old depressed man has been refusing The answer is C.
his meals. He believes there is no point eating
since he is dying. When the doctor reviews him in
Explanation: This patient exhibits
the evening, he says “I was dead in the morning
till afternoon”. Which of the following terms BEST most severe form of delusion of
describe his psychopathology? nihilism and believed that he was
dead. Delusion of nihilism is
A. Delusion of hypochondriasis associated with depression.
B. Delusion of identity
C. Delusion of nihilism

1
D. Delusion of persecution Author: Roger Ho
E. Delusion of reference Year: 2015
Cognitive assessment
5. Which of the following cognitive impairments are The answer is D.
MOST likely to be associated with subcortical
dementia? Explanation: Parietal lobe is
responsible for calculation and
A. Impairment in calculation and mood disorder occipital lobe is responsible
B. Impairment in calculation and visuospatial abilities visuospatial abilities. Temporal lobe
C. Impairment in visuospatial abilities and memory loss is responsible for memory and
D. Difficulty in articulation and mood disorder Broca’s area on the frontal lobe is
E. Memory loss and language problem responsible for language. Dysarthria
and depression are most likely to be
associated with subcortical dementia.

Author: Roger Ho
Year: 2015
Psychiatric epidemiology
6. The percentage of children with autistic disorder The answer is E.
also suffering from mental retardation is?
Explanation: Previous
epidemiological study has shown
A. 10% that 70% of children with autism
B. 20% have mental retardation.
C. 30% Approximately 30% has mild to
D. 50% moderate mental retardation, and
E. 70%. around 40% have severe to profound
mental retardation.

Author: Roger Ho
Year: 2015
Psychiatric aetiology and pathology
7. A 30-year-old schizophrenia patient is interested The answer is D.
in Indian dance. She needs to imagine and act for
different characters (e.g. princess, animal etc)
Explanation: This patient exhibits
during the performance. She has difficulty putting
herself in those imagined situations. Damages in difficulty in abstract thinking as a
which of the following neuroanatomical areas is result of damage in the pre-frontal
MOST likely to cause her difficulty? cortex which is a common pathology
found in schizophrenia.
A. Basal ganglia
B. Occipital lobe Author: Roger Ho
C. Parietal lobe
D. Frontal lobe Year: 2015
E. Temporal lobe.

8. A 30-year-old woman went through multiple The answer is A.


lumbar punctures which caused severe back pain
in the procedure room. Since then, whenever she
Explanation: This phenomenon is
enters the procedure room, she feels pain on her
back. Which of the following BEST explains the best explained by classical
above phenomenon? conditioning. The lumbar puncture is
unconditional stimulus. The
A. Classical conditioning procedure room is conditional
B. Cognitive learning stimulus. Back pain is the conditional
C. Observational learning
response.

2
D. Operant conditioning
E. Secondary gain. Author: Roger Ho
Year: 2015
9. Which of the following neuroanatomical areas is The answer is C.
LEAST likely to be affected in Alzheimer's
disease? Explanation: Nucleus accumbens
is associated with addiction and
A. Basal nucleus of Meynert
least likely to be affected in
B. Hippocampus
C. Nucleus accumbens Alzheimer’s disease compared to
D. Parietal lobe other structures. Basal nucleus of
E. Temporal lobe Meynert produces acetylcholine.
Hippocampus is affected due to
memory impairment in
Alzheimer’s disease.
Hippocampus is found in the
medial temporal lobe. Parietal
lobe is affected and resulted in
constructional apraxia.

Author: Roger Ho
Year: 2015.
10. Based on previous research evidence, which of the The answer is A.
following personality disorders is MOST commonly
seen in an individual who has past psychiatric Explanation: Previous research has
history of conduct disorder? demonstrated that around 40% of
young people with conduct disorder
A. Antisocial personality disorder go on and develop antisocial
B. Borderline personality disorder personality disorder in adulthood.
C. Histrionic personality disorder
D. Narcissistic personality disorder Author: Roger Ho
E. Passive aggressive personality disorder. Year: 2015.

General adult psychiatry


11. A 30-year-old woman suffering from obsessive The answer is B.
compulsive disorder has recurrent thoughts of
harming her husband. She does not have a
Explanation: Patients suffering from
concrete plan. She describes her idea as silly and
she is unlikely to harm her husband. She wants OCD may have thought of harming
to seek help from a doctor. Which of the following others (e.g. poisoning others). It is
course of action is MOST appropriate? important to obtain more details and
perform a proper risk assessment
A. Breach the confidentiality and call her husband before applying the Tarasoff’s rule
immediately.
(e.g. breach of confidentiality) or
B. Explore obsessions and compulsions in more details
and perform a proper risk assessment. send the patient to the IMH for
C. Infuse intravenous clomipramine to stop her assessment.
obsession.
D. Report her case to the police immediately. Author: Roger Ho
E. Stop the interview and send the patient to the Year: 2015.
Institute of Mental Health immediately.

12. Which of the following statements regarding post- The answer is A


traumatic stress disorder (PTSD) is CORRECT:
Explanation: Compensation neurosis
A. Compensation neurosis is an important perpetuating is an important perpetuating factor

3
factor for PTSD symptoms. and maintains PTSD symptoms. It is
B. Dissociation is not seen in patients suffering from important to assess the legal status,
PTSD. especially in PTSD related to road
C. Eye movement desensitization reprocessing (EMDR) traffic accident.
shows the most effective results.
D. The duration of PTSD can be as short as 48 hours Author: Roger Ho
after a traumatic event. Year: 2015
E. PTSD has unique clinical features and patients
cannot suffer from comorbid anxiety or depression.

13. A 50-year-old woman has been going to various The answer is D.


dermatologists requesting examination of her skin.
She complains there are worms crawling under her Explanation: This patients suffers
skin, causing her to feel a weird tingling sensation from delusional infestation or
over her limbs. Despite all investigations being parasitosis, which is a delusional
negative, she is not reassured and insists that the disorder in which she firmly believes
worms are real. Which one of the following that she is infested with living
statements is CORRECT? organisms. Most patients have poor
insight as they believe that their
A. This condition usually runs in family and her family infestations are real. Patients may
members experience similar symptom. have experienced an actual
B. This condition is transient and disappears on its own. infestation or exposure to organisms
C. This patient will respond to high dose of selective in the past and there is usually a
serotonin reuptake inhibitor (SSRI). specific precipitating event.
D. This patient will respond to antipsychotic drug. Antipsychotics are useful to reduce
E. This patient has good insight. the severity of delusions. This is not
body dysmorphic disorder and does
not respond to high dose SSRI.

Author: Roger Ho
Year: 2015
14. Which of the following conditions is least likely to The answer is C.
be a co-morbidity of OCD?
Explanation:
A. Anankastic (Obessive compulsive) personality trait Among all the options, pathological
B. Paediatric autoimmune neuropsychiatric disorder gambling is least likey to be a
associated with Streptococcus comorbidity of obsessive compulsive
C. Pathological gambling disorder (OCD). PANDAS is one of the
D. Pathological hoarding causes of paediatric OCD. Tourette’s
E. Tourette’s syndrome syndrome is common in young
people with OCD (incidence: 5 to 7%;
20-30% have history of tics).
Pathological hoarding is part the
OCD spectrum disorder. Some of the
OCD patients have obsessive
compulsive personality trait.

Author: Roger Ho
Year: 2015

15. Which one of the following statements regarding The answer is C

4
borderline personality disorder is FALSE?
Explanation: Impulsivity in borderline
A. Dialectical behaviour therapy and mentalisation- personality disorder improves
based therapy are evidence-based psychotherapy. significantly over time while affective
B. Environmental factors play more important role than symptoms have the least
genetic factors. improvement.
C. Impulsivity worsens when the patients get older.
D. Projective identification is a common defence Author: Roger Ho
mechanism. Year: 2015
E. The majority of patients are women.

16. Which of the following is the MOST SIGNIFICANT The answer is E.


risk factor predicting suicide in schizophrenia
patients? Explanation: About 10% of
schizophrenia patients commit
A. Discharge from the psychiatric ward 6 months ago suicide and, for most, it happens
B. Employment when they regain their insight when
C. Female gender psychotic symptoms subside. Other
D. Low educational attainment before onset of the risk factors for suicide include male
illness gender, young, unemployment, high
E. Regaining insight when psychotic symptoms subside. educational attainment before onset
of the illness, akathisia, abruptly
stopping medication and recent
discharge from psychiatric ward.

Author: Roger Ho
Year: 2015
17. A 30-year-old woman suffers from depression The answer is B.
due to infertility. Further inquiry reveals that she
suffers from spastic contraction of her vagina
Explanation: This woman suffers
during sexual intercourse. Which of the following
is MOST appropriate management? from vaginismus. It is not uncommon
that woman suffering from
A. Apply dynamic psychotherapy and explore vaginismus is mistaken as a
unconscious conflicts depressed or anxious patient and
B. Apply behaviour therapy and gradual approach of being offered with psychiatric
using dilator of increasing size.
treatments without treating the
C. Prescribe muscle relaxant such as bromazepam.
underlying condition. The most
D. Prescribe selective serotonin reuptake inhibitor such
as fluvoxamine. evidenced treatment is applying
E. Prescribe as norepinephrine-dopamine disinhibitor dilator in a gradual approach as part
such as filbanserin. of the behaviour therapy.

Author: Roger Ho
Year: 2015

Old age psychiatry


18. Which of the following statements is MOST likely to The answer is B.
be impaired in elderly suffering from depression
without dementia? Explanation: Option B refers to
pseudo-dementia, commonly seen in
A. Arithmetic old people suffering from
B. Attention and motivation depression.
C. Naming

5
D. Orientation
E. Visual-spatial organization. Author: Roger Ho
Year:2015

19. A 50-year-old woman had a hysterectomy. Two The answer is C.


days later, she was confused, with disorientation,
fluctuating level of consciousness, paranoid Explanation, If patient suffers from
ideation, and visual hallucinations. Investigations dementia, delusional disorder and
showed negative findings. Five days later, all stroke, the symptoms would not
symptoms have disappeared and her cognitive disappear in such a short period of
function has returned to normal. She has little time. Delirium is the most likely
recollection of events, except that it was like a diagnosis.
dream and she still believes that she saw ghosts in
the ward. The MOST likely diagnosis is: Author: Roger Ho
Year: 2015
A. Brief psychotic disorder
B. Cerebrovascular accident
C. Delirium
D. Delusional disorder
E. Early dementia

20. Memantine is used to treat severe Alzheimer’s The answer is C.


disease. Which of the following statements BEST
describes its pharmacodynamics actions? Explanation: Memantine works on
the glutaminergic and NMDA
A. Memantine increases cholinergic neurotransmission receptors as an antagonist to
B. Memantine reduces dopaminergic neurotransmission improve cognitive function.
C. Memantine reduces glutaminergic neurotransmission
D. Memantine increases noradrenergic
neurotransmission Author: Roger Ho
E. Memantine increases serotinergic neurotransmission Year: 2015

21. Which of the following medication is MOST suitable The answer is C.


to treat psychosis in a 65-year-old man suffering
from Parkinson’s disorder? Explanation: Quetiapine has the least
extrapyramidal side effect compared
A. Chlorpromazine to other antipsychotic drugs and is
B. Haloperidol most suitable in this case.
C. Quetiapine
D. Olanzapine Author: Roger Ho
E. Risperidone. Year: 2015

22. Which of the following sleep problems is MOST The answer is C.


likely to be associated with Lewy body
Explanation:
dementia? REM sleep behaviour disorder is
associated with Lewy body dementia.
A. Bedwetting Option A, B, C and E are associated
with slow wave sleep. Nightmare is
B. Nocturnal leg cramp the only option associated with REM
C. Nightmare sleep behaviour disorder.
D. Night terror
E. Sleep walking Author: Roger Ho
Year: 2015

6
Child and adolescent psychiatry
23. Which of the following childhood psychiatric The answer is E.
disorders has the LOWEST heritability?
Explanation: Genetic factors are
A. Asperger’s syndrome implicated in all of the above
B. Attention deficit and hyperactivity disorder conditions with the exception of
C. Autism school refusal. For ADHD, first-
D. Enuresis degree relatives are at high risk of
E. School refusal. developing ADHD. For autism,
evidence suggests that two regions
on chromosome 2 and 7 contain
genes related to autism. Similarly,
70% of children with enuresis have a
first degree relative who suffered
from enuresis.

Author: Roger Ho
Year: 2015
24. Which one of the following medications is MOST The answer is A.
indicated for a child diagnosed with attention
deficit and hyperactivity disorder (ADHD) but has a Explanation: Atomoxetine is
previous history of Tourette’s syndrome? indicated for treatment of a child
diagnosed with ADHD and with a
A. Atomoxetine previous history of Tourette’s
B. Fluoxetine syndrome. The indications for the
C. Methylphenidate usage of non-stimulants include
D. Risperidone inability to tolerate the side effects
E. Omega-3 fatty acids. associated with stimulants,
unsatisfactory response to
stimulants and history of substance
misuse.

Author: Roger Ho
Year: 2015
25. Which of the following symptoms ONLY occurs in The answer is D.
children with conduct disorder but not oppositional
defiant disorder? Explanation: Children suffering from
oppositional defiant disorder tend to
A. Annoy other people and like to argue have temper tantrums, being angry
B. Being spiteful and defy rules and spiteful, argue with adults, defy
C. Blame others for their own mistakes and refuse to rules and blame others for their
take responsibility mistakes. Children suffering from
D. Damage property and threaten the basic rights of oppositional defiant disorder do not
other people damage properties and threaten the
E. Refuse to go to school and have poor academic basic rights of other people.
performance.
Author: Roger Ho
Year: 2015
Substance abuse
26. A 30-year-old man was admitted to the The answer is C.
medical ward due to epilepsy. Currently, he

7
does not have access to the drug which he Explanation: Midazolam is a
often used prior to admission. He complains benzodiazepine and it is one of the
of insomnia, anxiety and shakiness. He is not benzodiazepine most commonly
depressed. Which of the following being misused by patients due to its
substances is MOST likely to cause the rapid onset of action. The
above symptoms? withdrawal symptoms include
insomnia, anxiety, shakiness and
A. Cocaine withdrawal fit. Withdrawal from
B. Methylphenidate
cocaine will lead to crash or
C. Midazolam
D. Nicotine depression.
E. Quetiapine
Author: Roger Ho
Year: 2015
27. Which of the medication is LEAST likely to be The answer is C.
used to treat patients suffering from substance
abuse in Singapore?
Explanation: Buprenorphine
A. Acamprosate for alcohol dependence and (Subutex) is banned in Singapore
mefenamic acid for opioid withdrawal. after widespread diversion between
B. Bupropion for nicotine dependence and methadone 2004 to 2006. Disulfiram is not used
for opioid dependence worldwide because of potential
C. Buprenorphine for opioid dependence and disulfiram serious cardiovascular complications
for alcohol dependence when mixed with alcohol.
D. Diazepam for benzodiazepine withdrawal and
vareniciline for nicotine dependence
E. Hydroxyzine for opioid withdrawal and naltrexone for Author: Roger Ho
pathological gambling. Year: 2015

28. Which of the following physical conditions is The answer is C.


LEAST likely to be associated with internet
addiction?
Explanation: Internet addiction is
A. Carpal tunnel syndrome associated with orthopaedic
B. Cervical spondylosis conditions including carpal tunnel
C. Hypothyroidism syndrome, cervical spondylosis and
D. Metabolic syndrome low back pain. The reduction in
E. Physical exhaustion and cardiovascular collapse. exercise may lead to metabolic
syndrome. It is not uncommon for
people with internet addiction
collapsed in internet café due to
physical exhaustion and lack of
sleep. Hypothyroidism is not an
aetiology or complication associated
with internet addiction.

Author: Roger Ho
Year: 2015
Psychopharmacology
29. Regarding bupropion, which of the following The answer is B.
statements is FALSE?
Explanation: Bupropion is less likely
A. A common side effect is weight loss. to cause sexual dysfunction as
B. It leads to the same degree of sexual dysfunction as compared to other selective
fluoxetine. serotonin reuptake inhibitors.

8
C. It blocks dopamine reuptake.
D. It is contraindicated for patients with history of Author: Roger Ho
epilepsy. Year: 2015
E. It is used in the treatment of attention deficit and
hyperactivity disorder

30. A 30-year-old depressed man wants to change The answer is C.


his antidepressant from fluoxetine to
moclobemide (monoamine oxidase inhibitor).
Explanation: Fluoxetine has long half
Which of the following advices is MOST
appropriate? –life. It is safer to stop fluoxetine for
several weeks to avoid drug
A. Moclobemide is not available in Singapore interaction with moclobemide and
B. Recommend patient to stop fluoxetine for several this may lead to serotonin syndrome.
days and then switch to moclobemide.
C. Recommend patient to stop fluoxetine for several
Author: Roger Ho
weeks and then switch to moclobemide.
D. Recommend patient to take fluoxetine in the morning Year: 2015
and moclobemide in the evening.
E. Recommend patient to take moclobemide in the
morning and fluoxetine in the evening.

31. A 40-year-old woman complains of depression The answer is B.


and poor sleep. She needs to wake up early in the
morning. Her body mass index (BMI) is 30. She
Explanation: Agomelatine works on
has multiple episodes of suicide attempts. Which
of the following antidepressants is MOST M1/M2 melatonin receptors and
appropriate? 5HT2C receptors. It helps to treat
both depression and insomnia but
A. Amitriptyline does not worsen BMI and does not
B. Agomelatine cause excessive sedation in the
C. Mirtazapine
morning. As a result, agomelatine is
D. Olanzapine
E. Vortioxetine. the best option among all choices.

Author: Roger Ho
Year: 2015
32. Regarding lamotrigine, which of the following The answer is B.
statements is CORRECT?
Explanation: Besides option B, the
A. During the first week of initiation, there should be other options are incorrect. During
rapid increase in dose after initiation to achieve rapid the first week of initiations, slow
control of mood symptoms. introduction of the drug, with gradual
B. Lamotrigine has acute and prophylactic weekly increases in dose will reduce
antidepressant effects in bipolar depression. the risk of rash. Although H.S.A. does
C. The risk of serious rashes appears to be much lower not require genetic test HLA B*1502
in Singaporean patients as compared to other test before initiation for lamotrigine,
countries. this does not mean lamotrigine does
D. The serum levels of lamotrigine are needed to be not cause Steven Johnson
monitored for treatment. Syndrome. Quite a number of local
E. There is no need to conduct genetic test before Singaporean patients develop Steven
starting lamotrigine because Stevens-Johnson Johnson Syndrome after taking
syndrome is not known to occur. lamotrigine.

Author: Roger Ho
Year: 2015
33. Which of the following is the MOST The answer is C.
APPROPRIATE indication for prescribing
clozapine? Explanation: Clozapine may cause

9
agranulocytosis and requires
A. Failed trial of chlorpromazine and sodium valproate frequent full blood count monitoring.
with adequate dose and duration of treatment. A psychiatrist only prescribes
B. Failed trail of fluanxol, a long-acting depot clozapine if a patient fails to respond
antipsychotic drug with adequate dose and duration. to a first generation and second
C. Failed trial of haloperidol and olanzapine with generation antipsychotic drug with
adequate dose and duration of treatment. adequate dosage and duration of
D. Failed trial of risperidone and quetiapine with treatment.
adequate dose and duration of treatment.
E. Recurrent schizoaffective symptoms and suicidal Author: Roger Ho
ideation and does not respond to quetiapine and Year: 2015
fluoxetine

34. Based on the Health Science Authority report The answer is A.


(2013), which of the following drugs have the
highest risk of causing leucopenia?
Explanation: carbamazepine is used
A. Carbamazapine as a mood stabiliser. Besides causing
B. Lamotrigine skin rash, it is associated with
C. Quetiapine leukopenia and requires regular full
D. Risperidone blood count monitoring.
E. Sodium valproate
Author: Roger Ho
Year: 2015

This is an advanced level question.


35. You are a general practitioner. A 30-year-old man The answer is D.
requests you to prescribe diazepam on a daily
basis and he will take diazepam in the afternoon
Explanation: Diazepam is not
to reduce his anxiety. He request 3-month supply
of diazepam. Which of the following is MOST recommended to be used regularly
APPROPRIATE recommendation? in the day time because it will cause
addiction and daytime drowsiness.
A. I agree to prescribe diazepam because it comes with Alprazolam has short half-life and it
2mg tablet which is a low dose and safe for the is more addictive than diazepam.
patient. I will prescribe 3-month supply.
Diazepam is available in tablet form.
B. I agree to prescribe diazepam because it is least
likely to cause withdrawal symptoms.
C. I disagree to prescribe diazepam because the patient Author: Roger Ho
should consider alprazolam which is less addictive Year: 2015
than diazepam.
D. I disagree to prescribe diazepam because its long
half-life will result in excessive daytime drowsiness.
E. I will inform the patient that diazepam is only
available as intravenous route and cannot be
prescribed for outpatients.
36. Compared to other second generation The answer is B.
antipsychotic drugs, risperidone has the
HIGHEST risk of causing which of the following
Explanation: Risperidone has the
side effects?
highest risk of causing amenorrhoea
A. Impaired liver function in female patients by increasing the
B. Increase in prolactin levels of prolactin.
C. Orthostatic hypotension
D. Weight gain Author: Roger Ho
E. Side effects associated with anti-histamine
Year: 2015
receptor

37. A 60-year-old man suffers from manic symptoms, The answer is D.

10
and his family doctor has started a medication to
treat his symptoms. He develops jaundice and Explanation: This patient develops
dark urine after taking the medication. Which of
idiopathic fatal hepatotoxicity after
taking sodium valproate.
the following medications is MOST LIKELY to be
responsible? Author: Roger Ho
Year: 2015
A. Haloperidol
B. Lamotrigine
C. Lithium
D. Sodium valproate
E. Sulpiride.

Psychotherapy
38. A 40-year-old man is admitted to the medical The answer is D.
ward because of upper gastrointestinal tract
bleeding. He has 30-year history of alcohol Explanation: This man tries to justify
dependence. His amylase level is high. During the
interview, he mentions that, „Compared to people
his alcohol dependence with a
who died of famine and war, I am very lucky plausible explanation rather than
indeed. There is no big deal about drinking examining the real cause of upper
alcohol. We should fulfil our dream when we are gastrointestinal bleeding.
alive. I prefer to drink and will continue to drink.‟
What is his defence mechanism? Author: Roger Ho
Year: 2015
A. Acting out
B. Denial
C. Projection
D. Rationalization
E. Reaction formation.

39. A 30-year-old woman was a victim of The answer is B.


childhood abuse. During psychotherapy, she cannot
recall any detail related to her childhood abuse. Explanation: This patient is using
Which of the following defence mechanisms is defence mechanism, repression to
responsible for this phenomenon? forget painful memories
unconsciously.
A. Denial
B. Repression Author: Roger Ho
C. Splitting Year: 2015
D. Projection
E. Projective identification.

40. A woman suffers from depression as a result of The answer is E.


marriage problems. She has been a housewife for
20 years. Her children did well in their studies
Explanation: This patient only
and graduated from universities. She cannot
identify any positive aspect of her marriage but focuses on the negative aspects but
only focus on the negative aspects. Which of the forget the positive aspects of her
following cognitive errors BEST describes her marriage.
view?
Author: Roger Ho
A. Black and white thinking
Year: 2015
B. Catastrophic thinking
C. Overgeneralisation
D. Personalisation

11
E. Selective abstraction

Ethics and laws


The answer is C.
41. You are a resident working in the Accident and
Emergency Department. You have seen a 40-year-
old man suffering from severe depression with Explanation: Up to now, involuntary
strong suicidal ideation. Which of the following hospitalisation can only take place at
statements regarding involuntary hospitalisation the IMH in Singapore and Form 1 is
is correct? only available in IMH but not other
hospitals. Form 1 only allows the
A. After signing Form 1, the patient needs to stay in the
patient to be kept for 72 hours.
psychiatric ward for a minimum of 1 week.
B. Electroconvulsive therapy can be administered
without patient’s consent during involuntary Author: Roger Ho
hospitalisation in Singapore. Year: 2015
C. Involuntary hospitalisation can only be applied at the
Institute of Mental Health in Singapore.
D. Form 1, the form for involuntary hospitalisation can
be found in all Accident and Emergency Departments
in Singapore.
E. Suicide is the only indication for involuntary
hospitalisation in Singapore.

The answer is B.
42. Which one of the following is the BEST predictor of
future violent behaviour? Explanation: A history of past
violence is the best predictor of
A. History of conduct disorder future violence among the 5 options.
B. History of past violence
C. History of substance abuse Author: Roger Ho
D. Low intelligence. Year: 2015
E. Presence of first rank symptoms

43. You are a General Practitioner. You have been The answer is D.
managing a 40-year-old businessman who suffers
from depression and he is undergoing a divorce.
Explanation: Based on the scenario,
His wife, who is not staying with him, calls your
clinic and requests a medical report on behalf of this man is 40-year-old, working as a
her husband. She is very upset that he has businessman and suffering from
prepared a will and decided to give his properties depression, he has capacity to make
to his parents. His wife claims that her lawyer decision. His wife may have hidden
requests for a medical report to certify her agenda for the arrangement of
husband suffers from depression and lacks the
properties. Based on the PDPA, you
capacity to write a will. His wife believes that he
is suicidal but does not want the property to be cannot issue medical report without
transferred to his parents if he would commit her husband’s consent even it is
suicide. She does not communicate with her requested by her lawyer.
husband. Which of the following actions is MOST
appropriate? Author: Roger Ho
Year: 2015
A. Call the patient immediately and sends him to the
nearest Accident and Emergency Department for
assessment.
B. Issue the medical report to his wife and keep the
discussion with his wife confidential.
C. Issue the medical report to his wife because the
medical report is requested by her lawyer.
D. Refuse to issue the medical report and inform his

12
wife that his medical information is protected by the
Personal Data Protection Act.
E. Refuse to issue the medical report and inform his
wife that her husband is protected by the Mental
Capacity Act.

Liaison and neuropsychiatry


44. A 40-year-old man suffers from depression due to The answer is C.
failure in his marriage. He sent a message to his aunt
that he is going to end his life. His aunt called the police Explanation: This man suffers from
and he was found unconscious in his room. There was a carbon monoxide poisoning and
bucket of burned charcoal next to the bed. Which of the delayed neuropsychiatric
following is the MOST SPECIFIC complication complications such as amnesia or
associated with the above method of suicide attempt but Parkinson’s feature is characteristic.
not with other suicide methods?
Author: Roger Ho
A. Confusion Year: 2015
B. Difficulty of coordination
C. Delayed occurrence of amnesia
D. Seizure
E. Death

45. A 30-year-old woman who is 28-week pregnant, The answer is B.


suffers from chronic depression. She is highly
suicidal and wants to jump from height with her
Explanation: This woman has high
foetus because she believes there is no point
living. Which of the following is MOST suicide risk and ECT is the most
appropriate? effective and safest to the foetus.
rTMS, augmentation with olanzapine
A. Augment antidepressant with olanzapine or intensive counselling are not
B. Admission with electroconvulsive therapy effective to prevent severe
C. Induction of labour to prevent harm to her foetus
depression and suicide.
D. Augment antidepressant with intensive antenatal
counselling
E. Repetitive transcranial magnetic stimulation Author: Roger Ho
Year: 2015
46. Which of the following is not a classical feature of The answer is D.
non-epileptic seizure or pseudoseizure?
Explanation: With regards the
A. Asymmetrical seizure movement diagnosis of non-epileptic seizure,
B. Lack of injury psychiatrists should rule out organic
C. Presence of primary and secondary gains causes of seizure first. The classical
D. Raised serum prolactin level immediately after the features of non-epileptic seizure
seizure. include the presence of past
E. Usually when other people are around. psychiatric disorders, presence of
on-going precipitants including
intense current psychosocial
stressors, and the presence of
underlying motives such as primary
and secondary gains. In addition, the
characteristics of non-epileptic
seizure include seizures occurring in
the daytime, usually when other

13
people are around and there is
gradual onset. In addition, there is
the presence of wide range of
atypical presentation such as pelvic
thrusting, lack of tonic and clonic
movements, one-sided
somatosensory symptom and
reactive papillary light reflex.

Author: Roger Ho
Year: 2015
47. Which of the following psychiatric conditions is The answer is B.
most prevalent after a head injury?
Explanation: The most common
A. Anorexia, motor retardation and terminal insomnia psychiatric condition after a head
B. Anxiety, irritability and sensitive to noises injury is post-concussion syndrome
C. Depression, simple auditory hallucination and with a prevalence rate of 50% after 2
paranoia months and 12% after 1 year.
D. Panic attack, phobia and interpersonal rejection Anxiety, irritability and sensitive to
sensitivity noises are symptoms of post-
E. Reactive mood, change in appetite and leaden concussion syndrome. Option A
paralysis suggests melancholia. Option E
suggests atypical depression.

Author: Roger Ho
Year: 2015
48. A 60-year-old woman presents with uncontrollable The answer is C.
episodes of laughing or crying. Which of the
following conditions is LEAST likely to cause such Explanation: This woman suffers
phenomenon? from pathological crying. Common
causes include dementia, multiple
A. Bipolar disorder sclerosis, stroke and severe head
B. Cerebrovascular accident injury. Lability associated with
C. Chronic schizophrenia bipolar disorder may present with
D. Dementia uncontrollable episodes of laughing
E. Multiple sclerosis and crying. Chronic schizophrenia is
least likely to cause this
phenomenon and associated with
blunted affect.

Author: Roger Ho
Year: 2015

49. A 40-year-old woman was admitted to the medical The answer is A.


ward and she was found to suffer from the
syndrome of inappropriate antidiuretic hormone Explanation: Agomelatine was found
secretion Her serum sodium is 110 (normal range: to be free from hyponatremia.
135- 145 mEq/L). She has a past history of Besides SSRI, carbamazapine,
psychiatric illness. The following is the list of her enalapril and thiazide are known to
medications. Which of the following medications is be associated with hyponatremia.
LEAST likely to cause her condition?
Author: Roger Ho

14
A. Agomelatine Year: 2015
B. Carbamazepine
C. Enalapril This is an advanced level question.
D. Fluoxetine
E. Thiazide

50. A 30-year-old woman came to see you because she The answer is A.
worries that her nose is too large and crooked since
the age of 15 years. She is increasingly worried Explanation: This woman suffers
about her nose deformity despite assurance from from body dysmorphic disorder.
various doctors. Which of the following is the first- Patients with this disorder have
line medication for her condition? preoccupation with perceived
defects or flaws in physical
A. Fluoxetine appearance that are not observable
B. Lithium to others. They perform repetitive
C. Lorazepam behaviours or mental acts (e.g.
D. Risperidone comparing their appearance with
E. Valproate that of others) in response to their
appearance concerns. Selective
serotonin re-uptake inhibitors (SSRIs)
antidepressants are first-line
treatment. About 50% of patients
respond to SSRIs but if the response
is poor, augmentation with a second-
generation antipsychotic is
recommended.

Author: Roger Ho
Year: 2015

EMI Paper 13

Extended matching items

Question 51 – 53

15
Options:

A. Alzheimer's dementia
B. Antisocial personality disorder
C. Acute manic episode
D. Exhibitionism
E. Schizophrenia

Lead-in: Select the most appropriate diagnosis from the above for the following clinical
scenario.

51. A 32-year-old Chinese man is arrested after lurking in the sand dunes, exposing his
flaccid penis to sunbathing females in the Sentosa beach. (Choose 1 option) D

52. A 40-year-old Malay man is arrested after running naked down the Orchard Road
shouting that he is the God of Love. (Choose 1 option) C

53. A 65-year-old Indian man is arrested after exposing himself at a local park. His family
reported to the police that he went missing. He could not find his way home and slept in the
local park. (Choose 1 option) A

Question 54 – 57

Options:

A. Catatonia
B. Conversion disorder
C. Elective mutism
D. Psychomotor retardation
E. Thought blocking

Lead-in: Select the most appropriate diagnosis from the above for the following clinical
scenario.

54. A 10-year-old boy is brought by his mother for not speaking to her for the past four
months, following the arrival of a stepfather in his house. He has a history of multiple
parental separations. He studies at a neigbourhood primary school and the school report
shows good academic performance in all subjects. (Choose 1 option) C

55. A 20-year-old lady took some unknown medication to lose weight. She has lost 20 kg
over 3 months but she develops mutism, rigidity, echopraxia and negativism. (Choose 1
option) A

16
56. The speech of a 30-year-old man is so delayed that he begins to reply only when the
interviewer asks the next question. He makes no eye contact and has lost considerable weight.
(Choose 1 option) D

57. A 40-year-old woman suddenly develops an inability to see during a bitter family
argument over her late father's will and properties. Her family bring her to the family doctor.
She herself does not seem distressed and able to find her way out of the clinic. (Choose 1
option) B

Question 58 – 60

Options:

A. Diminished responsibility
B. Fitness to plead
C. Informed consent
D. Lasting power of attorney
E. Torasoff’s rule: Duty to warn

Lead-in: Select the most appropriate concept from the above for the following clinical
scenario.

58. A 20-year-old man tells his therapist that he and her girlfriend are "fated to be together".
He is very upset with her for breaking off with him and plans to stab her. (Choose 1 option)
E

59. A 40-year-old woman assaulted a delivery man whom she believed he was sent by Satan.
The man suffered from mild injury and she was arrested by police. After starting clozapine
during her hospitalisation at the Institute of Mental Health, the patient no longer believes the
victim was sent by Satan. The Court asks if the case can proceed. (Choose 1 option)
B

60. A 60-year-old woman agrees to appoint her son and daughter to be the donees and make
decision for her when she develops dementia. (Choose 1 option)
D

17
MCQ exam 14

History, mental state exam and psychopathology


1. A 25-year-old man came to see you due to The answer is D.
memory problems. He was sponsored by the
Singapore Government to study in a top Explanation: This man is too
University in the United Kingdom. In the young for organic causes of
past 3 months, he cannot remember the name dementia and all investigations are
of the university which he graduated from. normal. He has marked difficulty
He cannot recall any email message. He has with recall of important
started a new job for 3 months. He finds his information that cannot be
co-workers do not respect his qualification. explained by normal forgetting.
He finds people who have just joined the
department have better ability. He realizes
that he does not any interest and hobby. Full Author: Roger Ho
blood count, liver function tests, renal Year: 2015
function tests, thyroid function test, vitamin
B12, folate, syphilis screen and CT brain scan
reveal normal findings. What is the most
likely diagnosis?

A. Conversion disorder
B. Early onset Alzheimer’s disease
C. Fronto-temporal lobe dementia
D. Dissociative amnesia
E. Schizotypal personality disorder.

2. Which of the following is not considered to be The answer is B.


formal thought disorders in schizophrenia?
Explanation: The formal thought
A. Derailment disorders in schizophrenia
B. Echolalia
C. Interruptions in train of thought
comprises of interruption in the
D. Loosening of association train of thought, derailment,
E. Neologism. loosening of association and
neologism.

Author: Roger Ho
Year: 2015
3. A 40-year-old German woman residing in The answer is B.
Singapore reported hearing her neighbour
talking about her for the past two weeks. She Explanation: She suffers from acute
claims two male voices getting angry and psychosis due to stress. She reports
making racist remarks about her and saying pseudohallucinations. As a result,
that she should leave Singapore, while the she has good prognosis and benefit
female voice took her side and defended her. from short course oral antipsychotic
She can hear their voices inside her head and regular follow-up.
everywhere in her house at any time of the
day. She does not have any suicidal or Author: Roger Ho
homicidal ideation. Her other struggles at Year: 2015
this time included stress of taking care of two
children when her husband went overseas.
Which of the following statements is correct?
A. She exhibits negative symptoms of schizophrenia.
B. She has good prognosis.
C. She has poor prognosis.
D. She requires depot antipsychotic treatment.
E. She suffers from paranoid personality disorder.
4. Which of the following subtypes of delusion is the The answer is D.
LEAST frequent?
Explanation: Shared delusion or folie
A. Grandiose a deux is rare because it requires
B. Jealous two persons to become patients.
C. Persecutory
D. Shared Explanation: Roger Ho
E. Somatic. Year: 2015

Cognitive assessment
The answer is C.
5. Which of the following features is more
suggestive of dementia rather than pseudo- Explanation: In genuine dementia
dementia? like Alzheimer’s disease, memory is
worse for recent events. Option A, B,
A. Abrupt onset of cognitive impairment D and E are more suggestive of
B. Depressed mood before onset of cognitive pseudodementia.
impairment
C. Memory worse for recent events as compared to Author: Roger Ho
remote events Year: 2015
D. No confabulation
E. Fluctuation of cognitive ability

6. A 60-year-old man is brought by his wife due to The answer is A


recent changes in behaviour after head injury.
He likes to hug female strangers and causes
Explanation: Frontal lobe
embarrassment. He appears to be overfamiliar
and disinhibited during the interview. Injury in impairment is associated with
which of the following neuroanatomical areas is behavioural changes such as
most likely to be implicated? disinhibition and overfamiliarity
rather than memory loss.
A. Frontal lobe
B. Hippocampus
Author: Roger Ho
C. Parietal lobe
D. Occipital lobe Year: 2015
E. Temporal lobe

Psychiatric epidemiology
7. Which one of the following is LEAST likely to be a The answer is B.
psychiatric comorbidity of trichotillomania?
Explanation: Trichotillomania is the
A. Anxiety disorder compulsion to pull out one’s hair.
B. Antisocial personality disorder The peak age at onset is 12–13 years,
C. Depressive disorder and the disorder is often chronic and
D. Deliberate self-harm difficult to treat Trichotillomania is
E. Obsessive compulsive disorder typically confined to one or two
sites. It most frequently affects the
scalp. Patients tend to be highly
secretive about the condition and to
regard their behaviour as shameful.
The common comorbid psychiatric
disorders in trichotillomania include
mood and anxiety disorder,
obsessive compulsive disorder
substance and eating disorders.
Antisocial personality disorder is not
associated with trichotillomania.

Author: Roger Ho
Year: 2015

Psychiatric aetiology, diagnosis and classification


8. Which of the following statements regarding The answer is D.
hoarding is FALSE?
Explanation: Hoarding can lead to
A. Personal protective equipment (PPE) is required poor hygiene and make home
when performing a home visit of hoarders environment not inhabitable.
B. Hoarding runs in family due to heritability. Hoarding is most commonly found in
C. Hoarding is associated with obsessive compulsive people with OCD. About 18–42% of
disorder. people with OCD report hoarding
D. Hoarding seldom occurs in patients suffering from and saving compulsions. Hoarding
dementia due to their impaired ability to keep items. symptoms show autosomal recessive
E. Hoarding may respond to selective serotonin inheritance pattern. Psychiatric
reuptake inhibitors. comorbidities include schizophrenia,
learning disability, dementia, autism-
spectrum disorders, eating disorder
and impulse control disorder.

Author: Roger Ho
Year: 2015

9. Which of the following disorders is LEAST likely to The answer is D.


be associated with the development of autism?
Explanation: Women with Turner’s
A. Fragile X syndrome syndrome have very mild intellectual
B. Phenylketonuria disability and near normal
C. Neurofibromatosis intelligence. West syndrome refers
D. Turner’s syndrome to an infantile spasm which is a
E. West syndrome. specific type of seizure seen in an
epilepsy syndrome of infancy and
childhood. All of the above disorders
have been found to be associated
with the development of
autism except option E.

Author: Roger Ho
Year: 2015
10. All of the following is the MOST IMPORTANT The answer is E.
protective factor against suicide in Singapore?
Explanation: Religion is most
A. Low social class due to lower social expectation important protective factor against
B. Male gender due to commitment to look after family suicide in Singapore. For suicide, the
C. Overinvolved spouse who looks after patient all the male to female gender ratio is 3:1
time and the rate of women is rising.
D. People suffering from borderline personality There is higher risk in the low and
disorder who often harm themselves instead of high social classes and lower risk in
killing themselves the middle social class. Suicide rates
E. Having faith in a religion which condemns suicide in the divorced and widowed are
higher than that of people who are
married. Unemployment, low
socioeconomic status, and certain
occupations such as bar owners,
doctors, and pharmacists are risk
factors. Suicide attempts occur after
significant life events are common
among people with poor social
support and living alone.

Author: Roger Ho
Year: 2015

11. Which of the following is MOST likely to cause The answer is C.


relapse of schizophrenia after hospitalisation?
Explanation: Option C refers to high
A. Return to work in the office after discharge expressed emotion. Previous research
B. Return to stay with family which has financial work done by Vaughn and Leff found
problems an increased relapse rate of
C. Return to stay with family who are overinvolved and schizophrenia in those who lived
critical with families which displayed high
D. Return to stay with family who has strong religious levels of expressed emotions.
belief
E. Return to study in the university after discharge Author: Roger Ho
Year: 2015

General adult psychiatry


12. Which of the following is NOT a positive The answer is E.
prognostic factor for a patient who has
recovered from the first episode of Explanation Young males have
schizophrenia? poor prognosis as compared to
older females. Option A, B and C
A. Acute onset are good prognostic factors.
B. Family history of a mood disorder
C. Presence of a precipitating factor Author: Roger Ho
D. Older female
E. Young male. Year: 2015

13. You are a medical officer working in the The answer is B.


Singapore Armed Force. A 19-year-old male is
recently enlisted for National Service and his
Explanation: This man suffers from
parents inform you that he often develops
sudden loss of muscle tone and excessive narcolepsy. Among all the options,
daytime sleepiness. The patient also complains fluoxetine is most appropriate by
of hearing voices when he falls asleep. Which of suppressing rapid eye movement
the following management recommendations is sleep.
MOST appropriate?
Author: Roger Ho
A. Amitriptyline
Year: 2015
B. Fluoxetine
C. Relaxation exercise
D. Sleep deprivation
E. Supportive psychotherapy.
14. Which of the following is NOT a poor prognostic The answer is E.
factor for borderline personality disorder?
Explanation: Chronicity of symptoms,
A. Chronicity of symptoms early childhood sexual abuse, early
B. Early childhood sexual abuse first psychiatric contact and high
C. Early first psychiatric contact affective instability are poor
D. High affective instability prognostic factors of borderline
E. High level of intelligence personality disorder.

Author: Roger Ho
Year: 2015

15. A 30-year-old man works in the Changi The answer is B.


airport. He likes to steal uniform and
underwear of air stewardess from their Explanation: Fetishism is defined as
luggage because he has intense sexually recurrent and intense sexual arousal
arousal when masturbates with their from the usage of non-living objects
clothing. One week ago, he was arrested by (i.e. uniform and underwear of air
the airport police for stealing a luggage which stewardess). He stole the item for his
belongs to an air stewardess. What is the sexual needs. As a result, his case is
most likely psychiatric diagnosis? not considered to be kleptomania
because the stole items which serve
A. Automatism a purpose for this patient.
B. Fetishism
C. Masochism Author: Roger Ho
D. Kleptomania Year: 2015
E. Voyeurism

16. A 30-year-old woman is undergoing a divorce. The answer is C.


She seeks attention compulsively. Her husband
cannot tolerate her acting out and hysterical
Explanation: This woman suffers
scenes when she fails in achieving her role as a
mother. She is markedly dysphoric and from histrionic personality disorder
uncomfortable at the centre of attention. She has as evidenced by self-dramatization,
to be the centre of the party. During the shallow and labile affectivity,
interview, she laughs in the beginning and cries continual seeking for excitement and
towards the end of the interview. What of the
following is MOST LIKELY psychiatric discomfort when she is not centre of
diagnosis? attention.

A. Borderline personality disorder Author: Roger Ho


B. Dependent personality disorder Year: 2015
C. Histrionic personality disorder
D. Narcissistic personality disorder
E. Obsessive compulsive personality disorder

17. A 30-year-old woman called Anita was brought in The answer is C.


by her family to the Accident and Emergency
Department. The family reported that another Explanation: The patient called Anita
woman called Cindy has taken over her. She exhibited discontinuity of sense of
wanted to strangle her boyfriend when she was self and changes in consciousness
Cindy. In the Accident and Emergency and memory. There were recurrent
Department, she was unconscious for 30 minutes. episodes for 6 months. The most
After she woke up, she could not recall anything appropriate diagnosis is dissociative
related to Cindy. Similar episodes have occurred identity.
more than 30 times in the past 6 months. Which
of the following diagnosis is MOST relevant? Author: Roger Ho
Year: 2015
A. Dissociative amnesia
B. Dissociative fugue
C. Dissociative identity
D. Dissociative stupor
E. Ganser’s syndrome

Old age psychiatry


18. A 70-year-old man suffering from severe The answer is B.
depressive disorder requests for electroconvulsive
therapy. Which of the following statements is Explanation: Older people have
true? higher seizure threshold and require
higher energy level to induce seizure.
A. General anaesthesia should be avoided due to his Old people require GA as young
old age. people. Male gender is associated
B. His age increases the seizure threshold. with higher seizure threshold.
C. His gender decreases the seizure threshold. Bilateral ECT is associated with more
D. The doctors should stop antidepressant prior to ECT cognitive impairment and the
to avoid confusion. doctors should not stop
E. Unilateral ECT causes more memory impairment as antidepressants prior to ECT as this
compared to bilateral ECT. will not cause confusion. Patients
need to continue antidepressants
during ECT.

Author: Roger Ho
Year: 2015

19. Which of the following would be most helpful for a The answer is C.
patient suffering from delirium after liver
transplant? Explanation: Prednisolone is
associated with delirium and
A. Being placed in a room far away from the nursing psychosis. Amitriptyline is associated
station with anticholinergic effect.
B. Being placed in a shared room with four other
patients Author: Roger Ho
C. Decreasing the dose of prednisolone Year: 2015
D. Increasing the dose of amitriptyline
E. Increasing the dose of anticholinergic medications

The answer is D.
20. You are seeing a 60-year-old cleaner. He
complains of the following, “I am losing my Explanation: He suffers from mild
ability to remember and learn new information. cognitive impairment as evidenced
I forgot that today is my wedding anniversary by the following:
until my wife tested me. I put the peanut butter
1. MMSE >24
in the fridge when it should actually be left on
the dining table. I tried reading the newspaper 2. Subjective or objective
but could not understand. I got confused over memory loss
the PIN number for my ATM card and needed 3. ADL: independent
to write it down. I am able to function in my 4. Amnesia type of MCI
work”. He scored 25 out of 30 in the Mini 5. He is able to function at
Mental State Examination (MMSE). What of the work.
following is the most appropriate diagnosis?
Author: Roger Ho
A. Alzheimer’s disease
B. Fronto-temporal lobe dementia Year: 2015
C. Generalised anxiety disorder
D. Mild cognitive impairment
E. Pseudo dementia
21. A 77-year-old woman suffering from major The answer is E.
depressive disorder. She was prescribed with
sertraline. The dose of sertraline was titrated up Explanation: This elderly woman
to 150mg per day. One week later, she presents suffers from hyponatremia. Low
nausea and malaise, lethargy, decreased level of sodium levels result in intracerebral
consciousness and headache. She is afebrile. osmotic fluid shifts and brain
Which of the following investigation is MOST oedema. As a result, she complains
RELEVANT? of headache.

A. Creatinine kinase level Author: Roger Ho


B. CT Brain Scan Year: 2015
C. Electrocardiogram
D. Liver Function Tests
E. Sodium Level

22. A 6 8- yea r- old ret ire d man is concerned The answer is E.


about t he f inancial s it uat ion at home. He
w as brought in by his w ife to the
Explanation: This case is modified
Ac c ident and Emerg enc y Department. He
sent SM S messages t o his w ife that he from a real tragedy which an elderly
has suicidal t houghts. His son does not man committed suicide with his
underst and w hy he is sui cid al as he grandson by jumping from height.
spends most of his t ime looking after his Medical students and future doctors
13- mont h old grandson . H is daughter - in- need to appreciate the potential risk
is t he sole b read w inner and the famil y
in elderly. They need to understand
has a lot of debt s . T he patient likes his
grandson v ery much. T hey all sta y in a 4 - the case beyond superficial facts to
room HDB f lat . Bot h pat ient and his son avoid future tragedy. The elderly
st rongly r ef use admi ssion and treatment . man may commit suicide with his
W hich t he f ollow ing st at ements is M O ST grandson because he may believe
appropriat e ? that no one looks after his grandson
after his suicide. Elderly men are at
A. His s uic id e r is k is low bec aus e th e pat ien t does high risk of committing suicide due
not thi nk he needs tr e atm ent. to genuine financial problem as his
B. His s uic id e r is k is low bec aus e h is grands on is
a protec ti ve f ac tor . daughter in law is the sole
C. His s uic ide r is k is low bec aus e h e s ta ys with h is breadwinner. The explanations
f am il y in th e s am e HDB f lat and t he y of f er good stated in Option A and C are not
s upport. protective factors against suicide.
D. His s uic ide r is k is high d ue to r et irem ent. Retirement is not a risk factor for
E. His s uic id e r is k is high a nd he m a y c om m it suicide.
s uic ide wi th h is gr ands on.
Author: Roger Ho
Year: 2015

Child and adolescent psychiatry


23. Regarding autism, all of the following statements The answer is D.
are true EXCEPT:
Explanation: The onset of autism is
A. Behavioural interventions will not lead to around 3 years of age.
improvement in cognitive and social functioning.
B. Intelligence is a predictor factor for prognosis. Author: Roger Ho
C. The male to female ratio is about 3:1. Year: 2015
D. The onset is around 10 years of age.
E. Thirty percent of patients suffering from autism have
seizures.

24. Which the following is the MOST common age of The answer is A.
onset for oppositional defiant disorder?
Explanation: The prevalence of
A. 8-year- old oppositional defiant disorder has
B. 10-year-old been estimated to be between 6 and
C. 12-year- old 16 percent. The age of onset of
D. 14-year-old oppositional defiant disorder is
E. 16-year-old usually before the age of 10 years.
This differs from that of conduct
disorder. The age of onset of
conduct disorder begins earlier in
boys (10-12 years) as compared to
girls (14-16 years).

Author: Roger Ho
Year: 2015

25. Which of the following is the MOST common cause The answer is A.
of intellectual disability?
Explanation: Down syndrome is the
A. Down syndrome most common cause of intellectual
B. Prader-Willi syndrome disability, with the risk increasing
C. Foetal alcohol syndrome with maternal age. It accounts for
D. Fragile X syndrome 30% of all children with intellectual
E. Turner syndrome. disability. In contrast, fragile X
syndrome is the most common
inherited cause of intellectual
disability. Women with fragile X
syndrome suffer from mild
intellectual disability, while men
suffer from moderate to severe
intellectual disability.

Author: Roger Ho
Year: 2015

26. Which of the following is MOST likely to be The answer is D.


associated with juvenile delinquency?
Explanation: Juvenile delinquency is
A. Conduct problems in parents when they were younger defined as law-breaking behaviour
B. Family history of depression by young people aged 10 to 20 years.
C. High intelligence The aetiology is multifactorial and is
D. Parental criminality not associated with an established
E. Small family size. psychiatric disorder. Factors
associated with the development of
delinquency include unsatisfactory
child rearing, low IQ, conduct
disorder in childhood, parental
criminality and large family size.
Approximately 50% will have
stopped their delinquent behaviour
by age 19.

Author: Roger Ho
Year: 2015

Substance abuse
27. According to the Central Narcotics Bureau, The answer is C.
possession and consumption of the following
drugs would NOT lead to imprisonment or fine in
Explanation: Methadone is partial
Singapore?
opioid agonist and an opioid
A. Buprenorphine substitute for patients who are
B. Ketamine addicted to heroin. Although it is not
C. Methadone illegal to possess or consume
D. Methamphetamine methadone, it is only prescribed by
E. Nimetazepam.
addiction specialists at the National
Addiction Management Service,
Institute of Mental Health.

Author: Roger Ho
Year: 2015

28. A 25-year-man is brought by his wife because he The answer is B.


seems to be addicted to the internet and online
gaming. His wife is the sole breadwinner for the
Explanation: This is a modification
family. This man is unemployed and he has a 2-
year-old son. All the following clinical features from a real case. In this real case, the
strongly suggest the diagnosis of internet young son suffers from starvation
addiction EXCEPT: due to neglect from his father who is
addicted to online game. Addiction
A. Continue to focus on online game despite long term may not just to neglect to oneself
unemployment but others including young children.
B. Compulsion to upload his photos onto the Facebook This is an important risk assessment.
with frequent updates
C. Poor care for his son. Option B alone does not qualify for
D. Spending a large amount of money to trade weapons the diagnosis of internet addiction
with other players online and it is more common in people
E. Uneasiness when he has no access to internet or with narcissistic personality.
online game
Author: Roger Ho
Year: 2015

29. A 28-year-old part-time female model has been The answer is A.


using an illicit drug to keep her body weight low
and makes her thin. She refuses to disclose the
Explanation: Among all the options,
name of drug. Which of the following drugs is
MOST likely to achieve the above effect? amphetamine is most likely to be
associated with weight loss.
A. Amphetamine
B. Cannabis Author: Roger Ho
C. Magic mushroom Year: 2015
D. Heroin
E. Phencyclidine

30. A 23-year-old university student presents to the The answer is A.


emergency department with psychotic features.
His parent suspects he might use illicit drugs Explanation: Explanation: Psychotic-
during his overseas exchange. Which of the like state results from acute or
following clinical features suggest schizophrenia chronic consumption of illicit drugs. It
rather than drug- induced psychosis? leads to paranoia, hallucinations and
sometimes a delirium-like state. In
A. Alogia contrast to paranoid schizophrenia,
B. Amotivation drug-induced psychosis is associated
C. Confusion with visual hallucinations,
D. Paranoia hyperactivity, hyper-sexuality,
E. Visual hallucination appropriate affect and confusion.
Formal thought disorder and alogia
are not found in drug-induced
psychosis. Amotivational syndrome
occurs in both schizophrenia and
cannabis-abuse.

Author: Roger Ho
Year: 2015

Psychopharmacology
31. A 30-year-old man suffers from erectile The answer is C
dysfunction and requests to take sildenafil. You
are reviewing his medication lists. Which of the Explanation: Nitroglycerin is a
following medications would lead to the MOST nitrate. Sildenafil and nitrate both
dangerous drug interaction?
cause the muscles that control the
size of blood vessels to relax.
A. Fluoxetine
B. Metformin
When these muscles relax, the
vessels enlarge in diameter and
C. Nitroglycerin
may lead to hypotensive crisis.
D. Sodium valproate
E. Simvastatin
Author: Roger Ho
Year: 2015
32. Which of the following side effects is specific for The answer is C.
venlafaxine but not fluoxetine?
Explanation: Venlafaxine is
A. Dry mouth
B. Nausea associated with dose-related
C. Hypertension hypertension. Option A, B, D and E
D. Sexual dysfunction are associated with both fluoxetine
E. Risk of serotonin syndrome and venlafaxine.

Author: Roger Ho
Year: 2015

33. You are a gastroenterologist. You are seeing a The answer is C.


51-year-old woman who suffers from mixed
anxiety and depression. She also suffers from
Explanation: Due to 5HT3
functional dyspepsia. She complains of
recurrent nausea and gastric discomfort. Which antagonism, mirtazapine is not
of the following antidepressants is MOST associated with nausea, which is a
suitable for her? common side effect of
antidepressants listed in other
A. Escitalopram options.
B. Fluvoxamine
C. Mirtazapine
D. Sertraline Author: Roger Ho
E. Vorticoxetine Year: 2015

34. Which of the following psychotropic medications The answer is E.


is MOST likely to cause renal calculi in countries
with hot weather such as Singapore? Explanation: Previous research has
found that topiramate has increased
A. Carbamazapine the risk of renal calculi. This may
B. Lamotrigine affect Singaporeans when they are
C. Olanzapine dehydrated in hot weather.
D. Sodium valproate Topiramate is used as a mood
E. Topiramate. stabiliser by psychiatrists or migraine
medication by neurologists.

Author: Roger Ho
Year: 2015

This is an advanced level question.


35. A 25-year-old woman is treated by bupoprion for The answer is D.
depressive disorder. She gives you a list of
symptoms and attributes these symptoms to the
Explanation: Bupoprion is not
side effects of bupoprion. Which of the following
symptoms is LEAST likely to be side effect of associated with sexual dysfunction.
bupoprion?
Author: Roger Ho
A. Anxiety Year: 2015
B. Nausea
C. Seizure
D. Sexual dysfunction
E. Weight loss.

36. You are seeing a 50-year-old with history of The answer is A.


intravenous drug abuse and hepatitis C. He
suffers from liver failure and psychotic
Explanation: Agomelatine is a new
depression. Which of the following psychotropic
medication is LEAST appropriate? antidepressant which works on
melatonin agonist and 5HT2C
A. Agomelatine antagonist. It is associated with
B. Lithium
C. Paliperidone possible liver impairment and liver
D. Sulpiride function test is mandatory before
E. Venlafaxine. and shortly after initiating
agomelatine. Options B to E are
excreted by kidney.

Author: Roger Ho
Year: 2015

37. A 33-year-old woman is 20-week pregnant and The answer is A.


suffers from major depressive disorder. Which of
the following antidepressants is safest for her
Explanation: The oldest medication,
foetus based on medical evidences?
like tricyclic antidepressant has a lot
A. Amitriptyline of data and associated with least
B. Bupropion teratogenic effect as compared to
C. Escitalopram other psychotrophic medications.
D. Quetiapine
E. Trazodone Author: Roger Ho
Year: 2015

38. A 60-year-old man suffering from behaviour and The answer is C.


psychiatric symptoms associated with dementia
was admitted to the hospital one week ago. He
Explanation: Among all options,
was prescribed with a number of psychotropic
medications. The QTc interval was 500 ms on haloperidol is most likely to be
admission. It was reduced to 430 ms after the associated with prolonged QTc.
psychiatrist removed one of the psychotropic Option A, B, D and E have no effect
medications. Removal of which of the following on prolonged QTc.
medications is most likely to reduce the QTc
interval?
Author: Roger Ho
A. Fluoxetine Year: 2015
B. Gabapentin
C. Haloperidol This is an advanced level question.
D. Lorazepam
E. Sodium valproate

Psychotherapy
39. A 30-year-old woman with history of shoplifting The answer is E.
and recurrent self-harm due to anger and
frustration towards her foster parents. After
Explanation: The best answer is
release from the prison, she has applied for
volunteer programme. She wants to turn her sublimation because she expressed
anger and hatred to help needy people. Which of her unacceptable impulses (e.g.
the following BEST describes her defence anger and hatred) in an acceptable
mechanism? way (i.e. volunteer work).
A. Projection
Author: Roger Ho
B. Rationalisation
C. Reaction formation Year: 2015
D. Repression
E. Sublimation
40. A 32-year-old schizophrenia patient wants to The answer is D.
receive psychotherapy. Which of the following
types of psychotherapy is LEAST suitable for
Explanation: Schizophrenia patients
this patient?
suffer from hallucinations and
A. Cognitive behaviour therapy delusions which make them confuse
B. Interpersonal psychotherapy with reality. Schizophrenia patients
C. Problem solving psychotherapy will be confused by interpretations
D. Psychodynamic psychotherapy. offered by the psychotherapist.
E. Supportive psychotherapy
Author: Roger Ho
Year: 2015

41. A woman suffered from gestational diabetes and The answer is A.


received multiple insulin injections five years
ago. She also developed recurrent depressive
Explanation: Guilt and hopelessness
episodes after argument with her husband
during her pregnancy. As a result, she attributed are not recognised side effects of
guilt and hopelessness to insulin injection. insulin. In this example, she
Recently, the polyclinic doctor reviews her associated insulin and depressive
diabetic status. She does not show satisfactory symptoms such as guilt and
response to oral hypoglycaemics. The polyclinic hopelessness through classical
doctor suggests insulin. She strongly refuses
conditioning. The conditional
and claims that insulin caused guilt and
hopelessness based on her past experience. stimulus was argument with her
What is the MOST logical explanation? husband. The unconditional stimulus
was insulin injection. The conditional
A. Classical conditioning response was depressive symptoms
B. Operant conditioning such as guilt and hopelessness.
C. She has cognitive error such as selective abstraction
When her arguments with husband
D. Social learning
E. The association between insulin and bad feeling is paired with insulin injection, she
due to chance. associated insulin injection with
conditional responses which are
depressive symptoms.

Author: Roger Ho
Year: 2015

This is an advanced-level question.


Ethics and laws
42. A 35-year-old man, Mr. Tan was arrested for The answer is E.
assaulting his brother because his brother
refused to lend him money. Other family Explanation: He is a psychopath who
members describe Mr. Tan as an has no remorse with recurrent
irresponsible person who treats his brother offences. He is a dangerous person
like an “ATM machine”. Mr Tan has an and even assaults his family
extensive criminal record. 17 year ago, he members. People with psychopathy
tried to abduct a 10-year-old boy and
may exhibit grandiosity.
requested SGD 500,000 from his parents. He
was jailed for 15 years. He failed to learn
from his mistakes and seemed to be Author: Roger Ho
grandiose. In the prison, he attempted to Year: 2015
bribe prison officers to smuggle drugs into
the prison. Which of the following term best
describes his condition?
A. Antisocial personality disorder
B. Kleptomania
C. Malignant narcissistic personality disorder
D. Pseudologia fantastica
E. Psychopathy

43. A 30-year-old man suffering from schizophrenia The answer is C.


and thought of harming other people is admitted
to the Institute of Mental Health (IMH) under the
Explanation:
Form 1, Mental Health (Care and Treatment) Act.
His family wants to know the maximum period
which he will be kept at IMH under Form 1. Your Form 1: Maximum period: 72 hours
answer is: Form 2: 1 month
Form 3: 6 months.
A. 24 hours
B. 48 hours
Author: Roger Ho
C. 72 hours
D. 96 hours Year: 2015
E. 110 hours.
44. You are a resident working in the medical ward. The answer is A.
A 35-year-old male patient assaulted a nurse in
the ward. He was admitted due to a break-
Explanation: The patient would have
through seizure. Your consultant wants a
psychiatrist to assess this man. Which is the assaulted the nurse due to confusion
following is the MOST appropriate reason for associated with seizure. The legal
referral? defence is known as automatism and
the patient was not aware of his act
A. Assessment for automatism due to influence of epilepsy.
B. Assessment for depression
C. Assessment of his capacity
D. Assessment of his compliance to anticonvulsant Author: Roger Ho
E. Assessment of his insight. Year: 2015

Liaison and neuropsychiatry


45. Which of the following medical conditions is most The answer is D.
likely to present in a way which is MOST similar
to panic disorder? Explanation: Mitral valve prolapse
is characterised by irregular
A. Acromegaly heartbeat, dizziness,
B. Hypothyroidism lightheadedness, shortness of breath
C. Lung cancer and most resembles panic disorder
D. Mitral valve prolapse among all options. Other medical
differential diagnoses of panic
E. Pituitary gland tumour disorder include hypoparathyroidism,
phaeochromocytoma, COPD,
asthma, hypoglycaemia, diabetes,
thyrotoxicosis and anaemia.

Author: Roger Ho
Year: 2015
46. Which of the following neuropsychiatric sequelae The answer is A.
is more likely to occur after carbon monoxide
poisoning as compared to post-concussion Explanation: Option B to E are
syndrome? shared by both carbon monoxide
poisoning and PCS. Agnosia is more
A. Agnosia common in carbon monoxide
B. Dizziness poisoning.
C. Headache
D. Lethargy Author: Roger Ho
E. Slow mental processing speed. Year: 2015

47. You are a neurology resident and managing The answer is C.


psychotic symptoms of an epileptic patient. Which
of the following antipsychotic drugs is LEAST likely The recommended antipsychotic
to be epileptogenic based on medical evidences? drugs include that of haloperidol,
trifluoperazine and sulpiride. In
A. Chlorpromazine contrast, chlorpromazine, depot
B. Clozapine antipsychotics, clozapine and
C. Haloperidol zotepine are contraindicated in
D. Quetiapine epileptic patients. In particular,
E. Zotepine clozapine has been known to be
most epileptogenic among all
antipsychotic drugs. The evidence
for new antipsychotic drug is not as
strong as old antipsychotic drug like
haloperidol.

Author: Roger Ho
Year: 2015
48. Regarding depression in patients suffering from The answer is D.
Parkinson’s disease, which of the following is
LEAST likely to be an important predisposing Explanation: Female gender rather
factor? male gender is a predisposing factor
for depression in Parkinson’s
A. Early onset disease.
B. Family history of depression
C. Loss of functional independence Author: Roger Ho
D. Male gender Year: 2015
E. Presence of cognitive impairment.

49. Which of the following statements about The answer is B.


depression and ischemic heart disease (IHD) is
FALSE? Explanation: After an acute
myocardial infarction, major
A. Approximately 20% of patients with IHD suffer from depressive disorder predicts
comorbid depression. mortality in the first 6 months. The
B. After an acute myocardial infarction, major impact of depressive episode is
depressive disorder does not predict mortality. equivalent to the impact of a
C. Autonomic disturbances in depression may lead to previous infarct. 20% of patients
heart rate changes and arrhythmias. suffering from IHD have comorbid
D. Depression affects compliance to medical treatment depression. Major depressive
E. History of major depressive disorder is an disorder is an independent risk
independent risk factor for IHD. factor for IHD. Depression also
affects compliance to cardiac
treatment and cause autonomic
disturbance due to underlying
serotonin dysfunction.
Author: Roger Ho
Year: 2015

50. The medical team has diagnosed several depressed The answer is B.
patients presenting proximal muscle weakness as
somatisation disorder. The medical team did not Explanation: Hypothyroidism is
order any blood test prior to their diagnosis and associated with proximal muscle
could have missed important medical diagnosis. weakness and depression among all
Which of the following diagnosis is MOST likely to options. The medical team should
be associated with the above presentation? order thyroid function test and
conduct thorough investigations
A. Amphetamine withdrawal before concluding somatisation
B. Hypothyroidism disorder.
C. Insulinoma
D. Phaeochromocytoma Author: Roger Ho
E. Rheumatoid arthritis. Year: 2015

Extended Matching Items (51 – 60)

Question 51 – 54

Options

A. Amygdala
B. Broca’s area
C. Cerebellum
D. Corpus callosum
E. Tuberoinfundibular system

Lead-in: Select the most appropriate neuroanatomical areas from the above for the
following signs and symptoms.

51. Depressive disorder (Choose 1 option) Answer: A

52. Non-fluent aphasia (Choose 1 option) Answer: B or no answer: Give one mark

53. Galactoorhoea (Choose 1 option) Answer: E

54. Marchiafava bignami diseases (Choose 1 option) Answer: D

Question 55 – 57

Options

A. Anorexia nervosa
B. Delusional disorder
C. Obsessive compulsive disorder
D. Munchausen syndrome by proxy
E. Schizophrenia
Lead-in: Select the most appropriate clinical diagnosis from the above for the following
scenarios.

55. An 8-year-old girl is admitted due to diarrhoea and weight loss. Her mother has been
feeding her with laxative and hopes that she is admitted to the hospital as a patient.
(Choose 1option) Answer: D

56. An 18-year-old man is admitted for weight loss. He says he cannot eat because he has
heard multiple voices warning him that the demons are poisoning his food. He has
undergone spiritual warfare with the demons for 6 months and the demons are taking
his thoughts away. (Choose 1 option) Answer: E

57. A 28-year-old woman worries that all the food are contaminated by radiation and
antimicrobial drugs in agriculture. She has been washing her food in multiple of
twenty. As a result, she does not have time to eat and results in weight loss. (Choose
1 option)
Answer: C

Question 58 – 60

Options

A. Alcohol
B. Cannabis
C. Heorin
D. Solvent
E. Tobacoo

Lead-in: Select the most appropriate substances from above for the following situations.

58. Which type of addiction has the highest rate of mortality worldwide? (Choose
1 option) Answer: E

59. Which type of addiction is most likely to be associated with blood-borne viral
infection? (Choose 1 option) Answer: C

60. Which type of addiction is most likely to be abused by secondary school


students in Singapore? (Choose 1 option) Answer: D

Author: Dr. Roger Ho


Question 1
A 50-year-old man is admitted to the ward. He complains that there are a lot of bugs on
his skin and he needs to wash hands excessively. His wife wants to consult you how to
differentiate an obsession from a delusion.

1. Define an obsession (2 marks). Examiner’s use only: / 2 marks


Answer may vary. The examiner is advised to use his or her discretion to decide whether marks should be
awarded.
Obsession is a
 recurrent and persistent thoughts, impulses, or images that are experienced as intrusive and
inappropriate
 they cause anxiety and distress
 they are not simple excessive worries about real-life problems
 the person attempts to ignore or suppress the thoughts, impulses, or images, or to neutralize them
with some other thought or action
 the person recognizes them as products of his/her own mind.

2. Define a delusion (2 marks). Examiner’s use only: / 2 marks


Answer may vary. The examiner is advised to use his or her discretion to decide whether marks should be
awarded.
Delusion is a
 a false belief, based on an incorrect inference about external reality.
 the belief cannot be corrected by reasoning.
 the patient firmly believes that the false belief is true (close to 100% true).

3. List TWO (2) features that differentiate an obsession from a delusion (4 marks).
Examiner’s use only: / 4 marks
Answer may vary. The examiner is advised to use his or her discretion to decide whether marks should be
awarded.
Differences between obsessions and delusions:
 obsessions are ego-dystonic (disharmony with one’s own ego) while delusions are ego-syntonic (in
harmony with one’s own ego) 2 marks or
 obsessions involve intact reality testing, whereas in delusions reality testing is impaired (2 marks)
or
 obsessions cause anxiety and distress, whereas patients with delusion are usually calm (2marks)
or
 obsessions involve compulsive actions to neutralize them, whereas delusions do not (2 marks)
 obsessions are not held with absolute certainty whereas delusions are (2 marks).
This is an advanced – level question.
4. List TWO (2) types of evidence-based and commonly used treatment for obsessive compulsive
disorder (2 marks).
Examiner’s use only: / 2 marks

 Selective serotonin reuptake inhibitors or name of SSRI e.g. fluoxetine (1 mark); antidepressant (0.5
mark); other (0 mark)
 Cognitive behaviour therapy (1 mark); Exposure and response prevention (0.5 mark); Other (0 mark)
Administrator’s use only (Total): Administrator 1: / 10 marks Administrator 2: /10 marks
Question 3
A 3-year-old boy has delayed language development. He is of normal height and weight, and has no
obvious physical abnormalities.

1. List THREE (3) main clinical features which you would look for to support the diagnosis of
autistic disorder (3 marks)
Examiner’s use only: / 3 marks
Answer may vary. The examiner is advised to use his or her discretion to decide whether marks should be
awarded.
1. Restricted interests or hobbies or repetitive behaiour.
2. Communication difficulties (both verbal – language delay and non-vebal – lack of eye contact, failure
to understand emotional cues)
3. Stereotyped movements or social interaction difficulty.

2. List TWO other causes for his poor language skills besides autism (2 marks).
Examiner’s use only: / 2 marks
Answer may vary. The examiner is advised to use his or her discretion to decide whether marks should be
awarded. Any two of the following:
1. Maltreatment or abuse
2. Mental retardation
3. Neurological injury
4. Hearing impairment
5. Specific language impairment (such as expressive language disorder)
6. Genetic disorders (Fragile X syndrome)

3. Name THREE (3) investigations which you order to establish the underlying cause for language
delay. (3 marks)

Examiner’s use only: / 3 marks


Answer may vary. The examiner is advised to use his or her discretion to decide whether marks should be
awarded. Any two of the following

1. Intelligence quotient (IQ) testing


2. Diagnostic interview for Autism
3. Electroencephalogram (EEG)
4. Hearing assessment
5. Speech assessment
6. Genetic testing (e.g. look for fragile X site or non tandem repeats in X chromosome.

4. Name TWO (2) non-pharmacological interventions which are useful for autism. (2 marks)

Examiner’s use only: / 2 marks


Answer may vary. The examiner is advised to use his or her discretion to decide whether marks should be
awarded. Any two of the following
1. Behaviour therapy (e.g. intensive behaviour intervention)
2. Training to use alternative communication
3. Visual strategies or visual support for communication
4. Social skill training
5. Special education
6. Speech therapy
Administrator’s use only (Total): Administrator 1: / 10 marks Administrator 2: /10 marks
Question 4
You are a medical resident. Your hospital does not have a psychiatric ward and a 40-year-old woman
presenting with catatonia was admitted last night. No investigation and treatment was ordered.

1. List THREE (3) characteristic features of catatonia (3 marks)

Examiner’s use only: / 3 marks


Any three of the following:

 Motoric immobility as evidence by cataplesy (or waxy flexibility) or stupor


 Excessive Motoric Activity (purposeless activity, not influenced by external stimuli)
 Ambitendency
 Automatic obedience
 Negativism (as evidenced by motiveless resistance to all instructions or maintenance of a rigid posture
against attempts at being moved)
 Mutisn
 Stereotyped movements, prominent mannerisms or prominent grimacing)
Echolalia
 Echopraxia.

2. Name TWO (2) psychiatric disorder commonly associated with catatonia.(2 marks)

Examiner’s use only: / 2 marks


Any of the following:

 Schizophrenia
 Bipolar disorder
 Depressive disorder
 Substance abuse with acute intoxication of recreation drugs.

3. At NAME TWO (2) medical disorders that are known to be associated with catatonia. (2 marks)

Examiner’s use only: / 2 marks


Any of the following:

 Central nervous system infection: meningitis and encephalitis


 Central nervous system tumour
 Cerebrovascular accident
 Lethal catatonia
 Acute poisoning

4. List ONE (1) most important investigation which you would order to identify the underlying
neurological cause for catatonia.(1 mark)

Examiner’s use only: / 1 mark

Magnetic Resonance Imaging (MRI) brain scan (1 mark); Computerized Tomography (CT) brain scan
(1 mark); other answer (0 mark)

5. List ONE (1) pharmacological treatment for this patient. (1 mark)

Examiner’s use only: / 1 mark


Benzodiazepine or lorazepam (1 mark)
6. Name ONE (1) non-pharmacological management order for this patient (1 mark)

Examiner’s use only: / 1 mark

Any of the following:


1. Hydration
2. Nutrition
3. Early mobilization when condition is stable
4. Close observation and frequent monitoring for vital signs.
5. Transfer to ICU if patient deteriorates.

ECT = 0 mark as it involves general anaesthesia (pharmacological)

Administrator’s use only (Total): Administrator 1: / 10 marks Administrator 2: /10 marks


SAQ (Paper 2)
Question 1

You are a General Practitioner. A 30-year-old woman comes to your clinic and she complains of low
mood. You suspect that she suffers from depressive disorder. She read a lot of information on the internet
and has her personal views on treatment. She would like to discuss with you on various treatment options.
1. List Four (4) pieces of information which you would inquire from the history to support the
decision for immediately prescribing an antidepressant to this patient (4 marks).
Answer may vary. The examiner is advised to use his or her discretion to decide whether marks should be
awarded.
1. The severity of depression: her symptoms are severe (e.g. with the presence of suicidal thought and a
lot of biological symptoms).
2. Lack of precipitant in her depression: her depression seems to be endogeneous and it is not a reactive
depression.
3. Presence of family history of depression: suggest biological cause of depression.
4. Previous response to antidepressant or previous relapse after stopping antidepressant.

5. Severe impairment in functioning.


6. Failure of psychotherapy.
7. Past history of depression.
8. Patient’s preference
2. The patient would like to know more about psychotherapy. Name TWO (2) types of
psychotherapy with MOST evidence to demonstrate effectiveness for her condition (2 marks).
1. Cognitive behaviour therapy
2. Interpersonal therapy
Other types of psychotherapies (0 mark)

3. Based on your answer in question 2, select ONE (1) type of psychotherapy and explain
how it works. (4 marks)

Answer may vary. The examiner is advised to use his or her discretion to decide whether marks should be
awarded.

For students who chose to explain CBT:

The frequency of Cognitive behaviour therapy (CBT) is usually weekly or fortnightly. It requires 12 to 16
sessions. The cognitive therapy involves identifying negative automatic thoughts and use dysfunctional thought
diary to identify pattern between the time, events, negative thoughts and resulted emotions and behaviours. The
psychologist will read the diary and help patients to question the negative automatic thoughts. Behaviour
therapy involves activity scheduling (for those depressed patients with psychomotor retardation), relaxation
techniques (for those patients with mixed anxiety and depression).
For students who chose to explain IPT:
Interpersonal therapy (IPT) is held weekly or fortnightly. It involves 12 to 20 sessions. IPT is indicated for
depressed patients whom precipitating factor is interpersonal problems. The psychologist closely examines
interpersonal relationship and relates depression to the interpersonal event. The therapist helps the patient to
work on one of the interpersonal problem areas: 1) grief; 2) interpersonal role dispute; 3) role transition; 4)
interpersonal deficits. IPT helps the patient to develop new skills to deal with people and life situations
associated with depressive symptpms. IPT works with the patient to look at interpersonal relationship from
another angle to minimise impact on the mood. The therapist also uses role-play to improve communication
skills.

1
Question 2

You are a general practitioner and looks after a 30-year-old woman who is 14-week pregnant.
She has been drinking alcohol and has poor compliance to antenatal follow-up. Besides alcohol,
she does not use other drug or substance. She feels that it is completely harmless to drink
alcohol during pregnancy. You need to offer psychoeducation to her.

1. What is the name of the syndrome which is of MOST concerned to her foetus? (2 mark)

Foetal alcohol syndrome

2. List THREE (3) facial features associated with the syndrome described in Question 1. (3
marks)

Answer may vary. The examiner is advised to use his or her discretion to decide whether marks should be
awarded.
Any three of the following:

 Small head
 Small eyes
 Short nose
 Ear lobe abnormality
 Short palpebral fissures (measured from inner to outer canthus; reflect underlying brain growth)
 Thin upper lip
 Long and flat philtrum

3. If her baby is born with the syndrome described in question 1, list THREE (3)
psychiatric impairments which may occur during the childhood of her offspring (3
marks)

Answer may vary. The examiner is advised to use his or her discretion to decide whether marks should be
awarded.
Any three of the following:
 intellectual impairment
 attention deficit or hyperactivity
 difficulty with reasoning
 depression
 anxiety
 oppositional defiant or conduct disorder

4. State TWO (2) specialities which you would refer this patient to consult for her current
problems. Please indicate the reasons for referral. (2 marks)

Specialities Reasons for referral

2
Specialities Reasons for referral
Obstetrics and gynaecology (0.5 mark) Frequent ultrasound monitoring for foetal
abnormalities (0.5 mark)

Psychiatrist or addiction specialist (0.5 mark) For detoxification and treatment of alcohol
dependence (0.5 mark)

Other specialist: no mark.

For the reasons of referral, the answer may vary. The examiner is advised to use his or her discretion to decide
whether marks should be awarded.

This is an advanced-level question.

3
Question 3

You are a general practitioner (GP) and seeing a 65-year-old man who complains of memory loss.
He also suffers from depression. His daughter has read information from the internet that he is at
risk of developing Alzheimer’s disease due to his age. She is not certain about the impact of
depression on his memory.

1. Name FOUR (4) differences between dementia and pseudodementia (4 marks)

Dementia Pseudodementia
Example: Caused by Alzheimer’s disease Caused by depression
Aetiology

1.

2.

3.

4.

Answer may vary. The examiner is advised to use his or her discretion to decide whether marks should be
awarded.

Any 3 of the following categories:

Dementia Pseudo-dementia
Onset Insidious onset (1 mark) Acute onset (1 mark)

Progression Slow progression (1 mark) Rapid progression (1 mark)

Disability Minimize disability (1 mark) Emphasize on disability (1 mark)

Extent of memory Recent memory loss more severe. Equal loss of recent and remote events.
loss (1 mark) (1 mark)

Consistency Consistent pattern of memory loss Variable pattern of memory loss (1


(1 mark) mark)
Insight/ performance Deny memory loss or confabulate Focus on cognitive loss(1 mark) or

4
during Mini Mental to fill up memory gap Don’t know answer (1 mark)
State Examination Minimize disability (1 mark)

Variation within a Sun-downer effect (worse in the Diurnal variation (worse in the
day evening) (1 mark) morning) (1 mark)

Personal history or No or less likely (1 mark) Yes or more likely (1 mark)


family history of
depression

3. You have assessed the patient and he suffers from depression. You have decided to start
fluoxetine. Name ONE (1) common electrolyte abnormality found in old people taking
SSRI. (2 marks)

Hyponatremia.

4. Four months later, this patient has tried fluoxetine and it does not work. He took
benzodiazepine from another GP for insomnia. He develops suicidal thought. You have
referred this case to a geriatric psychiatrist. The geriatric psychiatrist recommends
electroconvulsive therapy (ECT). Based on the clinical information provided, name TWO (2)
factors which may increase the seizure threshold for this patient (2 marks) during the ECT.

Any 2 of the following:

1. Old age
2. Male gender
3. Baldness
4. Usage of benzodiazepine.

This is an advanced-level question.

5. His daughter is concerned about memory impairment after ECT. State TWO (2)
adjustments which can apply during the administration of ECT to reduce the risk of
cognitive impairment.

Any 2 of the following:

1. Unilateral ECT
2. Use lower energy level
3. Reduce frequency of ECT (two times per week instead of three times per week)

This is an advanced-level question.

5
Question 4

You are a specialist working in renal medicine. A 60-year-old woman suffers from chronic
renal failure and she requires life-long hemodialysis. She wants to discontinue dialysis. It is a
long weekend and the psychiatrist is not available in your hospital. You need to determine her
capacity to make a decision to discontinue dialysis.

1. List FOUR (4) questions which you would ask this patient to assess her capacity to
make a decision to discontinue dialysis (4 marks).

Answer may vary. The examiner is advised to use his or her discretion to decide whether marks should be
awarded. Any three of the following:

1) What is the nature of your medical condition? / What has the treatment team told you about your condition?/
What do you believe is wrong with your health?

2) What is the purpose of the dialysis? / What have you been told about the recommended treatment?

3) Can you tell me the benefits of dialysis? / What have you been told about benefits of dialysis?/ What is the
treatment likely to do for you?

4) Can you tell me the risks/side effects of dialysis? / What have you been told about the risks or discomforts
associated with dialysis?

5) What happens if you do not get dialysis?

6) Are there alternative to dialysis? / What have you been told about alternatives to dialysis?

7) Why do you want to discontinue dialysis? / What have you been told about risks and benefits of no
treatment?/ How did you decide to discontinue dialysis?

2. You are reviewing her past psychiatric record. State FOUR (4) psychiatric disorders
that may affect her capacity to give consent.(4 marks)

Answer may vary. The examiner is advised to use his or her discretion to decide whether marks should be
awarded.

Any three of the following:

1. Dementia (any form: Alzheimer’s disease, vascular dementia)


2. Delirium or acute confusional state
3. Schizophrenia
4. Severe depressive disorder with psychotic features (1 mark); depression (0.5mark)
5. Acute psychosis
6. Delusional disorder
7. Bipolar disorder with psychotic features (1 mark); bipolar disorder/mania/hypomania (0.5 mark)
8. Mental retardation

3. If this patient is free from psychiatric illness and has the capacity to make decision. You respect
her right to make a decision to discontinue dialysis. State the MOST RELEVANT ethical
principle which support your decision. (2 marks)

Respect patient’s autonomy

6
SAQ (Paper 3)

Question 1 (Total = 10 marks)

You are a general practitioner (GP). You are asked to see a 36-year-old woman for the treatment of major depression.
A trial of fluoxetine 40mg every morning by another GP was unsuccessful.

She was very upset with the remarks made by her colleagues. As a result, she quitted her job 6 months ago and
became a housewife. She feels bored as a housewife and has conflicts with her in-laws. She has become more
depressed in the past 6 months. Her husband is supportive and there is no marital conflict. She has a 10-year-old son
and he has good relationship with her. She does not need to worry about his studies as he has done well in school.
She has other chronic medical illnesses and she takes other medications. She forgot to bring the medication and
cannot remember the name of those medications.

She has no history of substance abuse or dependence. Her ECG is normal.

1. State THREE (2) MOST RELEVANT points of the history that you would like to clarify regarding previous trial
of fluoxetine? (2 marks) / 2 marks

Any two of the following:

1) Her previous adherence or compliance to fluoxetine (1 mark) or


2) Side effects associated with fluoxetine (1 mark) or
3) The duration of taking fluoxetine (1 mark).

2. State TWO (2) medications which are used to treat chronic medical illnesses and known to be associated
with depression? (2 marks) / 2 marks

Any two of the following:

1) Corticosteroids / steroids/ prednisolone (1 mark) or


2) Beta-blockers / atenolol / propranolol (1 mark) or
3) Clonidine (1 mark) or
4) Metoclopramide (1 mark) or
5) Theophylline (1 mark) or
6) Calicium channel blockers: Nifedipine (1 mark).

3. Based on the patient’s background, state THREE (3) common medical conditions from different body
systems that is known to be associated with depression. (3 marks) / 3 marks

Answers may vary. Examiners are advised to use his or her own discretion to decide whether marks should be
awarded. Any three of the following:
1) Cardiovascular disorders: hypertension (1 mark)
2) Endocrine disorders: diabetes, hypothyroidism or Cushing syndrome or Addison’s disease (1 mark)
3) Neurological disorders: epilepsy (1 mark)
4) Gastrointestinal system: peptic ulcer or irritable bowel syndrome or inflammatory bowel diseases (1 mark)
5) Immune system: systemic lupus erythematosus (1 mark)

Medical disorders which are more common in older adults or elderly e.g. Parkinson’s disease, cerebrovascular accidents,
chronic obstructive pulmonary disease = 0 mark. Myocardial infarction = 0 mark as ECG is normal. If all answers are from one
single system = 1 mark.

4. Based on the patient’s history, state ONE (1) MOST appropriate and evidence-based psychotherapy that is
suitable for this patient. (1 mark) / 1 mark

Interpersonal psychotherapy

Other psychotherapies = 0 mark.

5. Based on the patient’s history, state how the psychotherapy recommended by you (in Question 4) works for
her. (2 marks) /2 marks

Answers may vary. Examiners are advised to use his or her own discretion to decide whether marks should be
awarded. For students who did not state interpersonal psychotherapy in Q.4, they should only get a maximum of 1
mark for question 5.

Intepersonal psychotherapy can help the patient to analyze previous interpersonal problems in her work place.
Interpersonal psychotherapy involves role play and patient can improve her communication with her in-laws. The role
play can identify previous maladaptive communication patterns. Interpersonal psychotherapy can help her to deal
with the role transition from an administrative person to become a housewife.
Question 2 (Total = 10 marks)

You are a general practitioner. A 25-year-old man suffering from schizoaffective disorder is bought to the clinic by her mother. He
was diagnosed to suffer from schizoaffective disorder 6 months ago. Her mother is concerned about his recent weight gain of 10 kg
in 3 months. According to her mother, his psychiatrist from a restructured hospital started a new antipsychotic and mood stabilizer
to control his psychiatric symptoms.

1. State TWO (2) antipsychotics which are MOST likely to cause weight gain in this patient. (2 marks) /2 marks

Any two of the following:


 Olanzapine (1 mark) or
 Clozapine (1 mark) or
 Quetiapine (1 mark) or
 Risperidone (0.5 mark)
 Other antipsychotics = 0 mark

2. State TWO (2) mood stabilizers which are MOST likely to cause weight gain in this patient. (2 marks) /2 marks

Any two of the following:


 Lithium (1 mark) or
 Sodium valproate (1 mark) or
 Lamotrigine (1 mark) or
 Other mood stabilizers: no mark

3. State THREE (3) MOST relevant blood tests which you would like to order and abnormalities you would like to rule out for
this patient. (3 marks) / 3 marks

Investigations Abnormalities
/0.5 mark /0.5 mark

/ 0.5 mark / 0.5 mark

/ 0.5 mark / 0.5 mark

Investigations Abnormalities
Fasting glucose (0.5 mark) Rule out diabetes (0.5 mark)

Fasting lipid / cholesterol / HDL / Rule out hyperlipidaemia (0.5 mark)


LDL (0.5 mark)

Liver function tests (0.5 mark) Rule out raised AST and ALT / fatty liver / liver impairment
(0.5 mark)

Thyroid function test Rule out hypothyroidism and associated weight gain

4. State ONE (1) MOST RELEVANT AND SPECIFIC symptom in schizophrenia which may contribute to obesity (2 marks).
1. Avolition or ahehdonia or lack of interest to do exercise (2 marks)
2. Negative symptom (1 mark)
3. Other negative symptom e.g. apathy, alogia, attentional deficit (0 mark)

5. State ONE (1) antipsychotic that has the LOWEST RISK to cause weight gain. (1 mark)
Aripiprazole/ haloperidol (1 mark). Other antipsychotic = 0 mark
Question 3

You are a paediatric resident working in the Child and Accident Emergency Department. You
are seeing an 11-year-old boy who was brought in by his foster parents. According to his
foster parents, he has a lot of behavioural problems at home and school. He cannot sit still to
do his homework at home. He also breaks school rules. Tonight, he had a fight with his foster
parents. This boy is a victim of child abuse.

1. State TWO (2) MOST RELEVANT diagnoses (2 marks) that this boy may suffer from and
indicate THREE (3) additional clinical features (6 marks) which you will look for to
support your diagnosis. (total 8 marks)

Diagnosis 3 additional clinical features per diagnosis


1. 1.

2.

3.

/1 mark /3 marks

2. 1.

2.

3.

/1 mark / 3 marks

Diagnosis 3 additional clinical features per diagnosis


1. Attention deficit Any THREE of the following:
hyperactivity disorder (1  Starts tasks or activities but not able to follow through and
mark) finish.
Abbreviation = 0 mark  Organization of tasks or activities is impaired.
 Loses things necessary for tasks and activities such as
school assignments or stationary.
 Instructions are not followed.
 Distraction by external stimuli.
 Careless mistakes
 Forgetfulness in daily activities
 Waiting for in lines or await turns in game cause
frustration.
 On the move most of the time such as running and
climbing.
 Restlessness and jitteriness.
 Squirms on seat.
 Impulsivness
o Talk excessively without appropriate response to social
constraints.
o Fidgets with hands and feet.
o Answers are blurted out before questions.
o Interruption of other people’s conversations.
o Loud noise in playing.

2. Conduct disorder Any THREE of the following:


 Repetitive and persistent pattern of behaviour in which
either the basic rights of the others or major age
appropriate societal rules are violated
 Often telling lies and breaking promises
/1 mark  Refusing adults’ requests or defying rules
 Staying out after dark against parental prohibition
 Blaming others for his mistakes
 Initiating fights with the others
Oppositional defiant disorder = 0 mark  Using weapons to harm the others
 Exhibiting physical cruelty (also to animals)
 Confronting victims during a crime
 Forcing another person into sexual activity
 Frequently bulling the others.
 Truancy
 Involvement in gang.

/ 3 marks

2. If you have a chance to speak to the child protection officer, state TWO (2) MOST
relevant psychiatric disorders that his parents may suffer from. (2 marks)

/ 2 marks

Any two of the following:

1) Antisocial personality disorder (1 mark)


2) Substance abuse (1 mark)

Adult ADHD = 0 mark (It is unlikely MFS needs to foster out the child if parent has adult ADHD).
Question 4

You are a geriatric resident. A 70-year-old man presents with dementia. His son wants to find out
more about the treatment for dementia.

1. State the name of TWO (2) acetylcholinesterase inhibitors available in Singapore. (2 marks)
/ 2 marks
Any two of the following:
Donepezil (1 mark) or
Rivastigmine (1 mark) or
Galantamine (1 mark).

2. List TWO (2) types of dementia for which acetylcholinesterase inhibitors are indicated (2
marks) / 2 marks

Any two of the following:


Alzheimer’s disease (1 mark)
Lewy body dementia (1 mark)

3. List TWO (2) COMMON side effects of acetylcholinesterase inhibitors (2 marks)


/ 2 marks

Any two of the following:


 Diarrhoea (excessive cholinergic effects) 1 mark or
 Difficulty in sleeping (excessive cholinergic effects) 1 mark or
 Dizziness 1 mark or
 Feeling agitated 1 mark or
 Headache 1 mark or
 Loss of appetite 1 mark or
 Muscle cramps (excessive cholinergic effects) 1 mark or
 Tiredness 1 mark.
Uncommon side effects like gastrointestinal bleeding or bradycardia (0 mark)

4. List TWO (2) SPECIFIC contraindications to the use of acetylcholinesterase inhibitors (2


marks)
/ 2 marks
Any two of the following:
 Severe dementia or Score of Mini Mental State Examination < 10 or 12 out of 30 (1 mark)
 Bradycardia or taking beta blockers (1 mark)
 Gastrointestinal bleeding or peptic ulcer (1 mark)

Allergy to acetylcholinesterase inhibitors: not specific enough (0 mark)

5. His son wants to know potential benefits that can be expected from treatment with
acetylcholinesterase inhibitors. List TWO (2) potential benefits (2 marks).
/ 2 marks

Answers may vary. Examiners are requested to use his or her own discretion to decide whether
marks should be awarded.

1. Reduce the rate of deterioration of memory loss or improve cognition (1 mark)


2. Delay nursing home placement (1 mark)
3. Improve behavioural and psychiatric disorders associated with dementia (1 mark)
SAQ (Paper 4)

Question 1

You are the resident working in the Accident and Emergency Medicine Department. A 50-year-
old man is admitted as a result of paracetamol overdose. He was treated in the Institute of
Mental Health for alcohol dependence 7 years ago.

1. State Four (4) clinical features which you would elicit in this man during history taking
to establish the diagnosis of alcohol dependence. (4 marks) Examiner: /4 marks

Any 4 of the following criteria for substance dependence: WASTED/ CAGE


o Withdrawal
o Amounts of alcohol intake (large amount for long periods)
o Social activities replaced by time spent acquiring substance
o Tolerance: higher amount of alcohol to achieve the same effect
o Efforts to cut down unsuccessful
o Despite adverse consequences, keeps drinking alcohol

o Feel Annoyed by criticisms of other people on drinking


o Feel Guilty about drinking
o Eye opening: drinking alcohol after wake-up in the morning

2. State TWO (2) medical or surgical disorders from two different body systems which will
result from prolonged alcohol usage. (2 marks) Examiner: /2 marks

Body system Specific medical or surgical disorder (not sign/symptom)


/1 mark

/1 mark

Any 2 of the following (The list is not completed and examiner will use his or her discretion to decide
whether marks should be awarded).

 CNS complications: withdrawal fits/epilepsy, head injury as a result of fall, cerebellar damage
 Gastrointestinal system: cancer in the oral cavity (e.g. tongue, throat), oesophagitis, gastritis,
gastric cancer.
 Hepatobiliary system: pancreatic cancer, liver cirrhosis, alcohol hepatitis.
 Peripheral nervous system: peripheral neuropathy
 Rheumatology: gouty arthritis
 Reproductive system: erectile dysfunction.

3. State TWO (2) physical signs which you will elicit from two different body systems to
support the diagnosis of alcohol dependence. (2 marks) Examiner: /2 marks

Body system Specific sign


/1 mark

/1 mark
Any 2 of the following (The list is not completed and examiner will use his or her discretion to decide
whether marks should be awarded).

Endocrine system: gynaecomastia


Hepatobiliary system: spider angiomata/navei; ascites; signs of liver cirrhosis: hard liver
Central nervous system: cerebellar ataxia (unsteady and wide-based gait)
Rheumatology: painful big toe
Peripheral nervous system: less of peripheral sensation to different stimuli (e.g. pin or cotton wool)

4. Please explain the following laboratory finding (2 marks).

AST: 600 IU/L


ALT: 150 IU/L

This patient suffers from alcohol hepatitis due to AST: ALT ratio > 2.
Question 3

You are a general practitioner. A 33-year-old woman delivered a boy two


months ago. She is single and she broke up with her boyfriend recently. This is her first
pregnancy and it was unplanned. The infant was delivered by normal vaginal delivery. She is
unemployed and receives social assistance ($300 per month). She is not close to her family
members and stays alone in a 1-room HDB flat. She was seen by a psychiatrist at the Institute
of Mental Health 3 years ago and she did not return for further appointment. She complains of
low mood, poor sleep, poor appetite, poor energy and hopelessness for the past 6 weeks. You
suspect that she suffers from postnatal depression.

1. From her history, list FOUR (4) specific risk factors for postpartum depression. (4 marks)
Examiner: /4 marks

Any three of the following:


1. Single mother (the mother is not that old in this case).
2. Unplanned pregnancy
3. Recent break up/Unemployment / financial difficulties / low socio-economic status.
4. Poor support from family
5. History of depression
6. Negative confinement
7. Past psychiatric history

2. State TWO (2) MOST important psychotic features which you would elicit as part of the risk
assessment (2 marks) Examiner: /2 marks

Answer may vary. Examiner will use his or her discretion to decide whether marks should be
awarded.
1. Command hallucination which asks her to harm herself and her infant.
2. Delusion which suggests that the infant is malformed/abnormal or possessed by demons and
she need to attack the infant.

Other mood congruent delusions e.g. delusion of guilt or nihilistic delusion 0.5 marks

3. She has an appointment to see a psychiatrist in one month. She cannot wait and wants to
seek your opinion. She is very keen to continue breastfeeding. If she wants to take an
antidepressant such as a selective serotonin reuptake inhibitor (SSRI) and continues
breastfeeding, state TWO (2) advices to minimize SSRI exposure to the infant. (2 marks)
Examiner: / 2 marks

Answer may vary. Examiner will use his or her discretion to decide whether marks should be
awarded. Any two of the following:

1. Take the lowest effective dose of a SSRI


2. Take SSRI once per day instead of twice per day.
3. Breastfeed or pump out breast milk immediately after or just before taking the SSRI
4. Discard the first round of breast milk which may contain SSRI.
5. Take SSRI just before the longest period of sleep of the infant.
6. Combination of breast milk and formula milk.
7. Use SSRI with short half live.

Other answers:

Name specific SSRI without reasoning (0 mark)


SSRIs are harmful to the infant and she must not take SSRI (0 mark)
She must give up breastfeeding (0 mark)
4. If the patient does not want to take antidepressant, please recommend TWO (2) alternative
treatment options. (2 marks) Examiner: / 2 marks

1. Psychotherapy e.g. cognitive behaviour therapy or other forms of psychotherapy.


2. Electro-convulsive therapy

Suggest other medications: 0 mark

Question 4

You are a general practitioner. A mother brings her 13-year-old girl to see you. She complains
of low mood, poor sleep, poor appetite and recurrent self-harm. She often cuts her wrist with a
pen knife. She has difficulty to cope with her study in Secondary 1. She often brings a pen
knife to school and cuts herself in the toilet. Her mother breaks into tear in the middle of the
interview because her husband is dependent on alcohol and has history of violence. The
patient is very close to her mother. As a result, the patient and her mother have no
communication with her father.

Question 1: Her mother worries about her personality development. State ONE (1) personality
disorder which she will MOST likely develop if the above pattern continues without any
intervention? (2 marks) Examiner: / 2 marks

Answer: Borderline personality disorder (2 marks). Other diagnosis: no mark.

Question 2: Based on your answer in Question 1, state TWO (2) MOST relevant comorbid
psychiatric disorders. (2 marks) Examiner: / 2 marks

Any two of the following:


1. Bulimia nervosa
2. Depressive disorder
3. Post-traumatic stress disorder
4. Substance abuse or dependence

Question 3 Her mother hopes that you can prescribe a psychotropic medication to
stabilize her. State ONE (1) psychotropic medication which is most suitable to her. (2 marks)
Examiner: / 2 marks

Class of medication Specific name of medication

/1 mark /1 mark

Class of medication Specific name of medication


Selective serotonin reuptake inhibitor Fluoxetine

/1 mark /1 mark

Other answer: 0 mark.

Question 4 Besides cognitive behaviour therapy, state TWO (2) MOST relevant
psychotherapy which will benefit this patient. (2 marks)
Examiner: /2 marks
Any two of the following:
1. Dialectical behaviour therapy
2. Metalization based therapy
3. Interpersonal psychotherapy
4. Family therapy.

Question 5 Her mother is concerned about her safety in the next few weeks while waiting for
the effect of the medication. List TWO (2) MOST relevant strategies to manage safety issues in
this patient. (2 marks) Examiner: /2 marks

Examiner has to use his or her own discretion to decide whether marks should be awarded. Any two
of the following:

1. Discuss need for her mother and teacher to monitor for changes in behaviour or self-harm at
home and in school.
2. Remove sharp objects, weapons or pen knives or unnecessary medications from home and
school.
3. Provide information and access to crisis services in the emergency department.
4. Her mother needs to supervise and administer antidepressant to patient.
5. Frequent and regular review by doctors to monitor response and side effects
SAQ 2014 EOPT – First rotation

Question 1 (10 marks)


A 24-year-old female executive consults her family physician complaining that she has
faced problems at work for the past year. It started after a presentation when her slides
were mixed-up and she was at a loss of words. Since then, she feels tremulous and
stammers at presentations. She is increasingly fearful of doing presentations, worrying
that she may faint or vomit in front of her colleagues. Her fears have grown to the
extent that she has not gone to work for 2 weeks.

1. What is the most likely diagnosis? (1 mark)

Social phobia (1 mark) or diagnosis: 0 mark including agoraphobia. ( /1 mark)

2. List TWO (2) medications from different classes that can treat her symptoms? (4
marks)

Class of medications Name of the medication

( /1 mark) ( /1 mark)

( /1 mark) ( /1 mark)

Class of medications Name of the medication


Selective serotonin reuptake inhibitor 1. Paroxetine(1 mark) or
(SSRI) (1 mark) 2. Fluvoxamine (1 mark) or
3. Fluoxetine (1 mark) or
4. Escitaloporam (1 mark) or
5. Sertraline (1 mark)
Beta-blocker (1 mark) 1. Propranolol (1 mark)

Benzodiazepine (1 mark) 1. Alprazolam (1 mark)


2. Clonazepam (1 mark)
3. Lorazepam (1 mark)

3. She wants to receive psychotherapy. Please state the most evidence based
psychotherapy based on your diagnosis. (1 mark)

1
Cognitive behaviour therapy; other psychotherapy (0 mark) ( /1 mark)

4. She wants to know more how psychotherapy works. Based on your answer in
Question 3 and her clinical history, list TWO (2) specific strategies which can
help her. (4 marks)

Other logical answers are acceptable. Examiner is advised to use his or her discretion to
decide whether marks should be awarded.

Any two of the following:

1. Provide cognitive explanation for social phobia (2 marks) or


2. Exposure to simulated situations e.g. presentation with the psychologist (2 marks) or
3. Cognitive restructuring to teach control of maladaptive thoughts (e.g. humiliation,
vomiting in front of others). (2 marks) or
4. Social skill training (2 marks)

( / 4 marks)

2
Question 2 (10 marks)

A 40-year-old woman with a bipolar disorder was stable on sodium valproate till a
year ago when she suffered three episodes of relapses. Her psychiatrist recommends
switching to lithium. She uses recreational drugs, sometimes forgets to take her
medication when she is clubbing till 5 am and has had unprotected sex many times in
the past three months. A friend asks her to borrow $10,000 to invest in a food and
beverage business.

Question 1 She wants to know the systemic side effects of lithium when the dose is within
therapeutic range. State FOUR (4) body systems affected by lithium and Name ONE (1) specific
side effect of lithium under each system. (4 marks)

Systems Specific side effects

( /0.5 mark) ( /0.5 mark)

( /0.5 mark) ( /0.5 mark)

( /0.5 mark) ( /0.5 mark)

( /0.5 mark) ( /0.5 mark)

Other logical answers are acceptable. Examiner is advised to use his or her discretion to
decide whether marks should be awarded. Signs of lithium toxicity e.g. ataxia, coarse tremor,
cardiac arrest, respiratory difficulty does not count.

Systems Specific side effects

Endocrine system Hypothyroidism

( / 0.5 mark) ( /1 mark)

Renal system Renal failure or


Polyuria

( /0.5 mark)
( /1 mark)

Dermatological system Psoriasis

( /0.5 mark) ( /1 mark)

Gastrointestinal system Metallic taste or nausea

( /0.5 mark) ( /1 mark)

Nervous system Tremor

3
( /0.5 mark) ( /1 mark)

Question 2 Based on her history and the possibility of starting lithium, List THREE (3) most
relevant laboratory investigations (3 marks)

( / 3 marks)

Any three of the following:


1. Renal function tests (1 mark) or
2. Thyroid function test (1 mark) or
3. Urine drug screen (1 mark) or
4. Urine pregnancy test (1 mark) or
5. HIV test / screen for sexually transmitted diseases (1 mark) or
6. Valproate level (1 mark)

Question 3 Based on her current lifestyle, offer THREE (3) MOST relevant advices. (3
marks)

( / 3 marks)

Any three of the following:


1. Regular sleep time and wake-up time (1 mark) or
2. Avoid unprotected sex (1 mark) or
3. Avoid overspending or borrowing money or foolhardy investment (1mark) or
4. Avoid alcohol or recreational drug (1 mark) or
5. Ensure compliance to mood stabilizer (1 mark)

4
Question 3

You are an Emergency Room Resident assessing a 16-year-old girl brought by


ambulance after she was found lying in a corridor in a mall. The brief history given is
that she had left home 3 days ago after a break-up with her boyfriend. She had
overdosed with 50 tablets of paracetamol at the mall. She was found by passer-by and
then sent to the Accident and Emergency Department by ambulance.

Question 1 During your assessment, state FOUR (4) most relevant risk factors
suggesting high suicide risk which you would explore.
Other logical answers are acceptable. Examiner is advised to use his or her discretion to
decide whether marks should be awarded.
1. Poor relationship with parents and family members (1 mark) or
2. Previous suicidal ideation or attempts (1 mark) or
3. Belief about lethality of paracetamol overdose (1 mark) or
4. Circumstances of suicide (e.g. suicide notes, isolation, avoidance of discover) (1 mark) or
5. Current plans for further suicidality (1 mark) or
6. Strong preoccupation with the relationship/ suicide as a mean to salvage the relationship
(1 mark) or
7. Presence or absence of alcohol intoxication (1 mark) or
8. Family history of psychiatric illness (1 mark) or
9. Current or past history of major depression or other psychiatric illness/or chronic medical
illness (1 mark).
( / 4 marks)
Question 2 During your assessment, state TWO (2) most relevant protective factors
against suicide which you would explore.

Other logical answers are acceptable. Examiner is advised to use his or her discretion to
decide whether marks should be awarded.
1. Religion (1 mark) or
2. Seeing a counsellor or psychologist (1 mark) or
3. Regret over suicide attempt (1 mark) or
4. Positive plan in the near future (e.g. return to study) (1 mark).

( / 2 marks)

Question 3 State FOUR (4) most appropriate management orders for this patient in
the Accident and Emergency Department.

Other logical answers are acceptable. Examiner is advised to use his or her discretion to
decide whether marks should be awarded.

1. Order investigations including Full blood count, liver function tests, renal function
tests, paracetamol levels, urine drug screen, electrocardiogram etc (1 mark) or

5
2. Activate charcoal to reduce absorption (1 mark) or
3. Suicide precaution (1 mark) or
4. Start N-acetylcystine (NAC) (1 mark) or
5. Inform family about her suicide attempt or to obtain collateral information (1 mark) or
6. Inform on-call psychiatrist for assessment (1 mark) or
7. Admit to the medical ward for further management (1 mark).

( / 4 marks)

6
Question 4 (10 marks)

A 70-year-old man was bought in by his family because he claims he has been seeing
ghosts for the past few months and is now hearing voices asking him to jump from his
9th floor flat. He has hypertension which is well-controlled and has been in good health.

Question 1 During the assessment, state THREE (3) risk factors for late onset
psychosis. (3 marks)

Other logical answers are acceptable. Examiner is advised to use his or her discretion to
decide whether marks should be awarded.
1. Sensory impairment (e.g. deafness) (1 mark) or
2. Social isolation (1 mark) or
3. Paranoid or schizoid or schizotypal personality trait (1 mark) or
4. Past psychiatric illness (e.g. history of schizophrenia/ severe depression/ bipolar
disorder) (1 mark) or
5. Past medical illness or underlying medical illness (e.g. stroke, head injury, brain
tumour) or medication which causes psychosis – steroid induced psychosis/
Parkinson’s disease (1 mark)

( / 3 marks)

Question 2 State TWO (2) differences between psychotic features found in an old
person and psychotic features found in a young person suffering from schizophrenia (
/ 2 marks)

Other logical answers are acceptable. Examiner is advised to use his or her discretion to
decide whether marks should be awarded.

1. Elderly exhibits less first rank symptoms / the content of psychotic symptoms (e.g.
auditory hallucination) is often less elaborated in elderly (1 mark) or
2. Elderly exhibits less negative symptoms as compared to young people suffering from
chronic schizophrenia (1 mark) or
3. Elderly are more likely to exhibit more paranoid ideas or delusions compared to
young people (1 mark).

Question 3 Based on his history, state THREE (3) most likely psychiatric diagnoses
(3 marks)

Any 3 of the following:


1. Late onset schizophrenia/ schizophrenia (1 mark) or
2. Late onset depressive disorder with psychotic features/severe depressive disorder with
psychotic features (1 mark) or
3. Late onset bipolar disorder / bipolar disorder with psychotic features (1 mark) or
4. Dementia with psychotic features (1 mark) or
5. Delirium or acute confusional state (1 mark)

7
Question 4 The family informs you that a family doctor started risperidone and he
developed severe rigidity and tremor. State ONE (1) antipsychotic drug which is MOST
suitable for him. (2 marks)

Any 1 of the following


Quetiapine (2 marks) or
Olanzapine (1 mark) or
Aripiprazole (2 marks)

( / 2 marks)

8
SAQ – 2014 Rotation 2

Question 1

You are a general practitioner. Depression is a common mental health problem. You are about
to start a patient on an antidepressant.

1. List FOUR (4) factors related to the patients which you would consider in choosing an
antidepressant for a patient suffering from depression (4 marks). ( /4 marks)

Any four of the following:

 Previous trials of antidepressants (1 mark) or


 Drug-to-drug interaction (1 mark)
 Previous response to antidepressants / efficacy(1 mark) or
 Side effect profile/ tolerability/ safety profile of antidepressant (1 mark)
 Psychiatric comorbidity (1 mark) or e.g. sleep difficulties, comorbid anxiety
 Risk of suicide or overdose (1 mark) or
 Underlying medical conditions (1 mark) or
 Concurrent medications (1 mark) or
 Symptom profile of depression (1 mark) or
 Patient’s preference (1 mark) or
 Age of patient (1 mark) or
 Pregnancy of patient / breast feeding (1 mark) or
 Frequency of dose (only fluoxetine can be given every other day) (1mark) or
 Financial status or affordability by patients (1 mark).
 Other answers from students: Liver function (more important than kidney function),
current body weight and BMI, occupation (with explanation e.g. drowsiness when driving)

2. Besides selective serotonin reuptake inhibitors (SSRI), name two other


antidepressants available in Singapore and fill the following table. (6 marks)
( /6 marks)

Name of antidepressants Other therapeutic effects (besides Specific side effects associated with
antidepressant effect and anxiety antidepressant (besides gastric
reduction effect) discomfort, headache, giddiness)

( /1 mark) ( /1 mark) ( /1 mark)

( /1 mark) ( /1 mark) ( /1 mark)


Examples: mirtazapine, venlafaxine, trazodone, agomelatine, amitriptyline etc. The following is an
example of acceptable answer. Examiner has to use his or her own discretion to decide whether
marks should be awarded. Spelling error of the name of the drug (deduct 0.5 marks).

Name of antidepressants Other therapeutic effects (besides Specific side effects associated with
antidepressant effect and anxiety antidepressant (besides gastric
reduction effect) discomfort, headache, giddiness)

Mirtazapine Useful for sedation and insomnia Weight gain


( /1 mark) ( /1 mark) ( /1 mark)

Amitriptyline or Neuropathic pain or enuresis Constipation


imipramine ( /1 mark) ( /1 mark)
( /1 mark)
Bupropion Smoking cessation Seizure
Question 2

You are a general practitioner. A 25-year-old man presents with disorganized thinking and speech.
You have a few psychiatric differential diagnoses.

1. State and define ONE (1) common type of disorganised thinking that are found in people
without any mental illness.(4 marks) ( /4 marks)

Type of disorganised thinking Definition

( /2 marks) ( /2 marks)

2. State and define TWO(2) common types of disorganised thinking that are found in people
suffering from schizophrenia. (4 marks) ( /4 marks)

Type of disorganised thinking Definition

( /1 mark) ( /1 mark)

( /1 mark) ( /1 mark)

3. State and define ONE (1) common type of disorganised thinking that is found in people
suffering from bipolar disorder. (2 marks) ( /2 marks)

Type of disorganised thinking Definition

( /1 mark) ( /1 mark)
Answers to question 2

1. State and define ONE (1) common types of disorganised thinking that are found in people
without any mental illness.(4 marks) ( /4 marks)

Type of disorganised thinking Definition (Other definitions are acceptable. Examiners


will decide on the marks)

Circumstantiality Unwarranted detail, elaborate and irrelevant, but returns


to the original topic after a long time

(2 marks) (2 marks)

2. State and define THREE (3) specific types of disorganised thinking that are found in people
suffering from schizophrenia. (6 marks) ( /6 marks)

Any three of the following:


Type of disorganised thinking Definition
Thought Blocking (1 mark) Cessation or complete interruption in the flow of the stream
of thought (1 mark)
Loosening of association or Loss of logical connections between ideas (1 mark)
knight’s move thinking (1 mark)
Derailment (1 mark) Insertion of novel or irrelevant idea in mid-sentence (1
mark)
Thought insertion (1 mark) External thoughts are inserted into the patient’s mind which
does not belong to the patient. (1 mark)
Thought withdrawal (1 mark) Patients’ thoughts are being taken away by the others (1
mark)
Thought broadcasting (1 mark) Patient’s own thoughts are known to the others through
broadcasting like a radio or TV station. (1 mark)

3. State and define ONE (1) specific type of disorganised thinking that is found in people
suffering from bipolar disorder. (2 marks) ( /2 marks)

Type of disorganised thinking Definition

Flight of ideas A continuous speech which topics jump rapidly from one
(1 mark) to another and there is a logical link between ideas.
( 1 mark)

Question 3
You are a paediatric resident. You are seeing an 11-year-old boy with repetitive eye blinking
and this has caused social dysfunction. He exhibits facial tics and gets frequent facial tics
when he is anxious. He sometimes cannot control his hand movements which affect his
writing.

1. You suspect that he suffers from Tourette’s syndrome. State the most APPROPRIATE
question which you would ask to establish the diagnosis of Tourette’s syndrome (2
marks) ( / 2 marks)

Answer: Does he have vocal tics? (2 marks)

2. What distinguishes Tourette’s disorder from a chronic motor or vocal tic disorder? (2
marks) ( /2 marks)

Answer: Patients suffering from Tourette’s disorder present with both motor and vocal tics.(2
marks)

3. State ONE (1) psychiatric condition often comorbid with Tourette’s syndrome. (2
marks) ( /2 marks)

Answer: Obsessive compulsive disorder (2 marks)

4. State ONE (1) psychiatric medication that can be used to treat Tourette’s syndrome. (2
marks) ( /2 marks)

Answer: Risperidone (2 marks), sulpiride (2 marks), haloperidol (2 marks)


Clonidine (0 mark) – not psychiatric medication

5. State Two (2) psychological interventions that can be used to treat Tourette’s
syndrome. (2 marks) ( /2 marks)

Any two of the following:

 Behaviour therapy or behaviour modification (1 mark)


 Relaxation exercise (1 mark)
 Social skill training (1 mark)
 Self-esteem building (1 mark)
 Remedial academic help (1 mark)
 Mass practice (1 mark)
 Habit rehearsal (1 mark)

Question 4:
You are a general practitioner (GP). The son of a 60-year-old man is concerned because his father is
taking a medication called, Flunitrazepam (Rohypnol) for 2 years. The patient obtained Rohypnol
from another GP who had retired 1 month ago. Rophypnol is a very potent benzodiazepine and is a
derivative of nitrazepam. Rophypnol is highly addictive and is listed as a controlled drug by the
Central Narcotics Bureau. His son has noted the following symptoms: anxiety, aggression, getting
high, insomnia, memory loss, slurred speech, tremor in his father. The patient was admitted to the
hospital due to fits.

1. Based on the symptoms reported by his son (underlined), classify the symptoms as
intoxication or withdrawal symptoms associated with benzodiazepine (4 marks)
( / 4 marks)

Intoxication symptoms Withdrawal symptoms


1. 1.

( / 0.5 marks) ( / 0.5 marks)


2. 2.

( / 0.5 marks) ( / 0.5 marks)


3. 3.

( / 0.5 marks) ( / 0.5 marks)


4. 4.

( / 0.5 marks) ( / 0.5 marks)

Answers:

Intoxication symptoms Withdrawal symptoms


1. Aggression (0.5 marks) 1. Anxiety (0.5 marks)

2. Getting high (euphoria) (0.5 marks) 2. Insomnia (0.5 marks)

3. Memory loss (amnesia) (0.5 marks) 3. Tremor (0.5 marks)

4. Slurred speech (0.5 marks) 4. Fits (withdrawal fits) (0.5 marks)

2. State the MOST APPROPRIATE psychotropic medication to treat his withdrawals (2 marks).
( / 2 marks)

Diazepam (Valium) is the only acceptable answer. (2 marks)


3. After stabilisation, his son would like you to prescribe a non-benzodiazapine medication to
help his father to sleep. Name ONE sedative antidepressant and ONE other sedative
medication. 4 marks ( /4 marks)

Sedative antidepressant

( /2 marks)
Other sedative medication (non-
benzodiazapine)

( /2 marks)

Sedative antidepressant An y one of the f ollo wi n g:


Mirta zapi ne (2 m ark s )
Agom elatine (2 m ark s )
Am itript yli ne (2 m ark s )
Fluvox am ine (2 m ark s )
Trazodon e (2 m ark s )
Parox etin e (2 m ark s )
Sertrali ne (1 m ark )
O ther antid epres s ant i f appropriate: 1 m ark
Fluox etin e/ bu popri on: 0 m ark .

Other sedative medication (non- An y one of the f ollo wi n g:


benzodiazapine) H ydrox yzin e (2 m ark s )
Anti- h is tam ine (1 m ark )
Mela nton in (2 m ark s )
Anti ps yc hotic s e.g. qu etiap ine (2 m ark s),
c hlorprom azin e (2 marks), olan zapi ne (2 m ark s )
Non- ben zod ia zapi ne h yp not ic s : Zopic lo ne (2
m ark s ) / Zolpidem (2 m ark s )
EOPT Rotation 3

Question 1

You are a general practitioner. A 30-year-old man suffers from major


depressive disorder. You are about to prescribe a selective serotonin reuptake
inhibitor (SSRI). He read about the SSRI on internet. You need to address the
following concerns.

1. SSRI may cause erectile dysfunction. Can you name ONE (1) antidepressant that
has relatively less sexual side effect as compared to SSRI? (2 marks)

Mirtazapine (2 marks) or Bupropion (2 marks) or agomelatine (2 marks)

2. If he wants to continue SSRI, state TWO (2) management strategies to improve his
erectile function if he develops SSRI induced erectile dysfunction. (2 marks)
Any two of the following:
1) Reduce the dose or prescribe the lowest dose of SSRI (1 mark)
2) Drug holiday or stop SSRI during weekend or take a long acting SSRI on alternate
day (1 mark)
3) Add sildenafil or PDE5 inhibitor (1 mark)
4) Refer patient to sensate focus or sex therapy / vacuum pump (1 mark)
Not grip method – for premature ejaculation
3) He understands that the combination of SSRI and other medication may lead to
high fever. What is the name of such condition? (1 mark)
Serotonin syndrome (1 mark) Not Neuroleptic malignant syndrome
4) State ONE (1) medication that may cause serotonin syndrome if he takes together
with SSRI (1 mark)
Monoamine oxidase inhibitor or irreversible monoamine oxidase inhibitor (1 mark),
moclobemide (0.5 mark) MAOI (0.5 mark) (Isocaroxazid, phenelzine – irreversible MAOI
1 mark)

5) State FOUR (4) signs or symptoms associated with serotonin syndrome (4 marks)
Any 4 of the following:
 Rigidity, myoclonus, uncontrollable shivering
 Sweating or tremor or hyperthermia or coma
 Hyperkinesia or bradykinesia
 High blood pressure, labile blood pressure
 hyper-reflexia, headache
 dysphagia, dilated pupils
 cardiovascular collapse
 confusion (delirium)
 decreased level of consciousness/mutism
 disorientation
 status epilepticus
 ataxia
Question 2

You are a resident working in the Accident and Emergency Department (AED).
A 25-year-old man is brought in by his mother to see you after he reveals to
her about his great plan to fly to Europe by “changing the magnetic pole of the
earth”. He is very talkative and appears to be very excited. He disturbs
other patients in the AED.

1) State THREE (3) MOST LIKELY psychiatry diagnoses that you would
consider. (3 marks)

Bipolar disorder / Mania (1 mark)


Schizoaffective disorder (1 mark)
Substance abuse – stimulant (1 mark)

2) You are unable to perform any blood test as he is very agitated. Name TWO
(2) MOST relevant psychotropic medications to sedate this patient at the
AED. (2 marks)

Antipsychotics: risperidone, olanzapine, haloperidol (1 mark)


Benzodiazapine: diazepam, lorazepam (1 mark)

3) List TWO (2) most important laboratory investigations you would do before
beginning lithium (2 marks)

Renal function test (1 mark)


Thyroid function test (1 mark)

4) State THREE (3) common side effects associated with lithium. (3 marks)

Any one of the following:


Polyuria (1 mark) or Nephreogenic diabetes insipidus
Polydipsia (1 mark) or
Weight gain (1 mark) or
Cognitive impairment (1 mark) or
Fine tremor (1 mark) or
Metallic taste (1 mark) or
Hypothyroidism (1 mark) or
Sexual dysfunction (1 mark) or
Drowsiness (1 mark) or
Alopecia (1 mark)
Question 3

You are a resident working in surgery. A 45-year-old man with a dual diagnosis of schizophrenia
and alcohol dependence is admitted to the ward due to upper gastrointestinal bleeding. The on-
call consultant recommends urgent oesophagogastroduodenoscopy (OGD).

1. You need to take an informed consent from this patient. State THREE (3) important pieces
information you must inform this patient before seeking his consent. (3 marks)

1. Explain the potential diagnosis of Mallory-Weiss syndrome which is associated with alcohol
misuse (1 mark) or
2. Explain purpose and nature of the diagnostic procedure about OGD (e.g. passing an
endoscope to examine the stomach and identify bleeding site, may take some tissues) (1
mark) or
3. Explain risks and benefits of OGD (e.g. small risk of perforation of feeding tube; benefit: can
assist diagnosis) (1 mark) or
4. Alternative investigation (e.g. monitor haemoglobin levels but bleeding may continue) ( 1
mark) or
5. Prognosis if without this investigation (e.g. continue bleeding) (1 mark)

2. State THREE (3) criteria which you would like to assess in order to certify he has the capacity to
take consent. (3 marks)

Any 3 of the following:


1. He has a factual understanding of the information provided. ( 1 mark) or
2. He can appreciate the nature of the condition (1 mark)
3. He can appreciate consequences of accepting OGD (1 mark)
4. He can appreciate consequences of refusing OGD (1 mark)
5. He can manipulate information provided in a rational manner to reach a decision (1 mark).

3) After the OGD is done, his wife hopes that he can quit drinking. List Four (4) techniques of
motivational interviewing which you could use to maximize his motivation to quit drinking (4
marks)

4 Elements to the process of Motivational interviewing:

 Listen empathically (1 mark for any of the following)


1. Understand the patient’s unique perspective and experience
2. To put you and patient on equal ground
3. Recognition and justification and very important parts of the process
 Have patient’s voice their own reasons for change (1 mark for any of the following)
1. Articulate reasons for or not for changing
2. Look at what they find challenging
 Roll with resistance (1 mark)
1. Never put yourself in position of arguing, or of trying to persuade patient to take a difficult
position, or of telling the patient what must be done immediately
 Support Self-Efficacy (1 mark for any of the following)
1. Give patient hope, optimism, and provide tools to help patient succeed
2. Not to blame for failures
Short answer questions (2014/2015 rotation 4)

Question 1 (10 marks)

You are a general practitioner. A 65-year-old woman has taken haloperidol for 30 years is concerned
about a side effect called tardive dyskinesia (TD).

1. State FOUR (4) risk factors for TD (4 marks). ( / 4 marks)

Any 3 of the following:


- Old age (1 mark) or
- Female gender (1 mark) or
- Comorbid affective disorder / alcohol abuse (1 mark) or
- Comorbid organic brain disorder (1 mark) or
- Long exposure to first generation antipsychotic drug / potent dopamine receptor
blockade (1 mark) or
- Precipitation by anticholinergic drug (1 mark) or
- High antipsychotic dose / rapid increment in dose (1 mark) or
- Previous extra pyramidal side effect / history of tremor or Parkinson’s disease (1 mark)
or
- Genetic predisposition due to hypersensitive dopamine receptor / Family history of TD
(1 mark) or
- Afro-Caribbean race ( 1 mark)

2. You are about to perform a physical examination. State TWO (2) anatomical structures and
ONE (1) clinical feature under each anatomical structure which you would look for to
confirm the presence of TD (4 marks). ( / 4 marks)

Anatomical structure Clinical feature

( / 1 mark) ( / 1 mark)

( / 1 mark) ( / 1 mark)

Any two of the following:


Anatomical structure Clinical feature
Mouth Mouth opening
Eye brow Elevation or depression
Eye lids Blepharospasm/sustained, forced, involuntary closing of the eyelids
Lips pursuing/retraction to lateral angle/ lip smacking
Tongue Fly catching, tongue protrusion
Jaw Abnormal mastication-like movement / protrusion and deviation
Neck Torticollis
Upper limb/ lower Hypertonia/wrist flexion/ pilling rolling tremor
limb
Hand Choreiform hand pill rolling movements
3. If she indeed suffers from TD, state TWO (2) management strategies (2
marks) ( / 2 marks)

Any two of the following:


1. Change to second generation antipsychotics (1 mark) or
2. Add vitamin E or vitamin B (1 mark) or
3. Benzodiazapine e.g. clonazapine (1 mark) or
4. Reduce the dose of haloperidol to minimum effective dose in long run (1 mark).

Wrong answers:

There is not enough evidence to suggest anticholinergic as good treatment for TD.

Drug holiday may cause relapse in schizophrenia.


Question 2 (10 marks)

A 68-year-old woman presents with poor sleep and decreased appetite for two months.
She has stopped reading because she cannot concentrate, and she is no longer seeing
friends that she is close to because she does not have the energy. She tells you that her
symptoms started after her son moved out of the house two months ago. Prior to
that, she was an energetic person who took care of her affairs and enjoyed life.

1. Besides major depressive disorder, state ONE (1) other psychiatric diagnosis based
on the above information (1 mark) ( / 1 mark)

Adjustment disorder (1 mark), Other diagnosis: 0 mark

2. Based on your diagnosis stated in Q.1, state THREE (3) non-pharmacological


interventions (3 marks) ( /3 marks)

Any three of the following:


Supportive psychotherapy (1 mark) or
Problem solving therapy (1 mark) or
Cognitive behaviour therapy (1 mark) or
Interpersonal therapy (1 mark) or
Attending a day care centre (1 mark) or
Home visit by community psychiatric team (1 mark)

In the later part of interview, she mentions about the death of her husband three years ago.
She admits that she has kept everything in her late husband’s room exactly the same since
his death.

3. State the term which describes the above phenomenon. (2 marks) ( /2 marks)

Mummification (2 marks), other term = 0 mark

4. With the additional information, you suspect the patient may suffer from abnormal
grief reaction although she claims she has coped well with the death of her husband.
State FOUR (4) clinical features which are found in abnormal grief but not normal
grief (4 marks) ( /4 marks)
Any 4 of the following:
- Prolonged grief with duration longer than 6 months (1 mark) or
- Inhibited grief at the time of death (1 mark) or
- Delayed grief (i.e. no grief reaction at the time of death but appeared some time later) (1 mark)
- Strong suicidal ideation to join the deceased (1 mark) or
- Psychotic experience other than deceased (1 mark) or
- Severe psychomotor retardation (1 mark) or
- Severe feeling of worthlessness or hopelessness (1 mark)

Question 3 (10 marks)

The public relation department of your hospital has received queries from a local newspaper. A
reporter wants to write about an article about alcohol misuse. You are invited to address the following
questions raised by the reporter.

1. How does long-term alcohol misuse damage our body systems? Please state FOUR (4)
physiological systems and name ONE (1) complication under each system (4 marks)
( / 4 marks)

Physiological system Complication

( / 0.5 mark) ( /0.5 mark)

( / 0.5 mark) ( /0.5 mark)

( / 0.5 mark) ( /0.5 mark)

( / 0.5 mark) ( /0.5 mark)

Physiological system Complication (The candidate needs to state one complication under
each physiological system)
Nervous system Blackouts/head injury, withdrawal fits/epilepsy, acute confusional
state, subdural haematoma, degeneration of cerebellum/ataxia,
peripheral neuropathy/loss of pain sensation, alcohol dementia,
Wernicke encephalopathy, Korsakoff psychosis, central pontine
myelinolysis, Marchiafava-Bignami syndrome, depression, anxiety,
morbid jealousy
Opthalmological Optic atrophy, retrobulbar neuropathy, loss of central vision,
system bilateral central scotoma
Gastrointestinal Cancer of mouth, larynx, pharynx, oesophagus, upper
system gastrointestinal bleeding, gastritis, malabsorption syndrome
Cardiovascular Hypertension, cardiomyopathy, atrial fibrillation
system
Respiratory system Increase in risk in respiratory infection (e.g. pneumonia,
tuberculosis)
Hepatobiliary system Cirrhosis, liver cancer, alcohol hepatitis, fatty liver, pancreatitis,
hepatic encephalopathy
Urogenital system Erectile dysfunction, sexual dysfunction, infertility, loss of
secondary sexual characteristics
Musculoskeletal Myopathy, gouty arthritis, palmar erythema.
system
Endocrine Cushing syndrome
Haematological Macrocytic anaemia
system

2. If an alcoholic stops drinking for 1 day, what would happen to him/her? State TWO (2) common
symptoms. (2 marks) ( / 2 marks)

Any two of the following:


- Tremor (1 mark) or
- Anxiety (1 mark) or
- Agitation (1 mark) or
- Muscle pain (1 mark) or
- Sweating (1 mark) or
- Nausea (1 mark)
- Sleep disturbance (1 mark)
- Withdrawal fits (1 mark)

3. If an alcoholic wants to join peer support group, name ONE (1) organisation which can support
the person (2 marks) ( /2
marks)

Alcoholic anonymous (2 marks)


12-step programme (1 mark)
Other answer: 0 mark

4. If an alcoholic wants to be admitted to a hospital to detoxify from alcohol, what are the
indications. State TWO (2) indications. (2 marks) ( / 2 marks)

Any two of the following:


- History of delirium tremens or withdrawal fits (1 mark) or
- Current moderate to severe symptoms of alcohol withdrawal (1 mark) or
- Significant risk of self-harm, suicide or harming other people (1 mark) or
- Unable to tolerate oral medication (e.g. naltrexone) (1 mark) or
- Multiple past inpatient detoxification (1 mark) or
- Acute psychosis (1 mark) or
- Severe cognitive impairment (e.g. Wernicke encephalopathy) 1 mark or
- Poor social support (1 mark) or
- Pregnancy (1 mark) or
- Comorbid severe mental illness (1 mark)
Question 4

You are a medical resident. The accident and emergency department has sent a 55-year-old man to the
medical ward for further management due to change in mental state. When you assess him, he appears
to be manic.

1. State FIVE (5) neurological disorders which may cause mania. (5 marks) ( / 5marks)

Any 5 of the following:

o Multiple sclerosis (1 mark) or


o Subcortical dementia (e.g. Lewy body dementia) (1 mark) or
o Frontal lobe dementia (1 mark) or
o Temporal lobe epilepsy (1 mark) or
o Cerebrovascular accidents (1 mark) or
o Head injury (1 mark) or
o Encephalitis (virus e.g. HSV, HIV or parasite) (1 mark) or
o Huntington’s disease (1 mark) or
o Pseudobulbar palsy (1 mark) or
o Brain tumour (1 mark) or
o Neurosyphilis(1 mark)

2. State TWO (2) medications (not illicit drugs) which may cause mania (2 marks) ( /2
marks)

Any two of the following:

o Steroid / prednisolone
o Antidepressant
o Anticholinergic drug
o Levodopa / bomocrptine
o Bronchodilator

3. When you are clerking the patient, he suddenly becomes very aggressive and tries to run
away from the ward. You have tried verbal de-escalation but fails. You need to apply
physical restraint. State THREE (3) specific instructions to nursing staff to ensure the physical
restraint will be applied successfully. (3 marks) ( / 3 marks)

Any three of the following:

1. Call for additional manpower (e.g. call security guards for help, at least 4 additional staff) (1
mark) or
2. Obtain adequate physical restraints (e.g. 1 body jacket, 1 restraint per limb) (1 mark) or
3. Standby psychotropic medication (e.g. rapid dissolvable antipsychotics or intravenous
lorazepam / haloperidol) (1 mark) or
4. The doctor will ensure the airway is not blocked. Other staffs will focus on the body and four
limbs. (1 mark)
SAQ (Paper 11)

Question 1 (10 marks)

James is a 40-year-old man. He is unemployed because of his constant worries lasting for years
about simple daily events. He now lives with his parents. Although he was an overachiever
throughout his teaching career in a secondary school, his parents are disappointed with his recent
resignation from the Ministry of Education. James resigned because he kept sweating and having
tremor and diarrhoea during his classes. He mainly focused on the minor mistakes made in his
classes but ignored his good teaching performance. His parents find James lazy and unmotivated
to even find a job in a tuition centre. He likes to spend his time in crowded shopping malls instead.
Mental state examination shows that James is easily irritated, distracted and quite tense. He
complains of giddiness, dry mouth, nausea. At the end of the interview, he says, “I failed my job. I
fail everything in my life.”

Question 1 Based on the history provided, state the MOST likely psychiatric diagnosis? ( /1
mark)

Generalised anxiety disorder (1 mark)

Question 2 Based on the history provided, state TWO (2) MOST RELEVANT psychiatric
differential diagnosis? ( / 2 marks)

Any two of the following:


Social phobia (1 mark)
Panic disorder (1 mark)
Major depressive disorder (1 mark)
Mixed anxiety and depression (1 mark)
Somatisation disorder (1 mark)

Question 3 Stated ONE (1) MOST important blood test to rule out a medical diagnosis which
has similar clinical presentation as seen in James. ( / 1 mark)

Thyroid function test (1 mark)

Question 4 James wants to discuss the different types of medications that are available for
providing relief from his disorder. He wants to take benzodiazepines. What are the pros and cons
of prescribing benzodiazepines to James? ( / 2 marks)

Pros 1 mark Cons 1 mark

Pros 1 mark Cons 1 mark


Rapid relief of anxiety symptoms with short term Potential dependence and addiction with long
use term use
Question 5 State ONE (1) class of psychotropic medication that has the most evidence to treat
his condition and name ONE (1) example. ( /2 mark)

Class of psychotropic medication 1 mark Example 1 mark

Class of psychotropic medication 1 mark Example 1 mark


Selective serotonin reuptake inhibitor Fluvoxamine
Antidepressants (0.5 mark) Fluoxetine
Tricyclic antidepressants (0 mark) Escitalopram
Paroxetine
Sertraline

Question 6 James wants to have psychotherapy to figure out his problems. Based on his
history, name two cognitive errors which can be challenged by cognitive therapy. ( / 2 marks)

1. Selective abstraction ( 1 mark)


(Based on the statement: He mainly focuses on the minor mistakes he made in his lessons but
ignores his good teaching performance).
2. Overgeneralisation (1 mark)
(Based on the statement: I failed in my job. I fail everything in my life).

Other answers: 0 mark

Author: Roger Ho
Year: 2015
Question 2 (10 marks)

Dennis, a 30-year-old man was admitted to the surgical ward following the collision of his
car into a road barrier on the Ayer Rajah Expressway (AYE). He reported that he was not
under influence of alcohol, but that he was looking at his lips in the rearview mirror when
he lost control of his car. No one else was with him during the accident.

He sustained a fractured ankle which required internal fixation. While on the ward, Dennis
spent a lot of time in the bathroom picking at his lips, looking at his lips in the mirror, and
tried to manipulate his way out of physiotherapy sessions to spend his time looking at his
lips. He often has his hand over his mouth and wears a mask.

Dennis works as a freelance apps developer and he mainly works at home. He feels
rejected by other people because of his lips. He becomes convinced that he will be more
appealing to others if he has cosmetic operation on his lips.

Physical examination does not reveal any abnormality on his lips.

Question 1 What is the most likely psychiatric diagnosis? ( / 1 mark)

Body dysmorphic disorder (1 mark)

Question 2 State TWO (2) common psychiatric comorbidity or condition related to the
psychiatric diagnosis established in question. ( /2 marks)

Any two of the following:

 Social phobia (1 mark) or


 Substance abuse (1 mark) or
 Suicide attempt or suicide ideation or self-injury (1 mark) or
 Obsessive compulsive disorder (1 mark) or
 Depressive disorder (1 mark) or
 Post-traumatic stress disorder (1 mark)

Question 3 Besides his lips, state TWO (2) common anatomical areas commonly affected
in this condition. ( / 2 marks)

Any two of the following:


 Hair or scalp (1 mark) or
 Nose (1 mark) or
 Skin (1 mark) or
 Eye (1 mark) or
 Face/Facial muscle (1 mark) or
 Neck (1 mark)

Question 4 State the most important question to ask Dennis during risk assessment. (
/ 1 mark)

Will you operate on your lips if the doctors decline cosmetic surgery?
Question 5 State ONE (1) class of psychotropic medication that has the most evidence to treat
his condition and name ONE (1) example. ( /2 mark)

Class of psychotropic medication 1 mark Example 1 mark

Class of psychotropic medication 1 mark Example 1 mark


Selective serotonin reuptake inhibitor Fluvoxamine
Antidepressants or tricyclic antidepressants (No Fluoxetine
mark) Escitalopram
Paroxetine
Sertraline

Question 6 Dennis is keen to consider cognitive behaviour therapy. Explain how cognitive
behaviour therapy works in his case. ( /2 marks)

Answers may vary. Examiner needs to use his or her own discretion to decide the marks to be
awarded.

Cognitive therapy provides cognitive restructuring or challenges his belief that his lips are abnormal
although it is normal during physical examination. (1 mark)

Behaviour therapy involves exposure and response prevention such as avoiding frequent mirror
checking on his lips. (1 mark)

Author: Roger Ho
Question 3

You are going to assess a 75-year-old man whose daughter is concerned that he is
becoming forgetful and confused. He suffers from visual impairment and cannot see
clearly. He had secondary school education and worked as a technician in the past.

1. Name TWO (2) main differences in clinical features between dementia and
delirium? ( /2 marks)
Answer may vary. The examiner is advised to use his or her discretion to decide whether
marks should be awarded.
1. The mental state and cognitive function of delirious patients fluctuate more often than
dementia patients.
2. The main cognitive impairment in delirium involves attention while dementia
involves memory.
3. Delirious patients are more likely to experience visual hallucations or illusions as
compared to dementia.
4. The clinical course of illness in delirium is shorter and more likely to be reversible as
compared to dementia.

2. Based on the history provided, name THREE (3) cognitive tasks which he would
have difficulty to perform during the Mini-Mental State Examination. ( /3
marks)

Any three of the following:


1. Orientation to place (1 mark) or
2. Orientation to time (1 mark) or
3. Registration of 3 items (1 mark) or
4. Recall after 3 minutes (1 mark) or
5. Recognition of objects (e.g. watch or pen) (1 mark) or
6. Reading instruction (e.g. Close your eyes) (1 mark) or
7. Writing a sentence (1 mark) or
8. Draw a double pentagon ( 1 mark) or
9. 3-stage command (1 mark)

No ifs and or buts; serial 7; serial 3: No mark

3. State THREE (3) specific tests of frontal lobe functioning which can be
performed on this patient. ( / 3 marks)

Any three of the following:


1. Cognitive estimation (e.g. “How tall is an average Chinese woman?”/ “How many
elephants are there in Singapore?”) (1 mark) or
2. Assess judgement (e.g. “What would you do if there is a fire in this room?” (1 mark)
or
3. Assess verbal fluency / word generation.(1 mark) or
4. Assess similarities between 2 objects or interpretation of proverbs (1 mark)
4. State the most appropriate psychotropic medication to treat mild to moderate
Alzheimer’s disease. Please state the class of psychotropic medication and
provides an example.( / 2 marks)

Class of psychotropic medication 1 mark Example 1 mark

Class of psychotropic medication 1 mark Example 1 mark


Acetylcholinesterase inhibitor (1 mark) Donepezil (1 mark) or
Rivastigmine (1 mark) or
Galantamine (1 mark)

Author: Roger Ho
Question 4 A 16-year-old male is bought in by his family. He has refused to attend school
or sit for the “O” level examination. He has spent a lot of time playing online games and
surfing the internet. He has been verbally abusive toward his parents when they try to remove
the computer from him. His school has asked him to discontinue his study because he shows
no interest in his study.

Question 1 State five (5) questions which you would ask to screen or establish the
diagnosis of internet addiction. ( / 5 marks)

Answers may vary. Examiners are advised to use his or her discretions to decide on the marks
awarded.

Any 5 of the following:

1) Tolerance: Do you spend more or more time on the internet to get excitement? If so, was it
over a period of time? (1 mark) or
2) Withdrawal: Do you feel uneasy, depressed or agitated when you are not allowed to play
online game or use internet? (1 mark) or
3) Have you given up your interest or hobbies as a result of internet use? (1 mark) or
4) Do you continue to spend a lot of time online despite failure in academic study? (1 mark) or
5) Do you feel that you need to cut down on online gaming or internet usage? (1 marks) or
6) Do you feel annoyed by your family members when they criticise your internet usage? (1
mark) or
7) Do you feel that it is a compulsion or strong urge to play online game which you cannot stop
or control (1 mark) or
8) Do you ever need to tell lies about your excessive internet usage or gaming activities (1
mark) or
9) Do you spend more and more money to purchase items for the online game? If so, how
much (1 mark) or
10) Do you feel guilty about your excessive online gaming or internet usage? (1 mark) or
11) Do you play online game or surf internet first thing in the morning or after you wake up and
ignore other activities (1 mark)?

Question 2 State THREE (3) techniques to motivate him to consider quitting online game and
to receive treatment. ( / 3 marks)

Answers may vary. Examiners are advised to use his or her discretions to decide on the marks
awarded.

Any 3 of the following:


1) Listening with empathy about his addiction to online gaming and internet.
2) Provide feedback and a menu of options related to internet usage or online gaming
3) Allow patient to ventilate his feelings and reason for change/come up with his own reasons
to change
4) Roll with his resistance to change
5) Enhance self-efficacy in reducing internet usage or online gaming.
6) Encourage him to set achievable goals.

Question 3 State TWO (2) Most common physical complication associated with internet
addiction? ( / 2 marks)

Answers may vary. Examiners are advised to use his or her discretions to decide on the marks
awarded.

1. Musculoskeletal complications (e.g. low back pain, cervical spondylosis/myelopathy, carpal


tunnel syndrome/kphosis) (1 mark)
2. Physical exhaustion/ malnutrition/weight loss/dehydration / collapse (1 mark)
3. Eye problems: dry eye or myopia(1 mark)
4. Obesity (1 mark)
5. Deep vein thrombosis (1 mark)
6. Sleep disturbance / insomnia (1 mark)
SAQ (2015 Rotation 2)

Question 1 (Failure: 1, range of score 3.5 – 10)

You are a resident working at the Accident and Emergency Department (AED) of the
National University Hospital. A 25-year-old nurse was found unconscious by her
parents. Her parents suspected that she attempted suicide by taking unknown
medications. Her parents mentioned that she was stressed by interpersonal problems
at work.

1. Given the fact that she works as a nurse, state TWO (2) MOST fatal medications
which she can take from the hospital and administer to herself to attempt
suicide. (2 marks) Examiner: / 2 marks

The answers can be any two of the following. For other answers, examiner can use
his or her judgement to decide whether marks should be awarded.

Insulin (1 mark) or
Lignocaine (1 mark) or
Digoxin (1 mark) or
Fetanyl (1 mark) or
Succinylcholine (1 mark) or
Adrenaline (1 mark) or
Potassium Chloride injection (1 mark) or
Propofol (1 mark) or
Midazolam (1 mark) or
Phenobarbitone (1 mark) or
Sodium thiopental (1 mark) or
Morphine (1 mark) or
Warfarin (1 mark) or
Tricyclic antidepressant (1 mark)
Lithium (1 mark)

2. The police officers came to the AED and informed you that they found burnt
charcoal in her bed room and the windows were sealed. State TWO (2) MOST
relevant laboratory investigations which you should order in the Accident and
Emergency Department (AED) (2 marks)
Examiner: / 2 marks
The answers can be any two of the following. For other answers (e.g. thyroid function
test), mark will not be awarded because it is less relevant for this case.

Arterial blood gases (1 mark) or


Carboxyheamoglobin levels (1 mark) or
Renal function tests or electrolytes (1 mark) or
Liver function tests (1 mark) or
Blood lactate levels
Serum drug levels (1 mark); Urine drug level is difficult to obtain as patient is
unconscious or

Non laboratory investigations e.g. ECG, CT brain scan = 0 mark.


3. State TWO (2) MOST RELEVANT immediate treatment order based on the
additional information provided by the police.
Examiner: / 2 marks

The answers can be any two of the following. For other answers, examiner can use
his or her judgement to decide whether marks should be awarded.

• The immediate use of 100% oxygen mask (1 mark) or


• Strict bed rest to reduce oxygen demand and consumption (1 mark) or
• Incubation (1 mark) or
• Transferred the patient to Singapore General Hospital for Hyperbaric oxygen
treatment (1 mark).

4. List TWO (2) MOST relevant neuropsychiatric sequelae which can be found in
this patient when her condition is stable. (2 marks) Examiner: /2
marks

Answer: Any two of the following:

1. Memory loss (1 mark) or


2. Parkinsonism feature (1 mark)/ psychomotor retardation (0.5 mark) or
3. Cognitive impairment (0.5 mark) or
4. Personality change (1 mark) or
5. Psychosis (1 mark)
6. Slow mental processing (1 mark) or
7. Decreased intellectual function or IQ (1 mark) or
8. Apraxia (1 mark) or
9. Agnosia (1 mark) or
10. Aphasia (1 mark) or
11. Ataxia (1 mark) or
12. Obsession or compulsion (1 mark) or
13. Seizure (1 mark)

5. The patient has stayed in the ward for one month and recovered without any
neuropsychiatric sequelae. She is keen for psychotherapy. Please state the
MOST relevant psychotherapy (1 mark) and explain how it works. (1 mark)
Examiner: / 2 marks

Interpersonal psychotherapy (1 mark)

The answers can be any two of the following. For other answers, examiner can use his
or her judgement to decide whether marks should be awarded.

The psychologist will closely examine her interpersonal problems and works with the
patient to look at interpersonal relationship from another angle to miminise impact on her
mood. The psychologists can use role – play to improve her communication skills (1
mark)
If students answer other forms of psychotherapy e.g.cognitive behaviour therapy,
psychodynamic psychotherapy, marks will be awarded by it depends on the quality of the
answer.

Question 2 A 30-year-old man suffers from the first episode of schizophrenia and
he has tried risperidone 2mg daily. His first rank symptoms have subsided but
negative symptoms persist. He has read the internet and found a medication called
clozapine. He believes that clozapine is the ultimate treatment for schizophrenia.

Question 1 He does not want to waste time with other antipsychotics and wants to
try clozapine. What is your advice? ( / 2 marks)

Answer may vary. For other answers, examiner can use his or her judgement to decide
whether marks should be awarded.

 He should try another psychotic drug (e.g. first generation antipsychotic drug) before
considering clozapine (1 mark) or
 Clozapine is reserved for treatment resistant schizophrenia but not first episode of
schizophrenia (1 mark) or
 Clozapine requires some investigations (e.g. full blood count) before starting clozapine
and patients cannot go straight to clozapine (1 mark).

Question 2 What is the one major side effect of clozapine that requires regular blood test?
State the blood test required. ( / 2 marks)

Major side effect of clozapine (1 mark) Regular blood test (1 mark)

Answer:

Major side effect of clozapine Regular blood test


Agranulocytosis (1 mark) Full blood count (1 mark)

Question 3 State Four (4) systemic side effects of clozapine. ( /4 marks)

Physiological Systems List one specific side effect under each


physiological system

( /0.5 mark) ( /0.5 mark)


( /0.5 mark) ( /0.5 mark)

( /0.5 mark) ( /0.5 mark)

( /0.5 mark) ( /0.5 mark)

Answer:

Physiological Systems List one specific side effect under each


system
Gastrointestinal system (0.5 mark)  Sialorrhoea or hyperventilation (0.5 mark)
 Dry mouth (0.5 mark)
 Constipation (0.5 mark)
 Increase in appetite / weight gain (0.5
mark)
Cardiovascular system (0.5 mark)  Cardiomyopathy (0.5 mark)
 Tachycardia (0.5 mark)
 Changes in ECG (ST segment changes)
0.5 mark
 Myocarditis (0.5 mark)
 Hyperlipidemia (0.5 mark)
 Postural hypotension (0.5 mark)
Respiratory system (0.5 mark)  Pulmonary embolism (0.5 mark0
Neurological system (0.5 mark)  Seizure (0.5 mark)
 Sedation (0.5 mark)
Endocrine system (0.5 mark)  Worsening diabetes (0.5 mark)

Urological system (0.5 mark)  Urinary retention (0.5 mark)


Ophthalmological system/Eye (0.5 mark)  Blurred vision (0.5 mark)
Haematological system (0.5 mark)  Agranulocytosis (0.5 mark)

Question 4 State TWO (2) psychosocial interventions for negative symptoms (2 marks)

Any two of the following:

 Cognitive rehabilitation/Psychiatric rehabilitation (1 mark) or


 Behaviour therapy/ Cognitive behaviour therapy (1 mark) or
 Social skill training (1 mark) or
 Art therapy (1 mark)
Question 3

You are a paediatric resident working in the Child and Accident Emergency Department. You
are seeing an 11-year-old boy who was brought in by his foster parents. According to his
foster parents, he has a lot of behavioural problems at home and school. He cannot sit still to
do his homework at home. He also breaks school rules. Tonight, he had a fight with his foster
parents. This boy was abused by his biological parents.

1) You suspect that this boy suffers from attention deficit and hyperactivity disorder
(ADHD). Besides that fact that he cannot sit still to do homework, state THREE (3)
inattention symptoms and THREE (3) hyperactivity symptoms. ( / 6 marks)

Inattention symptoms Hyperactivity symptoms

( / 1 mark) ( / 1 mark)

( / 1 mark) ( / 1 mark)

( / 1 mark) ( / 1 mark)

Inattention symptoms Hyperactivity symptoms


Any two of the following: Any two of the following:
 Starts tasks or activities but not able to  On the move most of the time such as
follow through and finish (1 mark) or running and climbing (1 mark) or
 Organisation of tasks or activities is  Restlessness and jitteriness (1 mark) or
impaired (1 mark) or  Fidgets with hands and feet (1 mark) or
 Loses things necessary for tasks and  Loud noise in playing (1 mark) or
activities such as school assignments or  Talks excessively without appropriate
stationary (1 mark) or response (1 mark) or
 Instructions are not followed (1 mark) or  Answers are blurted out before questions
 Distraction by external stimuli (1 mark) or (1 mark) or
 Other features: careless mistakes,  Interruption of others’ people
forgetfulness in daily activities. (1 mark) conversation (1 mark)

2) State TWO (2) other psychiatric diagnoses which must be ruled out? ( /2
marks)

 Conduct disorder (1 mark)


 Post- traumatic stress disorder (1 mark)
3) State TWO (2) psychosocial interventions which are useful for ADHD. ( / 2 marks)

 Parent training (1 mark) or


 Psychoeducation programme (1 mark) or
 Behaviour therapy (1 mark) or
 Social skill training (1 mark)
Question 4

You are a general practitioner. A 25-year-old man has returned from the United States (US). He
has obtained a Master degree in business administration. He consults you because he has been
using a drug called oxycodone in the US. After coming back to Singapore, he has repetitive
thoughts about using oxycodone. He has imagined pain in his arm when he is not using
oxycodone. He hopes that you can prescribe oxycodone to him.

Question 1 Will you prescribe oxycodone to him? (1 mark) Please justify your reason (1
mark).

Will you prescribe oxycodone? Please justify your reason

Yes, I will prescribe oxycodone.

No, I will not prescribe oxycodone.

Please delete the incorrect answer

( /1 mark) ( /1 mark)

Will you prescribe oxycodone? Please justify your reason


Answer may vary. Examiner need to decide the
Yes, I will prescribe oxycodone. marks to be awarded.
This patient is dependent on oxycodone which is
No, I will not prescribe oxycodone. (1 mark) an opioid analgesic. (1 mark) or
I will not prescribe oxycodone because he will
Please delete the incorrect answer misuse it in Singapore (1 mark) or
It is a Class A drug in Singapore and misuse will
lead to severe penalty. (1 mark)

Question 2 State FOUR (4) SPECIFIC withdrawal symptoms associated with termination of
oxycodone use (4 marks). Please do not repeat the symptoms stated in the case vignette.

Any four of the following:

 Gastrointestinal disturbances: Nausea / Vomiting (1 mark) or


 Pain: abdominal pain (1 mark) or muscle ache (1 mark) or
 Insomnia (1 mark) or
 Anxiety (1 mark) or Panic Attacks (1 mark) or
 Lacrimation (1 mark) or rhinorrhoea (1 mark) or
 Flu-like Symptoms (1 mark) or
 Seizures/Tremors (1 mark) or
 Depression (1 mark) or
 Pilorection (1 mark) or sweating

Question 3 As a general practitioner, state TWO (2) medications which you can prescribe to
him to reduce his withdrawal symptoms (2 marks)

1. Nonsteroidal anti-inflammatory drugs (NSAIDs) such as mefenamic acid (1 mark)


2. Anti-histamine such as hydroxyzine (1 mark)

Other answers (e.g methadone, buprenorphine: no mark)

Question 4 What is the class status of oxycodone under the Drug of Misuse Act (Singapore)? (1
mark)

Class A (1 mark)

Question 5 State the specialist service which you will refer him to manage his condition (1
mark)

Name of specialist service (0.5 mark) Hospital which provides this service (0.5 mark)

Name of specialist service (0.5 mark) Hospital which provides this service (0.5 mark)
National Addictions Management Service (0.5 Institute of Mental Health (0.5 mark)
mark)
SAQ (Paper 13)

Question 1
You are a General Practitioner. A 30-year-old woman comes to your clinic and complains of low
mood and anxiety. She has read a lot of information on the internet and has her personal views on
treatment. She would like to discuss with you on various treatment options.
Highest mark: 10 Lowest mark: 3.5
1) List THREE (3) ways in which anxiety can be distinguished from depression.
Any three of the following and other answers are acceptable. Examiner needs to use his or her discretion to
decide whether marks should be awarded.

Answers from students: Depression: predominant low mood or other symptoms (e.g.anhedonia,
energy),psychomotor retardation VS anxiety not predominant low mood; concerns of minor issues; usually no
psychosis, muscle tension, anticipatory anxiety
 Mental State Examination: Anxiety: nervous and fearful on Mental State Examination; Depression: sad,
furrow eye brow, eye looking down to the floor, tearful, withdrawn on Mental State Examination (1 mark)
 Somatic symptoms: Anxiety: more often associated with palpitations, tachycardia, dizziness; Depression:
most often associated with chest pain, headache (1 mark)
 Pregnancy and avoid adverse effects of psychotropic medication on foetus (1 mark)
 Anxiety: react to phobia; depression reactive to negative life event (1 mark)
 Future: Anxiety: worry or apprehension; Depression: hopelessness (1 mark)
 Anxiety: no diurnal variation, depression: diurnal variation (1 mark)
 Guilt: Anxiety: no guilt; Depression: Guilt (1 mark)
 Suicide: Anxiety: less likely to have suicide thought/ideation; depression: more likely to have suicidal
thought/ideation (1 mark)
 Family history: Anxiety: family history of anxiety disorder; depression: family history of depressive
disorder (1 mark)
 Psychotic feature: Anxiety: less likely to have psychotic feature; more likely to have mood congruent
psychotic features (1 mark)
2) She wants to try psychotherapy instead of antidepressant. List THREE (3) pieces of information
which you would inquire from the history taking to support the decision to consider psychotherapy
for this patient instead of antidepressant (3 marks).
Any three of the following and other answers are acceptable. Examiner needs to use his or her discretion to
decide whether marks should be awarded.
 The aetiology of depression is due to psychosocial (e.g. an event) causes rather than biological causes.(1
mark)
 Previous side effects associated with antidepressants. (1 mark)
 The severity of depressive symptoms is mild (1 mark)
 Previous good response to psychotherapy (1 mark)
 Patient is very insistent on psychotherapy or agree to complete 12 sessions (1 mark)
 Patient is psychological minded / good motivation/ in touch with feelings; explore life history; (1 mark)
 Patient is willing to do homework / afford the time to go for psychotherapy (1 mark)
 Clear focus – cognitive bias( 1mark)
 Patient may be unable to make informed decision regarding pharmacotherapy after initial assessment, and
may wish to consider it further (1 mark)
 Stable in mental state / no strong suicidal ideation/ not floridly psychotic (1 mark)
 Cannot tolerate side effects of antidepressants (1 mark)
 Patient is keen to augment psychotherapy with antidepressant (1 mark)

3) List TWO (2) poor prognostic factors for depressive disorder which you would inquire from the
history. (2 marks)
Any two of the following:
 Insidious onset (1 mark)
 Early onset and long history of depression (1 mark)
 Endogenous depression (1 mark)
 Low self – esteem ( 1 mark)
 Psychotic symptoms
 Past and multiple hospitalizations for depression (1 mark)
 Past suicide attempts or strong suicidal ideation ( 1mark)
 co-morbid dysthymic disorder or physical illness (1 mark)
 substance or alcohol abuse (1 mark)
 comorbid anxiety disorder symptoms or personality disorder (1 mark)
 past history of recurrent depressive episodes or failure in antidepressant treatment (1 mark)
 poor inter-episode recovery (1 mark)
 Strong family history of depressive disorder (1 mark)
 Poor socio-economic status or social support (1mark)
 Non-compliance to past treatment or poor insight (1 mark)

4) She read the internet and discovered a medication called clonazepam. She believes
that clonazepam is effective in treating mixed anxiety and depression. What is your
advice to the patient (2 marks).

1) Inform patient that clonazepam is only indicated for short term use and it has addiction
potential (1 mark)
2) Advise patient to consider selective serotonin reuptake inhibitor (SSRI) as regular treatment
for mixed anxiety and depression (1 mark)
Question 2 You are a resident working in renal medicine. A 45-year-old woman
suffering from schizophrenia and chronic renal failure. Due to her deteriorating renal
function, she was admitted to the renal ward 3 days ago. She has been receiving
haemodialysis for the past but wants to discontinue haemodialysis from today onwards.

1. List FOUR (4) questions you would ask the patient to assess her capacity to make
decision to discontinue haemodialysis. (4 marks)

Answers:
Any four of the following and other answers are acceptable. Examiner needs to use his or her discretion to
decide whether marks should be awarded.

1) Does the patient know her medical condition (i.e. renal failure) or the indication for dialysis? (1
mark)
2) Does the patient know the consequence for stopping the haemodialysis / risks and benefits of
haemodialysis? (1 mark)
3) Can the patient retain information about her condition and haemodialysis and use those
information to make a decision? (1 mark)
4) Does she experience any schizophrenia symptom which may affect her understanding of her
condition and haemodialysis? (1 mark)
5) What is her reason for stopping the dialysis or is the patient able to communicate her decision
in written or verbal format? (1 mark)

2. What are the TWO (2) most important questions you would ask the ward nurse taking care of
her? Answer must be different from Q.1. (2 marks)

Answers:
Any two of the following and other answers are acceptable. Examiner needs to use his or her discretion to
decide whether marks should be awarded.
1) Does the patient exhibit any first rank symptom (e.g. third person auditory hallucination, thought
interference, passivity experience or delusion of persecution) in the ward? (1 mark)
2) Does the patient exhibit any sign of delirium (e.g. disorientation, visual hallucination) in the ward? (1
mark)
3) Does she experience any recent complication associated with haemodialysis? (1 mark)

3. What are the TWO (2) most important questions you would ask her family members?
Answer must be different from Q.1 and Q.2. (2 marks)

Answers:
Any two of the following and other answers are acceptable. Examiner needs to use his or her discretion to
decide whether marks should be awarded.
1) Does she encounter any financial difficulty associated with haemodialysis? (1 mark)
2) Is she compliant to antipsychotic medication? (1 mark)
3) Did she refuse haemodialysis in the past? (1 mark)

4. State one indication in which you might request an ethics consult for this case. (2 mark)

Other answers are acceptable. Examiner needs to use his or her discretion to decide whether marks should be
awarded.
The patient has the capacity to make decision and insists to refuse dialysis and her renal functions become
critical and she is in life threatening situation. (2 marks) or

There is a dilemma between autonomy and beneficence (2 marks)

Question 3 You are a general practitioner. A 70-year-old man came to your clinic and needs to be
certified that he is fit to drive. He has seen a private psychiatrist for 3 years but not certain about his
diagnosis. He is taking psychotropic medications. He has good past health and does not suffer from other
chronic medical illness.

1. List TWO (2) classes of psychotropic medications and adverse drug reactions that have a
negative impact on his ability to drive. (3 marks)

Classes of psychotropic medications Adverse drug reactions which have a negative impact on
driving

( / 0.5 mark) ( / 1 mark)

( / 0.5 mark) ( / 1 mark)

Answer: Any two of the following:


Classes of psychotropic medications Adverse drug reactions which have a negative impact on driving (1
mark)
Tricyclic antidepressants (0.5 mark) Anticholinergic side effects lead to blurred vision or (1 mark)
Antiadrnergic side effects lead to postural hypotension and giddiness (1
mark) or
Anti-histamine effects and lead to drowsiness and result in road traffic
accident (1 mark)
Selective serotonin reuptake inhibitor Hyponatremia and lead to confusion or fit. (1 mark)
(0.5 mark)
Antipsychotic drug (0.5 mark) Extra-pyramidal side effect leading to rigidity and affect reaction (1
mark) or
Antiadrnergic side effects lead to postural hypotension and giddiness (1
mark).
Noadrenaline and specific serotonin Anti-histamine effects and lead to drowsiness and result in road traffic
antidepressant (0.5 mark) accident (1 mark)
Benzodiazepine (0.5 mark) Drowsiness and oversedation and result in road traffic accident (1 mark)
2. State TWO (2) different types of psychiatric disorders which make this 70-year-old man
unfit to drive.(2 marks)
Any two of the following:
1) Alzheimer’s disease or (1 mark)
2) Vascular dementia or (1 mark)
3) Lewy body dementia or (1 mark)
4) Fronto-temporal lobe dementia or (1 mark)
5) Late onset psychosis or (1 mark)
6) Late onset bipolar disorder or (1 mark)
7) Alcohol dependence or Alcohol use disorder (1 mark)

3. List TWO (2) questions you would ask the patient to assess his safety record in driving
(2 marks)

Any three of the following and other answers are acceptable. Examiner needs to use his or her
discretion to decide whether marks should be awarded.
1) Changes in driving patterns in the past year (e.g. difficulty to drive on expressway) (1 mark)
2) Use of seatbelt (1 mark)
3) Number of road traffic accidents in the past (1 mark)

4. State THREE (3) different and relevant physiological systems which you would examine to assess
his fitness to drive. Please explain how to assess each physiological system. (3 marks)

Physiological system How to assess each system

( / 0.5 mark) ( / 0.5 mark)

( / 0.5 mark) ( / 0.5 mark)

( / 0.5 mark) ( / 0.5 mark)

Physiological system How to assess each system

Eye or vision (0.5 mark) Snellen eye chart for vision (0.5 mark)

Auditory (0.5 mark) Whisper test for hearing (0.5 mark)

Musculoskeletal system (0.5 Cervical spine, hip, and knee mobility test for arthritis or
mark) Get-up-and-go test (0.5 mark)

Neurological system (0.5 mark) Neurological exam for signs of Parkinson’s disease (0.5
mark) or
Look for signs associated with cerebellum dysfunction
such as performing the balance test (0.5 mark) or
Get-up-and-go test (0.5 mark)
Question 4

A 40-year-old man used to be a successful banker in Singapore. He moved to Hong Kong 3 years
ago and started misusing cocaine there. His sister has noticed changes in his behaviour after he
misused cocaine. His behavioural changes include depression, exhaustion and oversleeping.

1. What is the most common route of administration of cocaine? (1 mark)


Answer: Via the intranasal route or sniff cocaine via nose ( 1 mark)

2. State FOUR (4) acute effects after consumption of cocaine? (4 marks)


Any 4 of the following:
1. Impaired in judgement (1 mark)
2. Euphoria (1 mark)
3. Anger (1 mark)
4. Change in sociability (1 mark)
5. Interpersonal sensitivity (1 mark)
6. Paranoia (1 mark)
7. Hallucination (1 mark)

3. How would you explain to his sister regarding his behavioural change? (2 marks)

Other answers are acceptable. Examiner needs to use his or her discretion to decide whether marks should be
awarded.

It is due to psychological withdrawal from cocaine and he “crashes” into depression, exhaustion and
overlapping. (2 marks) or
It is due to psychological withdrawal from cocaine and the symptoms are opposite to the effect of cocaine
intoxication. (2 marks)

4. He returned to Singapore 6 months ago. He could not find cocaine in Singapore and started to
misuse alcohol. His sister requests inpatient detoxification to help him to quit drinking. State
THREE (3) indications for inpatient detoxification. (3 marks)

Any three of the following:


History of delirium tremens (1 mark) or
History of withdrawal seizure (1 mark) or
Moderate or severe alcohol withdrawal symptoms (1 mark) or
Severe risk of self-harm or suicide (1 mark) or
Multiple past detoxifications (1 mark) or
Psychosis (1 mark) or
Severe cognitive impairment (1 mark) or
Very poor support at home (1 mark) or
Comorbid severe mental illness (e.g. severe depression or bipolar disorder (1 mark)
SAQ (Paper 14)

Question 1 (10 marks)

A 35-year-old woman presented with elated mood, grandiosity, high sexual drive, meeting
multiple sexual partners and racing thoughts. She was admitted to the medical ward because
your hospital does not have a psychiatric ward. She has history of bipolar disorder with last
episode being depressed. The inpatient medication record (IMR) shows that she is treated with
lithium 800mg at night but the serum level is 0.2 mmol/L (normal level: 0.4- 0.8 mmol/L). Her
renal function and thyroid function tests are normal. The IMR also shows that she is treated
with high-dose of bupropion 450mg per day. She has been using illicit drugs lately.

1. State the MOST IMPORTANT immediate action to prevent her mood from getting more
elated. (2 marks) Examiner: / 2 marks

Answer: Stop the bupropion immediately. (2 marks). Other answer: No mark.

2. State THREE (3) MOST IMPORTANT investigations which you should order based on
the history provided (3 marks) Examiner: /3 marks

Answer: 1) Urine drug screen (1 mark) or


2) Urine pregnancy test (1 mark) or
3) Screen for sexually transmitted disease (1 mark)

3. She admits that has stopped taking lithium due to its side effects. State THREE (3)
common side effects of lithium which she may experience (excluding impairments on
renal and thyroid functions because the laboratory results show normal findings) (3
marks) Examiner: / 3 marks

Any 3 of the following:


1. Metallic taste (1 mark) or
2. Thirst and increase in frequency of urine (1 mark) or
3. Fine tremor (1 mark) or
4. Weight gain (1 mark) or
5. Poor concentration (1 mark)

4. Name TWO (2) illicit drugs which are MOST LIKELY to lead to the above clinical
features. (2 marks) Examiner: / 3 marks

1. Amphetamine (1 mark) or
2. Cocaine (1 mark) or
3. MDMA abbreviation is acceptable for MDMA (1 mark)

Question 2 (10 marks)

1
A 30-year-old Singaporean man survived a bomb attack overseas one week ago. He has returned
to Singapore for further inpatient treatment. He suffers from burn and multiple fractures. His girl-
friend was killed in the bomb attack. When you interviewed him, he recalled seeing a lot of dead
bodies at the scene immediately after the bomb attack. He complained of anxiety. He could not
sleep at night and slept around 1-2 hours per night. Then he burst into tears when talking about
the death of his girlfriend.

1. After your assessment, you need to enter a psychiatric diagnosis in the medical record. What is
the current psychiatric diagnosis? (1 mark) Examiner: / 1 mark

Acute stress disorder (1 mark)

2. Besides major depressive disorder, state TWO (2) most possible psychiatric diagnoses
which he will develop in the next 6 months (2 marks). For each diagnosis, state THREE (3)
specific clinical features under each diagnosis (3 marks). Examiner: / 5 marks

Diagnosis 1: Diagnosis 2:

(Examiner: /1 mark) (Examiner: /1 mark)


Specific clinical feature 1 (for diagnosis 1) Specific clinical feature 1 (for diagnosis 2)

(Examiner: / 0.5 mark) (Examiner: / 0.5 mark)


Specific clinical feature 2 (for diagnosis 1) Specific clinical feature 2 (for diagnosis 2)

(Examiner: / 0.5 mark) (Examiner: / 0.5 mark)


Specific clinical feature 3 (for diagnosis 1) Specific clinical feature 3 (for diagnosis 2)

(Examiner: / 0.5 mark) (Examiner: / 0.5 mark)

Answers (other answers are acceptable. Marks will be awarded based on examiner’s discretion

Diagnosis 1: Post traumatic stress disorder Diagnosis 2: Abnormal grief

(Examiner: 1 mark) (Examiner: 1 mark)


Specific clinical feature 1 (for diagnosis 1) Specific clinical feature 1 (for diagnosis 2)

Flashback/intrusive recollection Strong suicidal ideation to join his late girl friend

(Examiner: 0.5 mark) (Examiner: 0.5 mark)


Specific clinical feature 2 (for diagnosis 1) Specific clinical feature 2 (for diagnosis 2)

Nightmare Psychotic features not related to his late girl

2
friend
(Examiner: 0.5 mark)
(Examiner: 0.5 mark)
Specific clinical feature 3 (for diagnosis 1) Specific clinical feature 3 (for diagnosis 2)

Avoidance to talk about topic of bomb attack Prolonged grief (last for several years) and
and visit the city where the bomb attack took impair his function
place.
(Examiner: 0.5 mark)
(Examiner: 0.5 mark)
Other answers: Other answers:
Hypervigilance Severe psychomotor retardation
Emotional numbness Severe worthlessness or guilt
Derealisation
Depersonalisation Please note that delayed grief and inhibited grief
Dissociative amnesia are unlikely based on case scenario.
Exaggerated startle response/autonomic arousal
Avoid reminder of the bomb attack
Detachment from family and friends

3. Based on his symptoms, state TWO CLASSESS of psychiatric medications which are most
suitable for this patient. Please name one specific example which you would prescribe. (2
marks) Examiner: / 2 marks

Psychotropic medication 1 Psychotropic medication 2


Class of psychotropic medication Class of psychotropic medication

( / 0.5 mark) ( / 0.5 mark)


Name one example which you would prescribe Name one example which you would prescribe

( / 0.5 mark) ( / 0.5 mark)

Answer:
Psychotropic medication 1 Psychotropic medication 2
Class of psychotropic medication Class of psychotropic medication
Benzodiazepine (Short term use) Selective serotonin reuptake inhibitor

( / 0.5 mark) ( / 0.5 mark)


Name one example which you would prescribe Name one example which you would prescribe
Lorazepam or diazepam Fluoxetine or fluvoxamine or paroxetine,
escitalopram or sertraline
( / 0.5 mark)
( / 0.5 mark)

4. State ONE (1) type of psychotherapy which is most beneficial for the patient (1 mark) and
explain how it works for this patient (1 mark) Examiner: / 2 marks

3
Answer:

Trauma-focused cognitive behaviour therapy (1 mark)

During the cognitive therapy, the psychologist will identify his inaccurate beliefs related to the bomb
attack (e.g. the patient will be guilty for not offering enough protection to his late girl friend or
should cancel the trip beforehand), the psychologist will challenge such beliefs and inform him that
the bomb attack was inevitable.

During behaviour therapy, the psychologist will assist the patient to recall the traumatic memories
or have in-vivo exposure to the objects related to bomb attack or his late girlfriend. The psychologist
will help him to practise relaxation exercise to inoculate his anxiety. (1 mark)

Other psychotherapies such as Eye Movement Desensitization and Reprocessing (EMDR) and grief
therapy are acceptable answers. Marks will be awarded based on the explanation offered by
student.

Question 3 (10 marks)

12 Failed (lowest: 2- 9)

4
A 70-year-old woman attempted suicide because she could not tolerate the drinking behaviours of
her husband and her son. You realise that it is important to treat the alcohol dependence of her
husband and her son to avoid future suicide attempt in the wife.

1. Comparing elder alcoholics and younger alcoholics, state TWO (2) specific phenomenon
which are more common in elder alcoholics (2 marks).

Answer may vary. Examiners will decide whether marks should be awarded.
1) Elder alcoholics are more likely to use cheaper alcohol such as Chinese wine (1 mark) or
2) Elder alcoholics are more likely to develop complications associated with alcohol misuse
(e.g. fall, hallucination if they have sensory impairment, head injury, delirium tremens,
Wernicke/Korsakoff/alcohol dementia/ liver cirrhosis due to prolonged use) (1 mark)
3) Elder alcoholics are less violent or aggressive when intoxicated when compared to
younger people (1 mark).
4) Elder alcoholics are more likely to drink at home (alone) or nearby hawker centre rather
than social occasions (1 mark) due to retirement or
5) Elder alcoholics are less likely to join alcoholic anonymous/ less motivated to quit (1
mark).

2. Based on the Cloninger’s Classification System, which type of alcoholism do the father and
son belong to? (1 mark)

Type II alcoholism (1 mark)

3. Based on your answer in Q.2, state THREE (3) specific clinical features (not mentioned in
the clinical scenario) to elicit from the father and son to support the type of alcoholism
which you have chosen. (3 marks)

Any three of the following:


- High level of risk taking behaviour (1 mark) or
- Affecting other males in the family besides the father and son (e.g. uncles, other male
cousins) (1 mark) or
- Early onset (1 mark) or
- Not due to stress or use alcohol to reduce anxiety (1 mark) or
- Not due to work requirement (1 mark) or
- Aggressive (1 mark) or
- Antisocial personality disorder or forensic history (1 mark)
- Comorbid substance abuse (1 mark)
- Less guilt or remorse over drinking/ more difficult to quit (1 mark)
- High tolerant to alcohol (1 mark)

4. Do you recommend controlled drinking or total abstinence for the father and son. Please
delete the inappropriate answer (1 mark)

Recommend controlled drinking / Recommend total abstinence

Recommend controlled drinking / Recommend total abstinence

5. State ONE (1) most commonly used medication which can help the father and son to quit
alcohol. (1 mark)

5
Answer: Naltrexone (1 mark)

6. State TWO (2) common side effects associated with the medication chosen in Q.5 (2
marks)
Any two of the following:
1. Abdominal pain/cramps ( 1 mark)
2. Diarrhoea (1 mark)
3. Anxiety ( 1 mark)
4. Other acceptable answers: Nausea, Headache, dizziness, fatigue, insomnia, liver
impairments

6
Question 4 (10 marks)

A 70-year-old man is brought by his family to see you. In the past one year, he developed poor
memory, seeing ghosts in the house and frequent falls. Physical examination reveals cogwheel and
lead-pipe rigidity in his upper limbs.

1. State the pathological hallmark of his condition (1 mark)

Answer: Lewy body (1 mark)

2. Based on clinical presentation, how to differentiate his condition from dementia caused by
Parkinson’s disease (2 marks)

Answer: For dementia related to Parkinson’s disease, the cognitive impairment has to be
after 1 year interval. (2 marks)

3. The family claims that he is very disturbed by the ghosts he sees at home. Please
recommend the MOST appropriate psychotropic medication for this patient (1 mark) and
justify your selection (1 mark)

Answer: I would prescribe quetiapine (1 mark) because it has low risk to cause extra-pyramidal side
effects (1 mark)

4. The family wants to prevent his memory from further deterioration. Please recommend
the MOST appropriate psychotropic medication for this patient (1 mark)

Answer: Rivastigmine (1 mark)

5. Based on your answer in Question 4, state TWO (2) common side effects (2 marks)

Answer: Muscle cramp (1 mark), diarrhoea(1 mark), bronchospasm (1 mark), insomnia (1 mark),
nausea/vomiting (1 mark), bradycardia (1 mark), anorexia/weight loss (1 mark), anxiety/agitation (1
mark), incontinence (1 mark)

6. His family complains that he does not sleep well at night. What is your explanation? (2
marks)

Answer: He suffers from rapid eye movement (REM) sleep disturbance and experiences a lot of
nightmares at night (2 marks). REM sleep disturbance is associated with Lewy Body Dementia.

7
Objective Structured Clinical Examination (OSCE) – (M4-R1)

Station No. 3 Date of OSCE: 21 August 2014

This OSCE is for summative assessment

of medical undergraduates.

Type OSCE: Intervention

SP:
 Age range: 30 - 60
 Gender: Female/Male
 Level of training: Trained

Designed by: Dr. Roger Ho Date: 25 July 2014

Updated 26 September 2013 1


Objective Structured Clinical Examination (OSCE) – (M4-R1)

Station No. 3 Date of OSCE: 21 August 2014

Introduction

OSCE Title: Oppositional Defiant Disorder

Construct: This is a station designed to test the candidate’s ability to take a history
of oppositional defiant disorder.

Objectives of the OSCE station:

To observe and assess candidate’s ability in:


1. Eliciting symptoms of oppositional defiant disorder
2. Can distinguish oppositional defiant disorder and conduct disorder
3. Expressing empathy and demonstrating good interview skills

Time: 8 minutes

Updated 26 September 2013 2


Objective Structured Clinical Examination (OSCE) – (M4-R1)

Station No. 3 Date of OSCE: 21 August 2014

Instructions to Candidates

Parent’s name: Ms. Tan or Mr. Tan

You are a general practitioner. Mr/Mrs Tan comes to see you. His/her 11-year-old
son exhibits difficult behaviour such as being disrespectful, spiteful and
argumentative at home and school. His behaviour starts to concern his parents and
teachers.

Tasks:
1. Take a history from the parent to establish a diagnosis.
2. Perform a risk assessment
3. At the end of your interview with the parent, please write down your diagnosis
and pass it to the examiner.

Time: 8 minutes

Updated 26 September 2013 3


Objective Structured Clinical Examination (OSCE) – (M4-R1)

Station No. 3 Date of OSCE: 21 August 2014

Student’s sticker

Answer sheet

Please write down the diagnosis of this patient and pass it to your examiner.

___________________________________________________________________

Updated 26 September 2013 4


Objective Structured Clinical Examination (OSCE) – (M4-R1)

Station No. 3 Date of OSCE: 21 August 2014

Instructions to Simulated Patients

Background
 Your name is Ms. Tan. You are married and has a single child, an 11-year-old
son called Jason. You are currently staying in a 3-room HDB flat. You worked as
an administrative officer in a company and your spouse also works as an
administrative staff in another company.
 You have a 11-month-old son (Jason). When yourself and your spouse go to
work, he is looking after by a domestic helper at home. He goes to school by
school bus. However, there were no problems in the pregnancy and giving birth.
He has normal development (e.g. normal in speaking, hearing and walking)
 Jason has average results from Primary 1 to Primary 4. He did not have
behavioural problems in the past.
 He is your first child. Yourself and your spouse cannot afford to have a second
child.
 Yourself and your spouse have no past or family history of mental illness.
Yourself and your spouse do not have forensic history.

Symptoms
Main symptoms
Jason currently studies in Primary 5 and he finds the subjects in Primary 5 are
difficult to study. In the past one year, he has been disrespectful to you, your spouse,
domestic helper and his teachers. He is noted to be defiant in almost any situation
(at home, going out with family and school). He finds his temper is bad and worse
than other children of his age. He has been argumentative, spiteful and challenges
you and the teacher the reasons to do homework. He wanted to go for a buffet
dinner last weekend but the family wants to save money and ask him to wait until
next school holiday. He was very anger and throws things at home.

Your spouse is very exhausted and gives in and tells your son that, “Yes, you do not
need to do your homework.” Jason then argues, “There is no need to study”. This
type of behaviour, over and over, leaves you and your spouse feeling overwhelmed
and exhausted. Your domestic helper finds Jason very annoying and blames her for
his mistakes.

You met the teacher last week. In this academic year, the teacher finds Jason
struggling in school and keeping his old friends. Jason deliberately annoys his
classmates. The teacher finds him refuse to take responsibility and blame his
classmates for his mistakes. His academic performance was above average last
year and he got a borderline pass this year. His results are similar across subjects.
He does not have a favoured subject. He has no attention problems in school and he
can sit still to do his work. He have not broken any school rule and never being
suspended before.

He never abuses animals.

Updated 26 September 2013 5


Objective Structured Clinical Examination (OSCE) – (M4-R1)

Station No. 3 Date of OSCE: 21 August 2014

His mood is normal. He sleeps and eats well. No self-harm and no suicide.
He is not nervous. He always attends school and never skips school.
There is no history of truancy.
He does not have other unusual experiences such as hearing voices or seeing
things that are not there.
He does not use recreational drugs. He does not sniff glue.
His IQ should be normal. He studies in a normal neighbourhood school.
He does not have specific learning problem (e.g. he can read, write, spell and do
mathematics)
He has normal attention and can sit still to study at home and in school.
He has normal development and there is no delay in walking and speaking.
He likes to watch TV and plays computer games. He is not addicted to internet.

Your views about your parenting

 You have poor relationship with your spouse.


 You agree that your parenting is ineffective because yourself and your spouse
are inconsistent. You are very harsh but your spouse is always very lenient.
 Both yourself and your spouse never punish Jason physically. Jason was never
abused.
 You agree that Jason may be lack of supervision when you and your spouse are
working. The domestic helper spends a lot of time on cleaning the house and no
one supervises him.

Risks
 Although his behaviour is difficult to manage, there are some good things
about Jason. Jason does not violate more serious rules like running away
from home or breaking school rules. He has never been physically aggressive
toward other people (parents, classmates) and animals. Jason never initiates
a fight. He does not bully other children. He lies about minor things but never
tells a big lie. He never steals and never damages others’ properties. He has
no trouble with the police.
 He does not harm himself
 He does not harm other people.
 He seldom has accident. E.g. falling from height due to excessive climbing or
running too fast.

Other points:
 You have not consulted any doctor or counsellor on this matter.
 You want to seek advice from the GP.

Time: 8 minutes

Updated 26 September 2013 6


Objective Structured Clinical Examination (OSCE) – (M4-R1)

Station No. 3 Date of OSCE: 21 August 2014

Updated 26 September 2013 7


Objective Structured Clinical Examination (OSCE) – (M4-R1)

Station No. 3 Date of OSCE: 21 August 2014

EXAMINER’S REPORT

Student’s Name: _________________ Matriculation No.: _____________ Examiner’s Name: ______________

Objectives very well Appropri patchy clearly not


(excellen ate needs to inadeq done
t) improve uate

1. Opens interview appropriately (greet, introduce, seek


permission, explains purpose) 2 1.5 1 0.5 0
Closes interview (summarizes, allows patient to clarify + to ask
questions, refers to future arrangements if any)
2. Creates safe environment, develops and maintains rapport,
controls interview appropriately (open ended questions,
respect, concern, sensitivity, empathy, responsive, normalizing 2 1.5 1 0.5 0
statements, avoids jargon)

3. Enquires about symptoms of Oppositional Defiant Disorder


a. Duration (1 year in this case)
b. Annoying and argue with other people
c. Blame others for mistake 4 3 2 1 0
d. Compliance refusal (e.g. refuse to study)
e. Defiant or Spiteful and Angry
f. Temper tantrums
4. Assess for symptoms for Conduct Disorder
a. Violates the rights of other people/animals: e.g.
bullying, fighting and torturing animals.
b. Vandalism and destroy properties/fire setting
4 3 2 1 0
c. Break rules at home or school
d. Forensic history: shoplifting, use of weapon, trouble
with police.
e. Truancy / Substance abuse / other relevant symptoms
5. Assess for risk factors and impact of for ODD
a. Inconsistent parenting
b. Lack of supervision
4 3 2 1 0
c. Impact on school results
d. Impact on interpersonal relationship
e. Parental relationship
6. Assess for risk issues
a. Harm to self – accident, fall from height, self-harm. 3 2 1 0.5 0
b. Harm to others – aggression to other people/animals
7. Other relevant information (e.g. Comorbidity: ADHD, IQ,
learning difficulty, family history of psychiatric illness/ forensic 3 2 1 0.5 0
history of parents, developmental history etc)
8. Correct diagnosis: Oppositional defiant disorder (3 marks); Any
3 0 0 0 0
other diagnoses (0 marks)
Subtotal score (For administrator to enter). Examiner can leave it
blank.
DECISION (circle one): Pass / Fail (For failure, the total score must be less than 12.5).
Signature of Examiner: ____________________________ Date: ______________________

Total score: Administrator 1:______________________ Administrator 2: _________________

Updated 26 September 2013 8


Objective Structured Clinical Examination (OSCE) – (M4-R1)

Station No. 3 Date of OSCE: 21 August 2014

Updated 26 September 2013 9


Objective Structured Clinical Examination (OSCE) – (M4-R2)

Station No. 2 Date of OSCE: 2nd October, 2014

This OSCE is for summative assessment

of medical undergraduates.

Type OSCE: Intervention

SP:
 Age range: 30 - 50
 Gender: Female
 Level of training: Trained

Designed by: Dr. Roger Ho Date: 17th September 2014

Updated 26 September 2013 1


Objective Structured Clinical Examination (OSCE) – (M4-R2)

Station No. 2 Date of OSCE: 2nd October, 2014

Introduction

OSCE Title: Suicide risk assessment

Construct: This is a station designed to test the candidate’s ability to perform a


suicide risk assessment.

Objectives of the OSCE station:

To observe and assess candidate’s ability in:


1. To take a history of recent suicide attempt and assess suicide risk.
2. To assess depressive symptoms and other relevant history.
3. Recommend a management plan for suicide attempt.

Time: 8 minutes

Updated 26 September 2013 2


Objective Structured Clinical Examination (OSCE) – (M4-R2)

Station No. 2 Date of OSCE: 2nd October, 2014

Instructions to Candidates

Patient’s name: Ms. Gigi Lim

You are a resident working in the Accident and Emergency Department (AED). Ms.
Lim was bought to the AED by police because she tried to attempt suicide by
jumping from height. Your hospital is new and does not have a psychiatric ward.

Tasks:
1. Take a history from Ms Lim to assess her suicide attempt and current suicide
risk.
2. Assess depressive symptoms and obtain further relevant information.
3. At the end of your interview with Ms Lim, please select the MOST
APPROPRIATE management strategy and pass to the examiner.

Time: 8 minutes

Updated 26 September 2013 3


Objective Structured Clinical Examination (OSCE) – (M4-R2)

Station No. 2 Date of OSCE: 2nd October, 2014

Student’s sticker

Answer sheet

Please CIRCLE the MOST appropriate management strategy.

1. Discharge against medical advice.

2. Admit to the medical ward of your hospital.

3. Transfer the patient to the Institute of Mental Health.

Updated 26 September 2013 4


Objective Structured Clinical Examination (OSCE) – (M4-R2)

Station No. 2 Date of OSCE: 2nd October, 2014

Instructions to Simulated Patients

Background:
 You are Ms. Gigi Lim, a 35-year-old woman tried to jump down from the roof
of her HDB flat tonight.
 You are bought in by the police because someone spotted that you wanted to
jump down from a building and called the police who stopped you.
 You are an unemployed, recently tested to be pregnant in urine pregnancy
test 2 weeks ago. This is an unplanned pregnancy. You do not use
contraception.
 You are not married with your boyfriend. He is doing odd-job with unstable
income.

Suicide attempt
Why did you want to jump down tonight?
 After you had argued with your boyfriend tonight, you wanted to jump from
height. You have thought about jumping for 2 weeks since you know the urine
result is positive.
 Your relationship with him is not good in the past 1month. You have thought
of jumping down to end it all because he is not good to you and both of you
have no savings.
 You want to get married with him as you are two-week pregnant. You
discovered the urine pregnancy test was positive 2 weeks ago. You have
known this boyfriend for 1 year but relationship has been unstable. He is not
keen to get married. He works as an odd job worker with unstable income.
You feel hopeless because there will not be enough money to support the
baby after the baby is born.
 After you had argued with him, your boyfriend left the HDB flat.
 As your boyfriend does not look after you and will leave you alone while you
are pregnant, you think there is no way out but to die.

The incident of jumping from height


 You went up to the roof alone. You firmly believe that you would die by
jumping from the rooftop. You went there for a few times and planned for your
suicide if your boyfriend argues with you. Tonight, you are determined to die.
 You went up alone and thought that no one would discover.
 You drank two cans of beer to relax yourself so that it would be easier to jump
from height.
 You did not do other things to harm yourself (e.g. overdose of Panadol or cut
yourself).
 You sent a good bye sms message to your boyfriend and inform him that you
are going to die. You did not write a suicide letter.
 You stayed at the rooftop for 30 minutes. When you are about to jump, two
police officers came up and pulled you down.
 You were sent to the Accident and Emergency Department immediately.

Updated 26 September 2013 5


Objective Structured Clinical Examination (OSCE) – (M4-R2)

Station No. 2 Date of OSCE: 2nd October, 2014

Your current view of suicide:


 You still want to die as your boyfriend did not respond to your sms. I do not
think he cares about me.
 You do not feel sorry for the attempt.
 You want to die with the foetus in your body.
 If you are released from the AED, you will jump from height again and die with
your foetus to make your boy friend guilty.
 You do not want to be admitted to any hospital and insist to go back on your
own.

Your mood and other relevant information:


 In the past one month, due to frequent argument with your boy friend,
 Your mood is low (Throughout the day, no variation in the day).
 You lost your interest (e.g. shopping)/ not enjoy shopping nowadays
 Your sleep is poor (Cannot fall asleep, wake up 5 times at night, wake up at 5
am cannot fall asleep).
 Your appetite is poor.
 Your energy level is low.
 You have low sexual drive.
 You cannot concentrate when you watch TV.
 You feel hopeless.
 You feel guilty towards your foetus because you have no money to raise your
future baby.
 You feel very negative (hopeless) about the relationship with your boy friend
because he is an irresponsible person.
 You have no one to talk to when you are depressed.
 You have poor relationship with your family. You stay in your boy friend’s
HDB.
 Your boy friend gives you $50 per week.

Say “No” to the following:


 No past history of suicide
 You did not overdose with medication tonight.
 No stocking of medication or sharp objects at home.
 You did not cut yourself with pen knife.
 You have no religion.
 You have not attempted suicide before.
 You do not hear voices when stressed. No other false belief. (e.g. you
deserve severe punishment for minor mistake).
 No family history of suicide.

 You have not seen psychiatrist before.


 No admission to psychiatric ward in the past.
 No past medical problems (e.g. no thyroid problem, no chronic pain)
 No family history of mental illness
 No past history of mental illness

Updated 26 September 2013 6


Objective Structured Clinical Examination (OSCE) – (M4-R2)

Station No. 2 Date of OSCE: 2nd October, 2014

 Not seen by psychiatrist before


 Not on psychiatric medication
 No history of alcohol or sleeping pill use.
 Not seen by gynaecologist or obstetrician yet.

 No other relationship in the past.


 No past trauma
 No emptiness
 No fear of abandonment
 No identity disturbance
 No voices
 No friend to talk to
 Not in contact with family

Behaviour
 Depressed; Still suicidal
 If doctor says that he needs to transfer you to IMH, you do not look too happy
but you do not violently object.

Time: 8 minutes

Updated 26 September 2013 7


Objective Structured Clinical Examination (OSCE) – (M4-R2)

Station No. 2 Date of OSCE: 2nd October, 2014

Updated 26 September 2013 8


Objective Structured Clinical Examination (OSCE) – (M4-R2)

Station No. 2 Date of OSCE: 2nd October, 2014

EXAMINER’S REPORT

Student’s Name: _________________ Matriculation No.: _____________ Examiner’s Name: ______________

Objectives very well Appropri patchy clearly not


(excellen ate needs to inadeq done
t) improve uate

1. Opens interview appropriately (greet, introduce, seek


permission, explains purpose) 2 1.5 1 0.5 0
Closes interview (summarizes, allows patient to clarify + to ask
questions, refers to future arrangements if any)
2. Creates safe environment, develops and maintains rapport,
controls interview appropriately (open ended questions,
respect, concern, sensitivity, empathy, responsive, normalizing 2 1.5 1 0.5 0
statements, avoids jargon)

3. Enquires about the suicide attempt


a. Precipitant (Why the patient attempted suicide tonight,
what is the participant?)
b. Preparation of suicide (Plan, avoid discovery, sms
suicide message) 4 3 2 1 0
c. The suicide method (e.g. jumping, other methods e.g.
overdose, cutting and use of alcohol during suicide)
d. The consequences of suicide attempt (e.g. how was
she discovered?)
4. Assess for symptoms of depression and other relevant history
a. Mood symptoms (e.g. low mood, loss of interest)
b. Biological symptoms of depression (e.g. sleep,
appetite)
c. Cognitive & other symptoms of depression (e.g. guilty, 4 3 2 1 0
psychosis)
d. Other relevant history (past history of
depression/suicide, family history of
depression/suicide).
5. Assess current suicide risk at the AED
a. Her views towards the suicide attempt? (Any remorse)
2 1.5 1 0.5 0
b. Her current suicide plan / view on treatment (any plan
for further attempt)
6. Asses for the cause of current suicide attempt/ongoing stress
a. Poor relationship with boy friend and financial problem 2 1.5 1 0.5 0
b. 2-week pregnancy
7. Other relevant information (e.g. medical history: chronic pain,
family history of depression, substance use (benzodiazepine 2 1.5 1 0.5 0
use, alcohol), religion, social support, antenatal care.
8. Correct management: Send to IMH (2 marks); Other options (0
2 0 0 0 0
mark)
Subtotal score (For administrator to enter). Examiner can leave it
blank.

Updated 26 September 2013 9


Objective Structured Clinical Examination (OSCE) – (M4-R3)

Station No. 4 Date of OSCE: 13th November, 2014

This OSCE is for summative assessment

of medical undergraduates.

Type OSCE: Intervention

SP:
 Age range: 40-65
 Gender: Male or female

Designed by: Dr. Roger Ho Date: 3 November 2014

Updated 14 August 2012 1


Objective Structured Clinical Examination (OSCE) – (M4-R3)

Station No. 4 Date of OSCE: 13th November, 2014

Introduction

OSCE Title: Cognitive Assessment

Construct: This is a station designed to test the candidate’s ability to perform a


cognitive assessment and elicit signs of post-concussion syndrome

Objectives of the OSCE station:

To observe and assess candidate’s ability in:


1. Interview and communication skills
2. Carry out a cognitive assessment, in particular frontal lobe assessment and Mini-
mental state examination (MMSE)

Time: 8 minutes

Updated 14 August 2012 2


Objective Structured Clinical Examination (OSCE) – (M4-R3)

Station No. 4 Date of OSCE: 13th November, 2014

Instructions to Candidates

Patient’s Name: Mr. Tan

You are a General Practitioner. A 45-year-old man comes to see you today because
he injured his head after he was assaulted by another man in a pub.

Tasks:
1. Obtain the answer sheet (with your name label on it).
2. Take a brief history (less than 4 minutes) from the patient to assess post-
concussion syndrome, risk assessment and sequelae after head injury.
3. Perform cognitive tasks listed on the left hand side of the table found on your
answer sheet.
4. Fill the right hand side of the table and provide the scores for the patient and
indicate maximum score for each task.

You are NOT required to perform a physical examination.

Time: 8 minutes

Updated 14 August 2012 3


Objective Structured Clinical Examination (OSCE) – (M4-R3)

Station No. 4 Date of OSCE: 13th November, 2014

Instructions to Simulated Patients

Background of patient

 Your name is Mr. Tan, a 45-year-old married person, staying with your spouse and son.
You work as an account manager. You have university education.
 You have NO personal or family history of mental illness or dementia.
 You have good past health.
 You are a non-smoker and a social drinker. You go to drink in a pub once a while.
 One month ago, you went to a pub and drank with your friends. You had an argument
with another man in the pub. He used a baseball bat to hit the front part and both sides
of your head.
 You had a black-out for 30 minutes and your scalp was bleeding from superficial wound.
Your friends sent you to the hospital immediately. You can remember events before and
after the fight.
 You were treated at the Accident and Emergency Department. The doctor said the head
injury was mild. You were not required to stay in the hospital. The brain scan did not
show major internal bleeding in the head. No surgery was required. No bleeding from
ear, nose and throat. No bruising from eyes.
 The man who attacked you left the pub and police could not catch him. Insurance
company thinks the injury is mild and refuses to compensate.

Problems after head injury

 After head injury, you find your brain function not very good. You cannot concentrate at
accounting work and often make mistakes. Your supervisor asks you to take no pay
leave to consult a doctor. You cannot plan or make decision.
 In your head, you experience headache over the frontal part of your head and giddiness.
You lost your balance a few times after getting up from bed and almost fell down.
 For your eyes and ears, you are very sensitive and become irritable to light and noise.
You hear “wee” sound in your eyes (Tinnitus).
 For your mental functions, you are slow in mental speed and cannot think as fast as the
past. You have noticed increased difficulty in finding the right words to express yourself.
You are also unable to play crossword puzzles which you used to enjoy as you are
unable to come up with the right words.
 Your memory is failing you: misplacing things (wallet), forgetting recent conversations
you had with your family, forgetting what you just ate for meals.
 For your mood, you are sad all the time but sometimes becomes angry for no reason,
poor appetite, poor sleep (4 hours at night, cannot fall asleep), low energy level, loss of
interest (e.g. tennis), loss of sexual drive, poor concentration and attention, feels
hopeless after the head injury. Your mood swings from normal mood to anger.
 You do not have suicidal thought. You do not hear voices or see things that are not
there.
 You feel nervous most of the time and easily startled. You worry that the head injury
caused a lot of problems in your head. You cannot sit still, need to pace around in the

Updated 14 August 2012 4


Objective Structured Clinical Examination (OSCE) – (M4-R3)

Station No. 4 Date of OSCE: 13th November, 2014

house. No sweating, no diarrhea, no difficulty in breathing, no chest pain and no panic


attack.
 You forget to switch fire and almost caught the whole house on fire. Luckily, no flooding
at home so far.
 You can handle money and ATM machine.
 Your wife and son are concerned as you become more irritable and angry. This affect
your relationship with them. They want you to see a doctor. You are so irritable and
argued with your friends. Now, you do not want to go out to meet people. You mainly
stay at home most of the time.

 No nightmare, no flashbacks, no avoidance of pub, no reliving of experiences


 No difficulty in breathing, no shortness of breath, no panic attack
 No fear of losing control
 No fear of dying/going to die
 No fear to centre of attention, no fear of talking to other people.
 No repetitive checking or hand-washing
 No repetitive thoughts
 No suicidal thought
 Not hearing voices
 Not an anxious or angry person In the past
 You do not have a driving license and you do not drive.

No other treatment/ no other problems.


 No past medical problems (e.g. no stroke), no surgery done, no head injury
 No family history of mental illness
 No past history of mental illness, no memory problems
 Not seeing other doctors at this moment.
 Not seen by psychiatrist or psychologist before
 Not on psychiatric medication nor other medications.
 No past hospitalisation
During memory testing

 Be cooperative and helpful to the candidate.


 You are able to repeat three words (out of 3 words) named by student.
e.g. Student told you “apple, newspaper and pencil”. You say “apple, newspaper and
pencil”.
 Subtracting 7 from 100. You do this test fairly quickly. Able to say “93”, but get all the
remaining 4 wrong. For example, 100-7 = 93; 93-7 = 90: 90-7 = 85; 85 – 7 = 80; 80 – 7 =
75.
 If students asks you to spell the word “WORLD” backwards, say D and get all other
letters wrong.
 The student ask you to say English words after an alphabet. For example, letter F. Say a
3 words Friend, Fly, face and then repeat the 3 words again slowly.
 Unable to recall the three items which were told (e.g. apple, newspaper and pencil) and
you come up with another 3 items (orange, train and tea).

Time: 8 minutes

Updated 14 August 2012 5


Objective Structured Clinical Examination (OSCE) – (M4-R3)

Station No. 4 Date of OSCE: 13th November, 2014

Updated 14 August 2012 6


Objective Structured Clinical Examination (OSCE) – (M4-R3)

Station No. 4 Date of OSCE: 13th November, 2014

Student’s sticker

Answer sheet

1. Take a history (4 minutes) to assess post-concussion symptoms, sequelae after


head injury and risk assessments

2. Please perform the following cognitive assessments and fill the right side of the
table

Mental tasks Please indicate patient’s scores and


maximum scores for each task
Assess registration
Patient’s score:

Maximum score:

Assess attention and


concentration Patient’s score:

Maximum score:

Assess verbal fluency


with one alphabet in Number of words mentioned by patient:
one minute

Average number of words mention by a normal


person in one minute:

Assess short term


recall Patient’s score:

Maximum score:

3. Please pass this form to the examiner.

Updated 14 August 2012 7


Objective Structured Clinical Examination (OSCE) – (M4-R3)

Station No. 4 Date of OSCE: 13th November, 2014

Student’s Name: _________________ Matriculation No.: _____________ Examiner’s Name: ______________

Objectives very well Appropri patchy clearly not


(excellen ate needs to inadeq done
t) improve uate

Basic Communication Skills


1. Opens interview appropriately (greet, introduce, seek
permission, explains purpose), Creates safe environment 2 1.5 1 0.5 0
Closes interview (summarizes, allows patient to clarify + to ask
questions, refers to future arrangements if any)
Post-concussion symptoms
2. Take an appropriate history to assess background of head
injury (e.g. type of injury, treatment, duration of blackout, 2 1.5 1 0.5 0
amnesia before and after injury, duration of impairment.)
3. Assess neurological symptoms (e.g. headache, giddiness,
2 1.5 1 0.5 0
tinnitus)
4. Assess psychiatric symptoms (e.g. anxiety, depression,
2 1.5 1 0.5 0
irritability)
5. Assess cognitive symptoms (e.g. slow motor speed, poor
2 1.5 1 0.5 0
concentration, attention, memory loss )
6. Risk assessment (e.g. fall, forget to turn off stove/fire, flooding
at home) or background history: alcohol abuse, past medical 2 1.5 1 0.5 0
history/
Brief cognitive assessment
7. Competency to assess registration of 3 items and correct
scores: 2 1.5 1 0.5 0
Patient’s scores = 3 Maximum scores 3
8. Competency to perform serial 7 or WORLD backwards
2 2 1 0.5 0
Patient’s scores = 1 Maximum scores 3
9. Verbal fluency
2 1.5 1 0.5 0
Patient’s scores = 3 Average number of a normal person: 10-15
10. Competency to assess recall of 3 items
2 0 0 0 0
Patient’s scores = 0 ; Maximum scores 0
Subtotal score (For administrator to enter). Examiner can leave it
blank.

Updated 14 August 2012 8


Station No. 1 Date of OSCE: 8 January, 2015

This OSCE is for summative assessment

of medical undergraduates.

Type OSCE: Intervention

SP:
 Age range: 30 - 49
 Gender: Female
 Level of training: Trained

Designed by: Dr. Roger Ho Date: 5 January, 2015

Updated 26 September 2013 1


Station No. 1 Date of OSCE: 8 January, 2015

Introduction

OSCE Title: Borderline personality disorder

Construct: This is a station designed to test the candidate’s ability to take a history
of borderline personality disorder.

Objectives of the OSCE station:

To observe and assess candidate’s ability in:


1. Eliciting symptoms of borderline personality disorder
2. Assess common psychiatric comorbidity associated with borderline personality
disorder
3. Establishing a diagnosis
4. Expressing empathy and demonstrating good interview skills

Time: 8 minutes

Updated 26 September 2013 2


Station No. 1 Date of OSCE: 8 January, 2015

Instructions to Candidates

Patient’s name: Amy

You are a resident working at the Accident and Emergency Department (AED). Amy,
a 35-year-old woman comes to see you because she has cut her thighs. Your
colleague has put dressing on her thighs and the wounds are superficial.

Tasks:
1. Take a history from Amy to assess her personality and establish a diagnosis.
2. Assess common psychiatric comorbidity associated with her diagnosis.
3. At the end of your interview with Amy, please write down the most likely
psychiatric diagnosis and pass it to the examiner.

You are NOT required to examine the wound.

Time: 8 minutes

Updated 26 September 2013 3


Station No. 1 Date of OSCE: 8 January, 2015

Student’s sticker

Answer sheet

Please write down the most likely psychiatric diagnosis of this patient and
pass it to your examiner.

___________________________________________________________________

Updated 26 September 2013 4


Station No. 1 Date of OSCE: 8 January, 2015

Instructions to Simulated Patients

Background:
 Your name is Amy, a 35-year-old woman. You live alone in a one-room HDB flat.
You are estranged from your family, being an only child whose father died of
cancer when you were 6 years old, and who has a very difficult relationship with
her mother. You have a history of physical abuse in childhood and her mother
was the perpetuator and she caned you very often for no reason. You are not in
contact with your mother. There is no history of sexual abuse.
 You had a number of relatively short-lived jobs in the past as a caregiver for the
elderly in nursing home. You resigned from your last job after an argument with
the manager and have been unemployed for several years.
 You consult the accident and emergency department today because you are very
stressed and cut your thigh.
 You have no known medical illness. No known family history of mental illness.

Symptoms
Main symptoms:

 The doctor may ask you a question how to describe yourself. You tell the doctor
that you don’t know how to describe yourself. You are confused about who you
are. You are lost about yourself.

 The doctor may ask you how your friends describe you. You tell the doctor that
you have a small number of close female friends, but find that people often let
you down so you do not have a wide social network. You feel that people in
general do not like you. It is very hard to maintain relationship with others.

 The reason for cutting yourself is to re-experience the pain. The pain inflicted
by your mother during the childhood abuse. You are not suicidal at this moment.
You always cut your thighs with pen knife. You have no intention to kill yourself.

 Your experience the following symptoms since young:


o Feel very empty inside yourself since young, cutting yourself since
adolescence
o Fear of being abandoned (see details below)
o Impulsive, like to cut your thighs, binge eating (i.e. eat a lot of cookies)
when stressed but does not vomit out the food. You don’t regret about
cutting.
o For your mood, hard to say whether you are depression You have mood
swing or fluctuation: normal mood can become sudden anger and
irritability for no reason.
o You are confused about your own identity (e.g. you do not know who you
are).

Updated 26 September 2013 5


Station No. 1 Date of OSCE: 8 January, 2015

o You had suicidal thoughts in the past (see details below).


o You saw angels when you were stressed. The angels appeared in the
corner of your flat. They just stood there but did not say anything. When
doctors ask you to elaborate further about the angels. You can’t. You do
not hear voices when nobody is around. You are not paranoid (i.e. does
not believe any one wants to harm you)
o You do tend to classify people as either all bad or all good. For example, in
your previous work place, the manager was the bad guy and other co-
workers were all good. No one was in between.

Relationship: You have a history of brief unstable relationships. You had 10 boy
friends in the past who were physically abusive. You have not been in a close
relationship for several years and no recent sexual activities..

Past suicide attempts: You are well apart from the lacerations, with no significant
medical history. You have scars on your thighs from past superficial lacerations. You
have had several past psychiatric admissions for self-harm or suicidality by drug
overdose across the last few years.
You use emergency A&E services at times. You have recently been admitted with
overdoses twice in the last month. “I’m just fed up and I can’t go on.”

Abandonment by your psychiatrist and psychologist: You are supposed to be


having follow-up from the psychiatrist at a general hospital. You are currently saying
that the psychiatrist has abandoned her and don’t care. You are not able to talk to a
staff member at the psychiatric department who knew you, as the receptionist of the
clinic said that both your doctor and psychologist were away during Christmas.

Other problems:
 You have nightmare, the images of childhood abuse popping up into your mind.
 You have binge eating when under stress but no self-induced vomiting.
 For your mood, very unpredictable and always have mood swings.
 Your sleep, appetite and energy are normal.
 You are not in any relationship at this moment and no sexual activity recently.

Say “No” to the following:


 No body image problem. You never put yourself on diet or do excessive exercise.
 You are not special when compared to other people.
 You do not like to be centre of attention.
 No past history of stealing, no problem with police.
 You do not drink alcohol, smoke cigarettes or use sleeping pills or illicit drugs.
 You are not on any psychotropic medications currently.
 You have no memory problem. No past wandering from home.
 You do not have other unusual experiences e.g. being controlled by other people
or your thoughts being interfered.
 You do not feel low in mood. You do not have feelings of
hopelessness/worthlessness. You do not think that life is meaningless and have

Updated 26 September 2013 6


Station No. 1 Date of OSCE: 8 January, 2015

no suicidal thoughts. In other words, your mood is unpredictable, your appetite is


ok (no problem), your sleep is ok (no problems).
 You do not have much interest throughout your life.
 There is no previous history of accident.

 No other medical problem. When you were admitted, the blood tests show normal
results.
 No difficulty in breathing, no shortness of breath, no panic attack
 No fear of losing control
 No fear of dying/going to die
 No fear of going out of the house. No fear of using MRT or bus.
 No fear of going to a shopping mall or crowded place/enclosed space
 No fear to centre of attention, no fear of talking to other people.
 No fear of blood, height or spider.
 No repetitive checking or hand-washing
 No repetitive thoughts of contamination
 Not hearing voices

No other treatment/ no other problems.


 No past medical problems (e.g. no thyroid problem)
 No family history of mental illness
 Not seeing other doctors at this moment.
 Not on psychiatric medication nor other medications.

Behaviour
 You are cooperative during the interview.

Time: 8 minutes

Updated 26 September 2013 7


Station No. 1 Date of OSCE: 8 January, 2015

Updated 26 September 2013 8


Station No. 1 Date of OSCE: 8 January, 2015

EXAMINER’S REPORT

Student’s Name: _________________ Matriculation No.: _____________ Examiner’s Name: ______________

Objectives very well Appropri patchy clearly not


(excellen ate needs to inadeq done
t) improve uate

1. Opens interview appropriately (greet, introduce, seek


permission, explains purpose) 2 1.5 1 0.5 0
Closes interview (summarizes, allows patient to clarify + to ask
questions, refers to future arrangements if any)
2. Open questions to allow patient to comment on her own
personality (1 mark) (e.g. How would you describe yourself as 2 1.5 1 0.5 0
a person?) and explore reason for cutting (1 mark)
Enquires about symptoms of borderline personality disorder

3. Chronic feeling of emptiness (1 mark) and fear of abandonment


2 1.5 1 0.5 0
(1 mark)
4. Unstable emotion (explore what constitutes mood swing,
2 1.5 1 0.5 0
normal mood to anger and irritability
5. Impulsive behaviour (e.g. past self harm-1 mark , binge eating /
2 1.5 1 0.5 0
stealing 1 mark)
6. Suicide risk assessment (e.g. current and past intention to die-
2 1.5 1 0.5 0
1 mark, explore past hospitalisation due to suicide – 1 mark)
7. Other borderline symptoms e.g. past unstable relationship /
explore transient psychotic feature / identity disturbance 2 1.5 1 0.5 0
defence mechanism – idealisation and devaluation or splitting
8. Assess for common comorbidity e.g. PTSD, depression,
2 1.5 1 0.5 0
bulimia, substance abuse, stealing)
9. Other relevant information (e.g. past trauma, childhood abuse,
past psychiatric history, family history social support etc) and 2 1.5 1 0.5 0
general performance of candidate
10. Correct diagnosis: Borderline personality disorder (2 marks for
full term and correct spelling); BPD abbreviation = 0 mark Any 2 0 0 0 0
other diagnoses: (0 mark)

Updated 26 September 2013 9


Objective Structured Clinical Examination (OSCE) – (M4-R3)

Station No. 3 Date of OSCE: 8th January 2015

This OSCE is for summative assessment

of medical undergraduates.

Type OSCE: Data interpretation

Designed by: Dr. Roger Ho Date: 8th January 2015.

Updated 14 August 2012 1


Objective Structured Clinical Examination (OSCE) – (M4-R3)

Station No. 3 Date of OSCE: 8th January 2015

Instructions to Candidates

You are a paediatrician working in a private medical group. A mother bought her 15-
year-old daughter, Angela to see your paediatric colleague because of low body
weight and lethargy. Angela used to be a high academic achiever in school but she
was not able to sit for examination in November last year. Your colleague has
ordered a set of investigations but he needs to go off to attend some urgent matter.
He has asked you to review the laboratory results and electrocardiogram (ECG) on
his behalf.

Tasks:

After entering the room, please review your colleagues’ record, laboratory results
and ECG and identify abnormalities.

PLEASE DO NOT WRITE OR LEAVE ANY MARKS ON THE LABORATORY


RESULTS AND ECG. There is a calculator in the room. Please clear your
answer from the calculator after usage.

Please answer the questions stated on the answer sheet.

Please double check your name is correct before answering the questions.

Please pass your answer sheet to the circuit administrator at the end of the station.

You are required to stay in the room until the end of the 8th minute. You cannot
leave the room if you finish early.

Time: 8 minutes

Updated 14 August 2012 2


Objective Structured Clinical Examination (OSCE) – (M4-R3)

Station No. 3 Date of OSCE: 8th January 2015

Outpatient record
(To be placed on the desk of the candidate’s room)

Miss Angela Chan


S1234567X 15 Y 11 M Female Visit date: 8 January, 2015

Doctor: Dr. Albert Tan

History:
Angela was bought in by her mother due to low body weight
and lethargy.

Used to be a high achiever in exam.

Weight: 40 kg
Height: 170 cm

PE:
Her physical examination showed marked malnutrition
and paleness.

Plan:
Order full blood count, liver function test, renal function test,
thyroid function test and electrocardiogram.

To review results; then speak to patient and mother.

Laboratory results:

Updated 14 August 2012 3


Objective Structured Clinical Examination (OSCE) – (M4-R3)

Station No. 3 Date of OSCE: 8th January 2015

(To be placed on the desk of the candidate’s room)

Full Blood Count

Test Results Unit Reference interval


White Blood Cell 4.83 X 10 9/L 3.40 – 9.60
Red Blood Cell 2.60 X 10 9/L 4.30 – 5.70
Haemoglobin 10.0 g/dL 12.9 to 17.0
Mean corpuscular 113.5 fL 80.0 – 95.0
volume (MCV)
Mean corpsular 38.5 Pg 27.0 – 33.0
haemoglobin (MCH)
Platelets 140 X 10 9/L 132 - 372

Liver Function Test

Test Results Unit Reference interval


Albumin 29 g/L 38 – 48
Bilirubin, Total 20 umol/L 5 – 30
Bilirubin, Conj 4 Umol/L 0–5
Bilirubin, Unconj 6 Umol/L 5 – 25
Aspartate aminotransferase (AST) 30 U/L 10 – 50
Alanine aminotransferase (ALT) 35 U/L 10 – 70

Renal Function Test

Test Results Unit Reference interval


Sodium 143 mmol/L 135 – 145
Potassium 3.1 mmol/L 3.5 – 5.0
Urea 4.8 mmol/L 2.5 – 7.5
Creatinine 92 mmol/L 65 – 125

Thyroid Function Test

Test Results Unit Reference interval


Free thyroxine 15 pmol/L 10 – 23
Thyroid stimulating hormone (TSH) 3.50 mIU/L 0.45 – 4.50

Updated 14 August 2012 4


Objective Structured Clinical Examination (OSCE) – (M4-R3)

Station No. 3 Date of OSCE: 8th January 2015

Insert Student’s Sticker

1. Based on her height and weight, calculate her body mass index (BMI). (2 marks)
(Examiner’s Use only: /2 marks)

2. Please review the laboratory results and electrocardiogram. Identify THREE (3)
abnormalities. (6 marks) (Examiner’s Use only: /6 marks)

3. Based on the above information, state the MOST LIKELY psychiatric diagnosis. (2 marks)
(Examiner’s Use only: /2 marks)

(PLEASE TURN OVER)

Updated 14 August 2012 5


Objective Structured Clinical Examination (OSCE) – (M4-R3)

Station No. 3 Date of OSCE: 8th January 2015

4. Based on your psychiatric diagnosis, state FOUR (4) additional questions which you would
ask the patient to confirm your diagnosis (Examiner’s Use only: /4 marks)

5. Based on your psychiatric diagnosis, state FOUR (4) additional clinical signs which you
would look for during physical examination to confirm your diagnosis. (Examiner’s Use only:
/4 marks)

6. Based on your psychiatric diagnosis, state TWO (2) additional investigations which your
colleague forgot to order but relevant to this condition (2 marks) (Examiner’s Use only:
/2 marks)

Total marks: Marker: ______________ Verification by administrator: _____________

Updated 14 August 2012 6


Mastering Psychiatry
A Core Textbook for Undergraduates
- Now with a companion guide to help you master your undergraduate exams!

The comprehensive textbook covers common psychiatric conditions


encountered in adults, children, adolescents and old people. The main textbook
provides core information you need for undergraduate examination and
future clinical practices.

This companion guide includes past exam papers to help you practice for the following
components in the actual exam:
• Multiple choice questions
• Short Answer questions
• OSCE stations

This book is a joint effort between authors from Singapore & Authors from the University of
Toronto.

Melvyn WB Zhang MBBS (Singapore), Diploma (Cl Psychiatry) (RCP&S Ireland), MRPCPsych(UK)
Associate Consultant, National Healthcare Group

Cyrus SH Ho MBBS (Singapore), Diploma (Cl Psychiatry) (RCP&S Ireland), MRCPsych (UK)
Associate Consultant, National University Healthcare System (NUHS)

Roger CM Ho MBBS (Hong Kong), DPM (Psych) (Ireland) Diploma (Cl Psychiatry) (RCP&S Ireland), DIP
(Psychotherapy) (NUS), MMed (Psych) (Singapore), MRCPsych (UK), FRCPC
Associate Professor and Consultant Psychiatrist, Department of Psychological Medicine,
Yong Loo Lin School of Medicine, National University of Singapore

Our online companion: www.masteringpsychiatry.com


[ Website QR] [Apple Store] [Android Store]

ISBN (Book): 978-981-07-0163-5


ISBN (DVD): 978-981-07-0164-2
ISBN (Ebook/App): 978-981-07-1493-2

You might also like